17.07.2024

Как Π½Π°ΠΉΡ‚ΠΈ Π΅ΠΌΠΊΠΎΡΡ‚ΡŒ кондСнсатора: Как Π½Π°ΠΉΡ‚ΠΈ напряТСниС Π½Π° кондСнсаторС Ρ„ΠΎΡ€ΠΌΡƒΠ»Π°

Π‘ΠΎΠ΄Π΅Ρ€ΠΆΠ°Π½ΠΈΠ΅

Как Π½Π°ΠΉΡ‚ΠΈ напряТСниС Π½Π° кондСнсаторС Ρ„ΠΎΡ€ΠΌΡƒΠ»Π°

Одним ΠΈΠ· Π²Π°ΠΆΠ½Ρ‹Ρ… элСмСнтов элСктричСской Ρ†Π΅ΠΏΠΈ являСтся кондСнсатор, Ρ„ΠΎΡ€ΠΌΡƒΠ»Ρ‹ для ΠΊΠΎΡ‚ΠΎΡ€ΠΎΠ³ΠΎ ΠΏΠΎΠ·Π²ΠΎΠ»ΡΡŽΡ‚ Ρ€Π°ΡΡΡ‡ΠΈΡ‚Π°Ρ‚ΡŒ ΠΈ ΠΏΠΎΠ΄ΠΎΠ±Ρ€Π°Ρ‚ΡŒ Π½Π°ΠΈΠ±ΠΎΠ»Π΅Π΅ подходящий Π²Π°Ρ€ΠΈΠ°Π½Ρ‚. Основная функция Π΄Π°Π½Π½ΠΎΠ³ΠΎ устройства Π·Π°ΠΊΠ»ΡŽΡ‡Π°Π΅Ρ‚ΡΡ Π² Π½Π°ΠΊΠΎΠΏΠ»Π΅Π½ΠΈΠΈ ΠΎΠΏΡ€Π΅Π΄Π΅Π»Π΅Π½Π½ΠΎΠ³ΠΎ количСства элСктроэнСргии. ΠŸΡ€ΠΎΡΡ‚Π΅ΠΉΡˆΠ°Ρ систСма Π²ΠΊΠ»ΡŽΡ‡Π°Π΅Ρ‚ Π² сСбя Π΄Π²Π° элСктрода ΠΈΠ»ΠΈ ΠΎΠ±ΠΊΠ»Π°Π΄ΠΊΠΈ, Ρ€Π°Π·Π΄Π΅Π»Π΅Π½Π½Ρ‹Π΅ ΠΌΠ΅ΠΆΠ΄Ρƒ собой диэлСктриком.

Π’ Ρ‡Π΅ΠΌ измСряСтся Π΅ΠΌΠΊΠΎΡΡ‚ΡŒ кондСнсатора

Одной ΠΈΠ· Π²Π°ΠΆΠ½Π΅ΠΉΡˆΠΈΡ… характСристик кондСнсатора являСтся Π΅Π³ΠΎ Π΅ΠΌΠΊΠΎΡΡ‚ΡŒ. Π”Π°Π½Π½Ρ‹ΠΉ ΠΏΠ°Ρ€Π°ΠΌΠ΅Ρ‚Ρ€ опрСдСляСтся количСством элСктроэнСргии, Π½Π°ΠΊΠ°ΠΏΠ»ΠΈΠ²Π°Π΅ΠΌΠΎΠΉ этим ΠΏΡ€ΠΈΠ±ΠΎΡ€ΠΎΠΌ. НакоплСниС происходит Π² Π²ΠΈΠ΄Π΅ элСктронов. Π˜Ρ… количСство, ΠΏΠΎΠΌΠ΅Ρ‰Π°ΡŽΡ‰Π΅Π΅ΡΡ Π² кондСнсаторС, опрСдСляСт Π²Π΅Π»ΠΈΡ‡ΠΈΠ½Ρƒ Смкости ΠΊΠΎΠ½ΠΊΡ€Π΅Ρ‚Π½ΠΎΠ³ΠΎ устройства.

Для измСрСния Смкости примСняСтся Π΅Π΄ΠΈΠ½ΠΈΡ†Π° – Ρ„Π°Ρ€Π°Π΄Π°. Π•ΠΌΠΊΠΎΡΡ‚ΡŒ кондСнсатора Π² 1 Ρ„Π°Ρ€Π°Π΄Ρƒ соотвСтствуСт элСктричСскому заряду Π² 1 ΠΊΡƒΠ»ΠΎΠ½, Π° Π½Π° ΠΎΠ±ΠΊΠ»Π°Π΄ΠΊΠ°Ρ… Ρ€Π°Π·Π½ΠΎΡΡ‚ΡŒ ΠΏΠΎΡ‚Π΅Π½Ρ†ΠΈΠ°Π»ΠΎΠ² Ρ€Π°Π²Π½Π° 1 Π²ΠΎΠ»ΡŒΡ‚Ρƒ. Π­Ρ‚Π° классичСская Ρ„ΠΎΡ€ΠΌΡƒΠ»ΠΈΡ€ΠΎΠ²ΠΊΠ° Π½Π΅ ΠΏΠΎΠ΄Ρ…ΠΎΠ΄ΠΈΡ‚ для практичСских расчСтов, ΠΏΠΎΡΠΊΠΎΠ»ΡŒΠΊΡƒ Π² кондСнсаторС ΡΠΎΠ±ΠΈΡ€Π°ΡŽΡ‚ΡΡ Π½Π΅ заряды, Π° элСктроны. Π•ΠΌΠΊΠΎΡΡ‚ΡŒ любого кондСнсатора находится Π² прямой зависимости ΠΎΡ‚ объСма элСктронов, способных Π½Π°ΠΊΠ°ΠΏΠ»ΠΈΠ²Π°Ρ‚ΡŒΡΡ ΠΏΡ€ΠΈ Π½ΠΎΡ€ΠΌΠ°Π»ΡŒΠ½ΠΎΠΌ Ρ€Π°Π±ΠΎΡ‡Π΅ΠΌ Ρ€Π΅ΠΆΠΈΠΌΠ΅. Для обозначСния Смкости всС Ρ€Π°Π²Π½ΠΎ ΠΈΡΠΏΠΎΠ»ΡŒΠ·ΡƒΠ΅Ρ‚ΡΡ Ρ„Π°Ρ€Π°Π΄Π°, Π° количСствСнныС ΠΏΠ°Ρ€Π°ΠΌΠ΅Ρ‚Ρ€Ρ‹ ΠΎΠΏΡ€Π΅Π΄Π΅Π»ΡΡŽΡ‚ΡΡ ΠΏΠΎ Ρ„ΠΎΡ€ΠΌΡƒΠ»Π΅: Π‘ = Q / U, Π³Π΄Π΅ Π‘ ΠΎΠ·Π½Π°Ρ‡Π°Π΅Ρ‚ Π΅ΠΌΠΊΠΎΡΡ‚ΡŒ, Q – заряд Π² ΠΊΡƒΠ»ΠΎΠ½Π°Ρ…, Π° U являСтся напряТСниСм. Π’Π°ΠΊΠΈΠΌ ΠΎΠ±Ρ€Π°Π·ΠΎΠΌ, просматриваСтся взаимная связь заряда ΠΈ напряТСния, ΠΎΠΊΠ°Π·Ρ‹Π²Π°ΡŽΡ‰ΠΈΡ… влияниС Π½Π° ΡΠΏΠΎΡΠΎΠ±Π½ΠΎΡΡ‚ΡŒ кондСнсатора ΠΊ накоплСнию ΠΈ ΡƒΠ΄Π΅Ρ€ΠΆΠ°Π½ΠΈΡŽ ΠΎΠΏΡ€Π΅Π΄Π΅Π»Π΅Π½Π½ΠΎΠ³ΠΎ количСства элСктричСства.

Для расчСтов Смкости плоского кондСнсатора ΠΈΡΠΏΠΎΠ»ΡŒΠ·ΡƒΠ΅Ρ‚ΡΡ Ρ„ΠΎΡ€ΠΌΡƒΠ»Π°:
Π² ΠΊΠΎΡ‚ΠΎΡ€ΠΎΠΉ Ξ΅ = 8,854187817 Ρ… 10 -12 Ρ„/ΠΌ прСдставляСт собой ΠΏΠΎΡΡ‚ΠΎΡΠ½Π½ΡƒΡŽ Π²Π΅Π»ΠΈΡ‡ΠΈΠ½Ρƒ. ΠŸΡ€ΠΎΡ‡ΠΈΠ΅ Π²Π΅Π»ΠΈΡ‡ΠΈΠ½Ρ‹: Ξ΅ – являСтся диэлСктричСской ΠΏΡ€ΠΎΠ½ΠΈΡ†Π°Π΅ΠΌΠΎΡΡ‚ΡŒΡŽ диэлСктрика, находящСгося ΠΌΠ΅ΠΆΠ΄Ρƒ ΠΎΠ±ΠΊΠ»Π°Π΄ΠΊΠ°ΠΌΠΈ, S – ΠΎΠ·Π½Π°Ρ‡Π°Π΅Ρ‚ ΠΏΠ»ΠΎΡ‰Π°Π΄ΡŒ ΠΎΠ±ΠΊΠ»Π°Π΄ΠΊΠΈ, Π° d – Π·Π°Π·ΠΎΡ€ ΠΌΠ΅ΠΆΠ΄Ρƒ ΠΎΠ±ΠΊΠ»Π°Π΄ΠΊΠ°ΠΌΠΈ.

Π€ΠΎΡ€ΠΌΡƒΠ»Π° энСргии кондСнсатора

Π‘ Π΅ΠΌΠΊΠΎΡΡ‚ΡŒΡŽ самым тСсным ΠΎΠ±Ρ€Π°Π·ΠΎΠΌ связана другая Π²Π΅Π»ΠΈΡ‡ΠΈΠ½Π°, извСстная ΠΊΠ°ΠΊ энСргия заряТСнного кондСнсатора. ПослС зарядки любого кондСнсатора, Π² Π½Π΅ΠΌ образуСтся ΠΎΠΏΡ€Π΅Π΄Π΅Π»Π΅Π½Π½ΠΎΠ΅ количСство энСргии, ΠΊΠΎΡ‚ΠΎΡ€ΠΎΠ΅ Π² дальнСйшСм выдСляСтся Π² процСссС разрядки. Π‘ этой ΠΏΠΎΡ‚Π΅Π½Ρ†ΠΈΠ°Π»ΡŒΠ½ΠΎΠΉ энСргиСй Π²ΡΡ‚ΡƒΠΏΠ°ΡŽΡ‚ Π²ΠΎ взаимодСйствиС ΠΎΠ±ΠΊΠ»Π°Π΄ΠΊΠΈ кондСнсатора. Π’ Π½ΠΈΡ… ΠΎΠ±Ρ€Π°Π·ΡƒΡŽΡ‚ΡΡ Ρ€Π°Π·Π½ΠΎΠΈΠΌΠ΅Π½Π½Ρ‹Π΅ заряды, ΠΏΡ€ΠΈΡ‚ΡΠ³ΠΈΠ²Π°ΡŽΡ‰ΠΈΠ΅ΡΡ Π΄Ρ€ΡƒΠ³ ΠΊ Π΄Ρ€ΡƒΠ³Ρƒ.

Π’ процСссС зарядки происходит расходованиС энСргии внСшнСго источника для раздСлСния зарядов с ΠΏΠΎΠ»ΠΎΠΆΠΈΡ‚Π΅Π»ΡŒΠ½Ρ‹ΠΌ ΠΈ ΠΎΡ‚Ρ€ΠΈΡ†Π°Ρ‚Π΅Π»ΡŒΠ½Ρ‹ΠΌ Π·Π½Π°Ρ‡Π΅Π½ΠΈΠ΅ΠΌ, ΠΊΠΎΡ‚ΠΎΡ€Ρ‹Π΅, Π·Π°Ρ‚Π΅ΠΌ Ρ€Π°ΡΠΏΠΎΠ»Π°Π³Π°ΡŽΡ‚ΡΡ Π½Π° ΠΎΠ±ΠΊΠ»Π°Π΄ΠΊΠ°Ρ… кондСнсатора. ΠŸΠΎΡΡ‚ΠΎΠΌΡƒ Π² соотвСтствии с Π·Π°ΠΊΠΎΠ½ΠΎΠΌ сохранСния энСргии, ΠΎΠ½Π° Π½Π΅ исчСзаСт бСсслСдно, Π° остаСтся Π²Π½ΡƒΡ‚Ρ€ΠΈ кондСнсатора Π² Π²ΠΈΠ΄Π΅ элСктричСского поля, сосрСдоточСнного ΠΌΠ΅ΠΆΠ΄Ρƒ пластинами. Π Π°Π·Π½ΠΎΠΈΠΌΠ΅Π½Π½Ρ‹Π΅ заряды ΠΎΠ±Ρ€Π°Π·ΡƒΡŽΡ‚ взаимодСйствиС ΠΈ ΠΏΠΎΡΠ»Π΅Π΄ΡƒΡŽΡ‰Π΅Π΅ притяТСниС ΠΎΠ±ΠΊΠ»Π°Π΄ΠΎΠΊ ΠΌΠ΅ΠΆΠ΄Ρƒ собой.

КаТдая пластина кондСнсатора ΠΏΠΎΠ΄ дСйствиСм заряда создаСт Π½Π°ΠΏΡ€ΡΠΆΠ΅Π½Π½ΠΎΡΡ‚ΡŒ элСктричСского поля, Ρ€Π°Π²Π½ΡƒΡŽ Π•/2. ΠžΠ±Ρ‰Π΅Π΅ ΠΏΠΎΠ»Π΅ Π±ΡƒΠ΄Π΅Ρ‚ ΡΠΊΠ»Π°Π΄Ρ‹Π²Π°Ρ‚ΡŒΡΡ ΠΈΠ· ΠΎΠ±ΠΎΠΈΡ… ΠΏΠΎΠ»Π΅ΠΉ, Π²ΠΎΠ·Π½ΠΈΠΊΠ°ΡŽΡ‰ΠΈΡ… Π² ΠΊΠ°ΠΆΠ΄ΠΎΠΉ ΠΎΠ±ΠΊΠ»Π°Π΄ΠΊΠ΅ с ΠΎΠ΄ΠΈΠ½Π°ΠΊΠΎΠ²Ρ‹ΠΌΠΈ зарядами, ΠΈΠΌΠ΅ΡŽΡ‰ΠΈΠΌΠΈ ΠΏΡ€ΠΎΡ‚ΠΈΠ²ΠΎΠΏΠΎΠ»ΠΎΠΆΠ½Ρ‹Π΅ значСния.

Π’Π°ΠΊΠΈΠΌ ΠΎΠ±Ρ€Π°Π·ΠΎΠΌ, энСргия кондСнсатора выраТаСтся Ρ„ΠΎΡ€ΠΌΡƒΠ»ΠΎΠΉ: W=q(E/2)d. Π’ свою ΠΎΡ‡Π΅Ρ€Π΅Π΄ΡŒ, напряТСниС выраТаСтся с ΠΏΠΎΠΌΠΎΡ‰ΡŒΡŽ понятий напряТСнности ΠΈ расстояния ΠΈ прСдставляСтся Π² Π²ΠΈΠ΄Π΅ Ρ„ΠΎΡ€ΠΌΡƒΠ»Ρ‹ U=Ed. Π­Ρ‚ΠΎ Π·Π½Π°Ρ‡Π΅Π½ΠΈΠ΅, подставлСнноС Π² ΠΏΠ΅Ρ€Π²ΡƒΡŽ Ρ„ΠΎΡ€ΠΌΡƒΠ»Ρƒ, ΠΎΡ‚ΠΎΠ±Ρ€Π°ΠΆΠ°Π΅Ρ‚ ΡΠ½Π΅Ρ€Π³ΠΈΡŽ кондСнсатора Π² Ρ‚Π°ΠΊΠΎΠΌ Π²ΠΈΠ΄Π΅: W=qU/2. Для получСния ΠΎΠΊΠΎΠ½Ρ‡Π°Ρ‚Π΅Π»ΡŒΠ½ΠΎΠ³ΠΎ Ρ€Π΅Π·ΡƒΠ»ΡŒΡ‚Π°Ρ‚Π° Π½Π΅ΠΎΠ±Ρ…ΠΎΠ΄ΠΈΠΌΠΎ ΠΈΡΠΏΠΎΠ»ΡŒΠ·ΠΎΠ²Π°Ρ‚ΡŒ ΠΎΠΏΡ€Π΅Π΄Π΅Π»Π΅Π½ΠΈΠ΅ Смкости: C=q/U, ΠΈ Π² ΠΊΠΎΠ½Ρ†Π΅ ΠΊΠΎΠ½Ρ†ΠΎΠ² энСргия заряТСнного кондСнсатора Π±ΡƒΠ΄Π΅Ρ‚ Π²Ρ‹Π³Π»ΡΠ΄Π΅Ρ‚ΡŒ ΡΠ»Π΅Π΄ΡƒΡŽΡ‰ΠΈΠΌ ΠΎΠ±Ρ€Π°Π·ΠΎΠΌ: Wэл = CU 2 /2.

Π€ΠΎΡ€ΠΌΡƒΠ»Π° заряда кондСнсатора

Для выполнСния зарядки, кондСнсатор Π΄ΠΎΠ»ΠΆΠ΅Π½ Π±Ρ‹Ρ‚ΡŒ ΠΏΠΎΠ΄ΠΊΠ»ΡŽΡ‡Π΅Π½ ΠΊ Ρ†Π΅ΠΏΠΈ постоянного Ρ‚ΠΎΠΊΠ°. Π‘ этой Ρ†Π΅Π»ΡŒΡŽ ΠΌΠΎΠΆΠ΅Ρ‚ ΠΈΡΠΏΠΎΠ»ΡŒΠ·ΠΎΠ²Π°Ρ‚ΡŒΡΡ Π³Π΅Π½Π΅Ρ€Π°Ρ‚ΠΎΡ€. Π£ ΠΊΠ°ΠΆΠ΄ΠΎΠ³ΠΎ Π³Π΅Π½Π΅Ρ€Π°Ρ‚ΠΎΡ€Π° имССтся Π²Π½ΡƒΡ‚Ρ€Π΅Π½Π½Π΅Π΅ сопротивлСниС. ΠŸΡ€ΠΈ Π·Π°ΠΌΡ‹ΠΊΠ°Π½ΠΈΠΈ Ρ†Π΅ΠΏΠΈ происходит зарядка кондСнсатора. ΠœΠ΅ΠΆΠ΄Ρƒ Π΅Π³ΠΎ ΠΎΠ±ΠΊΠ»Π°Π΄ΠΊΠ°ΠΌΠΈ появляСтся напряТСниС, Ρ€Π°Π²Π½ΠΎΠ΅ элСктродвиТущСй силС Π³Π΅Π½Π΅Ρ€Π°Ρ‚ΠΎΡ€Π°: Uc = E.

Обкладка, ΠΏΠΎΠ΄ΠΊΠ»ΡŽΡ‡Π΅Π½Π½Π°Ρ ΠΊ ΠΏΠΎΠ»ΠΎΠΆΠΈΡ‚Π΅Π»ΡŒΠ½ΠΎΠΌΡƒ ΠΏΠΎΠ»ΡŽΡΡƒ Π³Π΅Π½Π΅Ρ€Π°Ρ‚ΠΎΡ€Π°, заряТаСтся ΠΏΠΎΠ»ΠΎΠΆΠΈΡ‚Π΅Π»ΡŒΠ½ΠΎ (+q), Π° другая ΠΎΠ±ΠΊΠ»Π°Π΄ΠΊΠ° ΠΏΠΎΠ»ΡƒΡ‡Π°Π΅Ρ‚ Ρ€Π°Π²Π½ΠΎΠ·Π½Π°Ρ‡Π½Ρ‹ΠΉ заряд с ΠΎΡ‚Ρ€ΠΈΡ†Π°Ρ‚Π΅Π»ΡŒΠ½ΠΎΠΉ Π²Π΅Π»ΠΈΡ‡ΠΈΠ½ΠΎΠΉ (- q). Π’Π΅Π»ΠΈΡ‡ΠΈΠ½Π° заряда q находится Π² прямой ΠΏΡ€ΠΎΠΏΠΎΡ€Ρ†ΠΈΠΎΠ½Π°Π»ΡŒΠ½ΠΎΠΉ зависимости с Π΅ΠΌΠΊΠΎΡΡ‚ΡŒΡŽ кондСнсатора Π‘ ΠΈ напряТСниСм Π½Π° ΠΎΠ±ΠΊΠ»Π°Π΄ΠΊΠ°Ρ… Uc. Π­Ρ‚Π° Π·Π°Π²ΠΈΡΠΈΠΌΠΎΡΡ‚ΡŒ выраТаСтся Ρ„ΠΎΡ€ΠΌΡƒΠ»ΠΎΠΉ: q = C x Uc.

Π’ процСссС зарядки ΠΎΠ΄Π½Π° ΠΈΠ· ΠΎΠ±ΠΊΠ»Π°Π΄ΠΎΠΊ кондСнсатора ΠΏΡ€ΠΈΠΎΠ±Ρ€Π΅Ρ‚Π°Π΅Ρ‚, Π° другая тСряСт ΠΎΠΏΡ€Π΅Π΄Π΅Π»Π΅Π½Π½ΠΎΠ΅ количСство элСктронов. Они пСрСносятся ΠΏΠΎ внСшнСй Ρ†Π΅ΠΏΠΈ ΠΏΠΎΠ΄ влияниСм элСктродвиТущСй силы Π³Π΅Π½Π΅Ρ€Π°Ρ‚ΠΎΡ€Π°. Π’Π°ΠΊΠΎΠ΅ ΠΏΠ΅Ρ€Π΅ΠΌΠ΅Ρ‰Π΅Π½ΠΈΠ΅ являСтся элСктричСским Ρ‚ΠΎΠΊΠΎΠΌ, извСстным Π΅Ρ‰Π΅ ΠΊΠ°ΠΊ зарядный Смкостной Ρ‚ΠΎΠΊ (IΠ·Π°Ρ€).

Π’Π΅Ρ‡Π΅Π½ΠΈΠ΅ зарядного Ρ‚ΠΎΠΊΠ° Π² Ρ†Π΅ΠΏΠΈ происходит практичСски Π·Π° тысячныС Π΄ΠΎΠ»ΠΈ сСкунды, Π΄ΠΎ Ρ‚ΠΎΠ³ΠΎ ΠΌΠΎΠΌΠ΅Π½Ρ‚Π°, ΠΏΠΎΠΊΠ° напряТСниС кондСнсатора Π½Π΅ станСт Ρ€Π°Π²Π½Ρ‹ΠΌ элСктродвиТущСй силС Π³Π΅Π½Π΅Ρ€Π°Ρ‚ΠΎΡ€Π°. НапряТСниС увСличиваСтся ΠΏΠ»Π°Π²Π½ΠΎ, Π° ΠΏΠΎΡ‚ΠΎΠΌ постСпСнно замСдляСтся. Π”Π°Π»Π΅Π΅ Π·Π½Π°Ρ‡Π΅Π½ΠΈΠ΅ напряТСния кондСнсатора Π±ΡƒΠ΄Π΅Ρ‚ постоянным. Π’ΠΎ врСмя зарядки ΠΏΠΎ Ρ†Π΅ΠΏΠΈ Ρ‚Π΅Ρ‡Π΅Ρ‚ зарядный Ρ‚ΠΎΠΊ. Π’ самом Π½Π°Ρ‡Π°Π»Π΅ ΠΎΠ½ достигаСт максимальной Π²Π΅Π»ΠΈΡ‡ΠΈΠ½Ρ‹, Ρ‚Π°ΠΊ ΠΊΠ°ΠΊ напряТСниС кондСнсатора ΠΈΠΌΠ΅Π΅Ρ‚ Π½ΡƒΠ»Π΅Π²ΠΎΠ΅ Π·Π½Π°Ρ‡Π΅Π½ΠΈΠ΅. Богласно Π·Π°ΠΊΠΎΠ½Π° Ома IΠ·Π°Ρ€ = Π•/Ri, ΠΏΠΎΡΠΊΠΎΠ»ΡŒΠΊΡƒ ΠΊ ΡΠΎΠΏΡ€ΠΎΡ‚ΠΈΠ²Π»Π΅Π½ΠΈΡŽ Ri ΠΏΡ€ΠΈΠ»ΠΎΠΆΠ΅Π½Π° вся Π­Π”Π‘ Π³Π΅Π½Π΅Ρ€Π°Ρ‚ΠΎΡ€Π°.

Π€ΠΎΡ€ΠΌΡƒΠ»Π° Ρ‚ΠΎΠΊΠ° ΡƒΡ‚Π΅Ρ‡ΠΊΠΈ кондСнсатора

Π’ΠΎΠΊ ΡƒΡ‚Π΅Ρ‡ΠΊΠΈ кондСнсатора Π²ΠΏΠΎΠ»Π½Π΅ ΠΌΠΎΠΆΠ½ΠΎ ΡΡ€Π°Π²Π½ΠΈΡ‚ΡŒ с воздСйствиСм ΠΏΠΎΠ΄ΠΊΠ»ΡŽΡ‡Π΅Π½Π½ΠΎΠ³ΠΎ ΠΊ Π½Π΅ΠΌΡƒ рСзистора с ΠΊΠ°ΠΊΠΈΠΌ-Π»ΠΈΠ±ΠΎ сопротивлСниСм R. Π’ΠΎΠΊ ΡƒΡ‚Π΅Ρ‡ΠΊΠΈ тСсно связан с Ρ‚ΠΈΠΏΠΎΠΌ кондСнсатора ΠΈ качСством ΠΈΡΠΏΠΎΠ»ΡŒΠ·ΡƒΠ΅ΠΌΠΎΠ³ΠΎ диэлСктрика. ΠšΡ€ΠΎΠΌΠ΅ Ρ‚ΠΎΠ³ΠΎ, Π²Π°ΠΆΠ½Ρ‹ΠΌ Ρ„Π°ΠΊΡ‚ΠΎΡ€ΠΎΠΌ становится конструкция корпуса ΠΈ ΡΡ‚Π΅ΠΏΠ΅Π½ΡŒ Π΅Π³ΠΎ загрязнСнности.

НСкоторыС кондСнсаторы ΠΈΠΌΠ΅ΡŽΡ‚ Π½Π΅Π³Π΅Ρ€ΠΌΠ΅Ρ‚ΠΈΡ‡Π½Ρ‹ΠΉ корпус, Ρ‡Ρ‚ΠΎ ΠΏΡ€ΠΈΠ²ΠΎΠ΄ΠΈΡ‚ ΠΊ ΠΏΡ€ΠΎΠ½ΠΈΠΊΠ½ΠΎΠ²Π΅Π½ΠΈΡŽ Π²Π»Π°Π³ΠΈ ΠΈΠ· Π²ΠΎΠ·Π΄ΡƒΡ…Π° ΠΈ Π²ΠΎΠ·Ρ€Π°ΡΡ‚Π°Π½ΠΈΡŽ Ρ‚ΠΎΠΊΠ° ΡƒΡ‚Π΅Ρ‡ΠΊΠΈ. Π’ ΠΏΠ΅Ρ€Π²ΡƒΡŽ ΠΎΡ‡Π΅Ρ€Π΅Π΄ΡŒ это касаСтся устройств, Π³Π΄Π΅ Π² качСствС диэлСктрика использована промаслСнная Π±ΡƒΠΌΠ°Π³Π°. Π—Π½Π°Ρ‡ΠΈΡ‚Π΅Π»ΡŒΠ½Ρ‹Π΅ Ρ‚ΠΎΠΊΠΈ ΡƒΡ‚Π΅Ρ‡ΠΊΠΈ Π²ΠΎΠ·Π½ΠΈΠΊΠ°ΡŽΡ‚ ΠΈΠ·-Π·Π° сниТСния элСктричСского сопротивлСния изоляции. Π’ Ρ€Π΅Π·ΡƒΠ»ΡŒΡ‚Π°Ρ‚Π΅ Π½Π°Ρ€ΡƒΡˆΠ°Π΅Ρ‚ΡΡ основная функция кондСнсатора – ΡΠΏΠΎΡΠΎΠ±Π½ΠΎΡΡ‚ΡŒ ΠΏΠΎΠ»ΡƒΡ‡Π°Ρ‚ΡŒ ΠΈ ΡΠΎΡ…Ρ€Π°Π½ΡΡ‚ΡŒ заряд элСктричСского Ρ‚ΠΎΠΊΠ°.

Основная Ρ„ΠΎΡ€ΠΌΡƒΠ»Π° для расчСта выглядит ΡΠ»Π΅Π΄ΡƒΡŽΡ‰ΠΈΠΌ ΠΎΠ±Ρ€Π°Π·ΠΎΠΌ: IΡƒΡ‚ = U/Rd, Π³Π΄Π΅ IΡƒΡ‚, – это Ρ‚ΠΎΠΊ ΡƒΡ‚Π΅Ρ‡ΠΊΠΈ, U – напряТСниС, ΠΏΡ€ΠΈΠ»Π°Π³Π°Π΅ΠΌΠΎΠ΅ ΠΊ кондСнсатору, Π° Rd – сопротивлСниС изоляции.

По Π½Π°Π·Π½Π°Ρ‡Π΅Π½ΠΈΡŽ кондСнсатор ΠΌΠΎΠΆΠ½ΠΎ ΡΡ€Π°Π²Π½ΠΈΡ‚ΡŒ с Π±Π°Ρ‚Π°Ρ€Π΅ΠΉΠΊΠΎΠΉ. Но имССтся ΠΏΡ€ΠΈΠ½Ρ†ΠΈΠΏΠΈΠ°Π»ΡŒΠ½ΠΎΠ΅ ΠΎΡ‚Π»ΠΈΡ‡ΠΈΠ΅ Π² Ρ€Π°Π±ΠΎΡ‚Π΅ Π΄Π°Π½Π½Ρ‹Ρ… элСмСнтов. Π‘ΡƒΡ‰Π΅ΡΡ‚Π²ΡƒΡŽΡ‚ отличия Π² ΠΏΡ€Π΅Π΄Π΅Π»ΡŒΠ½ΠΎΠΉ Смкости ΠΈ скорости зарядки кондСнсатора ΠΈ Π±Π°Ρ‚Π°Ρ€Π΅ΠΉΠΊΠΈ.

Π€ΠΎΡ€ΠΌΡƒΠ»Π° заряда кондСнсатора

Π’Π΅Π»ΠΈΡ‡ΠΈΠ½Π° заряда кондСнсатора (q) связана с Π΅Π³ΠΎ Π΅ΠΌΠΊΠΎΡΡ‚ΡŒΡŽ (C) ΠΈ Ρ€Π°Π·Π½ΠΎΡΡ‚ΡŒΡŽ ΠΏΠΎΡ‚Π΅Π½Ρ†ΠΈΠ°Π»ΠΎΠ² (U) ΠΌΠ΅ΠΆΠ΄Ρƒ Π΅Π³ΠΎ ΠΎΠ±ΠΊΠ»Π°Π΄ΠΊΠ°ΠΌΠΈ ΠΊΠ°ΠΊ:

Π³Π΄Π΅ q – Π²Π΅Π»ΠΈΡ‡ΠΈΠ½Π° заряда ΠΎΠ΄Π½ΠΎΠΉ ΠΈΠ· ΠΎΠ±ΠΊΠ»Π°Π΄ΠΎΠΊ кондСнсатора, Π° – Ρ€Π°Π·Π½ΠΎΡΡ‚ΡŒ ΠΏΠΎΡ‚Π΅Π½Ρ†ΠΈΠ°Π»ΠΎΠ² ΠΌΠ΅ΠΆΠ΄Ρƒ Π΅Π³ΠΎ ΠΎΠ±ΠΊΠ»Π°Π΄ΠΊΠ°ΠΌΠΈ.

Π­Π»Π΅ΠΊΡ‚Ρ€ΠΎΠ΅ΠΌΠΊΠΎΡΡ‚ΡŒ кондСнсатора β€” это Π²Π΅Π»ΠΈΡ‡ΠΈΠ½Π°, которая зависит Ρ‚ΠΎ Ρ€Π°Π·ΠΌΠ΅Ρ€ΠΎΠ² ΠΈ устройства кондСнсатора.

Заряд Π½Π° пластинах плоского кондСнсатора Ρ€Π°Π²Π΅Π½:

Π³Π΄Π΅ – элСктричСская постоянная; – ΠΏΠ»ΠΎΡ‰Π°Π΄ΡŒ ΠΊΠ°ΠΆΠ΄ΠΎΠΉ (ΠΈΠ»ΠΈ наимСньшСй) пластины; – расстояниС ΠΌΠ΅ΠΆΠ΄Ρƒ пластинами; – диэлСктричСская ΠΏΡ€ΠΎΠ½ΠΈΡ†Π°Π΅ΠΌΠΎΡΡ‚ΡŒ диэлСктрика, ΠΊΠΎΡ‚ΠΎΡ€Ρ‹ΠΉ находится ΠΌΠ΅ΠΆΠ΄Ρƒ пластинами кондСнсатора.

Заряд Π½Π° ΠΎΠ±ΠΊΠ»Π°Π΄ΠΊΠ°Ρ… цилиндричСского кондСнсатора вычисляСтся ΠΏΡ€ΠΈ ΠΏΠΎΠΌΠΎΡ‰ΠΈ Ρ„ΠΎΡ€ΠΌΡƒΠ»Ρ‹:

Π³Π΄Π΅ l – высота Ρ†ΠΈΠ»ΠΈΠ½Π΄Ρ€ΠΎΠ²; – радиус внСшнСй ΠΎΠ±ΠΊΠ»Π°Π΄ΠΊΠΈ; – радиус Π²Π½ΡƒΡ‚Ρ€Π΅Π½Π½Π΅ΠΉ ΠΎΠ±ΠΊΠ»Π°Π΄ΠΊΠΈ.

Заряд Π½Π° ΠΎΠ±ΠΊΠ»Π°Π΄ΠΊΠ°Ρ… сфСричСского кондСнсатора Π½Π°ΠΉΠ΄Π΅ΠΌ ΠΊΠ°ΠΊ:

Π³Π΄Π΅ – радиусы ΠΎΠ±ΠΊΠ»Π°Π΄ΠΎΠΊ кондСнсатора.

Заряд кондСнсатора связан с энСргиСй поля (W) Π²Π½ΡƒΡ‚Ρ€ΠΈ Π½Π΅Π³ΠΎ:

Из Ρ„ΠΎΡ€ΠΌΡƒΠ»Ρ‹ (6) слСдуСт, Ρ‡Ρ‚ΠΎ заряд ΠΌΠΎΠΆΠ½ΠΎ Π²Ρ‹Ρ€Π°Π·ΠΈΡ‚ΡŒ ΠΊΠ°ΠΊ:

Рассмотрим ΠΏΠΎΡΠ»Π΅Π΄ΠΎΠ²Π°Ρ‚Π΅Π»ΡŒΠ½ΠΎΠ΅ соСдинСниС ΠΈΠ· N кондСнсаторов ( рис. 1).

Π—Π΄Π΅ΡΡŒ (рис.1) ΠΏΠΎΠ»ΠΎΠΆΠΈΡ‚Π΅Π»ΡŒΠ½Π°Ρ ΠΎΠ±ΠΊΠ»Π°Π΄ΠΊΠ° ΠΎΠ΄Π½ΠΎΠ³ΠΎ кондСнсатора соСдиняСтся с ΠΎΡ‚Ρ€ΠΈΡ†Π°Ρ‚Π΅Π»ΡŒΠ½ΠΎΠΉ ΠΎΠ±ΠΊΠ»Π°Π΄ΠΊΠΎΠΉ ΡΠ»Π΅Π΄ΡƒΡŽΡ‰Π΅Π³ΠΎ кондСнсатора. ΠŸΡ€ΠΈ Ρ‚Π°ΠΊΠΎΠΌ соСдинСнии, ΠΎΠ±ΠΊΠ»Π°Π΄ΠΊΠΈ сосСдних кондСнсаторов ΡΠΎΠ·Π΄Π°ΡŽΡ‚ Π΅Π΄ΠΈΠ½Ρ‹ΠΉ ΠΏΡ€ΠΎΠ²ΠΎΠ΄Π½ΠΈΠΊ. Π£ всСх кондСнсаторов, соСдинСнных ΠΏΠΎΡΠ»Π΅Π΄ΠΎΠ²Π°Ρ‚Π΅Π»ΡŒΠ½ΠΎ Π½Π° ΠΎΠ±ΠΊΠ»Π°Π΄ΠΊΠ°Ρ… ΠΈΠΌΠ΅ΡŽΡ‚ΡΡ Ρ€Π°Π²Π½Ρ‹Π΅ ΠΏΠΎ Π²Π΅Π»ΠΈΡ‡ΠΈΠ½Π΅ заряды.

ΠŸΡ€ΠΈ ΠΏΠ°Ρ€Π°Π»Π»Π΅Π»ΡŒΠ½ΠΎΠΌ соСдинСнии кондСнсаторов (рис.2), ΡΠΎΠ΅Π΄ΠΈΠ½ΡΡŽΡ‚ ΠΎΠ±ΠΊΠ»Π°Π΄ΠΊΠΈ, ΠΈΠΌΠ΅ΡŽΡ‰ΠΈΠ΅ заряды ΠΎΠ΄Π½ΠΎΠ³ΠΎ Π·Π½Π°ΠΊΠ°. Π‘ΡƒΠΌΠΌΠ°Ρ€Π½Ρ‹ΠΉ заряд соСдинСния (q) Ρ€Π°Π²Π΅Π½ суммС зарядов кондСнсаторов.

ΠŸΡ€ΠΈΠΌΠ΅Ρ€Ρ‹ Ρ€Π΅ΡˆΠ΅Π½ΠΈΡ Π·Π°Π΄Π°Ρ‡ ΠΏΠΎ Ρ‚Π΅ΠΌΠ΅ «Заряд кондСнсатора»

Π—Π°Π΄Π°Π½ΠΈΠ΅ΠšΠ°ΠΊΠΎΠ²Ρ‹ заряды Π½Π° ΠΎΠ±ΠΊΠ»Π°Π΄ΠΊΠ°Ρ… кондСнсаторов, Ссли ΠΎΠ½ΠΈ ΠΈΠΌΠ΅ΡŽΡ‚ Смкости Π€ ΠΈ Π€, соСдинСны ΠΏΠΎΡΠ»Π΅Π΄ΠΎΠ²Π°Ρ‚Π΅Π»ΡŒΠ½ΠΎ ΠΈ присоСдинСны ΠΊ Π±Π°Ρ‚Π°Ρ€Π΅Π΅ с Π­Π”Π‘ Ρ€Π°Π²Π½ΠΎΠΉ Π’ (рис.3)? Π§Π΅ΠΌΡƒ Ρ€Π°Π²Π΅Π½ суммарный заряд соСдинСния?
Π Π΅ΡˆΠ΅Π½ΠΈΠ΅Π Π°Π·Π½ΠΎΡΡ‚ΠΈ ΠΏΠΎΡ‚Π΅Π½Ρ†ΠΈΠ°Π»ΠΎΠ² Π½Π° ΠΎΠ±ΠΊΠ»Π°Π΄ΠΊΠ°Ρ… кондСнсаторов Π±ΡƒΠ΄ΡƒΡ‚ ΠΏΡ€ΠΈ Ρ‚Π°ΠΊΠΎΠΌ соСдинСнии Ρ€Π°Π²Π½Ρ‹:

Заряд Π½Π° ΠΏΠ΅Ρ€Π²ΠΎΠΌ кондСнсаторС ΠΏΡ€ΠΈ этом Ρ€Π°Π²Π΅Π½:

Заряд Π½Π° ΠΎΠ±ΠΊΠ»Π°Π΄ΠΊΠ°Ρ… Π²Ρ‚ΠΎΡ€ΠΎΠ³ΠΎ кондСнсатора:

Π‘ΡƒΠΌΠΌΠ°Ρ€Π½Ρ‹ΠΉ заряд систСмы ΠΌΠΎΠΆΠ½ΠΎ Π½Π°ΠΉΡ‚ΠΈ ΠΊΠ°ΠΊ:

Π’ΠΎΠ³Π΄Π° суммарный заряд Ρ€Π°Π²Π΅Π½:

ΠžΡ‚Π²Π΅Ρ‚ Кл; Кл; Кл
Π—Π°Π΄Π°Π½ΠΈΠ΅Π•ΠΌΠΊΠΎΡΡ‚ΡŒ пускового устройства элСктричСского двигатСля Ρ€Π°Π²Π½Π° C. Π­Π½Π΅Ρ€Π³ΠΈΠΈ ΠΈΠΌΠ΅ΡŽΡ‰Π΅ΠΉΡΡ Π² кондСнсаторС достаточно для Ρ‚ΠΎΠ³ΠΎ Ρ‡Ρ‚ΠΎΠ±Ρ‹ ΠΏΠΎΠ΄Π½ΡΡ‚ΡŒ Π³Ρ€ΡƒΠ· массы m Π½Π° высоту h. Π§Π΅ΠΌΡƒ Ρ€Π°Π²Π΅Π½ заряд кондСнсатора?
Π Π΅ΡˆΠ΅Π½ΠΈΠ΅ΠŸΡ€ΠΈ поднятии Π³Ρ€ΡƒΠ·Π° Π½Π° высоту h происходит ΠΏΠ΅Ρ€Π΅Ρ…ΠΎΠ΄ энСргии поля кондСнсатора () Π² ΠΏΠΎΡ‚Π΅Π½Ρ†ΠΈΠ°Π»ΡŒΠ½ΡƒΡŽ ΡΠ½Π΅Ρ€Π³ΠΈΡŽ Ρ‚Π΅Π»Π° (), поднятого Π½Π°Π΄ Π—Π΅ΠΌΠ»Π΅ΠΉ, поэтому запишСм:

Π­Π½Π΅Ρ€Π³ΠΈΡŽ Π½Π°ΠΉΠ΄Π΅ΠΌ ΠΊΠ°ΠΊ:

Π­Π½Π΅Ρ€Π³ΠΈΡŽ элСктричСского поля кондСнсатора Π±ΡƒΠ΄Π΅Ρ‚ ΡƒΠ΄ΠΎΠ±Π½Π΅Π΅ Π²Ρ‹Ρ€Π°Π·ΠΈΡ‚ΡŒ:

ΠŸΠΎΠ΄ΡΡ‚Π°Π²ΠΈΠΌ Π² Π²Ρ‹Ρ€Π°ΠΆΠ΅Π½ΠΈΠ΅ (2.1) ΠΏΡ€Π°Π²Ρ‹Π΅ части (2.2) ΠΈ (2.3), ΠΈΠΌΠ΅Π΅ΠΌ:

Один ΠΈΠ· Π½Π°ΠΈΠ±ΠΎΠ»Π΅Π΅ Π²Π°ΠΆΠ½Ρ‹Ρ… эффСктов, ΠΈΡΠΏΠΎΠ»ΡŒΠ·ΡƒΠ΅ΠΌΡ‹Ρ… Π² элСктроникС, β€” Ρ‘ΠΌΠΊΠΎΡΡ‚ΡŒ кондСнсаторов. Π‘ΠΏΠΎΡΠΎΠ±Π½ΠΎΡΡ‚ΡŒ Π½Π°ΠΊΠ°ΠΏΠ»ΠΈΠ²Π°Ρ‚ΡŒ ΠΈ Ρ…Ρ€Π°Π½ΠΈΡ‚ΡŒ элСктричСский заряд нашла ΠΏΡ€ΠΈΠΌΠ΅Π½Π΅Π½ΠΈΠ΅ практичСски Π²ΠΎ всСх Π°Π½Π°Π»ΠΎΠ³ΠΎΠ²Ρ‹Ρ… цСпях ΠΈ логичСских схСмах. ΠŸΠ°ΡΡΠΈΠ²Π½Ρ‹Π΅ устройства, Π·Π°ΠΏΠ°ΡΠ°ΡŽΡ‰ΠΈΠ΅ ΡΠ½Π΅Ρ€Π³ΠΈΡŽ Π² Π²ΠΈΠ΄Π΅ элСктричСского поля, Π½Π°Π·Ρ‹Π²Π°Π»ΠΈ кондСнсаторами ΡƒΠΆΠ΅ Π² Ρ‚Π΅ Π²Ρ€Π΅ΠΌΠ΅Π½Π°, ΠΊΠΎΠ³Π΄Π° ΡƒΡ‡Ρ‘Π½Ρ‹Π΅ Π΅Ρ‰Ρ‘ ΠΎΡ‡Π΅Π½ΡŒ ΠΌΠ°Π»ΠΎ Π·Π½Π°Π»ΠΈ ΠΎ ΠΏΡ€ΠΈΡ€ΠΎΠ΄Π΅ элСктричСства.

Π˜ΡΡ‚ΠΎΡ€ΠΈΡ Π½Π°ΠΊΠΎΠΏΠΈΡ‚Π΅Π»Π΅ΠΉ заряда

Π‘Π°ΠΌΠΎΠ΅ Ρ€Π°Π½Π½Π΅Π΅ письмСнноС ΡΠ²ΠΈΠ΄Π΅Ρ‚Π΅Π»ΡŒΡΡ‚Π²ΠΎ получСния зарядов с ΠΏΠΎΠΌΠΎΡ‰ΡŒΡŽ трСния ΠΏΡ€ΠΈΠ½Π°Π΄Π»Π΅ΠΆΠΈΡ‚ ΡƒΡ‡Ρ‘Π½ΠΎΠΌΡƒ ЀалСсу ΠΈΠ· ΠœΠΈΠ»Π΅Ρ‚Π° (635β€”543 Π³Π³. Π΄ΠΎ Π½. э.), ΠΊΠΎΡ‚ΠΎΡ€Ρ‹ΠΉ описал трибоэлСктричСский эффСкт ΠΎΡ‚ взаимодСйствия янтаря ΠΈ сухой ΡˆΠ΅Ρ€ΡΡ‚ΠΈ. Для ΠΏΡ€ΠΈΠ±Π»ΠΈΠ·ΠΈΡ‚Π΅Π»ΡŒΠ½ΠΎ 2300 ΠΏΠΎΡΠ»Π΅Π΄ΡƒΡŽΡ‰ΠΈΡ… Π»Π΅Ρ‚ любоС ΠΏΠΎΠ»ΡƒΡ‡Π΅Π½ΠΈΠ΅ элСктричСства Π·Π°ΠΊΠ»ΡŽΡ‡Π°Π»ΠΎΡΡŒ Π² Ρ‚Ρ€Π΅Π½ΠΈΠΈ Π΄Π²ΡƒΡ… Ρ€Π°Π·Π»ΠΈΡ‡Π½Ρ‹Ρ… ΠΌΠ°Ρ‚Π΅Ρ€ΠΈΠ°Π»ΠΎΠ² Π΄Ρ€ΡƒΠ³ ΠΎ Π΄Ρ€ΡƒΠ³Π°.

ΠšΠ°Ρ‡Π΅ΡΡ‚Π²Π΅Π½Π½Ρ‹ΠΉ Ρ€Ρ‹Π²ΠΎΠΊ Π² знаниях ΠΎ зарядах ΠΏΡ€ΠΎΠΈΠ·ΠΎΡˆΡ‘Π» Π² эпоху ΠŸΡ€ΠΎΡΠ²Π΅Ρ‰Π΅Π½ΠΈΡ β€” ΠΏΠ΅Ρ€ΠΈΠΎΠ΄ Ρ€Π΅Π²ΠΎΠ»ΡŽΡ†ΠΈΠΎΠ½Π½ΠΎΠ³ΠΎ развития Π½Π°ΡƒΡ‡Π½ΠΎΠΉ мысли Π² ΠΎΠ±Ρ€Π°Π·ΠΎΠ²Π°Π½Π½Ρ‹Ρ… ΠΊΡ€ΡƒΠ³Π°Ρ…. Π’ это врСмя элСктричСство становится популярной Ρ‚Π΅ΠΌΠΎΠΉ, Π° энтузиастами Π±Ρ‹Π»ΠΎ ΠΏΡ€ΠΎΠΈΠ·Π²Π΅Π΄Π΅Π½ΠΎ Π½Π΅ΠΌΠ°Π»ΠΎ ΠΎΠΏΡ‹Ρ‚ΠΎΠ² ΠΈ экспСримСнтов с Π³Π΅Π½Π΅Ρ€Π°Ρ‚ΠΎΡ€Π°ΠΌΠΈ Π½Π° основС трСния.

ΠŸΠ΅Ρ€Π²ΠΎΠ΅ устройство для хранСния ΠΏΠΎΠ»ΡƒΡ‡Π΅Π½Π½Ρ‹Ρ… зарядов Π±Ρ‹Π»ΠΎ создано Π² 1745 Π³. двумя элСктриками (Ρ‚Π°ΠΊ Ρ‚ΠΎΠ³Π΄Π° Π½Π°Π·Ρ‹Π²Π°Π»ΠΈ людСй, ΠΈΠ·ΡƒΡ‡Π°ΡŽΡ‰ΠΈΡ… ΠΏΡ€ΠΈΡ€ΠΎΠ΄Ρƒ статичСского элСктричСства), Ρ€Π°Π±ΠΎΡ‚Π°ΡŽΡ‰ΠΈΠΌΠΈ нСзависимо Π΄Ρ€ΡƒΠ³ ΠΎΡ‚ Π΄Ρ€ΡƒΠ³Π°: Эвальдом Ρ„ΠΎΠ½ ΠšΠ»Π΅ΠΉΡΡ‚ΠΎΠΌ, Π΄Π΅ΠΊΠ°Π½ΠΎΠΌ собора Π² ΠŸΡ€ΡƒΡΡΠΈΠΈ, ΠΈ ΠŸΠΈΡ‚Π΅Ρ€ΠΎΠΌ Π²Π°Π½ ΠœΡŽΡΡΠ΅Π½Π±Ρ€ΡƒΠΊΠΎΠΌ, профСссором ΠΌΠ°Ρ‚Π΅ΠΌΠ°Ρ‚ΠΈΠΊΠΈ ΠΈ Ρ„ΠΈΠ·ΠΈΠΊΠΈ Π² унивСрситСтС Π›Π΅ΠΉΠ΄Π΅Π½Π°.

ΠžΡ‚ΠΊΡ€Ρ‹Ρ‚ΠΈΠ΅ явлСния ΠΏΡ€ΠΎΠΈΠ·ΠΎΡˆΠ»ΠΎ Π²ΠΎ врСмя ΠΎΠΏΡ‹Ρ‚ΠΎΠ² Ρƒ ΠΎΠ±ΠΎΠΈΡ… экспСримСнтаторов, Π½ΠΎ с Ρ‚ΠΎΠΉ Ρ€Π°Π·Π½ΠΈΡ†Π΅ΠΉ, Ρ‡Ρ‚ΠΎ ΠœΡŽΡΡΠ΅Π½Π±Ρ€ΡƒΠΊ, Π²ΠΎ-ΠΏΠ΅Ρ€Π²Ρ‹Ρ…, сдСлал Π½Π΅ΠΌΠ°Π»ΠΎ ΡƒΡΠΎΠ²Π΅Ρ€ΡˆΠ΅Π½ΡΡ‚Π²ΠΎΠ²Π°Π½ΠΈΠΉ ΠΏΠ΅Ρ€Π²ΠΎΠ½Π°Ρ‡Π°Π»ΡŒΠ½ΠΎ созданного оборудования, Π° Π²ΠΎ-Π²Ρ‚ΠΎΡ€Ρ‹Ρ…, письмСнно сообщил ΠΊΠΎΠ»Π»Π΅Π³Π°ΠΌ ΠΎ своих достиТСниях. ΠŸΡ€ΠΎΡˆΠ»ΠΎ совсСм Π½Π΅ΠΌΠ½ΠΎΠ³ΠΎ Π²Ρ€Π΅ΠΌΠ΅Π½ΠΈ ΠΈ ΡƒΡ‡Ρ‘Π½Ρ‹Π΅ ΠΌΠΈΡ€Π° стали ΡΠΎΠ·Π΄Π°Π²Π°Ρ‚ΡŒ Π½Π°ΠΊΠΎΠΏΠΈΡ‚Π΅Π»ΠΈ зарядов собствСнных конструкций. Π­Ρ‚ΠΎ Π±Ρ‹Π»ΠΈ ΠΏΠ΅Ρ€Π²Ρ‹Π΅ шаги Π² ΡΠ²ΠΎΠ»ΡŽΡ†ΠΈΠΈ кондСнсаторов, ΠΏΡ€ΠΎΠ΄ΠΎΠ»ΠΆΠ°ΡŽΡ‰Π΅ΠΉΡΡ ΠΈ Π² наши Π΄Π½ΠΈ. ΠžΡΠ½ΠΎΠ²Π½Ρ‹Π΅ Π΄Π°Ρ‚Ρ‹ Ρ…Ρ€ΠΎΠ½ΠΎΠ»ΠΎΠ³ΠΈΠΈ появлСния устройств для хранСния зарядов:

  • 1746 Π³. β€” ΠΈΠ·ΠΎΠ±Ρ€Π΅Ρ‚Π΅Π½ΠΈΠ΅ лСйдСнской Π±Π°Π½ΠΊΠΈ Π² Ρ€Π΅Π·ΡƒΠ»ΡŒΡ‚Π°Ρ‚Π΅ экспСримСнтов ΠΏΠΎ Π΄ΠΎΡ€Π°Π±ΠΎΡ‚ΠΊΠ΅ устройства ΠšΠ»Π΅ΠΉΡΡ‚Π°;
  • 1750 Π³. β€” ΠΎΠΏΡ‹Ρ‚Ρ‹ Π‘Π΅Π½Π΄ΠΆΠ°ΠΌΠΈΠ½Π° Π€Ρ€Π°Π½ΠΊΠ»ΠΈΠ½Π° с батарСями кондСнсаторов;
  • 1837 Π³. β€” публикация Майклом Π€Π°Ρ€Π°Π΄Π΅Π΅ΠΌ Ρ‚Π΅ΠΎΡ€ΠΈΠΈ диэлСктричСской поляризации β€” Π½Π°ΡƒΡ‡Π½ΠΎΠΉ основы Ρ€Π°Π±ΠΎΡ‚Ρ‹ Π½Π°ΠΊΠΎΠΏΠΈΡ‚Π΅Π»Π΅ΠΉ;
  • ΠΊΠΎΠ½Π΅Ρ† XIX Π². β€” Π½Π°Ρ‡Π°Π»ΠΎ практичСского примСнСния лСйдСнских Π±Π°Π½ΠΎΠΊ вмСстС с ΠΏΠ΅Ρ€Π²Ρ‹ΠΌΠΈ устройствами постоянного Ρ‚ΠΎΠΊΠ°;
  • Π½Π°Ρ‡Π°Π»ΠΎ XX Π². β€” ΠΈΠ·ΠΎΠ±Ρ€Π΅Ρ‚Π΅Π½ΠΈΠ΅ ΡΠ»ΡŽΠ΄ΡΠ½Ρ‹Ρ… ΠΈ кСрамичСских кондСнсаторов.

Π€ΠΈΠ·ΠΈΠΊΠ° ёмкостных характСристик

Устройства, ΠΎΠ±Π»Π°Π΄Π°ΡŽΡ‰ΠΈΠ΅ ΡΠΏΠΎΡΠΎΠ±Π½ΠΎΡΡ‚ΡŒΡŽ хранСния энСргии Π² Ρ„ΠΎΡ€ΠΌΠ΅ элСктричСского заряда ΠΈ производящиС ΠΏΡ€ΠΈ этом Ρ€Π°Π·Π½ΠΎΡΡ‚ΡŒ ΠΏΠΎΡ‚Π΅Π½Ρ†ΠΈΠ°Π»ΠΎΠ², Π½Π°Π·Ρ‹Π²Π°ΡŽΡ‚ кондСнсаторами. Π’ ΠΏΡ€ΠΎΡΡ‚Π΅ΠΉΡˆΠ΅ΠΌ Π²ΠΈΠ΄Π΅ ΠΎΠ½ΠΈ состоят ΠΈΠ· Π΄Π²ΡƒΡ… ΠΈΠ»ΠΈ Π±ΠΎΠ»Π΅Π΅ ΠΏΠ°Ρ€Π°Π»Π»Π΅Π»ΡŒΠ½Ρ‹Ρ… проводящих пластин, находящихся Π½Π° нСбольшом расстоянии Π΄Ρ€ΡƒΠ³ ΠΎΡ‚ Π΄Ρ€ΡƒΠ³Π°, Π½ΠΎ элСктричСски Ρ€Π°Π·Π΄Π΅Π»Ρ‘Π½Π½Ρ‹Ρ… Π»ΠΈΠ±ΠΎ Π²ΠΎΠ·Π΄ΡƒΡ…ΠΎΠΌ, Π»ΠΈΠ±ΠΎ ΠΊΠ°ΠΊΠΈΠΌ-Π»ΠΈΠ±ΠΎ Π΄Ρ€ΡƒΠ³ΠΈΠΌ изоляционным ΠΌΠ°Ρ‚Π΅Ρ€ΠΈΠ°Π»ΠΎΠΌ, Π½Π°ΠΏΡ€ΠΈΠΌΠ΅Ρ€, Π²ΠΎΡ‰Ρ‘Π½ΠΎΠΉ Π±ΡƒΠΌΠ°Π³ΠΎΠΉ, слюдой, ΠΊΠ΅Ρ€Π°ΠΌΠΈΠΊΠΎΠΉ, пластмассой ΠΈΠ»ΠΈ ΡΠΏΠ΅Ρ†ΠΈΠ°Π»ΡŒΠ½Ρ‹ΠΌ Π³Π΅Π»Π΅ΠΌ.

Если ΠΏΠΎΠ΄ΠΊΠ»ΡŽΡ‡ΠΈΡ‚ΡŒ ΠΊ пластинам источник напряТСния, Ρ‚ΠΎ ΠΎΠ΄Π½Π° ΠΈΠ· Π½ΠΈΡ… ΠΏΠΎΠ»ΡƒΡ‡ΠΈΡ‚ ΠΈΠ·Π±Ρ‹Ρ‚ΠΎΠΊ элСктронов, Π° Π½Π° Π΄Ρ€ΡƒΠ³ΠΎΠΉ сформируСтся ΠΈΡ… Π΄Π΅Ρ„ΠΈΡ†ΠΈΡ‚. Π˜ΠΎΠ½Ρ‹ ΠΈ элСктроны Π½Π° ΠΊΠ°ΠΆΠ΄ΠΎΠΉ ΠΈΠ· этих пластин ΠΏΡ€ΠΈΡ‚ΡΠ³ΠΈΠ²Π°ΡŽΡ‚ΡΡ Π΄Ρ€ΡƒΠ³ ΠΊ Π΄Ρ€ΡƒΠ³Ρƒ, Π½ΠΎ благодаря диэлСктричСскому Π±Π°Ρ€ΡŒΠ΅Ρ€Ρƒ ΠΎΠ½ΠΈ Π½Π΅ ΡΠΎΠ΅Π΄ΠΈΠ½ΡΡŽΡ‚ΡΡ, Π° Π½Π°ΠΊΠ°ΠΏΠ»ΠΈΠ²Π°ΡŽΡ‚ΡΡ Π½Π° плоскостях ΠΏΡ€ΠΎΠ²ΠΎΠ΄Π½ΠΈΠΊΠΎΠ². Π’ Ρ€Π΅Π·ΡƒΠ»ΡŒΡ‚Π°Ρ‚Π΅ пСрвая пластина (элСктрод) окаТСтся заряТСнной ΠΎΡ‚Ρ€ΠΈΡ†Π°Ρ‚Π΅Π»ΡŒΠ½ΠΎ, Π° вторая β€” ΠΏΠΎΠ»ΠΎΠΆΠΈΡ‚Π΅Π»ΡŒΠ½ΠΎ. НСподвиТныС заряды ΡΠΎΠ·Π΄Π°ΡŽΡ‚ постоянноС элСктричСскоС ΠΏΠΎΠ»Π΅, тСорСтичСски сохраняСмоС Π½Π΅ΠΎΠ³Ρ€Π°Π½ΠΈΡ‡Π΅Π½Π½ΠΎΠ΅ количСство Π²Ρ€Π΅ΠΌΠ΅Π½ΠΈ Π² Π½Π΅Π·Π°ΠΌΠΊΠ½ΡƒΡ‚ΠΎΠΉ элСктричСской Ρ†Π΅ΠΏΠΈ.

ΠŸΠΎΡ‚ΠΎΠΊ элСктронов Π½Π° пластины называСтся зарядным Ρ‚ΠΎΠΊΠΎΠΌ, ΠΏΡ€ΠΎΠ΄ΠΎΠ»ΠΆΠ°ΡŽΡ‰ΠΈΠΌ ΠΏΡ€ΠΈΡΡƒΡ‚ΡΡ‚Π²ΠΎΠ²Π°Ρ‚ΡŒ Π΄ΠΎ Ρ‚Π΅Ρ… ΠΏΠΎΡ€, ΠΏΠΎΠΊΠ° напряТСниС Π½Π° пластинах Π½Π΅ сравняСтся с ΠΏΡ€ΠΈΠ»ΠΎΠΆΠ΅Π½Π½Ρ‹ΠΌ. Π’ этот ΠΌΠΎΠΌΠ΅Π½Ρ‚ кондСнсатор считаСтся ΠΏΠΎΠ»Π½ΠΎΡΡ‚ΡŒΡŽ заряТСнным, Ρ‚ΠΎ Π΅ΡΡ‚ΡŒ зарядов Π½Π° пластинах становится Ρ‚Π°ΠΊ ΠΌΠ½ΠΎΠ³ΠΎ, Ρ‡Ρ‚ΠΎ ΠΎΠ½ΠΈ ΠΎΡ‚Ρ‚Π°Π»ΠΊΠΈΠ²Π°ΡŽΡ‚ вновь ΠΏΠΎΡΡ‚ΡƒΠΏΠ°ΡŽΡ‰ΠΈΠ΅. ΠŸΡ€ΠΈ ΠΏΠΎΠ΄ΠΊΠ»ΡŽΡ‡Π΅Π½ΠΈΠΈ ΠΊ заряТСнному устройству Π½Π°Π³Ρ€ΡƒΠ·ΠΊΠΈ элСктроны ΠΈ ΠΈΠΎΠ½Ρ‹ находят Π½ΠΎΠ²Ρ‹ΠΉ ΠΏΡƒΡ‚ΡŒ Π΄Ρ€ΡƒΠ³ ΠΊ Π΄Ρ€ΡƒΠ³Ρƒ. Π’ этом случаС кондСнсатор Ρ€Π°Π±ΠΎΡ‚Π°Π΅Ρ‚ ΠΊΠ°ΠΊ источник Ρ‚ΠΎΠΊΠ° Π΄ΠΎ ΠΌΠΎΠΌΠ΅Π½Ρ‚Π° ΠΏΠΎΡ‚Π΅Ρ€ΠΈ разности ΠΏΠΎΡ‚Π΅Π½Ρ†ΠΈΠ°Π»ΠΎΠ² Π½Π° элСктродах.

Π‘ΠΏΠΎΡΠΎΠ±Π½ΠΎΡΡ‚ΡŒ кондСнсатора Ρ…Ρ€Π°Π½ΠΈΡ‚ΡŒ заряд Q (измСряСтся Π² ΠΊΡƒΠ»ΠΎΠ½Π°Ρ…) Π½Π°Π·Ρ‹Π²Π°ΡŽΡ‚ Ρ‘ΠΌΠΊΠΎΡΡ‚ΡŒΡŽ. Π§Π΅ΠΌ большС ΠΏΠ»ΠΎΡ‰Π°Π΄ΡŒ пластин ΠΈ мСньшС расстояниС ΠΌΠ΅ΠΆΠ΄Ρƒ Π½ΠΈΠΌΠΈ (благодаря ΡƒΡΠΈΠ»Π΅Π½ΠΈΡŽ эффСкта притяТСния зарядов ΠΌΠ΅ΠΆΠ΄Ρƒ ΠΎΠ±ΠΊΠ»Π°Π΄ΠΊΠ°ΠΌΠΈ), Ρ‚Π΅ΠΌ большая Ρ‘ΠΌΠΊΠΎΡΡ‚ΡŒ устройства. Π‘Ρ‚Π΅ΠΏΠ΅Π½ΡŒ приблиТСния пластин ограничиваСтся ΡΠΏΠΎΡΠΎΠ±Π½ΠΎΡΡ‚ΡŒΡŽ диэлСктрика ΡΠΎΠΏΡ€ΠΎΡ‚ΠΈΠ²Π»ΡΡ‚ΡŒΡΡ разрядкС ΠΏΡ€ΠΎΠ±ΠΎΠ΅ΠΌ ΠΌΠ΅ΠΆΠ΄Ρƒ Π½ΠΈΠΌΠΈ. Π’Π°ΠΊΠΈΠΌ ΠΎΠ±Ρ€Π°Π·ΠΎΠΌ, Ρ‚Ρ€ΠΈ характСристики ΠΎΠΏΡ€Π΅Π΄Π΅Π»ΡΡŽΡ‚ ΠΏΡ€ΠΎΠΈΠ·Π²ΠΎΠ΄ΠΈΡ‚Π΅Π»ΡŒΠ½ΠΎΡΡ‚ΡŒ кондСнсатора:

  • гСомСтрия пластин;
  • расстояниС ΠΌΠ΅ΠΆΠ΄Ρƒ Π½ΠΈΠΌΠΈ;
  • диэлСктричСский ΠΌΠ°Ρ‚Π΅Ρ€ΠΈΠ°Π» ΠΌΠ΅ΠΆΠ΄Ρƒ пластинами.

Π•Π΄ΠΈΠ½ΠΈΡ†Π° ΠΈ Ρ„ΠΎΡ€ΠΌΡƒΠ»Ρ‹ расчёта

ΠΠΌΠΊΠΎΡΡ‚ΡŒ Π² Π²ΠΈΠ΄Π΅ элСктричСского свойства, способного Ρ…Ρ€Π°Π½ΠΈΡ‚ΡŒ заряды, измСряСтся Π² Ρ„Π°Ρ€Π°Π΄Π°Ρ… (Π€) ΠΈ обозначаСтся Π‘. Π’Π΅Π»ΠΈΡ‡ΠΈΠ½Π° Π½Π°Π·Π²Π°Π½Π° Π² Ρ‡Π΅ΡΡ‚ΡŒ английского Ρ„ΠΈΠ·ΠΈΠΊΠ° Майкла ЀарадСя. ΠšΠΎΠ½Π΄Π΅Π½ΡΠ°Ρ‚ΠΎΡ€ Ρ‘ΠΌΠΊΠΎΡΡ‚ΡŒΡŽ 1 Ρ„Π°Ρ€Π°Π΄ способСн Ρ…Ρ€Π°Π½ΠΈΡ‚ΡŒ заряд Π² 1 ΠΊΡƒΠ»ΠΎΠ½ Π½Π° пластинах с напряТСниСм 1 Π²ΠΎΠ»ΡŒΡ‚. Π—Π½Π°Ρ‡Π΅Π½ΠΈΠ΅ Π‘ всСгда ΠΏΠΎΠ»ΠΎΠΆΠΈΡ‚Π΅Π»ΡŒΠ½ΠΎ.

ΠœΠ°Ρ‚Π΅ΠΌΠ°Ρ‚ΠΈΡ‡Π΅ΡΠΊΠΎΠ΅ Π²Ρ‹Ρ€Π°ΠΆΠ΅Π½ΠΈΠ΅ Ρ„Π°Ρ€Π°Π΄Π°

ΠΠΌΠΊΠΎΡΡ‚ΡŒ кондСнсатора β€” постоянная Π²Π΅Π»ΠΈΡ‡ΠΈΠ½Π°, ΠΎΠ·Π½Π°Ρ‡Π°ΡŽΡ‰Π°Ρ ΠΏΠΎΡ‚Π΅Π½Ρ†ΠΈΠ°Π»ΡŒΠ½ΡƒΡŽ ΡΠΏΠΎΡΠΎΠ±Π½ΠΎΡΡ‚ΡŒ Ρ…Ρ€Π°Π½ΠΈΡ‚ΡŒ ΡΠ½Π΅Ρ€Π³ΠΈΡŽ. ΠšΠΎΠ»ΠΈΡ‡Π΅ΡΡ‚Π²ΠΎ заряда, Ρ…Ρ€Π°Π½ΠΈΠΌΠΎΠ΅ Π² ΠΎΡ‚Π΄Π΅Π»ΡŒΠ½ΠΎ взятый ΠΌΠΎΠΌΠ΅Π½Ρ‚, опрСдСляСтся ΡƒΡ€Π°Π²Π½Π΅Π½ΠΈΠ΅ΠΌ Q=CV, Π³Π΄Π΅ V β€” ΠΏΡ€ΠΈΠ»ΠΎΠΆΠ΅Π½Π½ΠΎΠ΅ напряТСниС. Π’Π°ΠΊΠΈΠΌ ΠΎΠ±Ρ€Π°Π·ΠΎΠΌ, рСгулируя напряТСниС Π½Π° пластинах, ΠΌΠΎΠΆΠ½ΠΎ ΡƒΠ²Π΅Π»ΠΈΡ‡ΠΈΠ²Π°Ρ‚ΡŒ ΠΈΠ»ΠΈ ΡƒΠΌΠ΅Π½ΡŒΡˆΠ°Ρ‚ΡŒ заряд. Π­Ρ‚Π° Ρ„ΠΎΡ€ΠΌΡƒΠ»Π° ёмкости Π² Π²ΠΈΠ΄Π΅ C=Q/V Π² Π΅Π΄ΠΈΠ½ΠΈΡ‡Π½Ρ‹Ρ… значСниях опрСдСляСт, Π² Ρ‡Ρ‘ΠΌ измСряСтся Ρ‘ΠΌΠΊΠΎΡΡ‚ΡŒ кондСнсатора Π² БИ, ΠΈ являСтся матСматичСским Π²Ρ‹Ρ€Π°ΠΆΠ΅Π½ΠΈΠ΅ΠΌ Ρ„Π°Ρ€Π°Π΄Π°.

БпСциалисты ΠΏΠΎ элСктроникС Π΅Π΄ΠΈΠ½ΠΈΡ†Ρƒ Π² ΠΎΠ΄ΠΈΠ½ Ρ„Π°Ρ€Π°Π΄ ΡΡ‡ΠΈΡ‚Π°ΡŽΡ‚ Π½Π΅ совсСм ΠΏΡ€Π°ΠΊΡ‚ΠΈΡ‡Π½ΠΎΠΉ, ΠΏΠΎΡΠΊΠΎΠ»ΡŒΠΊΡƒ ΠΎΠ½Π° прСдставляСт собой ΠΎΠ³Ρ€ΠΎΠΌΠ½ΠΎΠ΅ Π·Π½Π°Ρ‡Π΅Π½ΠΈΠ΅. Π”Π°ΠΆΠ΅ 1/1000 F β€” это ΠΎΡ‡Π΅Π½ΡŒ большая Ρ‘ΠΌΠΊΠΎΡΡ‚ΡŒ. Как ΠΏΡ€Π°Π²ΠΈΠ»ΠΎ, для Ρ€Π΅Π°Π»ΡŒΠ½Ρ‹Ρ… элСктричСских ΠΊΠΎΠΌΠΏΠΎΠ½Π΅Π½Ρ‚ΠΎΠ² ΠΏΡ€ΠΈΠΌΠ΅Π½ΡΡŽΡ‚ ΡΠ»Π΅Π΄ΡƒΡŽΡ‰ΠΈΠ΅ Π²Π΅Π»ΠΈΡ‡ΠΈΠ½Ρ‹:

  • ΠΏΠΈΠΊΠΎΡ„Π°Ρ€Π°Π΄ β€” 10β€”12 Π€;
  • Π½Π°Π½ΠΎΡ„Π°Ρ€Π°Π΄ β€” 10β€”9 Π€;
  • ΠΌΠΈΠΊΡ€ΠΎΡ„Π°Ρ€Π°Π΄ β€” 10β€”6 Π€.

ДиэлСктричСская ΠΏΡ€ΠΎΠ½ΠΈΡ†Π°Π΅ΠΌΠΎΡΡ‚ΡŒ

Π€Π°ΠΊΡ‚ΠΎΡ€, благодаря ΠΊΠΎΡ‚ΠΎΡ€ΠΎΠΌΡƒ изолятор опрСдСляСт Ρ‘ΠΌΠΊΠΎΡΡ‚ΡŒ кондСнсатора, называСтся диэлСктричСской ΠΏΡ€ΠΎΠ½ΠΈΡ†Π°Π΅ΠΌΠΎΡΡ‚ΡŒΡŽ. ΠžΠ±ΠΎΠ±Ρ‰Ρ‘Π½Π½Π°Ρ Ρ„ΠΎΡ€ΠΌΡƒΠ»Π° расчёта ёмкости кондСнсатора с ΠΏΠ°Ρ€Π°Π»Π»Π΅Π»ΡŒΠ½Ρ‹ΠΌΠΈ пластинами прСдставлСна Π²Ρ‹Ρ€Π°ΠΆΠ΅Π½ΠΈΠ΅ΠΌ C= Ξ΅ (A / d), Π³Π΄Π΅:

  • А β€” ΠΏΠ»ΠΎΡ‰Π°Π΄ΡŒ мСньшСй пластины;
  • d β€” расстояниС ΠΌΠ΅ΠΆΠ΄Ρƒ Π½ΠΈΠΌΠΈ;
  • Ξ΅ β€” Π°Π±ΡΠΎΠ»ΡŽΡ‚Π½Π°Ρ ΠΏΡ€ΠΎΠ½ΠΈΡ†Π°Π΅ΠΌΠΎΡΡ‚ΡŒ ΠΈΡΠΏΠΎΠ»ΡŒΠ·ΡƒΠ΅ΠΌΠΎΠ³ΠΎ диэлСктричСского ΠΌΠ°Ρ‚Π΅Ρ€ΠΈΠ°Π»Π°.

ДиэлСктричСская ΠΏΡ€ΠΎΠ½ΠΈΡ†Π°Π΅ΠΌΠΎΡΡ‚ΡŒ Π²Π°ΠΊΡƒΡƒΠΌΠ° Ξ΅0 являСтся константой ΠΈ ΠΈΠΌΠ΅Π΅Ρ‚ Π·Π½Π°Ρ‡Π΅Π½ΠΈΠ΅ 8,84Ρ…10β€”12 Ρ„Π°Ρ€Π°Π΄ Π½Π° ΠΌΠ΅Ρ‚Ρ€. Как ΠΏΡ€Π°Π²ΠΈΠ»ΠΎ, проводящиС пластины Ρ€Π°Π·Π΄Π΅Π»Π΅Π½Ρ‹ слоСм изоляционного ΠΌΠ°Ρ‚Π΅Ρ€ΠΈΠ°Π»Π°, Π° Π½Π΅ Π²Π°ΠΊΡƒΡƒΠΌΠ°. Π§Ρ‚ΠΎΠ±Ρ‹ Π½Π°ΠΉΡ‚ΠΈ Ρ‘ΠΌΠΊΠΎΡΡ‚ΡŒ кондСнсатора, пластины ΠΊΠΎΡ‚ΠΎΡ€ΠΎΠ³ΠΎ находятся Π² Π²ΠΎΠ·Π΄ΡƒΡ…Π΅, ΠΌΠΎΠΆΠ½ΠΎ Π²ΠΎΡΠΏΠΎΠ»ΡŒΠ·ΠΎΠ²Π°Ρ‚ΡŒΡΡ Π·Π½Π°Ρ‡Π΅Π½ΠΈΠ΅ΠΌ Ξ΅0. Π Π°Π·Π½ΠΈΡ†Π΅ΠΉ диэлСктричСской проницаСмости атмосфСры ΠΈ Π²Π°ΠΊΡƒΡƒΠΌΠ° ΠΌΠΎΠΆΠ½ΠΎ ΠΏΡ€Π΅Π½Π΅Π±Ρ€Π΅Ρ‡ΡŒ, ΠΏΠΎΡΠΊΠΎΠ»ΡŒΠΊΡƒ ΠΈΡ… значСния ΠΎΡ‡Π΅Π½ΡŒ Π±Π»ΠΈΠ·ΠΊΠΈ.

На ΠΏΡ€Π°ΠΊΡ‚ΠΈΠΊΠ΅ Π² Ρ„ΠΎΡ€ΠΌΡƒΠ»Π°Ρ… нахоТдСния ёмкости кондСнсатора ΠΈΡΠΏΠΎΠ»ΡŒΠ·ΡƒΠ΅Ρ‚ΡΡ ΠΎΡ‚Π½ΠΎΡΠΈΡ‚Π΅Π»ΡŒΠ½Π°Ρ диэлСктричСская ΠΏΡ€ΠΎΠ½ΠΈΡ†Π°Π΅ΠΌΠΎΡΡ‚ΡŒ Π² качСствС коэффициСнта, ΠΎΠ·Π½Π°Ρ‡Π°ΡŽΡ‰Π°Ρ, насколько элСктричСскоС ΠΏΠΎΠ»Π΅ ΠΌΠ΅ΠΆΠ΄Ρƒ зарядами ΡƒΠΌΠ΅Π½ΡŒΡˆΠ°Π΅Ρ‚ΡΡ Π² диэлСктрикС ΠΏΠΎ ΡΡ€Π°Π²Π½Π΅Π½ΠΈΡŽ с Π²Π°ΠΊΡƒΡƒΠΌΠΎΠΌ. НСкоторыС значСния этой Π²Π΅Π»ΠΈΡ‡ΠΈΠ½Ρ‹ для Ρ€Π°Π·Π»ΠΈΡ‡Π½Ρ‹Ρ… ΠΌΠ°Ρ‚Π΅Ρ€ΠΈΠ°Π»ΠΎΠ²:

ΠŸΠΎΡΠΊΠΎΠ»ΡŒΠΊΡƒ ΡΡ„Ρ„Π΅ΠΊΡ‚ΠΈΠ²Π½ΠΎΡΡ‚ΡŒ кондСнсатора зависит ΠΎΡ‚ примСняСмого Π² Π½Ρ‘ΠΌ изолятора, Π΅Π³ΠΎ качСство ΠΊΠ°ΠΊ накопитСля ΠΌΠΎΠΆΠ½ΠΎ ΠΎΠΏΡ€Π΅Π΄Π΅Π»ΠΈΡ‚ΡŒ Ρ‡Π΅Ρ€Π΅Π· ΡƒΠ΄Π΅Π»ΡŒΠ½ΡƒΡŽ Ρ‘ΠΌΠΊΠΎΡΡ‚ΡŒ β€” Π²Π΅Π»ΠΈΡ‡ΠΈΠ½Ρƒ, Ρ€Π°Π²Π½ΡƒΡŽ ΠΎΡ‚Π½ΠΎΡˆΠ΅Π½ΠΈΡŽ ёмкости ΠΊ ΠΎΠ±ΡŠΡ‘ΠΌΡƒ диэлСктрика.

ΠŸΡ€Π°ΠΊΡ‚ΠΈΡ‡Π΅ΡΠΊΠΈΠ΅ измСрСния

Π—Π½Π°Ρ‡Π΅Π½ΠΈΠ΅ ёмкости кондСнсатора обозначаСтся Π½Π° корпусС Π² Π΄Ρ€ΠΎΠ±Π½Ρ‹Ρ… Ρ„Π°Ρ€Π°Π΄Π°Ρ… ΠΈΠ»ΠΈ с ΠΏΠΎΠΌΠΎΡ‰ΡŒΡŽ Ρ†Π²Π΅Ρ‚ΠΎΠ²ΠΎΠ³ΠΎ ΠΊΠΎΠ΄Π°. Но со Π²Ρ€Π΅ΠΌΠ΅Π½Π΅ΠΌ ΠΊΠΎΠΌΠΏΠΎΠ½Π΅Π½Ρ‚Ρ‹ способны ΠΏΠΎΡ‚Π΅Ρ€ΡΡ‚ΡŒ свои качСства, поэтому для Π½Π΅ΠΊΠΎΡ‚ΠΎΡ€Ρ‹Ρ… критичСских случаСв послСдствия ΠΌΠΎΠ³ΡƒΡ‚ Π±Ρ‹Ρ‚ΡŒ Π½Π΅ΠΏΡ€ΠΈΠ΅ΠΌΠ»Π΅ΠΌΡ‹ΠΌΠΈ. Π‘ΡƒΡ‰Π΅ΡΡ‚Π²ΡƒΡŽΡ‚ ΠΈ Π΄Ρ€ΡƒΠ³ΠΈΠ΅ ΠΎΠ±ΡΡ‚ΠΎΡΡ‚Π΅Π»ΡŒΡΡ‚Π²Π°, Ρ‚Ρ€Π΅Π±ΡƒΡŽΡ‰ΠΈΠ΅ ΠΈΠ·ΠΌΠ΅Ρ€Π΅Π½ΠΈΠΉ. НапримСр, Π½Π΅ΠΎΠ±Ρ…ΠΎΠ΄ΠΈΠΌΠΎΡΡ‚ΡŒ Π·Π½Π°Ρ‚ΡŒ ΠΎΠ±Ρ‰ΡƒΡŽ Ρ‘ΠΌΠΊΠΎΡΡ‚ΡŒ Ρ†Π΅ΠΏΠΈ ΠΈΠ»ΠΈ части элСктрооборудования. ΠŸΡ€ΠΈΠ±ΠΎΡ€ΠΎΠ², ΠΎΡΡƒΡ‰Π΅ΡΡ‚Π²Π»ΡΡŽΡ‰ΠΈΡ… нСпосрСдствСнноС считываниС ёмкости, Π½Π΅ сущСствуСт, Π½ΠΎ Π·Π½Π°Ρ‡Π΅Π½ΠΈΠ΅ ΠΌΠΎΠΆΠ΅Ρ‚ Π±Ρ‹Ρ‚ΡŒ вычислСно Π²Ρ€ΡƒΡ‡Π½ΡƒΡŽ ΠΈΠ»ΠΈ ΠΈΠ½Ρ‚Π΅Π³Ρ€ΠΈΡ€ΠΎΠ²Π°Π½Π½Ρ‹ΠΌΠΈ Π² ΠΈΠ·ΠΌΠ΅Ρ€ΠΈΡ‚Π΅Π»ΡŒΠ½Ρ‹Π΅ устройства процСссорами.

Для обнаруТСния фактичСской ёмкости Π½Π΅Ρ€Π΅Π΄ΠΊΠΎ ΠΈΡΠΏΠΎΠ»ΡŒΠ·ΡƒΡŽΡ‚ осциллограф ΠΊΠ°ΠΊ срСдство измСрСния постоянной Π²Ρ€Π΅ΠΌΠ΅Π½ΠΈ (Ρ‚). Π­Ρ‚Π° Π²Π΅Π»ΠΈΡ‡ΠΈΠ½Π° ΠΎΠ±ΠΎΠ·Π½Π°Ρ‡Π°Π΅Ρ‚ врСмя Π² сСкундах, Π·Π° ΠΊΠΎΡ‚ΠΎΡ€ΠΎΠ΅ кондСнсатор заряТаСтся Π½Π° 63%, ΠΈ Ρ€Π°Π²Π½Π° ΠΏΡ€ΠΎΠΈΠ·Π²Π΅Π΄Π΅Π½ΠΈΡŽ сопротивлСния Ρ†Π΅ΠΏΠΈ Π² ΠΎΠΌΠ°Ρ… Π½Π° Ρ‘ΠΌΠΊΠΎΡΡ‚ΡŒ Ρ†Π΅ΠΏΠΈ Π² Ρ„Π°Ρ€Π°Π΄Π°Ρ…: Ρ‚=RC. ΠžΡΡ†ΠΈΠ»Π»ΠΎΠ³Ρ€Π°Ρ„ позволяСт Π»Π΅Π³ΠΊΠΎ ΠΎΠΏΡ€Π΅Π΄Π΅Π»ΠΈΡ‚ΡŒ ΠΏΠΎΡΡ‚ΠΎΡΠ½Π½ΡƒΡŽ Π²Ρ€Π΅ΠΌΠ΅Π½ΠΈ ΠΈ Π΄Π°Ρ‘Ρ‚ Π²ΠΎΠ·ΠΌΠΎΠΆΠ½ΠΎΡΡ‚ΡŒ с ΠΏΠΎΠΌΠΎΡ‰ΡŒΡŽ расчётов Π½Π°ΠΉΡ‚ΠΈ ΠΈΡΠΊΠΎΠΌΡƒΡŽ Ρ‘ΠΌΠΊΠΎΡΡ‚ΡŒ.

БущСствуСт Ρ‚Π°ΠΊΠΆΠ΅ Π½Π΅ΠΌΠ°Π»ΠΎ ΠΌΠΎΠ΄Π΅Π»Π΅ΠΉ Π»ΡŽΠ±ΠΈΡ‚Π΅Π»ΡŒΡΠΊΠΎΠ³ΠΎ ΠΈ ΠΏΡ€ΠΎΡ„Π΅ΡΡΠΈΠΎΠ½Π°Π»ΡŒΠ½ΠΎΠ³ΠΎ элСктронного ΠΈΠ·ΠΌΠ΅Ρ€ΠΈΡ‚Π΅Π»ΡŒΠ½ΠΎΠ³ΠΎ оборудования, оснащённого функциями для тСстирования кондСнсаторов. МногиС Ρ†ΠΈΡ„Ρ€ΠΎΠ²Ρ‹Π΅ ΠΌΡƒΠ»ΡŒΡ‚ΠΈΠΌΠ΅Ρ‚Ρ€Ρ‹ ΠΎΠ±Π»Π°Π΄Π°ΡŽΡ‚ Π²ΠΎΠ·ΠΌΠΎΠΆΠ½ΠΎΡΡ‚ΡŒΡŽ ΠΎΠΏΡ€Π΅Π΄Π΅Π»ΡΡ‚ΡŒ Ρ‘ΠΌΠΊΠΎΡΡ‚ΡŒ. Π­Ρ‚ΠΈ устройства способны ΠΊΠΎΠ½Ρ‚Ρ€ΠΎΠ»ΠΈΡ€ΡƒΠ΅ΠΌΠΎ Π·Π°Ρ€ΡΠΆΠ°Ρ‚ΡŒ ΠΈ Ρ€Π°Π·Ρ€ΡΠΆΠ°Ρ‚ΡŒ кондСнсатор извСстным Ρ‚ΠΎΠΊΠΎΠΌ ΠΈ, анализируя нарастаниС Ρ€Π΅Π·ΡƒΠ»ΡŒΡ‚ΠΈΡ€ΡƒΡŽΡ‰Π΅Π³ΠΎ напряТСния, Π²Ρ‹Π΄Π°Π²Π°Ρ‚ΡŒ довольно Ρ‚ΠΎΡ‡Π½Ρ‹ΠΉ Ρ€Π΅Π·ΡƒΠ»ΡŒΡ‚Π°Ρ‚. ЕдинствСнный нСдостаток Π±ΠΎΠ»ΡŒΡˆΠΈΠ½ΡΡ‚Π²Π° Ρ‚Π°ΠΊΠΈΡ… ΠΏΡ€ΠΈΠ±ΠΎΡ€ΠΎΠ² β€” ΡΡ€Π°Π²Π½ΠΈΡ‚Π΅Π»ΡŒΠ½ΠΎ ΡƒΠ·ΠΊΠΈΠΉ Π΄ΠΈΠ°ΠΏΠ°Π·ΠΎΠ½ измСряСмых Π²Π΅Π»ΠΈΡ‡ΠΈΠ½.

Π‘ΠΎΠ»Π΅Π΅ слоТныС ΠΈ спСциализированныС инструмСнты β€” мостовыС ΠΈΠ·ΠΌΠ΅Ρ€ΠΈΡ‚Π΅Π»ΠΈ, ΠΈΡΠΏΡ‹Ρ‚Ρ‹Π²Π°ΡŽΡ‰ΠΈΠ΅ кондСнсаторы Π² мостовой схСмС. Π­Ρ‚ΠΎΡ‚ ΠΌΠ΅Ρ‚ΠΎΠ΄ косвСнного измСрСния обСспСчиваСт Π²Ρ‹ΡΠΎΠΊΡƒΡŽ Ρ‚ΠΎΡ‡Π½ΠΎΡΡ‚ΡŒ. Π‘ΠΎΠ²Ρ€Π΅ΠΌΠ΅Π½Π½Ρ‹Π΅ устройства Ρ‚Π°ΠΊΠΎΠ³ΠΎ Ρ‚ΠΈΠΏΠ° оснащСны Ρ†ΠΈΡ„Ρ€ΠΎΠ²Ρ‹ΠΌΠΈ дисплСями ΠΈ Π²ΠΎΠ·ΠΌΠΎΠΆΠ½ΠΎΡΡ‚ΡŒΡŽ Π°Π²Ρ‚ΠΎΠΌΠ°Ρ‚ΠΈΠ·ΠΈΡ€ΠΎΠ²Π°Π½Π½ΠΎΠ³ΠΎ использования Π² производствСнной срСдС, ΠΎΠ½ΠΈ ΠΌΠΎΠ³ΡƒΡ‚ Π±Ρ‹Ρ‚ΡŒ сопряТСны с ΠΊΠΎΠΌΠΏΡŒΡŽΡ‚Π΅Ρ€Π°ΠΌΠΈ ΠΈ ΡΠΊΡΠΏΠΎΡ€Ρ‚ΠΈΡ€ΠΎΠ²Π°Ρ‚ΡŒ показания для внСшнСго контроля.

ИдСя супСркондСнсатора

ЭлСктричСство β€” Ρ‡Ρ€Π΅Π·Π²Ρ‹Ρ‡Π°ΠΉΠ½ΠΎ ΡƒΠ½ΠΈΠ²Π΅Ρ€ΡΠ°Π»ΡŒΠ½Ρ‹ΠΉ Π²ΠΈΠ΄ энСргии, ΠΎΠ±Π»Π°Π΄Π°ΡŽΡ‰ΠΈΠΉ ΠΎΠ΄Π½ΠΈΠΌ нСдостатком β€” Π΅Π³ΠΎ Ρ‚Ρ€ΡƒΠ΄Π½ΠΎ ΡΠ°ΠΊΠΊΡƒΠΌΡƒΠ»ΠΈΡ€ΠΎΠ²Π°Ρ‚ΡŒ быстро. Π₯имичСскиС Π±Π°Ρ‚Π°Ρ€Π΅ΠΈ способны ΡΠΎΡ…Ρ€Π°Π½ΡΡ‚ΡŒ большоС количСство энСргии, Π½ΠΎ Ρ‚Ρ€Π΅Π±ΡƒΡŽΡ‚ Π½Π΅ΡΠΊΠΎΠ»ΡŒΠΊΠΈΡ… часов для ΠΏΠΎΠ»Π½ΠΎΠΉ зарядки. Π­Ρ‚ΠΎΠ³ΠΎ нСдостатка Π»ΠΈΡˆΠ΅Π½Ρ‹ кондСнсаторы β€” ΠΎΠ½ΠΈ ΠΌΠΎΠ³ΡƒΡ‚ Π·Π°Ρ€ΡΠΆΠ°Ρ‚ΡŒΡΡ практичСски ΠΌΠ³Π½ΠΎΠ²Π΅Π½Π½ΠΎ. Но ΠΈΡ… Ρ‘ΠΌΠΊΠΎΡΡ‚ΡŒ Π½Π΅ позволяСт Ρ…Ρ€Π°Π½ΠΈΡ‚ΡŒ большоС количСство энСргии, поэтому вСсьма Π·Π°ΠΌΠ°Π½Ρ‡ΠΈΠ²ΠΎΠΉ выглядит идСя супСркондСнсатора, ΡΠΎΡ‡Π΅Ρ‚Π°ΡŽΡ‰Π΅Π³ΠΎ Π»ΡƒΡ‡ΡˆΠΈΠ΅ качСства химичСских ΠΈ элСктростатичСских Π½Π°ΠΊΠΎΠΏΠΈΡ‚Π΅Π»Π΅ΠΉ элСктричСства.

НСсмотря Π½Π° Ρ„ΡƒΠ½ΠΊΡ†ΠΈΠΎΠ½Π°Π»ΡŒΠ½ΡƒΡŽ ΡΡ…ΠΎΠΆΠ΅ΡΡ‚ΡŒ, аккумуляторныС Π±Π°Ρ‚Π°Ρ€Π΅ΠΈ ΠΈ кондСнсаторы устроСны ΡΠΎΠ²Π΅Ρ€ΡˆΠ΅Π½Π½ΠΎ ΠΏΠΎ-Ρ€Π°Π·Π½ΠΎΠΌΡƒ. Π“Π°Π»ΡŒΠ²Π°Π½ΠΈΡ‡Π΅ΡΠΊΠΈΠ΅ элСмСнты Ρ€Π°Π±ΠΎΡ‚Π°ΡŽΡ‚ Π½Π° ΠΏΡ€ΠΈΠ½Ρ†ΠΈΠΏΠ΅ высвобоТдСния элСктричСской энСргии Π²ΠΎ врСмя химичСской Ρ€Π΅Π°ΠΊΡ†ΠΈΠΈ вСщСств Π²Π½ΡƒΡ‚Ρ€ΠΈ Π½ΠΈΡ…. ΠŸΡ€ΠΈ истощСнии запаса Π°ΠΊΡ‚ΠΈΠ²Π½Ρ‹Ρ… Ρ€Π΅Π°Π³Π΅Π½Ρ‚ΠΎΠ² ΠΎΠ½ΠΈ ΠΏΡ€Π΅ΠΊΡ€Π°Ρ‰Π°ΡŽΡ‚ Π³Π΅Π½Π΅Ρ€ΠΈΡ€ΠΎΠ²Π°Ρ‚ΡŒ Ρ€Π°Π·Π½ΠΎΡΡ‚ΡŒ ΠΏΠΎΡ‚Π΅Π½Ρ†ΠΈΠ°Π»ΠΎΠ² ΠΈ для Π½ΠΎΠ²ΠΎΠ³ΠΎ Ρ†ΠΈΠΊΠ»Π° Ρ‚Ρ€Π΅Π±ΡƒΡŽΡ‚ инициирования Ρ‚ΠΎΠΊΠΎΠΌ ΠΎΠ±Ρ€Π°Ρ‚Π½Ρ‹Ρ… химичСских Ρ€Π΅Π°ΠΊΡ†ΠΈΠΉ для восстановлСния Π°ΠΊΡ‚ΠΈΠ²Π½Ρ‹Ρ… вСщСств. ΠžΡΠ½ΠΎΠ²Π½Ρ‹Π΅ нСдостатки аккумуляторов ΠΏΠΎ сравнСнии ΠΈ кондСнсаторами:

  • Π½Π΅ΠΏΡ€ΠΎΠ΄ΠΎΠ»ΠΆΠΈΡ‚Π΅Π»ΡŒΠ½Ρ‹ΠΉ ΠΆΠΈΠ·Π½Π΅Π½Π½Ρ‹ΠΉ Ρ†ΠΈΠΊΠ»;
  • нСвысокая ΡƒΠ΄Π΅Π»ΡŒΠ½Π°Ρ ΠΌΠΎΡ‰Π½ΠΎΡΡ‚ΡŒ;
  • ΡƒΠ·ΠΊΠΈΠΉ Π΄ΠΈΠ°ΠΏΠ°Π·ΠΎΠ½ Ρ‚Π΅ΠΌΠΏΠ΅Ρ€Π°Ρ‚ΡƒΡ€ зарядки ΠΈ разрядки;
  • Π½Π΅ΡΠΏΠΎΡΠΎΠ±Π½ΠΎΡΡ‚ΡŒ быстро ΠΎΡ‚Π΄Π°Ρ‚ΡŒ вСсь запас энСргии.

Π’Π΅ΠΌ Π½Π΅ ΠΌΠ΅Π½Π΅Π΅ ΠΎΠ±Ρ‹Ρ‡Π½Ρ‹Π΅ кондСнсаторы Π½Π΅ ΠΈΡΠΏΠΎΠ»ΡŒΠ·ΡƒΡŽΡ‚ΡΡ Π² качСствС Π°ΠΊΡ‚ΠΈΠ²Π½Ρ‹Ρ… источников напряТСния ΠΈΠ·-Π·Π° Π½ΠΈΠ·ΠΊΠΎΠΉ ёмкости. ВСорСтичСскиС ΠΈ практичСскиС супСркондСнсаторы (ΡƒΠ»ΡŒΡ‚Ρ€Π°ΠΊΠΎΠ½Π΄Π΅Π½ΡΠ°Ρ‚ΠΎΡ€Ρ‹) ΠΎΡ‚Π»ΠΈΡ‡Π°ΡŽΡ‚ΡΡ ΠΎΡ‚ ΠΎΠ±Ρ‹Ρ‡Π½Ρ‹Ρ… ΠΊΡ€Π°ΠΉΠ½Π΅ высокой Ρ‘ΠΌΠΊΠΎΡΡ‚ΡŒΡŽ ΠΏΡ€ΠΈ большой плотности Ρ…Ρ€Π°Π½ΠΈΠΌΠΎΠΉ энСргии, Ρ‡Ρ‚ΠΎ позволяСт ΠΈΡ… Ρ€Π°ΡΡΠΌΠ°Ρ‚Ρ€ΠΈΠ²Π°Ρ‚ΡŒ ΠΊΠ°ΠΊ Π°Π»ΡŒΡ‚Π΅Ρ€Π½Π°Ρ‚ΠΈΠ²Ρƒ химичСским элСмСнтам.

ΠšΡ€ΡƒΠΏΠ½Π΅ΠΉΡˆΠΈΠ΅ коммСрчСскиС устройства ΠΎΠ±Π»Π°Π΄Π°ΡŽΡ‚ Ρ‘ΠΌΠΊΠΎΡΡ‚ΡŒΡŽ Π΄ΠΎ Π½Π΅ΡΠΊΠΎΠ»ΡŒΠΊΠΈΡ… тысяч Ρ„Π°Ρ€Π°Π΄, Π½ΠΎ ΠΈΡ… возмоТности всё Ρ€Π°Π²Π½ΠΎ нСсопоставимы с аккумуляторами, поэтому ΠΏΠΎΠ΄ΠΎΠ±Π½Ρ‹Π΅ устройства ΠΈΡΠΏΠΎΠ»ΡŒΠ·ΡƒΡŽΡ‚ΡΡ для хранСния зарядов Π² Ρ‚Π΅Ρ‡Π΅Π½ΠΈΠ΅ ΠΎΡ‚Π½ΠΎΡΠΈΡ‚Π΅Π»ΡŒΠ½ΠΎ ΠΊΠΎΡ€ΠΎΡ‚ΠΊΠΎΠ³ΠΎ ΠΏΠ΅Ρ€ΠΈΠΎΠ΄Π° Π²Ρ€Π΅ΠΌΠ΅Π½ΠΈ. Они нашли ΡˆΠΈΡ€ΠΎΠΊΠΎΠ΅ ΠΏΡ€ΠΈΠΌΠ΅Π½Π΅Π½ΠΈΠ΅ Π² качСствС элСктричСских эквивалСнтов мСханичСских ΠΌΠ°Ρ…ΠΎΠ²ΠΈΠΊΠΎΠ², Ρ‡Ρ‚ΠΎΠ±Ρ‹ ΡΠ³Π»Π°ΠΆΠΈΠ²Π°Ρ‚ΡŒ напряТСниС источников питания, Π½Π°ΠΏΡ€ΠΈΠΌΠ΅Ρ€, Π² Π²Π΅Ρ‚Ρ€ΠΎΠ²Ρ‹Ρ… Ρ‚ΡƒΡ€Π±ΠΈΠ½Π°Ρ… ΠΈΠ»ΠΈ Ρ€Π΅ΠΊΡƒΠΏΠ΅Ρ€Π°Ρ‚ΠΈΠ²Π½Ρ‹Ρ… Ρ‚ΠΎΡ€ΠΌΠΎΠ·Π½Ρ‹Ρ… систСмах элСктричСских транспортных срСдств.

ΠŸΠ΅Ρ€Π²Ρ‹Π΅ ΡƒΠ»ΡŒΡ‚Ρ€Π°ΠΊΠΎΠ½Π΄Π΅Π½ΡΠ°Ρ‚ΠΎΡ€Ρ‹ появились Π² сСрСдинС ΠΏΡ€ΠΎΡˆΠ»ΠΎΠ³ΠΎ Π²Π΅ΠΊΠ° ΠΈ ΠΎΠ±Π»Π°Π΄Π°Π»ΠΈ Π½Π΅ ΠΎΡ‡Π΅Π½ΡŒ Π²ΠΏΠ΅Ρ‡Π°Ρ‚Π»ΡΡŽΡ‰ΠΈΠΌΠΈ ёмкостями. Π‘ Ρ‚Π΅Ρ… ΠΏΠΎΡ€ прогрСсс Π² ΡΠΎΠ²Π΅Ρ€ΡˆΠ΅Π½ΡΡ‚Π²ΠΎΠ²Π°Π½ΠΈΠΈ ΠΌΠ°Ρ‚Π΅Ρ€ΠΈΠ°Π»ΠΎΠ² ΠΏΡ€ΠΈΠ²Ρ‘Π» ΠΊ ΡƒΡ‚ΠΎΠ½ΡŒΡˆΠ΅Π½ΠΈΡŽ диэлСктричСского слоя Π΄ΠΎ ΠΎΠ΄Π½ΠΎΠΉ ΠΌΠΎΠ»Π΅ΠΊΡƒΠ»Ρ‹, Ρ‡Ρ‚ΠΎ ΠΏΠΎΠ·Π²ΠΎΠ»ΠΈΠ»ΠΎ ΡΠΎΠ·Π΄Π°Π²Π°Ρ‚ΡŒ устройства с Π²Ρ‹Π΄Π°ΡŽΡ‰ΠΈΠΌΠΈΡΡ характСристиками. Π”Π°Π»ΡŒΠ½Π΅ΠΉΡˆΠ΅Π΅ Ρ€Π°Π·Π²ΠΈΡ‚ΠΈΠ΅ наноиндустрии стало основой для Ρ„ΡƒΠ½Π΄Π°ΠΌΠ΅Π½Ρ‚Π°Π»ΡŒΠ½Ρ‹Ρ… ΠΏΠ΅Ρ€Π΅ΠΌΠ΅Π½ Π² Π½Π°ΠΊΠΎΠΏΠ»Π΅Π½ΠΈΠΈ элСктричСства. Π’ΠΎΠ·ΠΌΠΎΠΆΠ½ΠΎ, Π² скором Π²Ρ€Π΅ΠΌΠ΅Π½ΠΈ экологичСски опасныС ΠΈ ΠΊΠ°ΠΏΡ€ΠΈΠ·Π½Ρ‹Π΅ химичСскиС аккумуляторы замСнят супСркондСнсаторы Π½Π° основС молСкулярно структурированных пластин ΠΈ диэлСктричСского слоя.

Как Π½Π°ΠΉΡ‚ΠΈ напряТСниС кондСнсаторС зная Π΅ΠΌΠΊΠΎΡΡ‚ΡŒ. Π‘ΠΎΠΏΡ€ΠΎΡ‚ΠΈΠ²Π»Π΅Π½ΠΈΠ΅ кондСнсатора ΠΏΠ΅Ρ€Π΅ΠΌΠ΅Π½Π½ΠΎΠΌΡƒ Ρ‚ΠΎΠΊΡƒ

Π•ΠΌΠΊΠΎΡΡ‚ΡŒ кондСнсаторов
простой Ρ„ΠΎΡ€ΠΌΡ‹ ΠΌΠΎΠΆΠ½ΠΎ Π²Ρ‹Ρ‡ΠΈΡΠ»ΠΈΡ‚ΡŒ. Для этого
ΠΏΡ€Π΅Π΄ΠΏΠΎΠ»Π°Π³Π°ΡŽΡ‚, Ρ‡Ρ‚ΠΎ Π½Π° ΠΊΠ°ΠΆΠ΄ΠΎΠΉ ΠΈΠ· ΠΎΠ±ΠΊΠ»Π°Π΄ΠΎΠΊ
Π½Π°Ρ…ΠΎΠ΄ΠΈΡ‚ΡŒΡΡ Π½Π΅ΠΊΠΎΡ‚ΠΎΡ€Ρ‹ΠΉ заряд q,
ΠΈ Π²Ρ‹Ρ‡ΠΈΡΠ»ΡΡŽΡ‚ ΠΏΠΎΡ‚Π΅Π½Ρ†ΠΈΠ°Π» Π² элСктричСском
ΠΏΠΎΠ»Π΅ рассматриваСмого кондСнсатора
U(x,y,z).
Если удаСтся Ρ€Π΅ΡˆΠΈΡ‚ΡŒ эту Π·Π°Π΄Π°Ρ‡Ρƒ, Ρ‚ΠΎ ΠΎΡ‚ΡΡŽΠ΄Π°
получаСтся ΠΈ Π·Π½Π°Ρ‡Π΅Π½ΠΈΠ΅ напряТСния ΠΌΠ΅ΠΆΠ΄Ρƒ
ΠΎΠ±ΠΊΠ»Π°Π΄ΠΊΠ°ΠΌΠΈ кондСнсатора U.
ПослС этого Π΅ΠΌΠΊΠΎΡΡ‚ΡŒ ΠΌΠΎΠΆΠ½ΠΎ Π½Π°ΠΉΡ‚ΠΈ ΠΏΠΎ
Ρ„ΠΎΡ€ΠΌΡƒΠ»Π΅
.

ΠΠΌΠΊΠΎΡΡ‚ΡŒ плоского
кондСнсатора.

Π‘ΡƒΠ΄Π΅ΠΌ ΡΡ‡ΠΈΡ‚Π°Ρ‚ΡŒ, Ρ‡Ρ‚ΠΎ
Π·Π°Π·ΠΎΡ€ ΠΌΠ΅ΠΆΠ΄Ρƒ пластинами ΠΌΠ°Π» ΠΏΠΎ ΡΡ€Π°Π²Π½Π΅Π½ΠΈΡŽ
с ΠΈΡ… Ρ€Π°Π·ΠΌΠ΅Ρ€Π°ΠΌΠΈ, Ρ‚Π°ΠΊ Ρ‡Ρ‚ΠΎ ΠΊΡ€Π°Π΅Π²Ρ‹ΠΌΠΈ эффСктами
ΠΌΠΎΠΆΠ½ΠΎ ΠΏΡ€Π΅Π½Π΅Π±Ρ€Π΅Ρ‡ΡŒ. Если Π½Π° Π΅Π΄ΠΈΠ½ΠΈΡ†Π΅
повСрхности ΠΎΠ±ΠΊΠ»Π°Π΄ΠΎΠΊ имССтся заряд Οƒ
ΠΈ диэлСктриком являСтся Π²Π°ΠΊΡƒΡƒΠΌ, Ρ‚ΠΎ
ΠΏΠΎΠ»Π½ΠΎΠ΅ напряТСниС ΠΌΠ΅ΠΆΠ΄Ρƒ ΠΎΠ±ΠΊΠ»Π°Π΄ΠΊΠ°ΠΌΠΈ
ΠΌΠΎΠΆΠ½ΠΎ ΠΎΠΏΡ€Π΅Π΄Π΅Π»ΠΈΡ‚ΡŒ ΠΈΠ· распрСдСлСния
ΠΏΠΎΡ‚Π΅Π½Ρ†ΠΈΠ°Π»Π° Π² ΠΏΠΎΠ»Π΅ плоского кондСнсатора

,
Ρ‚ΠΎ

,

S
– ΠΏΠ»ΠΎΡ‰Π°Π΄ΡŒ ΠΊΠ°ΠΆΠ΄ΠΎΠΉ ΠΈΠ· пластин ΠΈΠ»ΠΈ мСньшСй
ΠΈΠ· Π½ΠΈΡ…, d
– расстояниС ΠΌΠ΅ΠΆΠ΄Ρƒ пластинами. ΠŸΠΎΠ»Π½Ρ‹ΠΉ
заряд пластины

.
Если диэлСктриком являСтся Π½Π΅ Π²Π°ΠΊΡƒΡƒΠΌ,
Π° вСщСство с диэлСктричСской ΠΏΡ€ΠΎΠ½ΠΈΡ†Π°Π΅ΠΌΠΎΡΡ‚ΡŒΡŽ
Ξ΅, Π·Π°ΠΏΠΎΠ»Π½ΡΡŽΡ‰Π΅Π΅ всС пространство, Π³Π΄Π΅
имССтся элСктричСскоС ΠΏΠΎΠ»Π΅ (пространство
ΠΌΠ΅ΠΆΠ΄Ρƒ ΠΎΠ±ΠΊΠ»Π°Π΄ΠΊΠ°ΠΌΠΈ), Ρ‚ΠΎ Π΅ΠΌΠΊΠΎΡΡ‚ΡŒ Π±ΡƒΠ΄Π΅Ρ‚ Π² Ξ΅
Ρ€Π°Π· большС:

.

ΠΠΌΠΊΠΎΡΡ‚ΡŒ плоского
многопластинчатого кондСнсатора
отличаСтся ΠΎΡ‚ ёмкости плоского
кондСнсатора Π·Π°ΠΌΠ΅Π½ΠΎΠΉ S
Π½Π° S
(n-1),
Π³Π΄Π΅ n
– число пластин (ΠΎΠ±ΠΊΠ»Π°Π΄ΠΎΠΊ).

.

ΠŸΡ€ΠΈ ΡƒΠΌΠ΅Π½ΡŒΡˆΠ΅Π½ΠΈΠΈ
расстояния d
ΠΌΠ΅ΠΆΠ΄Ρƒ ΠΎΠ±ΠΊΠ»Π°Π΄ΠΊΠ°ΠΌΠΈ Ρ‘ΠΌΠΊΠΎΡΡ‚ΡŒ увСличиваСтся.

ΠΠΌΠΊΠΎΡΡ‚ΡŒ
цилиндричСского кондСнсатора ΠΈ
коаксиального кабСля
:

ΠŸΡƒΡΡ‚ΡŒ кондСнсатор
состоит ΠΈΠ· Π΄Π²ΡƒΡ… ΠΊΠΎΠ°ΠΊΡΠΈΠ°Π»ΡŒΠ½Ρ‹Ρ… Ρ†ΠΈΠ»ΠΈΠ½Π΄Ρ€ΠΎΠ²
с радиусами r 2
(внСшний) ΠΈ r 1 (Π²Π½ΡƒΡ‚Ρ€Π΅Π½Π½ΠΈΠΉ). Π”Π»ΠΈΠ½Ρƒ
Ρ†ΠΈΠ»ΠΈΠ½Π΄Ρ€Π° Π±ΡƒΠ΄Π΅ΠΌ ΡΡ‡ΠΈΡ‚Π°Ρ‚ΡŒ вСсьма большой
ΠΏΠΎ ΡΡ€Π°Π²Π½Π΅Π½ΠΈΡŽ с Π·Π°Π·ΠΎΡ€ΠΎΠΌ ΠΌΠ΅ΠΆΠ΄Ρƒ Π½ΠΈΠΌΠΈ.
НапряТСниС ΠΌΠ΅ΠΆΠ΄Ρƒ ΠΎΠ±ΠΊΠ»Π°Π΄ΠΊΠ°ΠΌΠΈ

,

Π³Π΄Π΅ r 2
ΠΈ r 1 – радиусы внСшнСго
ΠΈ Π²Π½ΡƒΡ‚Ρ€Π΅Π½Π½Π΅Π³ΠΎ Ρ†ΠΈΠ»ΠΈΠ½Π΄Ρ€ΠΎΠ², l

– Π΄Π»ΠΈΠ½Π° Ρ†ΠΈΠ»ΠΈΠ½Π΄Ρ€Π°, q
– заряд Π²Π½ΡƒΡ‚Ρ€Π΅Π½Π½Π΅Π³ΠΎ Ρ†ΠΈΠ»ΠΈΠ½Π΄Ρ€Π° Π½Π° Π΅Π΄ΠΈΠ½ΠΈΡ†Ρƒ
Π΅Π³ΠΎ Π΄Π»ΠΈΠ½Ρ‹.

ΠŸΠΎΡΡ‚ΠΎΠΌΡƒ Ρ‘ΠΌΠΊΠΎΡΡ‚ΡŒ
цилиндричСского кондСнсатора Π² Π²Π°ΠΊΡƒΡƒΠΌΠ΅

,

Π­Ρ‚Π° Ρ„ΠΎΡ€ΠΌΡƒΠ»Π°
Π²Ρ‹Ρ€Π°ΠΆΠ°Π΅Ρ‚, Π² частности, Ρ‘ΠΌΠΊΠΎΡΡ‚ΡŒ кабСля,
ΠΊΠΎΡ‚ΠΎΡ€Ρ‹ΠΉ состоит ΠΈΠ· мСталличСского
ΠΏΡ€ΠΎΠ²ΠΎΠ΄Π°, ΠΎΠΊΡ€ΡƒΠΆΠ΅Π½Π½ΠΎΠ³ΠΎ слоСм изолятора
ΠΈ мСталличСской Π±Ρ€ΠΎΠ½Π΅ΠΉ; Π΄Π°Π½Π½ΠΎΠ΅ Π²Ρ‹Ρ€Π°ΠΆΠ΅Π½ΠΈΠ΅
слСдуСт ΡƒΠΌΠ½ΠΎΠΆΠΈΡ‚ΡŒ Π΅Ρ‰Π΅ Π½Π° Π΄ΠΈΡΠ»Π΅ΠΊΡ‚Ρ€ΠΈΡ‡Π΅ΡΠΊΡƒΡŽ
ΠΏΡ€ΠΎΠ½ΠΈΡ†Π°Π΅ΠΌΠΎΡΡ‚ΡŒ вСщСства изолятора

ΠΠΌΠΊΠΎΡΡ‚ΡŒ сфСричСского
кондСнсатора:

Если Π½Π° ΠΎΠ±ΠΊΠ»Π°Π΄ΠΊΠ°Ρ…
кондСнсатора имССтся заряд q,
Ρ‚ΠΎ напряТСниС ΠΌΠ΅ΠΆΠ΄Ρƒ ΠΎΠ±ΠΊΠ»Π°Π΄ΠΊΠ°ΠΌΠΈ Π² Π²Π°ΠΊΡƒΡƒΠΌΠ΅

,

Π³Π΄Π΅ r 2
ΠΈ r 1 – радиусы внСшнСй
ΠΈ Π²Π½ΡƒΡ‚Ρ€Π΅Π½Π½Π΅ΠΉ сфСр. Если диэлСктриком
являСтся Π½Π΅ Π²Π°ΠΊΡƒΡƒΠΌ, Π° вСщСство с
диэлСктричСской ΠΏΡ€ΠΎΠ½ΠΈΡ†Π°Π΅ΠΌΠΎΡΡ‚ΡŒΡŽ Ξ΅, Ρ‚ΠΎ

.

Если внСшний радиус
r 2
Π³ΠΎΡ€Π°Π·Π΄ΠΎ большС
Π²Π½ΡƒΡ‚Ρ€Π΅Π½Π½Π΅Π³ΠΎ r 1 ,
Ρ‚ΠΎ эта Ρ„ΠΎΡ€ΠΌΡƒΠ»Π° упрощаСтся

Π•ΠΌΠΊΠΎΡΡ‚ΡŒ
Π΄Π²ΡƒΡ…ΠΏΡ€ΠΎΠ²ΠΎΠ΄Π½ΠΎΠΉ Π»ΠΈΠ½ΠΈΠΈ:

Рассмотрим Π΄Π²Π°
ΠΏΠ°Ρ€Π°Π»Π»Π΅Π»ΡŒΠ½Ρ‹Ρ… цилиндричСских ΠΏΡ€ΠΎΠ²ΠΎΠ΄Π°
с радиусами r
ΠΈ расстояниСм ΠΌΠ΅ΠΆΠ΄Ρƒ осями d
(рис.5). Π‘ΡƒΠ΄Π΅ΠΌ ΡΡ‡ΠΈΡ‚Π°Ρ‚ΡŒ, Ρ‡Ρ‚ΠΎ всС ΠΎΡΡ‚Π°Π»ΡŒΠ½Ρ‹Π΅
Ρ‚Π΅Π»Π°, Π²ΠΊΠ»ΡŽΡ‡Π°Ρ ΠΈ зСмлю, находятся Π½Π°
расстояниях, Π±ΠΎΠ»ΡŒΡˆΠΈΡ… ΠΏΠΎ ΡΡ€Π°Π²Π½Π΅Π½ΠΈΡŽ с d,
ΠΈ поэтому Π±ΡƒΠ΄Π΅ΠΌ Ρ€Π°ΡΡΠΌΠ°Ρ‚Ρ€ΠΈΠ²Π°Ρ‚ΡŒ ΠΎΠ±Π°
ΠΏΡ€ΠΎΠ²ΠΎΠ΄Π° ΠΊΠ°ΠΊ простой кондСнсатор.
ΠŸΡ€Π΅Π΄ΠΏΠΎΠ»ΠΎΠΆΠΈΠΌ, Ρ‡Ρ‚ΠΎ d
>> a.
Π’ этом случаС ΠΎΠ±Π° Ρ†ΠΈΠ»ΠΈΠ½Π΄Ρ€Π° заряТСны
Ρ€Π°Π²Π½ΠΎΠΌΠ΅Ρ€Π½ΠΎ. Π’Π°ΠΊ ΠΊΠ°ΠΊ напряТСниС Π²
элСктростатичСском ΠΏΠΎΠ»Π΅ Π½Π΅ зависит ΠΎΡ‚
Ρ„ΠΎΡ€ΠΌΡ‹ ΠΏΡƒΡ‚ΠΈ, Ρ‚ΠΎ для Π΅Π³ΠΎ вычислСния Π²Ρ‹Π±Π΅Ρ€Π΅ΠΌ
ΠΏΡ€ΠΎΡΡ‚Π΅ΠΉΡˆΠΈΠΉ ΠΏΡƒΡ‚ΡŒ Π² Π²ΠΈΠ΄Π΅ прямой Π»ΠΈΠ½ΠΈΠΈ,
ΡΠΎΠ΅Π΄ΠΈΠ½ΡΡŽΡ‰Π΅ΠΉ оси ΠΏΡ€ΠΎΠ²ΠΎΠ΄ΠΎΠ² ΠΈ пСрпСндикулярной
ΠΊ ΠΈΡ… повСрхности. ΠŸΠΎΡΡ‚ΠΎΠΌΡƒ напряТСниС U
ΠΌΠ΅ΠΆΠ΄Ρƒ ΠΏΡ€ΠΎΠ²ΠΎΠ΄Π°ΠΌΠΈ

,

ΠΠΌΠΊΠΎΡΡ‚ΡŒ Π΄Π²ΡƒΡ…
ΠΏΡ€ΠΎΠ²ΠΎΠ΄Π½ΠΎΠΉ Π»ΠΈΠ½ΠΈΠΈ Π² Π²Π°ΠΊΡƒΡƒΠΌΠ΅

,

Π² диэлСктрикС

d
– расстояниС ΠΌΠ΅ΠΆΠ΄Ρƒ осями ΠΏΡ€ΠΎΠ²ΠΎΠ΄ΠΎΠ², r
– радиус ΠΏΡ€ΠΎΠ²ΠΎΠ΄ΠΎΠ², l

– Π΄Π»ΠΈΠ½Π° Π»ΠΈΠ½ΠΈΠΈ.

Для всСх Ρ‚ΠΈΠΏΠΎΠ²
кондСнсаторов сущСствуСт ΠΏΡ€ΠΎΠ±ΠΈΠ²Π½ΠΎΠ΅
напряТСниС – Ρ€Π°Π·Π½ΠΎΡΡ‚ΡŒ ΠΏΠΎΡ‚Π΅Π½Ρ†ΠΈΠ°Π»ΠΎΠ²
ΠΌΠ΅ΠΆΠ΄Ρƒ ΠΎΠ±ΠΊΠ»Π°Π΄ΠΊΠ°ΠΌΠΈ, ΠΏΡ€ΠΈ ΠΊΠΎΡ‚ΠΎΡ€ΠΎΠΉ происходит
элСктричСский разряд Ρ‡Π΅Ρ€Π΅Π· слой
диэлСктрика. ΠŸΡ€ΠΎΠ±ΠΈΠ²Π½ΠΎΠ΅ напряТСниС
зависит ΠΎΡ‚ Ρ‚ΠΎΠ»Ρ‰ΠΈΠ½Ρ‹ диэлСктрика, Π΅Π³ΠΎ
свойств ΠΈ Ρ„ΠΎΡ€ΠΌΡ‹ ΠΎΠ±ΠΊΠ»Π°Π΄ΠΎΠΊ. Π‘ ΡƒΠΌΠ΅Π½ΡŒΡˆΠ΅Π½ΠΈΠ΅ΠΌ
Ρ‚ΠΎΠ»Ρ‰ΠΈΠ½Ρ‹ диэлСктрика ΠΏΠ°Π΄Π°Π΅Ρ‚ ΠΏΡ€ΠΎΠ±ΠΈΠ²Π½ΠΎΠ΅
напряТСниС ΠΈ ΠΏΡ€ΠΈ Ρ‚ΠΎΠ»Ρ‰ΠΈΠ½Π΅ 1 ΠΌΠΊΠΌ ΠΏΡ€ΠΎΠ±ΠΈΠ²Π½ΠΎΠ΅
напряТСниС Π½Π΅ ΠΏΡ€Π΅Π²Ρ‹ΡˆΠ°Π΅Ρ‚ 10 Π’. Π£Π²Π΅Π»ΠΈΡ‡Π΅Π½ΠΈΠ΅
Смкости, ΠΏΡ€ΠΈ ΡƒΠΌΠ΅Π½ΡŒΡˆΠ΅Π½ΠΈΠΈ Ρ‚ΠΎΠ»Ρ‰ΠΈΠ½Ρ‹
диэлСктрика, происходит Π·Π° счСт сниТСния
Ρ€Π°Π±ΠΎΡ‡Π΅Π³ΠΎ напряТСния.

Π›Π΅ΠΊΡ†ΠΈΠΈ ΠΏΠΎ ВОЭ/
β„–21 Π‘ΠΈΠ½ΡƒΡΠΎΠΈΠ΄Π°Π»ΡŒΠ½Ρ‹ΠΉ Ρ‚ΠΎΠΊ Π² Смкости.

БистСма ΠΈΠ· Π΄Π²ΡƒΡ… проводящих Ρ‚Π΅Π», Ρ€Π°Π·Π΄Π΅Π»Π΅Π½Π½Ρ‹Ρ… диэлСктриком, ΠΎΠ±Ρ€Π°Π·ΡƒΠ΅Ρ‚ кондСнсатор. Π­Ρ‚ΠΈ проводящиС Ρ‚Π΅Π»Π° Π½Π°Π·Ρ‹Π²Π°ΡŽΡ‚ΡΡ ΠΎΠ±ΠΊΠ»Π°Π΄ΠΊΠ°ΠΌΠΈ. Если ΠΊ Π½ΠΈΠΌ ΠΏΠΎΠ΄ΠΊΠ»ΡŽΡ‡ΠΈΡ‚ΡŒ источник энСргии, Ρ‚ΠΎ Π½Π° Π½ΠΈΡ… Π±ΡƒΠ΄Π΅Ρ‚ Π½Π°ΠΊΠ°ΠΏΠ»ΠΈΠ²Π°Ρ‚ΡŒΡΡ заряд q
, ΠΏΡ€ΠΎΠΏΠΎΡ€Ρ†ΠΈΠΎΠ½Π°Π»ΡŒΠ½Ρ‹ΠΉ Π½Π°ΠΏΡ€ΡΠΆΠ΅Π½ΠΈΡŽ Π½Π° кондСнсаторС u c

ΠšΠΎΡΡ„Ρ„ΠΈΡ†ΠΈΠ΅Π½Ρ‚ ΠΏΡ€ΠΎΠΏΠΎΡ€Ρ†ΠΈΠΎΠ½Π°Π»ΡŒΠ½ΠΎΡΡ‚ΠΈ C ΠΌΠ΅ΠΆΠ΄Ρƒ зарядом ΠΈ напряТСниСм называСтся Π΅ΠΌΠΊΠΎΡΡ‚ΡŒΡŽ кондСнсатора. Π•Π΄ΠΈΠ½ΠΈΡ†Π° измСрСния Смкости – Ρ„Π°Ρ€Π°Π΄Π° (Π€). Она ΠΈΠΌΠ΅Π΅Ρ‚ ΡΠ»Π΅Π΄ΡƒΡŽΡ‰ΡƒΡŽ Ρ€Π°Π·ΠΌΠ΅Ρ€Π½ΠΎΡΡ‚ΡŒ: Кл/Π’=А*с/Π’=с/Ом=Ом -1 *с. Π•ΠΌΠΊΠΎΡΡ‚ΡŒ зависит ΠΎΡ‚ Ρ„ΠΎΡ€ΠΌΡ‹, Ρ€Π°Π·ΠΌΠ΅Ρ€ΠΎΠ² кондСнсатора ΠΈ ΠΎΡ‚ диэлСктричСской проницаСмости диэлСктрика ΠΌΠ΅ΠΆΠ΄Ρƒ ΠΎΠ±ΠΊΠ»Π°Π΄ΠΊΠ°ΠΌΠΈ. ΠŸΡƒΡΡ‚ΡŒ напряТСниС, ΠΏΠΎΠ΄Π°Π²Π°Π΅ΠΌΠΎΠ΅ источником Π½Π° кондСнсатор, измСняСтся ΠΏΠΎ Π·Π°ΠΊΠΎΠ½Ρƒ:

U c =U cmax sin(Ο‰t+ψ)

ΠŸΡ€ΠΈ Π΅Π³ΠΎ возрастании ΠΎΡ‚ нуля Π΄ΠΎ максимального значСния кондСнсатор заряТаСтся, Π½Π° Π΅Π³ΠΎ ΠΎΠ±ΠΊΠ»Π°Π΄ΠΊΠΈ ΠΎΡ‚ источника поступаСт элСктричСский заряд. ΠŸΡ€ΠΈ ΡƒΠΌΠ΅Π½ΡŒΡˆΠ΅Π½ΠΈΠΈ напряТСния ΠΎΡ‚ максимума Π΄ΠΎ нуля, заряд стСкаСт с кондСнсатора, ΠΎΠ½ разряТаСтся. Π’Π°ΠΊΠΈΠΌ ΠΎΠ±Ρ€Π°Π·ΠΎΠΌ, Π² ΠΏΡ€ΠΎΠ²ΠΎΠ΄Π°Ρ…, ΡΠΎΠ΅Π΄ΠΈΠ½ΡΡŽΡ‰ΠΈΡ… кондСнсатор с ΠΎΡΡ‚Π°Π»ΡŒΠ½ΠΎΠΉ Ρ†Π΅ΠΏΡŒΡŽ, постоянно двиТСтся элСктричСский заряд, Ρ‚.Π΅. ΠΏΡ€ΠΎΡ‚Π΅ΠΊΠ°Π΅Ρ‚ элСктричСский Ρ‚ΠΎΠΊ. Π’Ρ‹Π²ΠΎΠ΄ ΠΎ Π½Π°Π»ΠΈΡ‡ΠΈΠΈ элСктричСского Ρ‚ΠΎΠΊΠ° ΠΌΡ‹ Π΄Π΅Π»Π°Π΅ΠΌ, ΡΠΎΠ²Π΅Ρ€ΡˆΠ΅Π½Π½ΠΎ Π½Π΅ касаясь вопроса ΠΎ Ρ‚ΠΎΠΌ, ΠΊΠ°ΠΊΠΈΠ΅ процСссы происходят ΠΌΠ΅ΠΆΠ΄Ρƒ ΠΎΠ±ΠΊΠ»Π°Π΄ΠΊΠ°ΠΌΠΈ кондСнсатора. Π’Π΅Π»ΠΈΡ‡ΠΈΠ½Π° Ρ‚ΠΎΠΊΠ° опрСдСляСтся зарядом, ΠΏΡ€ΠΎΡˆΠ΅Π΄ΡˆΠΈΠΌ Π² Π΅Π΄ΠΈΠ½ΠΈΡ†Ρƒ Π²Ρ€Π΅ΠΌΠ΅Π½ΠΈ Ρ‡Π΅Ρ€Π΅Π· ΠΏΠΎΠΏΠ΅Ρ€Π΅Ρ‡Π½ΠΎΠ΅ сСчСниС ΠΏΡ€ΠΎΠ²ΠΎΠ΄Π½ΠΈΠΊΠ°:

Она зависит ΠΎΡ‚ Смкости ΠΈ скорости измСнСния ΠΏΠΈΡ‚Π°ΡŽΡ‰Π΅Π³ΠΎ напряТСния, Ρ‚.Π΅. ΠΎΡ‚ частоты. ΠžΡ‚ этих ΠΆΠ΅ Ρ„Π°ΠΊΡ‚ΠΎΡ€ΠΎΠ² зависит ΠΈ элСктричСская ΠΏΡ€ΠΎΠ²ΠΎΠ΄ΠΈΠΌΠΎΡΡ‚ΡŒ участка Ρ†Π΅ΠΏΠΈ с кондСнсатором. Π•Π΅ Π½Π°Π·Ρ‹Π²Π°ΡŽΡ‚ Смкостной ΠΏΡ€ΠΎΠ²ΠΎΠ΄ΠΈΠΌΠΎΡΡ‚ΡŒΡŽ ΠΈ ΠΎΠΏΡ€Π΅Π΄Π΅Π»ΡΡŽΡ‚ ΠΏΠΎ Ρ„ΠΎΡ€ΠΌΡƒΠ»Π΅:

Наклона ΠΊΠ°ΠΆΠ΄ΠΎΠ³ΠΎ Π²Π΅ΠΊΡ‚ΠΎΡ€Π° ΠΊ ΠΏΠΎΠ»ΠΎΠΆΠΈΡ‚Π΅Π»ΡŒΠ½ΠΎΠΌΡƒ Π½Π°ΠΏΡ€Π°Π²Π»Π΅Π½ΠΈΡŽ вСщСствСнной оси опрСдСляСтся Π½Π°Ρ‡Π°Π»ΡŒΠ½Ρ‹ΠΌΠΈ Ρ„Π°Π·Π°ΠΌΠΈ Π² выраТСниях Π²Ρ‹ΡˆΠ΅. Π’Π°ΠΊ ΠΊΠ°ΠΊ ΠΏΡ€ΠΈ ΠΎΠΏΡ€Π΅Π΄Π΅Π»Π΅Π½ΠΈΠΈ напряТСния U c ΠΌΡ‹ ΡƒΠΌΠ½ΠΎΠΆΠ°Π΅ΠΌ Ix c Π½Π° -j, Ρ‚ΠΎ Π²Π΅ΠΊΡ‚ΠΎΡ€ U c оказываСтся ΠΏΠΎΠ²Π΅Ρ€Π½ΡƒΡ‚Ρ‹ΠΌ ΠΎΡ‚Π½ΠΎΡΠΈΡ‚Π΅Π»ΡŒΠ½ΠΎ Π²Π΅ΠΊΡ‚ΠΎΡ€Π° Ρ‚ΠΎΠΊΠ° Π½Π° ΡƒΠ³ΠΎΠ» 90Π³Ρ€Π°Π΄. Π² ΠΎΡ‚Ρ€ΠΈΡ†Π°Ρ‚Π΅Π»ΡŒΠ½ΠΎΠΌ Π½Π°ΠΏΡ€Π°Π²Π»Π΅Π½ΠΈΠΈ, ΠΏΠΎ часовой стрСлкС. Как ΠΎΡ‚ΠΌΠ΅Ρ‡Π°Π»ΠΎΡΡŒ Ρ€Π°Π½ΡŒΡˆΠ΅, Π½Π°ΠΏΡ€Π°Π²Π»Π΅Π½ΠΈΠ΅ ΡƒΠ³Π»Π° Ο† Π½Π° Π΄ΠΈΠ°Π³Ρ€Π°ΠΌΠΌΠ΅ показываСтся ΠΎΡ‚ Π²Π΅ΠΊΡ‚ΠΎΡ€Π° Ρ‚ΠΎΠΊΠ° ΠΊ Π²Π΅ΠΊΡ‚ΠΎΡ€Ρƒ напряТСния.

ΠŸΡ€ΠΈΠΌΠ΅Ρ€ 2.6.

НапряТСниС Π½Π° кондСнсаторС u C = 100sin (1000t –30Β°). ΠΠ°ΠΏΠΈΡΠ°Ρ‚ΡŒ Π²Ρ‹Ρ€Π°ΠΆΠ΅Π½ΠΈΠ΅ ΠΌΠ³Π½ΠΎΠ²Π΅Π½Π½ΠΎΠ³ΠΎ значСния Ρ‚ΠΎΠΊΠ° Ρ‡Π΅Ρ€Π΅Π· кондСнсатор. Каким станСт Ρ‚ΠΎΠΊ, Ссли частота ΠΏΠΈΡ‚Π°ΡŽΡ‰Π΅Π³ΠΎ напряТСния увСличится Π²Π΄Π²ΠΎΠ΅? Π•ΠΌΠΊΠΎΡΡ‚ΡŒ кондСнсатора Π‘ = 50 ΠΌΠΊΠ€.

РСшСниС.

ΠžΠΏΡ€Π΅Π΄Π΅Π»ΡΠ΅ΠΌ СмкостноС сопротивлСниС:

Π–Π΅Π»Π°Π΅ΠΌ ΡƒΠ΄Π°Ρ‡Π½ΠΎΠ³ΠΎ изучСния ΠΌΠ°Ρ‚Π΅Ρ€ΠΈΠ°Π»Π° ΠΈ ΡƒΡΠΏΠ΅ΡˆΠ½ΠΎΠΉ сдачи!

Рассмотрим ΡΠ»Π΅ΠΊΡ‚Ρ€ΠΈΡ‡Π΅ΡΠΊΡƒΡŽ Ρ†Π΅ΠΏΡŒ, ΡΠΎΠ΄Π΅Ρ€ΠΆΠ°Ρ‰ΡƒΡŽ рСзистор с Π°ΠΊΡ‚ΠΈΠ²Π½Ρ‹ΠΌ сопротивлСниСм R
ΠΈ кондСнсатор Смкости C
, ΠΏΠΎΠ΄ΠΊΠ»ΡŽΡ‡Π΅Π½Π½ΡƒΡŽ ΠΊ источнику ΠΏΠ΅Ρ€Π΅ΠΌΠ΅Π½Π½ΠΎΠΉ Π­Π”Π‘ (рис. 653).

рис. 653

β€ƒΠšΠΎΠ½Π΄Π΅Π½ΡΠ°Ρ‚ΠΎΡ€, ΠΏΠΎΠ΄ΠΊΠ»ΡŽΡ‡Π΅Π½Π½Ρ‹ΠΉ ΠΊ источнику постоянной Π­Π”Π‘, ΠΏΠΎΠ»Π½ΠΎΡΡ‚ΡŒΡŽ прСпятствуСт прохоТдСния Ρ‚ΠΎΠΊΠ° βˆ’ Π·Π° Π½Π΅ΠΊΠΎΡ‚ΠΎΡ€Ρ‹ΠΉ ΠΏΡ€ΠΎΠΌΠ΅ΠΆΡƒΡ‚ΠΎΠΊ Π²Ρ€Π΅ΠΌΠ΅Π½ΠΈ кондСнсатор заряТаСтся, напряТСниС ΠΌΠ΅ΠΆΠ΄Ρƒ Π΅Π³ΠΎ ΠΎΠ±ΠΊΠ»Π°Π΄ΠΊΠ°ΠΌΠΈ становится Ρ€Π°Π²Π½Ρ‹ΠΌ Π­Π”Π‘ источника, послС Ρ‡Π΅Π³ΠΎ Ρ‚ΠΎΠΊ Π² Ρ†Π΅ΠΏΠΈ прСкращаСтся. Если ΠΆΠ΅ кондСнсатор Π²ΠΊΠ»ΡŽΡ‡Π΅Π½ Π² Ρ†Π΅ΠΏΡŒ ΠΏΠ΅Ρ€Π΅ΠΌΠ΅Π½Π½ΠΎΠ³ΠΎ Ρ‚ΠΎΠΊΠ°, Ρ‚ΠΎ Ρ‚ΠΎΠΊ Π² Ρ†Π΅ΠΏΠΈ Π½Π΅ прСкращаСтся βˆ’ фактичСски кондСнсатор пСриодичСски пСрСзаряТаСтся, заряды Π½Π° Π΅Π³ΠΎ ΠΎΠ±ΠΊΠ»Π°Π΄ΠΊΠ°Ρ… пСриодичСски ΠΈΠ·ΠΌΠ΅Π½ΡΡŽΡ‚ΡΡ ΠΊΠ°ΠΊ ΠΏΠΎ Π²Π΅Π»ΠΈΡ‡ΠΈΠ½Π΅, Ρ‚Π°ΠΊ ΠΈ ΠΏΠΎ Π·Π½Π°ΠΊΡƒ. ΠšΠΎΠ½Π΅Ρ‡Π½ΠΎ, Π½ΠΈΠΊΠ°ΠΊΠΈΠ΅ заряды Π½Π΅ ΠΏΡ€ΠΎΡ‚Π΅ΠΊΠ°ΡŽΡ‚ ΠΌΠ΅ΠΆΠ΄Ρƒ ΠΎΠ±ΠΊΠ»Π°Π΄ΠΊΠ°ΠΌΠΈ, элСктричСского Ρ‚ΠΎΠΊΠ° Π² строгом ΠΎΠΏΡ€Π΅Π΄Π΅Π»Π΅Π½ΠΈΠΈ ΠΌΠ΅ΠΆΠ΄Ρƒ Π½ΠΈΠΌΠΈ Π½Π΅Ρ‚. Но, часто Π½Π΅ вдаваясь Π² Π΄Π΅Ρ‚Π°Π»ΠΈ ΠΈ Π½Π΅ слишком ΠΊΠΎΡ€Ρ€Π΅ΠΊΡ‚Π½ΠΎ, говорят ΠΎ Ρ‚ΠΎΠΊΠ΅ Ρ‡Π΅Ρ€Π΅Π· кондСнсатор, подразумСвая ΠΏΠΎΠ΄ этим Ρ‚ΠΎΠΊ Π² Ρ†Π΅ΠΏΠΈ, ΠΊ ΠΊΠΎΡ‚ΠΎΡ€ΠΎΠΉ ΠΏΠΎΠ΄ΠΊΠ»ΡŽΡ‡Π΅Π½ кондСнсатор. Π’Π°ΠΊΠΎΠΉ ΠΆΠ΅ Ρ‚Π΅Ρ€ΠΌΠΈΠ½ΠΎΠ»ΠΎΠ³ΠΈΠ΅ΠΉ Π±ΡƒΠ΄Π΅ΠΌ ΠΏΠΎΠ»ΡŒΠ·ΠΎΠ²Π°Ρ‚ΡŒΡΡ ΠΈ ΠΌΡ‹.
β€ƒΠŸΠΎ-ΠΏΡ€Π΅ΠΆΠ½Π΅ΠΌΡƒ, для ΠΌΠ³Π½ΠΎΠ²Π΅Π½Π½Ρ‹Ρ… Π·Π½Π°Ρ‡Π΅Π½ΠΈΠΉ справСдлив Π·Π°ΠΊΠΎΠ½ Ома для ΠΏΠΎΠ»Π½ΠΎΠΉ Ρ†Π΅ΠΏΠΈ: Π­Π”Π‘ источника Ρ€Π°Π²Π½Π° суммС напряТСний Π½Π° всСх участках Ρ†Π΅ΠΏΠΈ. ΠŸΡ€ΠΈΠΌΠ΅Π½Π΅Π½ΠΈΠ΅ этого Π·Π°ΠΊΠΎΠ½Π° ΠΊ рассматриваСмой Ρ†Π΅ΠΏΠΈ ΠΏΡ€ΠΈΠ²ΠΎΠ΄ΠΈΡ‚ ΠΊ ΡƒΡ€Π°Π²Π½Π΅Π½ΠΈΡŽ

здСсь U R = IR
βˆ’ напряТСниС Π½Π° рСзисторС, U C = q/C
βˆ’ напряТСниС Π½Π° кондСнсаторС, q
βˆ’ элСктричСский заряд Π½Π° Π΅Π³ΠΎ ΠΎΠ±ΠΊΠ»Π°Π΄ΠΊΠ°Ρ…. Π£Ρ€Π°Π²Π½Π΅Π½ΠΈΠ΅ (1) содСрТит Ρ‚Ρ€ΠΈ ΠΈΠ·ΠΌΠ΅Π½ΡΡŽΡ‰ΠΈΡ…ΡΡ Π²ΠΎ Π²Ρ€Π΅ΠΌΠ΅Π½ΠΈ Π²Π΅Π»ΠΈΡ‡ΠΈΠ½Ρ‹ (ΠΈΠ·Π²Π΅ΡΡ‚Π½ΡƒΡŽ Π­Π”Π‘, ΠΈ ΠΏΠΎΠΊΠ° нСизвСстныС силу Ρ‚ΠΎΠΊΠ° ΠΈ заряд кондСнсатора), учитывая, Ρ‡Ρ‚ΠΎ сила Ρ‚ΠΎΠΊΠ° Ρ€Π°Π²Π½Π° ΠΏΡ€ΠΎΠΈΠ·Π²ΠΎΠ΄Π½ΠΎΠΉ ΠΏΠΎ Π²Ρ€Π΅ΠΌΠ΅Π½ΠΈ ΠΎΡ‚ заряда кондСнсатора I = q /
, это ΡƒΡ€Π°Π²Π½Π΅Π½ΠΈΠ΅ ΠΌΠΎΠΆΠ΅Ρ‚ Π±Ρ‹Ρ‚ΡŒ Ρ‚ΠΎΡ‡Π½ΠΎ Ρ€Π΅ΡˆΠ΅Π½ΠΎ. Π’Π°ΠΊ ΠΊΠ°ΠΊ Π­Π”Π‘ источника измСняСтся ΠΏΠΎ гармоничСскому Π·Π°ΠΊΠΎΠ½Ρƒ, Ρ‚ΠΎ ΠΈ напряТСниС Π½Π° кондСнсаторС ΠΈ сила Ρ‚ΠΎΠΊΠ° Π² Ρ†Π΅ΠΏΠΈ Ρ‚Π°ΠΊΠΆΠ΅ Π±ΡƒΠ΄ΡƒΡ‚ ΠΈΠ·ΠΌΠ΅Π½ΡΡ‚ΡŒΡΡ ΠΏΠΎ гармоничСским Π·Π°ΠΊΠΎΠ½Π°ΠΌ с Ρ‚ΠΎΠΉ ΠΆΠ΅ частотой βˆ’ это ΡƒΡ‚Π²Π΅Ρ€ΠΆΠ΄Π΅Π½ΠΈΠ΅ нСпосрСдствСнно слСдуСт ΠΈ уравнСния (1).
 Бначала установим связь ΠΌΠ΅ΠΆΠ΄Ρƒ силой Ρ‚ΠΎΠΊΠ° Π² Ρ†Π΅ΠΏΠΈ напряТСниСм Π½Π° кондСнсаторС. Π—Π°Π²ΠΈΡΠΈΠΌΠΎΡΡ‚ΡŒ напряТСния ΠΎΡ‚ Π²Ρ€Π΅ΠΌΠ΅Π½ΠΈ прСдставим Π² Π²ΠΈΠ΄Π΅

β€ƒΠŸΠΎΠ΄Ρ‡Π΅Ρ€ΠΊΠ½Π΅ΠΌ, Ρ‡Ρ‚ΠΎ Π² Π΄Π°Π½Π½ΠΎΠΌ случаС напряТСниС Π½Π° кондСнсаторС отличаСтся ΠΎΡ‚ Π­Π”Π‘ источника, ΠΊΠ°ΠΊ Π±ΡƒΠ΄Π΅Ρ‚ Π²ΠΈΠ΄Π½ΠΎ ΠΈΠ· дальнСйшСго излоТСния, ΠΌΠ΅ΠΆΠ΄Ρƒ этими функциями сущСствуСт Ρ‚Π°ΠΊΠΆΠ΅ ΠΈ Ρ€Π°Π·Π½ΠΎΡΡ‚ΡŒ Ρ„Π°Π·. ΠŸΠΎΡΡ‚ΠΎΠΌΡƒ ΠΏΡ€ΠΈ записи выраТСния (2), ΠΌΡ‹ Π²Ρ‹Π±ΠΈΡ€Π°Π΅ΠΌ ΠΏΡ€ΠΎΠΈΠ·Π²ΠΎΠ»ΡŒΠ½ΡƒΡŽ Π½Π°Ρ‡Π°Π»ΡŒΠ½ΡƒΡŽ Ρ„Π°Π·Ρƒ Π½ΡƒΠ»Π΅Π²ΠΎΠΉ, ΠΏΡ€ΠΈ Ρ‚Π°ΠΊΠΎΠΌ ΠΎΠΏΡ€Π΅Π΄Π΅Π»Π΅Π½ΠΈΠΈ Ρ„Π°Π·Ρ‹ Π­Π”Π‘, напряТСния Π½Π° рСзисторС ΠΈ силы Ρ‚ΠΎΠΊΠ° ΠΎΡ‚ΡΡ‡ΠΈΡ‚Ρ‹Π²Π°ΡŽΡ‚ΡΡ ΠΎΡ‚Π½ΠΎΡΠΈΡ‚Π΅Π»ΡŒΠ½ΠΎ Ρ„Π°Π·Ρ‹ ΠΊΠΎΠ»Π΅Π±Π°Π½ΠΈΠΉ напряТСния Π½Π° рСзисторС.
β€ƒΠ˜ΡΠΏΠΎΠ»ΡŒΠ·ΡƒΡ связь ΠΌΠ΅ΠΆΠ΄Ρƒ напряТСниСм ΠΈ зарядом кондСнсатора, запишСм Π²Ρ‹Ρ€Π°ΠΆΠ΅Π½ΠΈΠ΅ для зависимости послСднСго ΠΎΡ‚ Π²Ρ€Π΅ΠΌΠ΅Π½ΠΈ

ΠΊΠΎΡ‚ΠΎΡ€ΠΎΠ΅ позволяСт Π½Π°ΠΉΡ‚ΠΈ Π²Ρ€Π΅ΠΌΠ΅Π½Π½ΡƒΡŽ Π·Π°Π²ΠΈΡΠΈΠΌΠΎΡΡ‚ΡŒ силы Ρ‚ΠΎΠΊΠ° 1

Π½Π° послСднСм шагС использована тригономСтричСская Ρ„ΠΎΡ€ΠΌΡƒΠ»Π° привСдСния, для Ρ‚ΠΎΠ³ΠΎ, Ρ‡Ρ‚ΠΎΠ±Ρ‹ Π² явном Π²ΠΈΠ΄Π΅ Π²Ρ‹Π΄Π΅Π»ΠΈΡ‚ΡŒ сдвиг Ρ„Π°Π· ΠΌΠ΅ΠΆΠ΄Ρƒ Ρ‚ΠΎΠΊΠΎΠΌ ΠΈ напряТСниСм.
β€ƒΠ˜Ρ‚Π°ΠΊ, ΠΌΡ‹ ΠΏΠΎΠ»ΡƒΡ‡ΠΈΠ»ΠΈ, Ρ‡Ρ‚ΠΎ Π°ΠΌΠΏΠ»ΠΈΡ‚ΡƒΠ΄Π½ΠΎΠ΅ Π·Π½Π°Ρ‡Π΅Π½ΠΈΠ΅ силы Ρ‚ΠΎΠΊΠ° Ρ‡Π΅Ρ€Π΅Π· кондСнсатор связано с напряТСниСм Π½Π° Π½Π΅ΠΌ ΡΠΎΠΎΡ‚Π½ΠΎΡˆΠ΅Π½ΠΈΠ΅ΠΌ

Π° Ρ‚Π°ΠΊΠΆΠ΅ ΠΌΠ΅ΠΆΠ΄Ρƒ колСбаниями силы Ρ‚ΠΎΠΊΠ° ΠΈ напряТСния сущСствуСт Ρ€Π°Π·Π½ΠΎΡΡ‚ΡŒ Ρ„Π°Π·, Ρ€Π°Π²Π½Π° Δφ = Ο€/2
. Π­Ρ‚ΠΈ Ρ€Π΅Π·ΡƒΠ»ΡŒΡ‚Π°Ρ‚Ρ‹ суммированы Π½Π° рис. 654, Π³Π΄Π΅ Ρ‚Π°ΠΊΠΆΠ΅ прСдставлСна вСкторная Π΄ΠΈΠ°Π³Ρ€Π°ΠΌΠΌΠ° ΠΊΠΎΠ»Π΅Π±Π°Π½ΠΈΠΉ силы Ρ‚ΠΎΠΊΠ° ΠΈ напряТСния.

рис. 654

 Для Ρ‚ΠΎΠ³ΠΎ, Ρ‡Ρ‚ΠΎΠ±Ρ‹ ΡΠΎΡ…Ρ€Π°Π½ΠΈΡ‚ΡŒ Ρ„ΠΎΡ€ΠΌΡƒ Π·Π°ΠΊΠΎΠ½Π° Ома для участка Ρ†Π΅ΠΏΠΈ, вводят понятиС Смкостного сопротивлСния
, ΠΊΠΎΡ‚ΠΎΡ€ΠΎΠ΅ опрСдСляСтся ΠΏΠΎ Ρ„ΠΎΡ€ΠΌΡƒΠ»Π΅

 В этом случаС ΡΠΎΠΎΡ‚Π½ΠΎΡˆΠ΅Π½ΠΈΠ΅ (5) становится Ρ‚Ρ€Π°Π΄ΠΈΡ†ΠΈΠΎΠ½Π½Ρ‹ΠΌ для Π·Π°ΠΊΠΎΠ½Π° Ома

β€ƒΠŸΡ€ΠΈ ΠΈΠ·ΡƒΡ‡Π΅Π½ΠΈΠΈ Π·Π°ΠΊΠΎΠ½Π° Ома для Ρ†Π΅ΠΏΠ΅ΠΉ постоянного Ρ‚ΠΎΠΊΠ°, ΠΌΡ‹ ΡƒΠΊΠ°Π·Ρ‹Π²Π°Π»ΠΈ, Ρ‡Ρ‚ΠΎ элСктричСскоС ΠΏΠΎΠ»Π΅ заставляСт упорядочСнно Π΄Π²ΠΈΠ³Π°Ρ‚ΡŒΡΡ заряТСнныС частицы Π²Π½ΡƒΡ‚Ρ€ΠΈ ΠΏΡ€ΠΎΠ²ΠΎΠ΄Π½ΠΈΠΊΠ°, Ρ‚ΠΎ Π΅ΡΡ‚ΡŒ создаСт элСктричСский Ρ‚ΠΎΠΊ. Π˜Π½Ρ‹ΠΌΠΈ словами, «напряТСниС являСтся ΠΏΡ€ΠΈΡ‡ΠΈΠ½ΠΎΠΉ возникновСния Ρ‚ΠΎΠΊΠ°Β». Π’ Π΄Π°Π½Π½ΠΎΠΌ случаС ситуация обратная βˆ’ благодаря элСктричСскому Ρ‚ΠΎΠΊΡƒ Π½Π° ΠΎΠ±ΠΊΠ»Π°Π΄ΠΊΠ°Ρ… Π²ΠΎΠ·Π½ΠΈΠΊΠ°ΡŽΡ‚ элСктричСскиС заряды, ΡΠΎΠ·Π΄Π°ΡŽΡ‰ΠΈΠ΅ элСктричСскоС ΠΏΠΎΠ»Π΅, поэтому ΠΌΠΎΠΆΠ½ΠΎ ΡΠΊΠ°Π·Π°Ρ‚ΡŒ, Ρ‡Ρ‚ΠΎ Π² Π΄Π°Π½Π½ΠΎΠΌ случаС «сила Ρ‚ΠΎΠΊΠ° являСтся ΠΏΡ€ΠΈΡ‡ΠΈΠ½ΠΎΠΉ возникновСния напряТСния». Π₯отя, ΠΊ Π΄Π°Π½Π½Ρ‹ΠΌ рассуТдСниям слСдуСт ΠΎΡ‚Π½ΠΎΡΠΈΡ‚ΡŒΡΡ нСсколько скСптичСски, Ρ‚Π°ΠΊ Π΄Π²ΠΈΠΆΠ΅Π½ΠΈΠ΅ зарядов (элСктричСский Ρ‚ΠΎΠΊ) ΠΈ элСктричСскоС ΠΏΠΎΠ»Π΅ Β«ΠΏΠΎΠ΄ΡΡ‚Ρ€Π°ΠΈΠ²Π°ΡŽΡ‚ΡΡΒ» Π΄Ρ€ΡƒΠ³ ΠΊ Π΄Ρ€ΡƒΠ³Ρƒ, ΠΏΠΎΠΊΠ° ΠΌΠ΅ΠΆΠ΄Ρƒ Π½ΠΈΠΌΠΈ Π½Π΅ устанавливаСтся ΠΎΠΏΡ€Π΅Π΄Π΅Π»Π΅Π½Π½ΠΎΠ΅ ΡΠΎΠΎΡ‚Π½ΠΎΡˆΠ΅Π½ΠΈΠ΅, ΡΠΎΠΎΡ‚Π²Π΅Ρ‚ΡΡ‚Π²ΡƒΡŽΡ‰Π΅Π΅ ΡƒΡΡ‚Π°Π½ΠΎΠ²ΠΈΠ²ΡˆΠ΅ΠΌΡƒΡΡ Ρ€Π΅ΠΆΠΈΠΌΡƒ. Π’Π°ΠΊ ΠΏΡ€ΠΈ постоянном Ρ‚ΠΎΠΊΠ΅ условиСм стационарности являСтся условиС постоянства Ρ‚ΠΎΠΊΠ°. Π’ Ρ†Π΅ΠΏΠΈ ΠΏΠ΅Ρ€Π΅ΠΌΠ΅Π½Π½ΠΎΠ³ΠΎ Ρ‚ΠΎΠΊΠ° Π² ΡƒΡΡ‚Π°Π½ΠΎΠ²ΠΈΠ²ΡˆΠ΅ΠΌΡΡ Ρ€Π΅ΠΆΠΈΠΌΠ΅ ΡΠΎΠ³Π»Π°ΡΡƒΡŽΡ‚ΡΡ Π½Π΅ Ρ‚ΠΎΠ»ΡŒΠΊΠΎ Π°ΠΌΠΏΠ»ΠΈΡ‚ΡƒΠ΄Π½Ρ‹Π΅ значСния Ρ‚ΠΎΠΊΠΎΠ² ΠΈ напряТСний, Π½ΠΎ Ρ€Π°Π·Π½ΠΎΡΡ‚ΡŒ Ρ„Π°Π· ΠΌΠ΅ΠΆΠ΄Ρƒ Π½ΠΈΠΌΠΈ. Π˜Π½Ρ‹ΠΌΠΈ словами, обсуТдаСмый здСсь ΠΏΡ€ΠΈΡ‡ΠΈΠ½Π½ΠΎ-слСдствСнный вопрос ΠΏΠΎΠ΄ΠΎΠ±Π΅Π½ вопросу ΠΎ Ρ‚ΠΎΠΌ, Β«Ρ‡Ρ‚ΠΎ появилось Ρ€Π°Π½ΡŒΡˆΠ΅, ΠΊΡƒΡ€ΠΈΡ†Π° ΠΈΠ»ΠΈ яйцо?Β»
 Вак ΠΊΠ°ΠΊ ΠΌΠ΅ΠΆΠ΄Ρƒ Ρ‚ΠΎΠΊΠΎΠΌ ΠΈ напряТСниСм сущСствуСт сдвиг Ρ„Π°Π· Ρ€Π°Π²Π½Ρ‹ΠΉ Δφ = Ο€/2
, Ρ‚ΠΎ срСдняя ΠΌΠΎΡ‰Π½ΠΎΡΡ‚ΡŒ Ρ‚ΠΎΠΊΠ° Ρ‡Π΅Ρ€Π΅Π· кондСнсатор Ρ€Π°Π²Π½Π° Π½ΡƒΠ»ΡŽ. Π”Π΅ΠΉΡΡ‚Π²ΠΈΡ‚Π΅Π»ΡŒΠ½ΠΎ,

β€ƒΠ˜Π½Ρ‹ΠΌΠΈ словами, ΠΏΠΎΡ‚Π΅Ρ€ΡŒ энСргии ΠΏΡ€ΠΈ ΠΏΡ€ΠΎΡ‚Π΅ΠΊΠ°Π½ΠΈΠΈ Ρ‚ΠΎΠΊΠ° Ρ‡Π΅Ρ€Π΅Π· кондСнсатор Π² срСднСм Π½Π΅ происходит. ΠšΠΎΠ½Π΅Ρ‡Π½ΠΎ, кондСнсатор влияСт Π½Π° ΠΏΡ€ΠΎΡ‚Π΅ΠΊΠ°Π½ΠΈΠ΅ Ρ‚ΠΎΠΊΠ° Π² Ρ†Π΅ΠΏΠΈ. Π’ Ρ…ΠΎΠ΄Π΅ зарядки кондСнсатора энСргия элСктричСского Ρ‚ΠΎΠΊΠ° прСвращаСтся Π² ΡΠ½Π΅Ρ€Π³ΠΈΡŽ элСктростатичСского поля ΠΌΠ΅ΠΆΠ΄Ρƒ ΠΎΠ±ΠΊΠ»Π°Π΄ΠΊΠ°ΠΌΠΈ кондСнсатора, Π° ΠΏΡ€ΠΈ разрядкС кондСнсатор ΠΎΡ‚Π΄Π°Π΅Ρ‚ Π² Ρ†Π΅ΠΏΡŒ Π½Π°ΠΊΠΎΠΏΠ»Π΅Π½Π½ΡƒΡŽ ΡΠ½Π΅Ρ€Π³ΠΈΡŽ, ΠΏΡ€ΠΈ этом, срСдняя энСргия, потрСбляСмая кондСнсатором, остаСтся Ρ€Π°Π²Π½ΠΎΠΉ Π½ΡƒΠ»ΡŽ. ΠŸΠΎΡΡ‚ΠΎΠΌΡƒ СмкостноС сопротивлСниС Π½Π°Π·Ρ‹Π²Π°ΡŽΡ‚ Ρ€Π΅Π°ΠΊΡ‚ΠΈΠ²Π½Ρ‹ΠΌ.
 Графики зависимости силы Ρ‚ΠΎΠΊΠ°, напряТСния ΠΈ ΠΌΠ³Π½ΠΎΠ²Π΅Π½Π½ΠΎΠΉ мощности Ρ‚ΠΎΠΊΠ° Π² рассматриваСмой Ρ†Π΅ΠΏΠΈ ΠΏΠΎΠΊΠ°Π·Π°Π½Ρ‹ Π½Π° рис. 655.

рис. 655

 Заливкой Π²Ρ‹Π΄Π΅Π»Π΅Π½Ρ‹ ΠΏΡ€ΠΎΠΌΠ΅ΠΆΡƒΡ‚ΠΊΠΈ Π²Ρ€Π΅ΠΌΠ΅Π½ΠΈ, Π² Ρ‚Π΅Ρ‡Π΅Π½ΠΈΠΈ ΠΊΠΎΡ‚ΠΎΡ€Ρ‹Ρ… кондСнсатор Π½Π°ΠΊΠ°ΠΏΠ»ΠΈΠ²Π°Π΅Ρ‚ энСргия βˆ’ Π² этих ΠΏΡ€ΠΎΠΌΠ΅ΠΆΡƒΡ‚ΠΊΠ°Ρ… сила Ρ‚ΠΎΠΊΠ° ΠΈ напряТСниС ΠΈΠΌΠ΅ΡŽΡ‚ ΠΎΠ΄ΠΈΠ½ Π·Π½Π°ΠΊ.
β€ƒΠ£ΠΌΠ΅Π½ΡŒΡˆΠ΅Π½ΠΈΠ΅ Смкостного сопротивлСния ΠΏΡ€ΠΈ возрастании частоты ΠΎΡ‡Π΅Π²ΠΈΠ΄Π½Π° βˆ’ Ρ‡Π΅ΠΌ Π²Ρ‹ΡˆΠ΅ частота Ρ‚ΠΎΠΊΠ°, Ρ‚Π΅ΠΌ мСньший заряд Π½Π° кондСнсаторС успСваСт Π½Π°ΠΊΠΎΠΏΠΈΡ‚ΡŒΡΡ Π½Π° ΠΎΠ±ΠΊΠ»Π°Π΄ΠΊΠ°Ρ… кондСнсатора Π·Π° ΠΏΠΎΠ»ΠΎΠ²ΠΈΠ½Ρƒ ΠΏΠ΅Ρ€ΠΈΠΎΠ΄Π° (ΠΏΠΎΠΊΠ° Ρ‚ΠΎΠΊ ΠΈΠ΄Π΅Ρ‚ Π² ΠΎΠ΄Π½ΠΎΠΌ Π½Π°ΠΏΡ€Π°Π²Π»Π΅Π½ΠΈΠΈ), Ρ‚Π΅ΠΌ мСньшС напряТСниС Π½Π° Π½Π΅ΠΌ, Ρ‚Π΅ΠΌ мСньшС ΠΎΠ½ прСпятствуСт ΠΏΡ€ΠΎΡ…ΠΎΠΆΠ΄Π΅Π½ΠΈΡŽ Ρ‚ΠΎΠΊΠ° Π² Ρ†Π΅ΠΏΠΈ. АналогичныС рассуТдСния справСдливы ΠΈ для объяснСния зависимости этого сопротивлСния ΠΎΡ‚ Смкости кондСнсатора.
 ВСрнСмся ΠΊ Ρ€Π°ΡΡΠΌΠΎΡ‚Ρ€Π΅Π½ΠΈΡŽ Ρ†Π΅ΠΏΠΈ, ΠΏΠΎΠΊΠ°Π·Π°Π½Π½ΠΎΠΉ Π½Π° рис. 653, которая описываСтся ΡƒΡ€Π°Π²Π½Π΅Π½ΠΈΠ΅ΠΌ (1). ΠŸΡ€Π΅Π½Π΅Π±Ρ€Π΅Π³Π°Ρ Π²Π½ΡƒΡ‚Ρ€Π΅Π½Π½ΠΈΠΌ сопротивлСниСм источника, запишСм явноС Π²Ρ‹Ρ€Π°ΠΆΠ΅Π½ΠΈΠ΅ для напряТСния, создаваСмого источником

Π—Π΄Π΅ΡΡŒ U o
βˆ’ Π°ΠΌΠΏΠ»ΠΈΡ‚ΡƒΠ΄Π½ΠΎΠ΅ Π·Π½Π°Ρ‡Π΅Π½ΠΈΠ΅ напряТСния, Ρ€Π°Π²Π½ΠΎΠ΅ Π°ΠΌΠΏΠ»ΠΈΡ‚ΡƒΠ΄Π½ΠΎΠΌΡƒ Π·Π½Π°Ρ‡Π΅Π½ΠΈΡŽ Π­Π”Π‘ источника. ΠšΡ€ΠΎΠΌΠ΅ Ρ‚ΠΎΠ³ΠΎ, Ρ‚Π΅ΠΏΠ΅Ρ€ΡŒ ΠΌΡ‹ считаСм Π½Π°Ρ‡Π°Π»ΡŒΠ½ΡƒΡŽ Ρ„Π°Π·Ρƒ Π­Π”Π‘ источника Ρ€Π°Π²Π½ΠΎΠΉ Π½ΡƒΠ»ΡŽ (Ρ€Π°Π½Π΅Π΅ Π·Π° Π½ΡƒΠ»ΡŒ ΠΌΡ‹ ΠΏΡ€ΠΈΠ½ΠΈΠΌΠ°Π»ΠΈ Ρ„Π°Π·Ρƒ ΠΊΠΎΠ»Π΅Π±Π°Π½ΠΈΠΉ напряТСния Π½Π° рСзисторС).
β€ƒΠ˜ΡΠΏΠΎΠ»ΡŒΠ·ΡƒΡ это ΡƒΡ€Π°Π²Π½Π΅Π½ΠΈΠ΅ ΠΈ связь ΠΌΠ΅ΠΆΠ΄Ρƒ силой Ρ‚ΠΎΠΊΠ° ΠΈ зарядом кондСнсатора, Π½Π°ΠΉΠ΄Π΅ΠΌ явноС Π²Ρ‹Ρ€Π°ΠΆΠ΅Π½ΠΈΠ΅ для зависимости силы Ρ‚ΠΎΠΊΠ° Π² Ρ†Π΅ΠΏΠΈ ΠΎΡ‚ Π²Ρ€Π΅ΠΌΠ΅Π½ΠΈ. ΠŸΡ€Π΅Π΄ΡΡ‚Π°Π²ΠΈΠΌ эту Π·Π°Π²ΠΈΡΠΈΠΌΠΎΡΡ‚ΡŒ Π² Π²ΠΈΠ΄Π΅

Π³Π΄Π΅ I o
ΠΈ Ο†
βˆ’ ΠΏΠΎΠ΄Π»Π΅ΠΆΠ°Ρ‰ΠΈΠ΅ ΠΎΠΏΡ€Π΅Π΄Π΅Π»Π΅Π½ΠΈΡŽ Π°ΠΌΠΏΠ»ΠΈΡ‚ΡƒΠ΄Π½ΠΎΠ΅ Π·Π½Π°Ρ‡Π΅Π½ΠΈΠ΅ силы Ρ‚ΠΎΠΊΠ° ΠΈ разности Ρ„Π°Π· ΠΌΠ΅ΠΆΠ΄Ρƒ колСбаниями Ρ‚ΠΎΠΊΠ° ΠΈ напряТСния источника. Π›Π΅Π³ΠΊΠΎ Π·Π°ΠΌΠ΅Ρ‚ΠΈΡ‚ΡŒ, Ρ‡Ρ‚ΠΎ Π² этом случаС заряд кондСнсатора измСняСтся ΠΏΠΎ Π·Π°ΠΊΠΎΠ½Ρƒ

 Для ΠΏΡ€ΠΎΠ²Π΅Ρ€ΠΊΠΈ этого ΡΠΎΠΎΡ‚Π½ΠΎΡˆΠ΅Π½ΠΈΡ достаточно Π²Ρ‹Ρ‡ΠΈΡΠ»ΠΈΡ‚ΡŒ ΠΏΡ€ΠΎΠΈΠ·Π²ΠΎΠ΄Π½ΡƒΡŽ ΠΎΡ‚ ΠΏΡ€ΠΈΠ²Π΅Π΄Π΅Π½Π½ΠΎΠΉ Ρ„ΡƒΠ½ΠΊΡ†ΠΈΠΈ ΠΈ убСдится, Ρ‡Ρ‚ΠΎ ΠΎΠ½Π° совпадаСт с Ρ„ΡƒΠ½ΠΊΡ†ΠΈΠ΅ΠΉ (9).
β€ƒΠŸΠΎΠ΄ΡΡ‚Π°Π²ΠΈΠΌ эти выраТСния Π² ΡƒΡ€Π°Π²Π½Π΅Π½ΠΈΠ΅ (8)

ΠΈ ΠΏΡ€Π΅ΠΎΠ±Ρ€Π°Π·ΡƒΠ΅ΠΌ Ρ‚Ρ€ΠΈΠ³ΠΎΠ½ΠΎΠΌΠ΅Ρ‚Ρ€ΠΈΡ‡Π΅ΡΠΊΡƒΡŽ сумму

Π³Π΄Π΅ Ρ‡Π΅Ρ€Π΅Π· Ο† 1
ΠΎΠ±ΠΎΠ·Π½Π°Ρ‡Π΅Π½Π° Π²Π΅Π»ΠΈΡ‡ΠΈΠ½Π°, ΡƒΠ΄ΠΎΠ²Π»Π΅Ρ‚Π²ΠΎΡ€ΡΡŽΡ‰Π°Ρ ΡƒΡΠ»ΠΎΠ²ΠΈΡŽ

β€ƒΠ’Π΅ΠΏΠ΅Ρ€ΡŒ Π²ΠΈΠ΄Π½ΠΎ, Ρ‡Ρ‚ΠΎ для Ρ‚ΠΎΠ³ΠΎ, Ρ‡Ρ‚ΠΎΠ±Ρ‹ функция (9) являлась Ρ€Π΅ΡˆΠ΅Π½ΠΈΠ΅ уравнСния (8), Π½Π΅ΠΎΠ±Ρ…ΠΎΠ΄ΠΈΠΌΠΎ, Ρ‡Ρ‚ΠΎΠ±Ρ‹ Π΅Π΅ ΠΏΠ°Ρ€Π°ΠΌΠ΅Ρ‚Ρ€Ρ‹ ΠΏΡ€ΠΈΠ½ΠΈΠΌΠ°Π»ΠΈ значСния:
 Амплитуда

искомая Ρ€Π°Π·Π½ΠΎΡΡ‚ΡŒ Ρ„Π°Π· связана с появившимся ΠΏΠ°Ρ€Π°ΠΌΠ΅Ρ‚Ρ€ΠΎΠΌ Ο† 1
ΡΠΎΠΎΡ‚Π½ΠΎΡˆΠ΅Π½ΠΈΠ΅ΠΌ Ο† + Ο† 1 = 0
, Ρ‚ΠΎ Π΅ΡΡ‚ΡŒ

 Ваким ΠΎΠ±Ρ€Π°Π·ΠΎΠΌ, Π½Π°ΠΉΠ΄Π΅Π½Π° явная Π·Π°Π²ΠΈΡΠΈΠΌΠΎΡΡ‚ΡŒ силы Ρ‚ΠΎΠΊΠ° ΠΎΡ‚ Π²Ρ€Π΅ΠΌΠ΅Π½ΠΈ.
 В ΠΏΡ€ΠΈΠ½Ρ†ΠΈΠΏΠ΅ Ρ‚Π°ΠΊΠΈΠΌ ΠΌΠ΅Ρ‚ΠΎΠ΄ΠΎΠΌ, ΠΌΠΎΠΆΠ½ΠΎ Ρ€Π°ΡΡΡ‡ΠΈΡ‚Π°Ρ‚ΡŒ Π»ΡŽΠ±ΡƒΡŽ Ρ†Π΅ΠΏΡŒ ΠΏΠ΅Ρ€Π΅ΠΌΠ΅Π½Π½ΠΎΠ³ΠΎ Ρ‚ΠΎΠΊΠ°. Но Ρ‚Π°ΠΊΠΎΠΉ ΠΏΠΎΠ΄Ρ…ΠΎΠ΄ Ρ‚Ρ€Π΅Π±ΡƒΠ΅Ρ‚ Π³Ρ€ΠΎΠΌΠΎΠ·Π΄ΠΊΠΈΡ… тригономСтричСских ΠΈ алгСбраичСских ΠΏΡ€Π΅ΠΎΠ±Ρ€Π°Π·ΠΎΠ²Π°Π½ΠΈΠΉ. К Ρ‚Π΅ΠΌ ΠΆΠ΅ Ρ€Π΅Π·ΡƒΠ»ΡŒΡ‚Π°Ρ‚Π°ΠΌ ΠΌΠΎΠΆΠ½ΠΎ ΠΏΡ€ΠΈΠΉΡ‚ΠΈ Π³ΠΎΡ€Π°Π·Π΄ΠΎ ΠΏΡ€ΠΎΡ‰Π΅, ΠΈΡΠΏΠΎΠ»ΡŒΠ·ΡƒΡ Ρ„ΠΎΡ€ΠΌΠ°Π»ΠΈΠ·ΠΌ Π²Π΅ΠΊΡ‚ΠΎΡ€Π½Ρ‹Ρ… Π΄ΠΈΠ°Π³Ρ€Π°ΠΌΠΌ. ПокаТСм, ΠΊΠ°ΠΊ ΠΌΠ΅Ρ‚ΠΎΠ΄ Π²Π΅ΠΊΡ‚ΠΎΡ€Π½Ρ‹Ρ… Π΄ΠΈΠ°Π³Ρ€Π°ΠΌΠΌ примСняСтся ΠΊ рассматриваСмой Ρ†Π΅ΠΏΠΈ. Π‘Π°ΠΌΠΎΠ΅ Π²Π°ΠΆΠ½ΠΎΠ΅ ΠΏΡ€ΠΈ использовании этого ΠΌΠ΅Ρ‚ΠΎΠ΄Π° βˆ’ построСниС Π²Π΅ΠΊΡ‚ΠΎΡ€Π½ΠΎΠΉ Π΄ΠΈΠ°Π³Ρ€Π°ΠΌΠΌΡ‹, ΠΈΠ·ΠΎΠ±Ρ€Π°ΠΆΠ°ΡŽΡ‰Π΅ΠΉ колСбания Ρ‚ΠΎΠΊΠΎΠ² ΠΈ напряТСний Π½Π° Ρ€Π°Π·Π»ΠΈΡ‡Π½Ρ‹Ρ… участках Ρ†Π΅ΠΏΠΈ.
 Вак ΠΊΠ°ΠΊ кондСнсатор ΠΈ рСзистор соСдинСны ΠΏΠΎΡΠ»Π΅Π΄ΠΎΠ²Π°Ρ‚Π΅Π»ΡŒΠ½ΠΎ, Ρ‚ΠΎ силы Ρ‚ΠΎΠΊΠΎΠ² Ρ‡Π΅Ρ€Π΅Π· Π½ΠΈΡ… ΠΎΠ΄ΠΈΠ½Π°ΠΊΠΎΠ²Ρ‹ Π² любой ΠΌΠΎΠΌΠ΅Π½Ρ‚ Π²Ρ€Π΅ΠΌΠ΅Π½ΠΈ. Π˜Π·ΠΎΠ±Ρ€Π°Π·ΠΈΠΌ силу Ρ‚ΠΎΠΊΠ° Π² Π²ΠΈΠ΄Π΅ ΠΏΡ€ΠΎΠΈΠ·Π²ΠΎΠ»ΡŒΠ½ΠΎ Π½Π°ΠΏΡ€Π°Π²Π»Π΅Π½Π½ΠΎΠ³ΠΎ Π²Π΅ΠΊΡ‚ΠΎΡ€Π° (Π½Π°ΠΏΡ€ΠΈΠΌΠ΅Ρ€, Π³ΠΎΡ€ΠΈΠ·ΠΎΠ½Ρ‚Π°Π»ΡŒΠ½ΠΎ 2 , ΠΊΠ°ΠΊ Π½Π° рис. 656).

рис. 656

 ДалСС ΠΈΠ·ΠΎΠ±Ρ€Π°Π·ΠΈΠΌ Π²Π΅ΠΊΡ‚ΠΎΡ€Ρ‹ ΠΊΠΎΠ»Π΅Π±Π°Π½ΠΈΠΉ напряТСния Π½Π° рСзисторС U R
, ΠΊΠΎΡ‚ΠΎΡ€Ρ‹ΠΉ ΠΏΠ°Ρ€Π°Π»Π»Π΅Π»Π΅Π½ Π²Π΅ΠΊΡ‚ΠΎΡ€Ρƒ ΠΊΠΎΠ»Π΅Π±Π°Π½ΠΈΠΉ Ρ‚ΠΎΠΊΠ° (Ρ‚Π°ΠΊ ΠΊΠ°ΠΊ сдвиг Ρ„Π°Π· ΠΌΠ΅ΠΆΠ΄Ρƒ этими колСбаниями Ρ€Π°Π²Π΅Π½ Π½ΡƒΠ»ΡŽ) ΠΈ напряТСния Π½Π° кондСнсаторС U C
, ΠΊΠΎΡ‚ΠΎΡ€Ρ‹ΠΉ пСрпСндикулярСн Π²Π΅ΠΊΡ‚ΠΎΡ€Ρƒ ΠΊΠΎΠ»Π΅Π±Π°Π½ΠΈΠΉ Ρ‚ΠΎΠΊΠ° (Ρ‚Π°ΠΊ ΠΊΠ°ΠΊ сдвиг Ρ„Π°Π· ΠΌΠ΅Π΄Ρƒ Π½ΠΈΠΌΠΈ Ρ€Π°Π²Π΅Π½ Ο€/2
βˆ’ см. рис. 657).

рис. 657

 Бумма этих напряТСний Ρ€Π°Π²Π½Π° Π½Π°ΠΏΡ€ΡΠΆΠ΅Π½ΠΈΡŽ источника, поэтому Π²Π΅ΠΊΡ‚ΠΎΡ€ суммы Π²Π΅ΠΊΡ‚ΠΎΡ€ΠΎΠ², ΠΈΠ·ΠΎΠ±Ρ€Π°ΠΆΠ°ΡŽΡ‰ΠΈΡ… колСбания U R
ΠΈ U C
, ΠΈΠ·ΠΎΠ±Ρ€Π°ΠΆΠ°Π΅Ρ‚ колСбания напряТСния источника U(t)
.
 Если ΠΆΠ΅ Π’Ρ‹ настаиваСтС, Ρ‡Ρ‚ΠΎ Ρ„Π°Π·Π° суммарного напряТСния Ρ€Π°Π²Π½Π° Π½ΡƒΠ»ΡŽ (Ρ‚ΠΎ Π΅ΡΡ‚ΡŒ Π²Π΅ΠΊΡ‚ΠΎΡ€, ΠΈΠ·ΠΎΠ±Ρ€Π°ΠΆΠ°ΡŽΡ‰ΠΈΠΉ U
Π΄ΠΎΠ»ΠΆΠ΅Π½ Π±Ρ‹Ρ‚ΡŒ располоТСн Π³ΠΎΡ€ΠΈΠ·ΠΎΠ½Ρ‚Π°Π»ΡŒΠ½ΠΎ), Ρ‚ΠΎ ΠΏΠΎΠ²Π΅Ρ€Π½ΠΈΡ‚Π΅ ΠΏΠΎΡΡ‚Ρ€ΠΎΠ΅Π½Π½ΡƒΡŽ Π΄ΠΈΠ°Π³Ρ€Π°ΠΌΠΌΡƒ (рис. 657). Π’Π°ΠΊΠΈΠΌ Π΄ΠΎΠ³ΠΌΠ°Ρ‚ΠΈΠ·ΠΌΠΎΠΌ Π΄Π°Π»Π΅Π΅ ΠΌΡ‹ Π·Π°Π½ΠΈΠΌΠ°Ρ‚ΡŒΡΡ Π½Π΅ Π±ΡƒΠ΄Π΅ΠΌ!
β€ƒΠ˜Π· построСнной Π΄ΠΈΠ°Π³Ρ€Π°ΠΌΠΌΡ‹ слСдуСт, Ρ‡Ρ‚ΠΎ Π°ΠΌΠΏΠ»ΠΈΡ‚ΡƒΠ΄Π½Ρ‹Π΅ значСния рассматриваСмых напряТСний связаны ΡΠΎΠΎΡ‚Π½ΠΎΡˆΠ΅Π½ΠΈΠ΅ΠΌ (ΡΠ»Π΅Π΄ΡƒΡŽΡ‰ΠΈΠΌ ΠΈΠ· Ρ‚Π΅ΠΎΡ€Π΅ΠΌΡ‹ ΠŸΠΈΡ„Π°Π³ΠΎΡ€Π°)

 ВыраТая Π°ΠΌΠΏΠ»ΠΈΡ‚ΡƒΠ΄Ρ‹ напряТСний Ρ‡Π΅Ρ€Π΅Π· Π°ΠΌΠΏΠ»ΠΈΡ‚ΡƒΠ΄Ρƒ силы Ρ‚ΠΎΠΊΠ° с ΠΏΠΎΠΌΠΎΡ‰ΡŒΡŽ извСстных ΡΠΎΠΎΡ‚Π½ΠΎΡˆΠ΅Π½ΠΈΠΉ

ΠΈ

ΠΏΠΎΠ»ΡƒΡ‡Π°Π΅ΠΌ элСмСнтарноС ΡƒΡ€Π°Π²Π½Π΅Π½ΠΈΠ΅ для опрСдСлСния Π°ΠΌΠΏΠ»ΠΈΡ‚ΡƒΠ΄Ρ‹ силы Ρ‚ΠΎΠΊΠ°

ΠΈΠ· ΠΊΠΎΡ‚ΠΎΡ€ΠΎΠ³ΠΎ Π½Π°Ρ…ΠΎΠ΄ΠΈΠΌ Π°ΠΌΠΏΠ»ΠΈΡ‚ΡƒΠ΄Ρƒ силы Ρ‚ΠΎΠΊΠ° Π² Ρ†Π΅ΠΏΠΈ

Ρ‡Ρ‚ΠΎ, СстСствСнно, совпадаСт с Π²Ρ‹Ρ€Π°ΠΆΠ΅Π½ΠΈΠ΅ΠΌ (11), ΠΏΠΎΠ»ΡƒΡ‡Π΅Π½Π½Ρ‹ΠΌ Ρ€Π°Π½Π΅Π΅ Π³Ρ€ΠΎΠΌΠΎΠ·Π΄ΠΊΠΈΠΌ алгСбраичСским ΠΌΠ΅Ρ‚ΠΎΠ΄ΠΎΠΌ. ВСкторная Π΄ΠΈΠ°Π³Ρ€Π°ΠΌΠΌΠ° Ρ‚Π°ΠΊΠΆΠ΅ позволяСт Π»Π΅Π³ΠΊΠΎ ΠΎΠΏΡ€Π΅Π΄Π΅Π»ΠΈΡ‚ΡŒ сдвиг Ρ„Π°Π· ΠΌΠ΅ΠΆΠ΄Ρƒ колСбаниями силы Ρ‚ΠΎΠΊΠ° ΠΈ напряТСния источника

Ρ‡Ρ‚ΠΎ Ρ‚Π°ΠΊΠΆΠ΅ совпадаСт с ΠΏΠΎΠ»ΡƒΡ‡Π΅Π½Π½Ρ‹ΠΌ Ρ€Π°Π½Π΅Π΅.
β€ƒΠšΠ°ΠΊ Π²ΠΈΠ΄Π½ΠΎ, ΠΌΠ΅Ρ‚ΠΎΠ΄ Π²Π΅ΠΊΡ‚ΠΎΡ€Π½Ρ‹Ρ… Π΄ΠΈΠ°Π³Ρ€Π°ΠΌΠΌ позволяСт ΠΏΠΎΠ»Π½ΠΎΡΡ‚ΡŒΡŽ Ρ€Π°ΡΡΡ‡ΠΈΡ‚Π°Ρ‚ΡŒ характСристики Ρ†Π΅ΠΏΠ΅ΠΉ ΠΏΠ΅Ρ€Π΅ΠΌΠ΅Π½Π½ΠΎΠ³ΠΎ Ρ‚ΠΎΠΊΠ°, Π³ΠΎΡ€Π°Π·Π΄ΠΎ ΠΏΡ€ΠΎΡ‰Π΅, Ρ‡Π΅ΠΌ рассмотрСнным Π²Ρ‹ΡˆΠ΅ ΠΌΠ΅Ρ‚ΠΎΠ΄ΠΎΠΌ аналитичСского Ρ€Π΅ΡˆΠ΅Π½ΠΈΡ ΡΠΎΠΎΡ‚Π²Π΅Ρ‚ΡΡ‚Π²ΡƒΡŽΡ‰Π΅Π³ΠΎ уравнСния.
 БлСдуСт ΠΏΠΎΠ΄Ρ‡Π΅Ρ€ΠΊΠ½ΡƒΡ‚ΡŒ, Ρ‡Ρ‚ΠΎ физичСская ΡΡƒΡ‰Π½ΠΎΡΡ‚ΡŒ ΠΎΠ±ΠΎΠΈΡ… ΠΌΠ΅Ρ‚ΠΎΠ΄ΠΎΠ² ΠΎΠ΄Π½Π° ΠΈ Ρ‚Π° ΠΆΠ΅, ΠΎΠ½Π° выраТаСтся ΡƒΡ€Π°Π²Π½Π΅Π½ΠΈΠ΅ΠΌ (10), Ρ€Π°Π·Π»ΠΈΡ‡ΠΈΠ΅ Ρ‚ΠΎΠ»ΡŒΠΊΠΎ Π² матСматичСском языкС, Π½Π° ΠΊΠΎΡ‚ΠΎΡ€ΠΎΠΌ Ρ€Π΅ΡˆΠ°Π΅Ρ‚ΡΡ это ΡƒΡ€Π°Π²Π½Π΅Π½ΠΈΠ΅.
 РассчитаСм, ΡΡ€Π΅Π΄Π½ΡŽΡŽ ΠΌΠΎΡ‰Π½ΠΎΡΡ‚ΡŒ, Ρ€Π°Π·Π²ΠΈΠ²Π°Π΅ΠΌΡƒΡŽ источником. МгновСнноС Π·Π½Π°Ρ‡Π΅Π½ΠΈΠ΅ этой мощности Ρ€Π°Π²Π½ΠΎ ΠΏΡ€ΠΎΠΈΠ·Π²Π΅Π΄Π΅Π½ΠΈΡŽ Π­Π”Π‘ Π½Π° силу Ρ‚ΠΎΠΊΠ° P = EI
. ΠŸΠΎΠ΄ΡΡ‚Π°Π²Π»ΡΡ явныС значСния для этих Π²Π΅Π»ΠΈΡ‡ΠΈΠ½ ΠΈ проводя усрСднСниС, ΠΏΠΎΠ»ΡƒΡ‡ΠΈΠΌ

β€ƒΠžΠ±Ρ€Π°Ρ‚ΠΈΡ‚Π΅ Π²Π½ΠΈΠΌΠ°Π½ΠΈΠ΅, Ρ‡Ρ‚ΠΎ ΠΏΠΎΠ»ΡƒΡ‡Π΅Π½Π½ΠΎΠ΅ Π²Ρ‹Ρ€Π°ΠΆΠ΅Π½ΠΈΠ΅ для срСднСй мощности являСтся ΠΎΠ±Ρ‰ΠΈΠΌ для ΠΏΠ΅Ρ€Π΅ΠΌΠ΅Π½Π½ΠΎΠ³ΠΎ Ρ‚ΠΎΠΊΠ°: срСдняя ΠΌΠΎΡ‰Π½ΠΎΡΡ‚ΡŒ ΠΏΠ΅Ρ€Π΅ΠΌΠ΅Π½Π½ΠΎΠ³ΠΎ Ρ‚ΠΎΠΊΠ° Ρ€Π°Π²Π½Π° ΠΏΠΎΠ»ΠΎΠ²ΠΈΠ½Π΅ произвСдСния Π°ΠΌΠΏΠ»ΠΈΡ‚ΡƒΠ΄ силы Ρ‚ΠΎΠΊΠ°, напряТСния ΠΈ косинуса разности Ρ„Π°Π· ΠΌΠ΅ΠΆΠ΄Ρƒ Π½ΠΈΠΌΠΈ. Если ΠΈΡΠΏΠΎΠ»ΡŒΠ·ΠΎΠ²Π°Ρ‚ΡŒ Π½Π΅ Π°ΠΌΠΏΠ»ΠΈΡ‚ΡƒΠ΄Π½Ρ‹Π΅, Π° Π΄Π΅ΠΉΡΡ‚Π²ΡƒΡŽΡ‰ΠΈΠ΅ значСния силы Ρ‚ΠΎΠΊΠ° ΠΈ напряТСния, Ρ‚ΠΎ Ρ„ΠΎΡ€ΠΌΡƒΠ»Π° (16) ΠΏΡ€ΠΈΠΎΠ±Ρ€Π΅Ρ‚Π°Π΅Ρ‚ Π²ΠΈΠ΄

срСдняя ΠΌΠΎΡ‰Π½ΠΎΡΡ‚ΡŒ ΠΏΠ΅Ρ€Π΅ΠΌΠ΅Π½Π½ΠΎΠ³ΠΎ элСктричСского Ρ‚ΠΎΠΊΠ° Ρ€Π°Π²Π½Π° ΠΏΡ€ΠΎΠΈΠ·Π²Π΅Π΄Π΅Π½ΠΈΡŽ Π΄Π΅ΠΉΡΡ‚Π²ΡƒΡŽΡ‰ΠΈΡ… Π·Π½Π°Ρ‡Π΅Π½ΠΈΠΉ силы Ρ‚ΠΎΠΊΠ°, напряТСния ΠΈ косинуса разности Ρ„Π°Π· ΠΌΠ΅ΠΆΠ΄Ρƒ Π½ΠΈΠΌΠΈ
. Часто косинус сдвига Ρ„Π°Π· ΠΌΠ΅ΠΆΠ΄Ρƒ силой Ρ‚ΠΎΠΊΠ° ΠΈ напряТСниСм Π½Π°Π·Ρ‹Π²Π°ΡŽΡ‚ коэффициСнтом мощности
.
 В Ρ‚Π΅Ρ… случаях, ΠΊΠΎΠ³Π΄Π° ΠΏΠΎ элСктричСской Π»ΠΈΠ½ΠΈΠΈ трСбуСтся ΠΏΠ΅Ρ€Π΅Π΄Π°Ρ‚ΡŒ ΠΌΠ°ΠΊΡΠΈΠΌΠ°Π»ΡŒΠ½ΡƒΡŽ ΠΌΠΎΡ‰Π½ΠΎΡΡ‚ΡŒ, Π½Π΅ΠΎΠ±Ρ…ΠΎΠ΄ΠΈΠΌΠΎ ΡΡ‚Ρ€Π΅ΠΌΠΈΡ‚ΡŒΡΡ, Ρ‡Ρ‚ΠΎΠ±Ρ‹ сдвиг Ρ„Π°Π· ΠΌΠ΅ΠΆΠ΄Ρƒ Ρ‚ΠΎΠΊΠΎΠΌ ΠΈ напряТСниСм Π±Ρ‹Π» ΠΌΠΈΠ½ΠΈΠΌΠ°Π»ΡŒΠ½Ρ‹ΠΌ (ΠΎΠΏΡ‚ΠΈΠΌΠ°Π»ΡŒΠ½ΠΎ βˆ’ Π½ΡƒΠ»Π΅Π²Ρ‹ΠΌ), Ρ‚Π°ΠΊ ΠΊΠ°ΠΊ Π² этом случаС пСрСдаваСмая ΠΌΠΎΡ‰Π½ΠΎΡΡ‚ΡŒ Π±ΡƒΠ΄Π΅Ρ‚ максимальна.
β€ƒΠŸΡ€ΠΈΠΌΠ΅Π½ΠΈΠΌ ΠΏΠΎΠ»ΡƒΡ‡Π΅Π½Π½ΡƒΡŽ Ρ„ΠΎΡ€ΠΌΡƒΠ»Ρƒ для расчСта мощности Ρ‚ΠΎΠΊΠ° Π² рассматриваСмой Ρ†Π΅ΠΏΠΈ, для Ρ‡Π΅Π³ΠΎ Π²Ρ‹Ρ€Π°Π·ΠΈΠΌ косинус сдвига Ρ„Π°Π· ΠΈΠ· выраТСния (12) ΠΈ подставим Π² Ρ„ΠΎΡ€ΠΌΡƒΠ»Ρƒ (17), Π² Ρ€Π΅Π·ΡƒΠ»ΡŒΡ‚Π°Ρ‚Π΅ Ρ‡Π΅Π³ΠΎ ΠΏΠΎΠ»ΡƒΡ‡ΠΈΠΌ

β€ƒΠŸΡ€ΠΈ Π²Ρ‹Π²ΠΎΠ΄Π΅ этого ΡΠΎΠΎΡ‚Π½ΠΎΡˆΠ΅Π½ΠΈΡ использована Ρ„ΠΎΡ€ΠΌΡƒΠ»Π° (14) для Π°ΠΌΠΏΠ»ΠΈΡ‚ΡƒΠ΄Ρ‹ силы Ρ‚ΠΎΠΊΠ° Π² Ρ†Π΅ΠΏΠΈ. β€ƒΠŸΠΎΠ»ΡƒΡ‡Π΅Π½Π½Ρ‹ΠΉ Ρ€Π΅Π·ΡƒΠ»ΡŒΡ‚Π°Ρ‚ ΠΎΡ‡Π΅Π²ΠΈΠ΄Π΅Π½ βˆ’ срСдняя ΠΌΠΎΡ‰Π½ΠΎΡΡ‚ΡŒ, развиваСмая источником, Ρ€Π°Π²Π½Π° срСднСй мощности Ρ‚Π΅ΠΏΠ»ΠΎΡ‚Ρ‹, Π²Ρ‹Π΄Π΅Π»ΡΡŽΡ‰Π΅ΠΉΡΡ Π½Π° рСзисторС. Π­Ρ‚ΠΎΡ‚ Π²Ρ‹Π²ΠΎΠ΄ Π΅Ρ‰Π΅ Ρ€Π°Π· ΠΏΠΎΠ΄Ρ‚Π²Π΅Ρ€ΠΆΠ΄Π°Π΅Ρ‚, Ρ‡Ρ‚ΠΎ Π½Π° кондСнсаторС Π½Π΅ происходит ΠΏΠΎΡ‚Π΅Ρ€ΡŒ энСргии элСктричСского Ρ‚ΠΎΠΊΠ°.
 РасчСт мощности Ρ‚ΠΎΠΊΠ° Ρ‚Π°ΠΊΠΆΠ΅ ΠΌΠΎΠΆΠ½ΠΎ ΠΏΡ€ΠΎΠ²ΠΎΠ΄ΠΈΡ‚ΡŒ с ΠΏΠΎΠΌΠΎΡ‰ΡŒΡŽ построСнной Π²Π΅ΠΊΡ‚ΠΎΡ€Π½ΠΎΠΉ Π΄ΠΈΠ°Π³Ρ€Π°ΠΌΠΌΡ‹, ΠΈΠ· ΠΊΠΎΡ‚ΠΎΡ€ΠΎΠΉ слСдуСт, Ρ‡Ρ‚ΠΎ ΠΏΡ€ΠΎΠΈΠ·Π²Π΅Π΄Π΅Π½ΠΈΠ΅ Π°ΠΌΠΏΠ»ΠΈΡ‚ΡƒΠ΄Ρ‹ напряТСния источника Π½Π° косинус сдвига Ρ„Π°Π· Ρ€Π°Π²Π½ΠΎ Π°ΠΌΠΏΠ»ΠΈΡ‚ΡƒΠ΄Π΅ напряТСния Π½Π° рСзисторС

ΠΎΡ‚ΠΊΡƒΠ΄Π° сразу слСдуСт Ρ„ΠΎΡ€ΠΌΡƒΠ»Π° (18).
 Вак ΠΊΠ°ΠΊ Π°ΠΌΠΏΠ»ΠΈΡ‚ΡƒΠ΄Π½Ρ‹Π΅ ΠΈ Π΄Π΅ΠΉΡΡ‚Π²ΡƒΡŽΡ‰ΠΈΠ΅ значСния сил Ρ‚ΠΎΠΊΠΎΠ² ΠΈ напряТСний ΠΏΡ€ΠΎΠΏΠΎΡ€Ρ†ΠΈΠΎΠ½Π°Π»ΡŒΠ½Ρ‹ Π΄Ρ€ΡƒΠ³ Π΄Ρ€ΡƒΠ³Ρƒ, Ρ‚ΠΎ Π΄Π»ΠΈΠ½Ρ‹ Π²Π΅ΠΊΡ‚ΠΎΡ€ΠΎΠ² Π²Π΅ΠΊΡ‚ΠΎΡ€Π½Ρ‹Ρ… Π΄ΠΈΠ°Π³Ρ€Π°ΠΌΠΌ ΠΌΠΎΠΆΠ½ΠΎ ΡΡ‡ΠΈΡ‚Π°Ρ‚ΡŒ ΠΏΡ€ΠΎΠΏΠΎΡ€Ρ†ΠΈΠΎΠ½Π°Π»ΡŒΠ½Ρ‹ΠΌΠΈ Π΄Π΅ΠΉΡΡ‚Π²ΡƒΡŽΡ‰ΠΈΠΌ (Π° Π½Π΅ Π°ΠΌΠΏΠ»ΠΈΡ‚ΡƒΠ΄Π½Ρ‹ΠΌ) значСниям. ΠŸΡ€ΠΈ Ρ‚Π°ΠΊΠΎΠΌ ΠΎΠΏΡ€Π΅Π΄Π΅Π»Π΅Π½ΠΈΠΈ срСднСС ΠΏΡ€ΠΎΠΈΠ·Π²Π΅Π΄Π΅Π½ΠΈΠ΅ Π΄Π²ΡƒΡ… гармоничСских Ρ„ΡƒΠ½ΠΊΡ†ΠΈΠΉ Ρ€Π°Π²Π½ΠΎ скалярному ΠΏΡ€ΠΎΠΈΠ·Π²Π΅Π΄Π΅Π½ΠΈΡŽ Π²Π΅ΠΊΡ‚ΠΎΡ€ΠΎΠ², ΠΈΠ·ΠΎΠ±Ρ€Π°ΠΆΠ°ΡŽΡ‰ΠΈΡ… эти Ρ„ΡƒΠ½ΠΊΡ†ΠΈΠΈ.

1 Π—Π΄Π΅ΡΡŒ ΠΌΡ‹ ΠΈΡΠΏΠΎΠ»ΡŒΠ·ΡƒΠ΅ΠΌ ΠΌΠ°Ρ‚Π΅ΠΌΠ°Ρ‚ΠΈΡ‡Π΅ΡΠΊΡƒΡŽ ΠΎΠΏΠ΅Ρ€Π°Ρ†ΠΈΡŽ вычислСния ΠΏΡ€ΠΎΠΈΠ·Π²ΠΎΠ΄Π½ΠΎΠΉ Ρ„ΡƒΠ½ΠΊΡ†ΠΈΠΈ. Если ΠΆΠ΅ вас ΠΎΠ½Π° Π΅Ρ‰Π΅ ΠΏΡƒΠ³Π°Π΅Ρ‚ βˆ’ Π²ΠΎΡΠΏΠΎΠ»ΡŒΠ·ΡƒΠΉΡ‚Π΅ΡΡŒ Π°Π½Π°Π»ΠΎΠ³ΠΈΠ΅ΠΉ с мСханичСскими гармоничСскими колСбаниями: Π°Π½Π°Π»ΠΎΠ³ΠΎΠΌ заряда являСтся ΠΊΠΎΠΎΡ€Π΄ΠΈΠ½Π°Ρ‚Π°, Ρ‚ΠΎΠ³Π΄Π° Π°Π½Π°Π»ΠΎΠ³ΠΎΠΌ силы Ρ‚ΠΎΠΊΠ° слуТит мгновСнная ΡΠΊΠΎΡ€ΠΎΡΡ‚ΡŒ.
2 ΠœΡ‹ постоянно ΠΏΠΎΠ΄Ρ‡Π΅Ρ€ΠΊΠΈΠ²Π°Π΅ΠΌ, Ρ‡Ρ‚ΠΎ Π½Π°Ρ‡Π°Π»ΡŒΠ½Π°Ρ Ρ„Π°Π·Π° ΠΎΡ‚Π΄Π΅Π»ΡŒΠ½ΠΎΠ³ΠΎ колСбания, Π½ΠΈ Π² ΠΊΠ°ΠΊΠΈΡ… процСссах Π½Π΅ сущСствСнна, ΠΎΠ½Π° ΠΌΠΎΠΆΠ΅Ρ‚ Π±Ρ‹Ρ‚ΡŒ ΠΈΠ·ΠΌΠ΅Π½Π΅Π½Π° простым пСрСносом Π½Π°Ρ‡Π°Π»Π° отсчСта Π²Ρ€Π΅ΠΌΠ΅Π½ΠΈ. ЀизичСский смысл ΠΈΠΌΠ΅ΡŽΡ‚ разности Ρ„Π°Π· ΠΌΠ΅ΠΆΠ΄Ρƒ Ρ€Π°Π·Π»ΠΈΡ‡Π½Ρ‹ΠΌΠΈ Π²Π΅Π»ΠΈΡ‡ΠΈΠ½Π°ΠΌΠΈ, ΠΈΠ·ΠΌΠ΅Π½ΡΡŽΡ‰ΠΈΠΌΠΈΡΡ ΠΏΠΎ гармоничСским Π·Π°ΠΊΠΎΠ½Π°ΠΌ. Π—Π΄Π΅ΡΡŒ ΠΌΡ‹ ΠΊΠ°ΠΊ Π±Ρ‹, ΠΎΡ‡Π΅Ρ€Π΅Π΄Π½ΠΎΠΉ Ρ€Π°Π· измСняСм Β«Ρ‚ΠΎΡ‡ΠΊΡƒ ΠΎΡ‚Ρ‡Π΅Ρ‚Π°Β» Ρ„Π°Π·Ρ‹ βˆ’ ΠΏΡ€ΠΈ Π³ΠΎΡ€ΠΈΠ·ΠΎΠ½Ρ‚Π°Π»ΡŒΠ½ΠΎΠΌ располоТСнии Π²Π΅ΠΊΡ‚ΠΎΡ€Π° ΠΊΠΎΠ»Π΅Π±Π°Π½ΠΈΠΉ Ρ‚ΠΎΠΊΠ° ΠΌΡ‹ нСявно ΠΏΡ€ΠΈΠ½ΠΈΠΌΠ°Π΅ΠΌ Π½Π°Ρ‡Π°Π»ΡŒΠ½ΡƒΡŽ Ρ„Π°Π·Ρƒ ΠΊΠΎΠ»Π΅Π±Π°Π½ΠΈΠΉ силы Ρ‚ΠΎΠΊΠ° Ρ€Π°Π²Π½ΠΎΠΉ Π½ΡƒΠ»ΡŽ.

Π”Π°Π²Π°ΠΉΡ‚Π΅-ΠΊΠ° я Π²Π°ΠΌ напомню. ΠšΠΎΠ½Π΄Π΅Π½ΡΠ°Ρ‚ΠΎΡ€, ΠΎΠ½ ΠΆΠ΅ Π² Π½Π°Ρ€ΠΎΠ΄Π΅ «ΠΊΠΎΠ½Π΄Ρ‘Ρ€», состоит ΠΈΠ· Π΄Π²ΡƒΡ… ΠΈΠ·ΠΎΠ»ΠΈΡ€ΠΎΠ²Π°Π½Π½Ρ‹Ρ… ΠΎΠ±ΠΊΠ»Π°Π΄ΠΎΠΊ. ΠŸΡ€ΠΈ ΠΊΡ€Π°Ρ‚ΠΊΠΎΠ²Ρ€Π΅ΠΌΠ΅Π½Π½ΠΎΠΉ ΠΏΠΎΠ΄Π°Ρ‡Π΅ Π½Π° кондСнсатор постоянного напряТСния, ΠΎΠ½ заряТаСтся ΠΈ сохраняСт Π² сСбС этот заряд. Π•ΠΌΠΊΠΎΡΡ‚ΡŒ кондСнсатора зависит ΠΎΡ‚ Ρ‚ΠΎΠ³ΠΎ, Π½Π° сколько «ΠΌΠ΅ΡΡ‚» рассчитаны ΠΎΠ±ΠΊΠ»Π°Π΄ΠΊΠΈ, Π° Ρ‚Π°ΠΊΠΆΠ΅ смотря, ΠΊΠ°ΠΊΠΎΠ΅ расстояниС ΠΌΠ΅ΠΆΠ΄Ρƒ Π½ΠΈΠΌΠΈ. Π”Π°Π²Π°ΠΉΡ‚Π΅ рассмотрим ΠΏΡ€ΠΎΡΡ‚Π΅ΠΉΡˆΡƒΡŽ схСму ΡƒΠΆΠ΅ заряТСнного ΠΊΠΎΠ½Π΄Π΅Ρ€Π°:

Π˜Ρ‚Π°ΠΊ, ΠΌΡ‹ здСсь Π²ΠΈΠ΄ΠΈΠΌ Π½Π° ΠΎΠ΄Π½ΠΎΠΉ ΠΎΠ±ΠΊΠ»Π°Π΄ΠΊΠ΅ восСмь «ΠΏΠ»ΡŽΡΠΎΠ²», Π° Π½Π° Π΄Ρ€ΡƒΠ³ΠΎΠΉ ΡΡ‚ΠΎΠ»ΡŒΠΊΠΎ ΠΆΠ΅ ΠΈ «ΠΌΠΈΠ½ΡƒΡΠΎΠ²». Ну Π° ΠΊΠ°ΠΊ Π²Ρ‹ Π·Π½Π°Π΅Ρ‚Π΅, противополоТности ΠΏΡ€ΠΈΡ‚ΡΠ³ΠΈΠ²Π°ΡŽΡ‚ΡΡ) И Ρ‡Π΅ΠΌ мСньшС расстояниС ΠΌΠ΅ΠΆΠ΄Ρƒ ΠΎΠ±ΠΊΠ»Π°Π΄ΠΊΠ°ΠΌΠΈ, Ρ‚Π΅ΠΌ сильнСС «Π»ΡŽΠ±ΠΎΠ²ΡŒ. Π‘Π»Π΅Π΄ΠΎΠ²Π°Ρ‚Π΅Π»ΡŒΠ½ΠΎ, плюс «Π»ΡŽΠ±ΠΈΡ‚» минус, Π° Ρ‚Π°ΠΊ ΠΊΠ°ΠΊ любовь взаимная, Π·Π½Π°Ρ‡ΠΈΡ‚ ΠΈ минус Ρ‚ΠΎΠΆΠ΅ «Π»ΡŽΠ±ΠΈΡ‚» плюс)). ΠŸΠΎΡΡ‚ΠΎΠΌΡƒ, это притяТСниС Π½Π΅ Π΄Π°Π΅Ρ‚ Ρ€Π°Π·Ρ€ΡΠ΄ΠΈΡ‚ΡŒΡΡ ΡƒΠΆΠ΅ заряТСнному кондСнсатору.

Для Ρ‚ΠΎΠ³ΠΎ, Ρ‡Ρ‚ΠΎΠ±Ρ‹ Ρ€Π°Π·Ρ€ΡΠ΄ΠΈΡ‚ΡŒ кондСнсатор, достаточно ΠΏΡ€ΠΎΠ»ΠΎΠΆΠΈΡ‚ΡŒ «ΠΌΠΎΡΡ‚ΠΈΠΊ», Ρ‡Ρ‚ΠΎΠ±Ρ‹ «ΠΏΠ»ΡŽΡΡ‹» ΠΈ «ΠΌΠΈΠ½ΡƒΡΡ‹» Π²ΡΡ‚Ρ€Π΅Ρ‚ΠΈΠ»ΠΈΡΡŒ. Π’ΠΎ Π΅ΡΡ‚ΡŒ Ρ‚ΡƒΠΏΠΎ Π·Π°ΠΌΠΊΠ½ΡƒΡ‚ΡŒ Π΄Π²Π° Π²Ρ‹Π²ΠΎΠ΄Π° ΠΎΡ‚ ΠΏΡ€ΠΎΠΊΠ»Π°Π΄ΠΎΠΊ Ρ…ΠΎΡ€ΠΎΡˆΠΈΠΌ ΠΏΡ€ΠΎΠ²ΠΎΠ΄Π½ΠΈΠΊΠΎΠΌ . ΠšΠΎΠ½Π΄Π΅Π½ΡΠ°Ρ‚ΠΎΡ€Ρ‹ большой Смкости Π»ΡƒΡ‡ΡˆΠ΅ Ρ€Π°Π·Ρ€ΡΠΆΠ°Ρ‚ΡŒ Ρ‡Π΅Ρ€Π΅Π· сопротивлСниС , Ρ‚ΠΎ Π΅ΡΡ‚ΡŒ рСзистором.

Π‘ ΠΊΠΎΠ½Π΄Π΅Ρ€ΠΎΠΌ Π²Ρ€ΠΎΠ΄Π΅ Ρ€Π°Π·ΠΎΠ±Ρ€Π°Π»ΠΈΡΡŒ… А Π²ΠΎΡ‚ Ρ‡Ρ‚ΠΎ Ρ‚Π°ΠΊΠΎΠ΅ «Ρ†Π΅ΠΏΡŒ» ?

Π‘Ρ‹Π²Π°ΡŽΡ‚ вСлосипСдныС Ρ†Π΅ΠΏΠΈ, ΠΌΠΎΡ‚ΠΎΡ†ΠΈΠΊΠ»Π΅Ρ‚Π½Ρ‹Π΅, Ρ†Π΅ΠΏΠΈ для Π±Π΅Π½Π·ΠΎΠΏΠΈΠ»Ρ‹, Π° Π±Ρ‹Π²Π°ΡŽΡ‚ Π΅Ρ‰Π΅ Ρ‚Π°ΠΊ Π½Π°Π·Ρ‹Π²Π°Π΅ΠΌΡ‹Π΅ «ΡΠ»Π΅ΠΊΡ‚ричСскиС Ρ†Π΅ΠΏΠΈ». Π’ΠΎ Π΅ΡΡ‚ΡŒ это ΠΏΡ€ΠΎΠ²ΠΎΠ΄Π°, Π»Π°ΠΌΠΏΠΎΡ‡ΠΊΠΈ, Π³ΠΎΠ²ΠΎΡ€ΠΈΠ»ΠΊΠΈ, Ρ€Π°Π΄ΠΈΠΎΠ΄Π΅Ρ‚Π°Π»ΠΈ ΠΈ Ρ‚Π΄ соСдинСнныС Π² ΠΊΠ°ΠΊΠΎΠΉ Ρ‚ΠΎ ΠΏΠΎΡΠ»Π΅Π΄ΠΎΠ²Π°Ρ‚Π΅Π»ΡŒΠ½ΠΎΡΡ‚ΠΈ ΠΈ Ρ‡Π΅Ρ€Π΅Π· ΠΊΠΎΡ‚ΠΎΡ€Ρ‹Π΅ ΠΈΠ΄Π΅Ρ‚ ΠΈΠ»ΠΈ Π±ΡƒΠ΄Π΅Ρ‚ ΠΈΠ΄Ρ‚ΠΈ элСктричСский Ρ‚ΠΎΠΊ ΠΎΡ‚ источника питания. Π”Π° хотя Π±Ρ‹ Π΄Π°ΠΆΠ΅ ΠΎΡ‚ Π±Π°Ρ‚Π°Ρ€Π΅ΠΉΠΊΠΈ ΠΈΠ»ΠΈ Π‘Π»ΠΎΠΊΠ° питания .

Π”ΡƒΠΌΠ°ΡŽ, Π²Ρ‹ Π·Π½Π°Π΅Ρ‚Π΅, Ρ‡Ρ‚ΠΎ элСктричСский Ρ‚ΠΎΠΊ Π±Ρ‹Π²Π°Π΅Ρ‚ ΠΏΠ΅Ρ€Π΅ΠΌΠ΅Π½Π½Ρ‹ΠΌ ΠΈ постоянным. Π”Π°Π²Π°ΠΉΡ‚Π΅ ΠΆΠ΅ ΡƒΠ·Π½Π°Π΅ΠΌ, ΠΊΠ°ΠΊ Π²Π΅Π΄Π΅Ρ‚ сСбя кондСнсатор, ΠΊΠΎΠ³Π΄Π° Ρ‡Π΅Ρ€Π΅Π· Π½Π΅Π³ΠΎ ΠΏΡ€ΠΎΡ…ΠΎΠ΄ΠΈΡ‚ постоянный ΠΈ ΠΏΠ΅Ρ€Π΅ΠΌΠ΅Π½Π½Ρ‹ΠΉ Ρ‚ΠΎΠΊ?

ΠšΠΎΠ½Π΄Π΅Π½ΡΠ°Ρ‚ΠΎΡ€ Π² Ρ†Π΅ΠΏΠΈ постоянного Ρ‚ΠΎΠΊΠ°

Π˜Ρ‚Π°ΠΊ, Π±Π΅Ρ€Π΅ΠΌ Π±Π»ΠΎΠΊ питания постоянного напряТСния ΠΈ выставляСм Π½Π° Π΅Π³ΠΎ ΠΊΡ€ΠΎΠΊΠΎΠ΄ΠΈΠ»Π°Ρ… напряТСниС Π² 12 Π’ΠΎΠ»ΡŒΡ‚. Π›Π°ΠΌΠΏΠΎΡ‡ΠΊΡƒ Ρ‚ΠΎΠΆΠ΅ Π±Π΅Ρ€Π΅ΠΌ Π½Π° 12 Π’ΠΎΠ»ΡŒΡ‚. Π’Π΅ΠΏΠ΅Ρ€ΡŒ ΠΌΠ΅ΠΆΠ΄Ρƒ ΠΎΠ΄Π½ΠΈΠΌ Ρ‰ΡƒΠΏΠΎΠΌ Π±Π»ΠΎΠΊΠ° питания ΠΈ Π»Π°ΠΌΠΏΠΎΡ‡ΠΊΠΈ вставляСм кондСнсатор:

НС-Π°, Π½Π΅ Π³ΠΎΡ€ΠΈΡ‚.

А Π²ΠΎΡ‚ Ссли Π½Π°ΠΏΡ€ΡΠΌΡƒΡŽ ΡΠ΄Π΅Π»Π°Ρ‚ΡŒ, Ρ‚ΠΎ Π³ΠΎΡ€ΠΈΡ‚:

ΠžΡ‚ΡΡŽΠ΄Π° Π½Π°ΠΏΡ€Π°ΡˆΠΈΠ²Π°Π΅Ρ‚ΡΡ Π²Ρ‹Π²ΠΎΠ΄: постоянный Ρ‚ΠΎΠΊ Ρ‡Π΅Ρ€Π΅Π· кондСнсатор Π½Π΅ Ρ‚Π΅Ρ‡Π΅Ρ‚!

Ну Π½Π΅, Ссли чСстно, Ρ‚ΠΎ Π² самый Π½Π°Ρ‡Π°Π»ΡŒΠ½Ρ‹ΠΉ ΠΌΠΎΠΌΠ΅Π½Ρ‚ ΠΏΠΎΠ΄Π°Ρ‡ΠΈ напряТСния Ρ‚ΠΎΠΊ всС-Ρ‚Π°ΠΊΠΈ Ρ‚Π΅Ρ‡Π΅Ρ‚ Π½Π° Π΄ΠΎΠ»ΠΈ сСкунды. ВсС зависит ΠΎΡ‚ Смкости кондСнсатора. Но это Π² расчСт Π½Π΅ Π±Π΅Ρ€ΡƒΡ‚.

ΠšΠΎΠ½Π΄Π΅Π½ΡΠ°Ρ‚ΠΎΡ€ Π² Ρ†Π΅ΠΏΠΈ ΠΏΠ΅Ρ€Π΅ΠΌΠ΅Π½Π½ΠΎΠ³ΠΎ Ρ‚ΠΎΠΊΠ°


Π˜Ρ‚Π°ΠΊ, Ρ‡Ρ‚ΠΎΠ±Ρ‹ ΡƒΠ·Π½Π°Ρ‚ΡŒ, Ρ‚Π΅Ρ‡Π΅Ρ‚ Π»ΠΈ ΠΏΠ΅Ρ€Π΅ΠΌΠ΅Π½Π½Ρ‹ΠΉ Ρ‚ΠΎΠΊ Ρ‡Π΅Ρ€Π΅Π· кондСнсатор, Π½Π°ΠΌ Π½ΡƒΠΆΠ΅Π½ Π³Π΅Π½Π΅Ρ€Π°Ρ‚ΠΎΡ€ ΠΏΠ΅Ρ€Π΅ΠΌΠ΅Π½Π½ΠΎΠ³ΠΎ Ρ‚ΠΎΠΊΠ°. Π”ΡƒΠΌΠ°ΡŽ, этот Π³Π΅Π½Π΅Ρ€Π°Ρ‚ΠΎΡ€ частоты Π²ΠΏΠΎΠ»Π½Π΅ сойдСт:

Π’Π°ΠΊ ΠΊΠ°ΠΊ китайский Π³Π΅Π½Π΅Ρ€Π°Ρ‚ΠΎΡ€ Ρƒ мСня ΠΎΡ‡Π΅Π½ΡŒ слабСнький, Ρ‚ΠΎ ΠΌΡ‹ вмСсто Π½Π°Π³Ρ€ΡƒΠ·ΠΊΠΈ-Π»Π°ΠΌΠΏΠΎΡ‡ΠΊΠΈ Π±ΡƒΠ΄Π΅ΠΌ ΠΈΡΠΏΠΎΠ»ΡŒΠ·ΠΎΠ²Π°Ρ‚ΡŒ простой РСзистор Π½Π° 100 Ом. Π’Π°ΠΊΠΆΠ΅ возьмСм ΠΈ кондСнсатор Π΅ΠΌΠΊΠΎΡΡ‚ΡŒΡŽ Π² 1 ΠΌΠΈΠΊΡ€ΠΎΠ€Π°Ρ€Π°Π΄:

Π‘ΠΏΠ°ΠΈΠ²Π°Π΅ΠΌ ΠΊΠ°ΠΊ-Ρ‚ΠΎ Π²ΠΎΡ‚ Ρ‚Π°ΠΊ ΠΈ ΠΏΠΎΠ΄Π°Π΅ΠΌ сигнал с Π³Π΅Π½Π΅Ρ€Π°Ρ‚ΠΎΡ€Π° частоты:

Π”Π°Π»Π΅Π΅ Π·Π° Π΄Π΅Π»ΠΎ бСрСтся Π¦ΠΈΡ„Ρ€ΠΎΠ²ΠΎΠΉ осциллограф OWON SDS6062 . Π§Ρ‚ΠΎ Ρ‚Π°ΠΊΠΎΠ΅ осциллограф ΠΈ с Ρ‡Π΅ΠΌ Π΅Π³ΠΎ Сдят, Ρ‡ΠΈΡ‚Π°Π΅ΠΌ здСсь . Π‘ΡƒΠ΄Π΅ΠΌ ΠΈΡΠΏΠΎΠ»ΡŒΠ·ΠΎΠ²Π°Ρ‚ΡŒ сразу Π΄Π²Π° ΠΊΠ°Π½Π°Π»Π°, Π½Ρƒ Ρ‚ΠΎ Π΅ΡΡ‚ΡŒ Π½Π° ΠΎΠ΄Π½ΠΎΠΌ экранС Π±ΡƒΠ΄ΡƒΡ‚ Π²Ρ‹ΡΠ²Π΅Ρ‡ΠΈΠ²Π°Ρ‚ΡŒΡΡ сразу Π΄Π²Π° сигнала. Π—Π΄Π΅ΡΡŒ Π½Π° экранчикС ΡƒΠΆΠ΅ Π²ΠΈΠ΄Π½Ρ‹ Π½Π°Π²ΠΎΠ΄ΠΊΠΈ ΠΎΡ‚ сСти 220 Π’ΠΎΠ»ΡŒΡ‚. НС ΠΎΠ±Ρ€Π°Ρ‰Π°ΠΉΡ‚Π΅ Π²Π½ΠΈΠΌΠ°Π½ΠΈΠ΅.

Π‘ΡƒΠ΄Π΅ΠΌ ΠΏΠΎΠ΄Π°Π²Π°Ρ‚ΡŒ ΠΏΠ΅Ρ€Π΅ΠΌΠ΅Π½Π½ΠΎΠ΅ напряТСниС ΠΈ ΡΠΌΠΎΡ‚Ρ€Π΅Ρ‚ΡŒ сигналы, ΠΊΠ°ΠΊ говорят ΠΏΡ€ΠΎΡ„Π΅ΡΡΠΈΠΎΠ½Π°Π»ΡŒΠ½Ρ‹Π΅ элСктронщики, Π½Π° Π²Ρ…ΠΎΠ΄Π΅ ΠΈ Π½Π° Π²Ρ‹Ρ…ΠΎΠ΄Π΅. ΠžΠ΄Π½ΠΎΠ²Ρ€Π΅ΠΌΠ΅Π½Π½ΠΎ.

ВсС это Π±ΡƒΠ΄Π΅Ρ‚ Π²Ρ‹Π³Π»ΡΠ΄Π΅Ρ‚ΡŒ ΠΏΡ€ΠΈΠΌΠ΅Ρ€Π½ΠΎ Π²ΠΎΡ‚ Ρ‚Π°ΠΊ:

Π˜Ρ‚Π°ΠΊ, Ссли Ρƒ нас частота нулСвая, Ρ‚ΠΎ это Π·Π½Π°Ρ‡ΠΈΡ‚ постоянный Ρ‚ΠΎΠΊ. ΠŸΠΎΡΡ‚ΠΎΡΠ½Π½Ρ‹ΠΉ Ρ‚ΠΎΠΊ, ΠΊΠ°ΠΊ ΠΌΡ‹ ΡƒΠΆΠ΅ Π²ΠΈΠ΄Π΅Π»ΠΈ, кондСнсатор Π½Π΅ пропускаСт. Π‘ этим Π²Ρ€ΠΎΠ΄Π΅ Π±Ρ‹ Ρ€Π°Π·ΠΎΠ±Ρ€Π°Π»ΠΈΡΡŒ. Но Ρ‡Ρ‚ΠΎ Π±ΡƒΠ΄Π΅Ρ‚, Ссли ΠΏΠΎΠ΄Π°Ρ‚ΡŒ синусоиду с частотой Π² 100 Π“Π΅Ρ€Ρ†?

На дисплСС осциллографа я Π²Ρ‹Π²Π΅Π» Ρ‚Π°ΠΊΠΈΠ΅ ΠΏΠ°Ρ€Π°ΠΌΠ΅Ρ‚Ρ€Ρ‹, ΠΊΠ°ΠΊ частота сигнала ΠΈ Π΅Π³ΠΎ Π°ΠΌΠΏΠ»ΠΈΡ‚ΡƒΠ΄Π°: F

— это частота, Ma

— Π°ΠΌΠΏΠ»ΠΈΡ‚ΡƒΠ΄Π° (эти ΠΏΠ°Ρ€Π°ΠΌΠ΅Ρ‚Ρ€Ρ‹ ΠΏΠΎΠΌΠ΅Ρ‚ΠΈΠ» Π±Π΅Π»ΠΎΠΉ стрСлочкой). ΠŸΠ΅Ρ€Π²Ρ‹ΠΉ ΠΊΠ°Π½Π°Π» ΠΏΠΎΠΌΠ΅Ρ‡Π΅Π½ красным Ρ†Π²Π΅Ρ‚ΠΎΠΌ, Π° Π²Ρ‚ΠΎΡ€ΠΎΠΉ ΠΊΠ°Π½Π°Π» — ΠΆΠ΅Π»Ρ‚Ρ‹ΠΌ, для удобства восприятия.

ΠšΡ€Π°ΡΠ½Π°Ρ синусоида ΠΏΠΎΠΊΠ°Π·Ρ‹Π²Π°Π΅Ρ‚ сигнал, ΠΊΠΎΡ‚ΠΎΡ€Ρ‹ΠΉ Π²Ρ‹Π΄Π°Π΅Ρ‚ Π½Π°ΠΌ китайский Π³Π΅Π½Π΅Ρ€Π°Ρ‚ΠΎΡ€ частоты. ЖСлтая синусоида — это Ρ‚ΠΎ, Ρ‡Ρ‚ΠΎ ΠΌΡ‹ ΡƒΠΆΠ΅ ΠΏΠΎΠ»ΡƒΡ‡Π°Π΅ΠΌ Π½Π° Π½Π°Π³Ρ€ΡƒΠ·ΠΊΠ΅. Π’ нашСм случаС Π½Π°Π³Ρ€ΡƒΠ·ΠΊΠΎΠΉ являСтся рСзистор. Ну Π²ΠΎΡ‚, собствСнно, ΠΈ всС.

Как Π²Ρ‹ Π²ΠΈΠ΄ΠΈΡ‚Π΅ Π½Π° осциллограммС Π²Ρ‹ΡˆΠ΅, с Π³Π΅Π½Π΅Ρ€Π°Ρ‚ΠΎΡ€Π° я подаю ΡΠΈΠ½ΡƒΡΠΎΠΈΠ΄Π°Π»ΡŒΠ½Ρ‹ΠΉ сигнал с частотой Π² 100 Π“Π΅Ρ€Ρ† ΠΈ Π°ΠΌΠΏΠ»ΠΈΡ‚ΡƒΠ΄ΠΎΠΉ Π² 2 Π’ΠΎΠ»ΡŒΡ‚Π°. На рСзисторС ΠΌΡ‹ ΡƒΠΆΠ΅ Π²ΠΈΠ΄ΠΈΠΌ сигнал с Ρ‚Π°ΠΊΠΎΠΉ ΠΆΠ΅ частотой (ΠΆΠ΅Π»Ρ‚Ρ‹ΠΉ сигнал), Π½ΠΎ Π΅Π³ΠΎ Π°ΠΌΠΏΠ»ΠΈΡ‚ΡƒΠ΄Π° составляСт ΠΊΠ°ΠΊΠΈΡ…-Ρ‚ΠΎ 136 ΠΌΠΈΠ»Π»ΠΈΠ’ΠΎΠ»ΡŒΡ‚. Π”Π° Π΅Ρ‰Π΅ ΠΈ сигнал получился ΠΊΠ°ΠΊΠΎΠΉ-Ρ‚ΠΎ «Π»ΠΎΡ…ΠΌΠ°Ρ‚Ρ‹ΠΉ». Π­Ρ‚ΠΎ связано с Ρ‚Π°ΠΊ Π½Π°Π·Ρ‹Π²Π°Π΅ΠΌΡ‹ΠΌΠΈ «ΡˆΡƒΠΌΠ°ΠΌΠΈ». Π¨ΡƒΠΌ — это ΠΏΠΎ ΠΈΠ΄Π΅Π΅ сигнал с малСнькой Π°ΠΌΠΏΠ»ΠΈΡ‚ΡƒΠ΄ΠΎΠΉ ΠΈ бСспорядочным ΠΈΠ·ΠΌΠ΅Π½Π΅Π½ΠΈΠ΅ΠΌ напряТСния, Π»ΠΎΠ²ΠΈΠΌΡ‹ΠΉ ΠΈΠ· ΠΎΠΊΡ€ΡƒΠΆΠ°ΡŽΡ‰Π΅ΠΉ срСды. Π’Π°ΠΊΠΆΠ΅ радиоэлСмСнты Ρ‚ΠΎΠΆΠ΅ ΠΌΠΎΠ³ΡƒΡ‚ Π΄ΠΎΠ±Π°Π²Π»ΡΡ‚ΡŒ ΡˆΡƒΠΌ. НапримСр ΠΎΡ‡Π΅Π½ΡŒ Ρ…ΠΎΡ€ΠΎΡˆΠΎ «ΡˆΡƒΠΌΠΈΡ‚» рСзистор. Π—Π½Π°Ρ‡ΠΈΡ‚ «Π»ΠΎΡ…ΠΌΠ°Ρ‚ΠΎΡΡ‚ΡŒ» сигнала — это сумма синусоиды ΠΈ ΡˆΡƒΠΌΠ°.

Амплитуда ΠΆΠ΅Π»Ρ‚ΠΎΠ³ΠΎ сигнала стала мСньшС, Π΄Π° Π΅Ρ‰Π΅ ΠΈ Π³Ρ€Π°Ρ„ΠΈΠΊ ΠΆΠ΅Π»Ρ‚ΠΎΠ³ΠΎ сигнала сдвигаСтся Π²Π»Π΅Π²ΠΎ, Ρ‚ΠΎ Π΅ΡΡ‚ΡŒ ΠΎΠΏΠ΅Ρ€Π΅ΠΆΠ°Π΅Ρ‚ красный сигнал, ΠΈΠ»ΠΈ Π½Π°ΡƒΡ‡Π½Ρ‹ΠΌ языком, появляСтся сдвиг Ρ„Π°Π·
. ΠžΠΏΠ΅Ρ€Π΅ΠΆΠ°Π΅Ρ‚ ΠΈΠΌΠ΅Π½Π½ΠΎ Ρ„Π°Π·Π°, Π° Π½Π΅ сам сигнал.
Если Π±Ρ‹ ΠΎΠΏΠ΅Ρ€Π΅ΠΆΠ°Π» сам сигнал, Ρ‚ΠΎ Ρƒ нас Π±Ρ‹ Ρ‚ΠΎΠ³Π΄Π° ΠΏΠΎΠ»ΡƒΡ‡ΠΈΠ»ΠΎΡΡŒ, Ρ‡Ρ‚ΠΎ сигнал Π½Π° рСзисторС появлялся Π±Ρ‹ ΠΏΠΎ Π²Ρ€Π΅ΠΌΠ΅Π½ΠΈ Ρ€Π°Π½ΡŒΡˆΠ΅, Ρ‡Π΅ΠΌ сигнал, ΠΏΠΎΠ΄Π°Π½Ρ‹ΠΉ Π½Π° Π½Π΅Π³ΠΎ Ρ‡Π΅Ρ€Π΅Π· кондСнсатор. ΠŸΠΎΠ»ΡƒΡ‡ΠΈΠ»ΠΎΡΡŒ Π±Ρ‹ ΠΊΠ°ΠΊΠΎΠ΅-Ρ‚Π΅ ΠΏΠ΅Ρ€Π΅ΠΌΠ΅Ρ‰Π΅Π½ΠΈΠ΅ Π²ΠΎ Π²Ρ€Π΅ΠΌΠ΅Π½ΠΈ:-), Ρ‡Ρ‚ΠΎ ΠΊΠΎΠ½Π΅Ρ‡Π½ΠΎ ΠΆΠ΅, Π½Π΅Π²ΠΎΠ·ΠΌΠΎΠΆΠ½ΠΎ.

Π‘Π΄Π²ΠΈΠ³ Ρ„Π°Π·
— это Ρ€Π°Π·Π½ΠΎΡΡ‚ΡŒ ΠΌΠ΅ΠΆΠ΄Ρƒ Π½Π°Ρ‡Π°Π»ΡŒΠ½Ρ‹ΠΌΠΈ Ρ„Π°Π·Π°ΠΌΠΈ Π΄Π²ΡƒΡ… измСряСмых Π²Π΅Π»ΠΈΡ‡ΠΈΠ½
. Π’ Π΄Π°Π½Π½ΠΎΠΌ случаС напряТСния. Для Ρ‚ΠΎΠ³ΠΎ, Ρ‡Ρ‚ΠΎΠ±Ρ‹ произвСсти Π·Π°ΠΌΠ΅Ρ€ сдвига Ρ„Π°Π·, Π΄ΠΎΠ»ΠΆΠ½ΠΎ Π±Ρ‹Ρ‚ΡŒ условиС, Ρ‡Ρ‚ΠΎ Ρƒ этих сигналов ΠΎΠ΄Π½Π° ΠΈ Ρ‚Π° ΠΆΠ΅ частота
. Амплитуда ΠΌΠΎΠΆΠ΅Ρ‚ Π±Ρ‹Ρ‚ΡŒ любой. НиТС Π½Π° рисункС ΠΏΡ€ΠΈΠ²Π΅Π΄Π΅Π½ этот самый сдвиг Ρ„Π°Π· ΠΈΠ»ΠΈ, ΠΊΠ°ΠΊ Π΅Ρ‰Π΅ Π΅Π³ΠΎ Π½Π°Π·Ρ‹Π²Π°ΡŽΡ‚, Ρ€Π°Π·Π½ΠΎΡΡ‚ΡŒ Ρ„Π°Π·
:

Π”Π°Π²Π°ΠΉΡ‚Π΅ ΡƒΠ²Π΅Π»ΠΈΡ‡ΠΈΠΌ частоту Π½Π° Π³Π΅Π½Π΅Ρ€Π°Ρ‚ΠΎΡ€Π΅ Π΄ΠΎ 500 Π“Π΅Ρ€Ρ†

На рСзисторС ΡƒΠΆΠ΅ ΠΏΠΎΠ»ΡƒΡ‡ΠΈΠ»ΠΈ 560 ΠΌΠΈΠ»Π»ΠΈΠ’ΠΎΠ»ΡŒΡ‚Π°. Π‘Π΄Π²ΠΈΠ³ Ρ„Π°Π· ΡƒΠΌΠ΅Π½ΡŒΡˆΠ°Π΅Ρ‚ΡΡ.

Π£Π²Π΅Π»ΠΈΡ‡ΠΈΠ²Π°Π΅ΠΌ частоту Π΄ΠΎ 1 ΠšΠΈΠ»ΠΎΠ“Π΅Ρ€Ρ†Π°

На Π²Ρ‹Ρ…ΠΎΠ΄Π΅ Ρƒ нас ΡƒΠΆΠ΅ 1 Π’ΠΎΠ»ΡŒΡ‚

.

Амплитуда 1,84 Π’ΠΎΠ»ΡŒΡ‚Π° ΠΈ сдвиг Ρ„Π°Π· явно стаСт мСньшС

Амплитуда ΡƒΠΆΠ΅ ΠΏΠΎΡ‡Ρ‚ΠΈ такая ΠΆΠ΅ ΠΊΠ°ΠΊ ΠΈ Π½Π° Π²Ρ…ΠΎΠ΄Π΅. Π‘Π΄Π²ΠΈΠ³ Ρ„Π°Π· ΠΌΠ΅Π½Π΅Π΅ Π·Π°ΠΌΠ΅Ρ‚Π΅Π½.

Π‘Π΄Π²ΠΈΠ³Π° Ρ„Π°Π· ΠΏΠΎΡ‡Ρ‚ΠΈ Π½Π΅Ρ‚. Амплитуда ΠΏΠΎΡ‡Ρ‚ΠΈ такая ΠΆΠ΅, ΠΊΠ°ΠΊ ΠΈ Π½Π° Π²Ρ…ΠΎΠ΄Π΅, Ρ‚ΠΎ Π΅ΡΡ‚ΡŒ 2 Π’ΠΎΠ»ΡŒΡ‚Π°.

Π§Π΅ΠΌ большС частота, Ρ‚Π΅ΠΌ мСньшСС сопротивлСниС кондСнсатор ΠΎΠΊΠ°Π·Ρ‹Π²Π°Π΅Ρ‚ ΠΏΠ΅Ρ€Π΅ΠΌΠ΅Π½Π½ΠΎΠΌΡƒ Ρ‚ΠΎΠΊΡƒ. Π‘Π΄Π²ΠΈΠ³ Ρ„Π°Π· ΡƒΠ±Ρ‹Π²Π°Π΅Ρ‚ с ΡƒΠ²Π΅Π»ΠΈΡ‡Π΅Π½ΠΈΠ΅ΠΌ частоты ΠΏΠΎΡ‡Ρ‚ΠΈ Π΄ΠΎ нуля. На бСсконСчно Π½ΠΈΠ·ΠΊΠΈΡ… частотах Π΅Π³ΠΎ Π²Π΅Π»ΠΈΡ‡ΠΈΠ½Π° составляСт 90 градусов ΠΈΠ»ΠΈ

Ο€/2

.

Если ΠΏΠΎΡΡ‚Ρ€ΠΎΠΈΡ‚ΡŒ ΠΎΠ±Ρ€Π΅Π·ΠΎΠΊ Π³Ρ€Π°Ρ„ΠΈΠΊΠ°, Ρ‚ΠΎ получится Ρ‚ΠΈΠΏΠ° Ρ‡Ρ‚ΠΎ-Ρ‚ΠΎ этого:

По Π²Π΅Ρ€Ρ‚ΠΈΠΊΠ°Π»ΠΈ я ΠΎΡ‚Π»ΠΎΠΆΠΈΠ» напряТСниС, ΠΏΠΎ Π³ΠΎΡ€ΠΈΠ·ΠΎΠ½Ρ‚Π°Π»ΠΈ — частоту.

Π˜Ρ‚Π°ΠΊ, ΠΌΡ‹ с Π²Π°ΠΌΠΈ ΡƒΠ·Π½Π°Π»ΠΈ, Ρ‡Ρ‚ΠΎ сопротивлСниС кондСнсатора зависит ΠΎΡ‚ частоты. Но Ρ‚ΠΎΠ»ΡŒΠΊΠΎ Π»ΠΈ ΠΎΡ‚ частоты? Π”Π°Π²Π°ΠΉΡ‚Π΅ возьмСм кондСнсатор Π΅ΠΌΠΊΠΎΡΡ‚ΡŒΡŽ Π² 0,1 ΠΌΠΈΠΊΡ€ΠΎΠ€Π°Ρ€Π°Π΄, Ρ‚ΠΎ Π΅ΡΡ‚ΡŒ Π½ΠΎΠΌΠΈΠ½Π°Π»ΠΎΠΌ Π² 10 Ρ€Π°Π· мСньшС, Ρ‡Π΅ΠΌ ΠΏΡ€Π΅Π΄Ρ‹Π΄ΡƒΡ‰ΠΈΠΉ ΠΈ снова ΠΏΡ€ΠΎΠ³ΠΎΠ½ΠΈΠΌ ΠΏΠΎ этим ΠΆΠ΅ частотам.

Π’Π½ΠΈΠΌΠ°Ρ‚Π΅Π»ΡŒΠ½ΠΎ сравнитС Π°ΠΌΠΏΠ»ΠΈΡ‚ΡƒΠ΄Π½Ρ‹Π΅ значСния ΠΆΠ΅Π»Ρ‚ΠΎΠ³ΠΎ сигнала Π½Π° ΠΎΠ΄Π½ΠΎΠΉ ΠΈ Ρ‚ΠΎΠΉ ΠΆΠ΅ частотС, Π½ΠΎ с Ρ€Π°Π·Π½Ρ‹ΠΌΠΈ Π½ΠΎΠΌΠΈΠ½Π°Π»ΠΌΠΈ кондСнсатора. НапримСр, Π½Π° частотС Π² 100 Π“Π΅Ρ€Ρ† ΠΈ Π½ΠΎΠΌΠΈΠ½Π°Π»ΠΎΠΌ ΠΊΠΎΠ½Π΄Π΅Ρ€Π° Π² 1 ΠΌΠΊΠ€ Π°ΠΌΠΏΠ»ΠΈΡ‚ΡƒΠ΄Π° ΠΆΠ΅Π»Ρ‚ΠΎΠ³ΠΎ сигнала Ρ€Π°Π²Π½ΡΠ»Π°ΡΡŒ 136 ΠΌΠΈΠ»Π»ΠΈΠ’ΠΎΠ»ΡŒΡ‚, Π° Π½Π° этой ΠΆΠ΅ самой частотС Π°ΠΌΠΏΠ»ΠΈΡ‚ΡƒΠ΄Π° ΠΆΠ΅Π»Ρ‚ΠΎΠ³ΠΎ сигнала, Π½ΠΎ с ΠΊΠΎΠ½Π΄Π΅Ρ€ΠΎΠΌ Π² 0,1 ΠΌΠΊΠ€ ΡƒΠΆΠ΅ Π±Ρ‹Π»Π° 101 ΠΌΠΈΠ»Π»ΠΈΠ’ΠΎΠ»ΡŒΡ‚(Π² Ρ€Π΅Π°Π»ΡŒΠ½ΠΎΡΡ‚ΠΈ Π΅Ρ‰Π΅ мСньшС ΠΈΠ· Π·Π° ΠΏΠΎΠΌΠ΅Ρ…). На частотС 500 Π“Π΅Ρ€Ρ† — 560 ΠΌΠΈΠ»Π»ΠΈΠ’ΠΎΠ»ΡŒΡ‚ ΠΈ 106 ΠΌΠΈΠ»Π»ΠΈΠ’ΠΎΠ»ΡŒΡ‚ соотвСтствСнно, Π½Π° частотС Π² 1 ΠšΠΈΠ»ΠΎΠ“Π΅Ρ€Ρ† — 1 Π’ΠΎΠ»ΡŒΡ‚ ΠΈ 136 ΠΌΠΈΠ»Π»ΠΈΠ’ΠΎΠ»ΡŒΡ‚ ΠΈ Ρ‚Π°ΠΊ Π΄Π°Π»Π΅Π΅.

ΠžΡ‚ΡΡŽΠ΄Π° Π²Ρ‹Π²ΠΎΠ΄ Π½Π°ΠΏΡ€Π°ΡˆΠΈΠ²Π°Π΅Ρ‚ΡΡ сам собой: ΠΏΡ€ΠΈ ΡƒΠΌΠ΅Π½ΡŒΡˆΠ΅Π½ΠΈΠΈ Π½ΠΎΠΌΠΈΠ½Π°Π»Π° кондСнсатора Π΅Π³ΠΎ сопротивлСниС стаСт большС.

Π‘ ΠΏΠΎΠΌΠΎΡ‰ΡŒΡŽ Ρ„ΠΈΠ·ΠΈΠΊΠΎ-матСматичСских ΠΏΡ€Π΅ΠΎΠ±Ρ€Π°Π·ΠΎΠ²Π°Π½ΠΈΠΉ Ρ„ΠΈΠ·ΠΈΠΊΠΈ ΠΈ ΠΌΠ°Ρ‚Π΅ΠΌΠ°Ρ‚ΠΈΠΊΠΈ Π²Ρ‹Π²Π΅Π»ΠΈ Ρ„ΠΎΡ€ΠΌΡƒΠ»Ρƒ для расчСта сопротивлСния кондСнсатора. ΠŸΡ€ΠΎΡˆΡƒ Π»ΡŽΠ±ΠΈΡ‚ΡŒ ΠΈ ΠΆΠ°Π»ΠΎΠ²Π°Ρ‚ΡŒ:

П


постоянная ΠΈ равняСтся ΠΏΡ€ΠΈΠ±Π»ΠΈΠ·ΠΈΡ‚Π΅Π»ΡŒΠ½ΠΎ 3,14

Π’Π°ΠΊ Π²ΠΎΡ‚, ΠΏΠΎΡΡ‚Π°Π²ΡŒΡ‚Π΅ Π² эту Ρ„ΠΎΡ€ΠΌΡƒΠ»Ρƒ частоту Π² ноль Π“Π΅Ρ€Ρ†. Частота Π² ноль Π“Π΅Ρ€Ρ† — это ΠΈ Π΅ΡΡ‚ΡŒ постоянный Ρ‚ΠΎΠΊ. Π§Ρ‚ΠΎ получится? 1/0=Π±Π΅ΡΠΊΠΎΠ½Π΅Ρ‡ΠΎΡΡ‚ΡŒ ΠΈΠ»ΠΈ ΠΎΡ‡Π΅Π½ΡŒ большоС сопротивлСниС. ΠšΠΎΡ€ΠΎΡ‡Π΅ говоря, ΠΎΠ±Ρ€Ρ‹Π² Ρ†Π΅ΠΏΠΈ.

ЗабСгая Π²ΠΏΠ΅Ρ€Π΅Π΄, ΠΌΠΎΠ³Ρƒ ΡΠΊΠ°Π·Π°Ρ‚ΡŒ, Ρ‡Ρ‚ΠΎ Π² Π΄Π°Π½Π½ΠΎΠΌ ΠΎΠΏΡ‹Ρ‚Π΅ ΠΌΡ‹ ΠΏΠΎΠ»ΡƒΡ‡ΠΈΠ»ΠΈ Π€ΠΈΠ»ΡŒΡ‚Ρ€ Высокой Частоты (Π€Π’Π§). Π‘ ΠΏΠΎΠΌΠΎΡ‰ΡŒΡŽ простого кондСнсатора ΠΈ рСзистора, ΠΏΡ€ΠΈΠΌΠ΅Π½ΠΈΠ² Π³Π΄Π΅-Π½ΠΈΠ±ΡƒΠ΄ΡŒ Π² Π·Π²ΡƒΠΊΠΎΠ²ΠΎΠΉ Π°ΠΏΠΏΠ°Ρ€Π°Ρ‚ΡƒΡ€Π΅ Ρ‚Π°ΠΊΠΎΠΉ Ρ„ΠΈΠ»ΡŒΡ‚Ρ€ Π½Π° Π΄ΠΈΠ½Π°ΠΌΠΈΠΊ, Π² Π΄ΠΈΠ½Π°ΠΌΠΈΠΊΠ΅ ΠΌΡ‹ Π±ΡƒΠ΄Π΅Ρ‚ ΡΠ»Ρ‹ΡˆΠ°Ρ‚ΡŒ Ρ‚ΠΎΠ»ΡŒΠΊΠΎ писклявыС высокиС Ρ‚ΠΎΠ½Π°. А Π²ΠΎΡ‚ частоту баса ΠΊΠ°ΠΊ Ρ€Π°Π· ΠΈ Π·Π°Π³Π»ΡƒΡˆΠΈΡ‚ Ρ‚Π°ΠΊΠΎΠΉ Ρ„ΠΈΠ»ΡŒΡ‚Ρ€. Π—Π°Π²ΠΈΡΠΈΠΌΠΎΡΡ‚ΡŒ сопротивлСния кондСнсатора ΠΎΡ‚ частоты ΠΎΡ‡Π΅Π½ΡŒ ΡˆΠΈΡ€ΠΎΠΊΠΎ ΠΈΡΠΏΠΎΠ»ΡŒΠ·ΡƒΠ΅Ρ‚ΡΡ Π² радиоэлСктроникС, особСнно Π² Ρ€Π°Π·Π»ΠΈΡ‡Π½Ρ‹Ρ… Ρ„ΠΈΠ»ΡŒΡ‚Ρ€Π°Ρ…, Π³Π΄Π΅ Π½Π°Π΄ΠΎ ΠΏΠΎΠ³Π°ΡΠΈΡ‚ΡŒ ΠΎΠ΄Π½Ρƒ частоту ΠΈ ΠΏΡ€ΠΎΠΏΡƒΡΡ‚ΠΈΡ‚ΡŒ Π΄Ρ€ΡƒΠ³ΡƒΡŽ.

Π£Π΅Π΄ΠΈΠ½Π΅Π½Π½Ρ‹Π΅ ΠΏΡ€ΠΎΠ²ΠΎΠ΄Π½ΠΈΠΊΠΈ ΠΎΠ±Π»Π°Π΄Π°ΡŽΡ‚ нСбольшой Π΅ΠΌΠΊΠΎΡΡ‚ΡŒΡŽ. Π”Π°ΠΆΠ΅ ΡˆΠ°Ρ€ Ρ‚Π°ΠΊΠΈΡ… Ρ€Π°Π·ΠΌΠ΅Ρ€ΠΎΠ², ΠΊΠ°ΠΊ ЗСмля, ΠΈΠΌΠ΅Π΅Ρ‚ Π΅ΠΌΠΊΠΎΡΡ‚ΡŒ всСго лишь 700 ΠΌΠΊΠ€. ВмСстС с Ρ‚Π΅ΠΌ Π½Π° ΠΏΡ€Π°ΠΊΡ‚ΠΈΠΊΠ΅ Π±Ρ‹Π²Π°Π΅Ρ‚ ΠΏΠΎΡ‚Ρ€Π΅Π±Π½ΠΎΡΡ‚ΡŒ Π² устройствах, ΠΊΠΎΡ‚ΠΎΡ€Ρ‹Π΅ ΠΏΡ€ΠΈ нСбольшом ΠΎΡ‚Π½ΠΎΡΠΈΡ‚Π΅Π»ΡŒΠ½ΠΎ ΠΎΠΊΡ€ΡƒΠΆΠ°ΡŽΡ‰ΠΈΡ… Ρ‚Π΅Π» ΠΏΠΎΡ‚Π΅Π½Ρ†ΠΈΠ°Π»Π΅ Π½Π°ΠΊΠ°ΠΏΠ»ΠΈΠ²Π°Π»ΠΈ Π±Ρ‹ Π½Π° сСбС («кондСнсировали») Π·Π°ΠΌΠ΅Ρ‚Π½Ρ‹Π΅ ΠΏΠΎ Π²Π΅Π»ΠΈΡ‡ΠΈΠ½Π΅ заряды. Π’ основу Ρ‚Π°ΠΊΠΈΡ… устройств, Π½Π°Π·Ρ‹Π²Π°Π΅ΠΌΡ‹Ρ… кондСнсаторами, ΠΏΠΎΠ»ΠΎΠΆΠ΅Π½ Ρ‚ΠΎΡ‚ Ρ„Π°ΠΊΡ‚, Ρ‡Ρ‚ΠΎ ΡΠ»Π΅ΠΊΡ‚Ρ€ΠΎΠ΅ΠΌΠΊΠΎΡΡ‚ΡŒ ΠΏΡ€ΠΎΠ²ΠΎΠ΄Π½ΠΈΠΊΠ° возрастаСт ΠΏΡ€ΠΈ ΠΏΡ€ΠΈΠ±Π»ΠΈΠΆΠ΅Π½ΠΈΠΈ ΠΊ Π½Π΅ΠΌΡƒ Π΄Ρ€ΡƒΠ³ΠΈΡ… Ρ‚Π΅Π». Π­Ρ‚ΠΎ Π²Ρ‹Π·Π²Π°Π½ΠΎ Ρ‚Π΅ΠΌ, Ρ‡Ρ‚ΠΎ ΠΏΠΎΠ΄ дСйствиСм поля, создаваСмого заряТСнным ΠΏΡ€ΠΎΠ²ΠΎΠ΄Π½ΠΈΠΊΠΎΠΌ, Π½Π° поднСсСнном ΠΊ Π½Π΅ΠΌΡƒ Ρ‚Π΅Π»Π΅ Π²ΠΎΠ·Π½ΠΈΠΊΠ°ΡŽΡ‚ ΠΈΠ½Π΄ΡƒΡ†ΠΈΡ€ΠΎΠ²Π°Π½Π½Ρ‹Π΅ (Π½Π° ΠΏΡ€ΠΎΠ²ΠΎΠ΄Π½ΠΈΠΊΠ΅) ΠΈΠ»ΠΈ связанныС (Π½Π° диэлСктрикС) заряды. Заряды, ΠΏΡ€ΠΎΡ‚ΠΈΠ²ΠΎΠΏΠΎΠ»ΠΎΠΆΠ½Ρ‹Π΅ ΠΏΠΎ Π·Π½Π°ΠΊΡƒ заряду ΠΏΡ€ΠΎΠ²ΠΎΠ΄Π½ΠΈΠΊΠ° q, Ρ€Π°ΡΠΏΠΎΠ»Π°Π³Π°ΡŽΡ‚ΡΡ Π±Π»ΠΈΠΆΠ΅ ΠΊ ΠΏΡ€ΠΎΠ²ΠΎΠ΄Π½ΠΈΠΊΡƒ, Ρ‡Π΅ΠΌ ΠΎΠ΄Π½ΠΎΠΈΠΌΠ΅Π½Π½Ρ‹Π΅ с q, ΠΈ, ΡΠ»Π΅Π΄ΠΎΠ²Π°Ρ‚Π΅Π»ΡŒΠ½ΠΎ, ΠΎΠΊΠ°Π·Ρ‹Π²Π°ΡŽΡ‚ большоС влияниС Π½Π° Π΅Π³ΠΎ ΠΏΠΎΡ‚Π΅Π½Ρ†ΠΈΠ°Π». ΠŸΠΎΡΡ‚ΠΎΠΌΡƒ ΠΏΡ€ΠΈ поднСсСнии ΠΊ заряТСнному ΠΏΡ€ΠΎΠ²ΠΎΠ΄Π½ΠΈΠΊΡƒ ΠΊΠ°ΠΊΠΎΠ³ΠΎ-Π»ΠΈΠ±ΠΎ Ρ‚Π΅Π»Π° ΠΏΠΎΡ‚Π΅Π½Ρ†ΠΈΠ°Π» ΠΏΡ€ΠΎΠ²ΠΎΠ΄Π½ΠΈΠΊΠ° ΡƒΠΌΠ΅Π½ΡŒΡˆΠ°Π΅Ρ‚ΡΡ ΠΏΠΎ Π°Π±ΡΠΎΠ»ΡŽΡ‚Π½ΠΎΠΉ Π²Π΅Π»ΠΈΡ‡ΠΈΠ½Π΅. Богласно Ρ„ΠΎΡ€ΠΌΡƒΠ»Π΅ (26.2) это ΠΎΠ·Π½Π°Ρ‡Π°Π΅Ρ‚ ΡƒΠ²Π΅Π»ΠΈΡ‡Π΅Π½ΠΈΠ΅ Смкости ΠΏΡ€ΠΎΠ²ΠΎΠ΄Π½ΠΈΠΊΠ°.

ΠšΠΎΠ½Π΄Π΅Π½ΡΠ°Ρ‚ΠΎΡ€Ρ‹ Π΄Π΅Π»Π°ΡŽΡ‚ Π² Π²ΠΈΠ΄Π΅ Π΄Π²ΡƒΡ… ΠΏΡ€ΠΎΠ²ΠΎΠ΄Π½ΠΈΠΊΠΎΠ², ΠΏΠΎΠΌΠ΅Ρ‰Π΅Π½Π½Ρ‹Ρ… Π±Π»ΠΈΠ·ΠΊΠΎ Π΄Ρ€ΡƒΠ³ ΠΊ Π΄Ρ€ΡƒΠ³Ρƒ. ΠžΠ±Ρ€Π°Π·ΡƒΡŽΡ‰ΠΈΠ΅ кондСнсатор ΠΏΡ€ΠΎΠ²ΠΎΠ΄Π½ΠΈΠΊΠΈ Π½Π°Π·Ρ‹Π²Π°ΡŽΡ‚ Π΅Π³ΠΎ ΠΎΠ±ΠΊΠ»Π°Π΄ΠΊΠ°ΠΌΠΈ. Π§Ρ‚ΠΎΠ±Ρ‹ внСшниС Ρ‚Π΅Π»Π° Π½Π΅ ΠΎΠΊΠ°Π·Ρ‹Π²Π°Π»ΠΈ влияния Π½Π° Π΅ΠΌΠΊΠΎΡΡ‚ΡŒ кондСнсатора, ΠΎΠ±ΠΊΠ»Π°Π΄ΠΊΠ°ΠΌ ΠΏΡ€ΠΈΠ΄Π°ΡŽΡ‚ Ρ‚Π°ΠΊΡƒΡŽ Ρ„ΠΎΡ€ΠΌΡƒ ΠΈ Ρ‚Π°ΠΊ Ρ€Π°ΡΠΏΠΎΠ»Π°Π³Π°ΡŽΡ‚ ΠΈΡ… Π΄Ρ€ΡƒΠ³ ΠΎΡ‚Π½ΠΎΡΠΈΡ‚Π΅Π»ΡŒΠ½ΠΎ Π΄Ρ€ΡƒΠ³Π°, Ρ‡Ρ‚ΠΎΠ±Ρ‹ ΠΏΠΎΠ»Π΅, создаваСмоС Π½Π°ΠΊΠ°ΠΏΠ»ΠΈΠ²Π°Π΅ΠΌΡ‹ΠΌΠΈ Π½Π° Π½ΠΈΡ… зарядами, Π±Ρ‹Π»ΠΎ сосрСдоточСно Π²Π½ΡƒΡ‚Ρ€ΠΈ кондСнсатора. Π­Ρ‚ΠΎΠΌΡƒ ΡƒΡΠ»ΠΎΠ²ΠΈΡŽ ΡƒΠ΄ΠΎΠ²Π»Π΅Ρ‚Π²ΠΎΡ€ΡΡŽΡ‚ (см. Β§ 14) Π΄Π²Π΅ пластинки, располоТСнныС Π±Π»ΠΈΠ·ΠΊΠΎ Π΄Ρ€ΡƒΠ³ ΠΊ Π΄Ρ€ΡƒΠ³Ρƒ, Π΄Π²Π° ΠΊΠΎΠ°ΠΊΡΠΈΠ°Π»ΡŒΠ½Ρ‹Ρ… Ρ†ΠΈΠ»ΠΈΠ½Π΄Ρ€Π° ΠΈ Π΄Π²Π΅ концСнтричСскиС сфСры. БоотвСтствСнно Π±Ρ‹Π²Π°ΡŽΡ‚ плоскиС, цилиндричСскиС ΠΈ сфСричСскиС кондСнсаторы. ΠŸΠΎΡΠΊΠΎΠ»ΡŒΠΊΡƒ ΠΏΠΎΠ»Π΅ Π·Π°ΠΊΠ»ΡŽΡ‡Π΅Π½ΠΎ Π²Π½ΡƒΡ‚Ρ€ΠΈ кондСнсатора, Π»ΠΈΠ½ΠΈΠΈ элСктричСского смСщСния Π½Π°Ρ‡ΠΈΠ½Π°ΡŽΡ‚ΡΡ Π½Π° ΠΎΠ΄Π½ΠΎΠΉ ΠΎΠ±ΠΊΠ»Π°Π΄ΠΊΠ΅ ΠΈ Π·Π°ΠΊΠ°Π½Ρ‡ΠΈΠ²Π°ΡŽΡ‚ΡΡ Π½Π° Π΄Ρ€ΡƒΠ³ΠΎΠΉ. Π‘Π»Π΅Π΄ΠΎΠ²Π°Ρ‚Π΅Π»ΡŒΠ½ΠΎ, сторонниС заряды, Π²ΠΎΠ·Π½ΠΈΠΊΠ°ΡŽΡ‰ΠΈΠ΅ Π½Π° ΠΎΠ±ΠΊΠ»Π°Π΄ΠΊΠ°Ρ…, ΠΈΠΌΠ΅ΡŽΡ‚ ΠΎΠ΄ΠΈΠ½Π°ΠΊΠΎΠ²ΡƒΡŽ Π²Π΅Π»ΠΈΡ‡ΠΈΠ½Ρƒ ΠΈ Ρ€Π°Π·Π»ΠΈΡ‡Π½Ρ‹ ΠΏΠΎ Π·Π½Π°ΠΊΡƒ.

Основной характСристикой кондСнсатора являСтся Π΅Π³ΠΎ Π΅ΠΌΠΊΠΎΡΡ‚ΡŒ, ΠΏΠΎΠ΄ ΠΊΠΎΡ‚ΠΎΡ€ΠΎΠΉ ΠΏΠΎΠ½ΠΈΠΌΠ°ΡŽΡ‚ Π²Π΅Π»ΠΈΡ‡ΠΈΠ½Ρƒ, ΠΏΡ€ΠΎΠΏΠΎΡ€Ρ†ΠΈΠΎΠ½Π°Π»ΡŒΠ½ΡƒΡŽ заряду q ΠΈ ΠΎΠ±Ρ€Π°Ρ‚Π½ΠΎ ΠΏΡ€ΠΎΠΏΠΎΡ€Ρ†ΠΈΠΎΠ½Π°Π»ΡŒΠ½ΡƒΡŽ разности ΠΏΠΎΡ‚Π΅Π½Ρ†ΠΈΠ°Π»ΠΎΠ² ΠΌΠ΅ΠΆΠ΄Ρƒ ΠΎΠ±ΠΊΠ»Π°Π΄ΠΊΠ°ΠΌΠΈ:

Π Π°Π·Π½ΠΎΡΡ‚ΡŒ ΠΏΠΎΡ‚Π΅Π½Ρ†ΠΈΠ°Π»ΠΎΠ² Π½Π°Π·Ρ‹Π²Π°ΡŽΡ‚ напряТСниСм ΠΌΠ΅ΠΆΠ΄Ρƒ ΡΠΎΠΎΡ‚Π²Π΅Ρ‚ΡΡ‚Π²ΡƒΡŽΡ‰ΠΈΠΌΠΈ Ρ‚ΠΎΡ‡ΠΊΠ°ΠΌΠΈ. ΠœΡ‹ Π±ΡƒΠ΄Π΅ΠΌ ΠΎΠ±ΠΎΠ·Π½Π°Ρ‡Π°Ρ‚ΡŒ напряТСниС Π±ΡƒΠΊΠ²ΠΎΠΉ U.

Π’ΠΎΡΠΏΠΎΠ»ΡŒΠ·ΠΎΠ²Π°Π²ΡˆΠΈΡΡŒ этим ΠΎΠ±ΠΎΠ·Π½Π°Ρ‡Π΅Π½ΠΈΠ΅ΠΌ, ΠΌΠΎΠΆΠ½ΠΎ ΠΏΡ€ΠΈΠ΄Π°Ρ‚ΡŒ Ρ„ΠΎΡ€ΠΌΡƒΠ»Π΅ (27.1) Π²ΠΈΠ΄

Π—Π΄Π΅ΡΡŒ U — напряТСниС ΠΌΠ΅ΠΆΠ΄Ρƒ ΠΎΠ±ΠΊΠ»Π°Π΄ΠΊΠ°ΠΌΠΈ.

Π•ΠΌΠΊΠΎΡΡ‚ΡŒ кондСнсаторов измСряСтся Π² Ρ‚Π΅Ρ… ΠΆΠ΅ Π΅Π΄ΠΈΠ½ΠΈΡ†Π°Ρ…, Ρ‡Ρ‚ΠΎ ΠΈ Π΅ΠΌΠΊΠΎΡΡ‚ΡŒ ΡƒΠ΅Π΄ΠΈΠ½Π΅Π½Π½Ρ‹Ρ… ΠΏΡ€ΠΎΠ²ΠΎΠ΄Π½ΠΈΠΊΠΎΠ² (см. ΠΏΡ€Π΅Π΄Ρ‹Π΄ΡƒΡ‰ΠΈΠΉ ΠΏΠ°Ρ€Π°Π³Ρ€Π°Ρ„).

Π’Π΅Π»ΠΈΡ‡ΠΈΠ½Π° Смкости опрСдСляСтся Π³Π΅ΠΎΠΌΠ΅Ρ‚Ρ€ΠΈΠ΅ΠΉ кондСнсатора (Ρ„ΠΎΡ€ΠΌΠΎΠΉ ΠΈ Ρ€Π°Π·ΠΌΠ΅Ρ€Π°ΠΌΠΈ ΠΎΠ±ΠΊΠ»Π°Π΄ΠΎΠΊ ΠΈ Π²Π΅Π»ΠΈΡ‡ΠΈΠ½ΠΎΠΉ Π·Π°Π·ΠΎΡ€Π° ΠΌΠ΅ΠΆΠ΄Ρƒ Π½ΠΈΠΌΠΈ), Π° Ρ‚Π°ΠΊΠΆΠ΅ диэлСктричСскими свойствами срСды, Π·Π°ΠΏΠΎΠ»Π½ΡΡŽΡ‰Π΅ΠΉ пространство ΠΌΠ΅ΠΆΠ΄Ρƒ ΠΎΠ±ΠΊΠ»Π°Π΄ΠΊΠ°ΠΌΠΈ. НайдСм Ρ„ΠΎΡ€ΠΌΡƒΠ»Ρƒ для Смкости плоского кондСнсатора. Если ΠΏΠ»ΠΎΡ‰Π°Π΄ΡŒ ΠΎΠ±ΠΊΠ»Π°Π΄ΠΊΠΈ S, Π° заряд Π½Π° Π½Π΅ΠΉ q, Ρ‚ΠΎ Π½Π°ΠΏΡ€ΡΠΆΠ΅Π½Π½ΠΎΡΡ‚ΡŒ поля ΠΌΠ΅ΠΆΠ΄Ρƒ ΠΎΠ±ΠΊΠ»Π°Π΄ΠΊΠ°ΠΌΠΈ Ρ€Π°Π²Π½Π°

Π’ соотвСтствии с (8.6) Ρ€Π°Π·Π½ΠΎΡΡ‚ΡŒ ΠΏΠΎΡ‚Π΅Π½Ρ†ΠΈΠ°Π»ΠΎΠ² ΠΌΠ΅ΠΆΠ΄Ρƒ ΠΎΠ±ΠΊΠ»Π°Π΄ΠΊΠ°ΠΌΠΈ Ρ€Π°Π²Π½Π°

ΠžΡ‚ΡΡŽΠ΄Π° для Смкости плоского кондСнсатора получаСтся Ρ„ΠΎΡ€ΠΌΡƒΠ»Π°

Π³Π΄Π΅ S — ΠΏΠ»ΠΎΡ‰Π°Π΄ΡŒ ΠΎΠ±ΠΊΠ»Π°Π΄ΠΊΠΈ, d — Π²Π΅Π»ΠΈΡ‡ΠΈΠ½Π° Π·Π°Π·ΠΎΡ€Π° ΠΌΠ΅ΠΆΠ΄Ρƒ ΠΎΠ±ΠΊΠ»Π°Π΄ΠΊΠ°ΠΌΠΈ, — диэлСктричСская ΠΏΡ€ΠΎΠ½ΠΈΡ†Π°Π΅ΠΌΠΎΡΡ‚ΡŒ вСщСства, Π·Π°ΠΏΠΎΠ»Π½ΡΡŽΡ‰Π΅Π³ΠΎ Π·Π°Π·ΠΎΡ€.

ΠžΡ‚ΠΌΠ΅Ρ‚ΠΈΠΌ, Ρ‡Ρ‚ΠΎ Π΅ΠΌΠΊΠΎΡΡ‚ΡŒ Ρ€Π΅Π°Π»ΡŒΠ½ΠΎΠ³ΠΎ плоского кондСнсатора опрСдСляСтся Ρ„ΠΎΡ€ΠΌΡƒΠ»ΠΎΠΉ (27.3) с Ρ‚Π΅ΠΌ большСй Ρ‚ΠΎΡ‡Π½ΠΎΡΡ‚ΡŒΡŽ, Ρ‡Π΅ΠΌ мСньшС Π·Π°Π·ΠΎΡ€ d ΠΏΠΎ ΡΡ€Π°Π²Π½Π΅Π½ΠΈΡŽ с Π»ΠΈΠ½Π΅ΠΉΠ½Ρ‹ΠΌΠΈ Ρ€Π°Π·ΠΌΠ΅Ρ€Π°ΠΌΠΈ ΠΎΠ±ΠΊΠ»Π°Π΄ΠΎΠΊ.

Из Ρ„ΠΎΡ€ΠΌΡƒΠ»Ρ‹ (27.3) слСдуСт, Ρ‡Ρ‚ΠΎ Ρ€Π°Π·ΠΌΠ΅Ρ€Π½ΠΎΡΡ‚ΡŒ элСктричСской постоянной Ρ€Π°Π²Π½Π° размСрности Смкости, Π΄Π΅Π»Π΅Π½Π½ΠΎΠΉ Π½Π° Ρ€Π°Π·ΠΌΠ΅Ρ€Π½ΠΎΡΡ‚ΡŒ Π΄Π»ΠΈΠ½Ρ‹. Π’ соотвСтствии с этим измСряСтся Π² Ρ„Π°Ρ€Π°Π΄Π°Ρ… Π½Π° ΠΌΠ΅Ρ‚Ρ€ (см. (4.2)).

Если ΠΏΡ€Π΅Π½Π΅Π±Ρ€Π΅Ρ‡ΡŒ рассСяниСм поля Π²Π±Π»ΠΈΠ·ΠΈ ΠΊΡ€Π°Π΅Π² ΠΎΠ±ΠΊΠ»Π°Π΄ΠΎΠΊ, Π½Π΅Ρ‚Ρ€ΡƒΠ΄Π½ΠΎ ΠΏΠΎΠ»ΡƒΡ‡ΠΈΡ‚ΡŒ для Смкости цилиндричСского кондСнсатора Ρ„ΠΎΡ€ΠΌΡƒΠ»Ρƒ

Π³Π΄Π΅ l — Π΄Π»ΠΈΠ½Π° кондСнсатора, — радиусы Π²Π½ΡƒΡ‚Ρ€Π΅Π½Π½Π΅ΠΉ ΠΈ внСшнСй ΠΎΠ±ΠΊΠ»Π°Π΄ΠΎΠΊ.

Π­Ρ‚Π° Ρ„ΠΎΡ€ΠΌΡƒΠ»Π° опрСдСляСт Π΅ΠΌΠΊΠΎΡΡ‚ΡŒ Ρ€Π΅Π°Π»ΡŒΠ½ΠΎΠ³ΠΎ кондСнсатора с Ρ‚Π΅ΠΌ большСй Ρ‚ΠΎΡ‡Π½ΠΎΡΡ‚ΡŒΡŽ, Ρ‡Π΅ΠΌ мСньшС Π·Π°Π·ΠΎΡ€ ΠΌΠ΅ΠΆΠ΄Ρƒ ΠΎΠ±ΠΊΠ»Π°Π΄ΠΊΠ°ΠΌΠΈ ΠΏΠΎ ΡΡ€Π°Π²Π½Π΅Π½ΠΈΡŽ с .

Помимо Смкости ΠΊΠ°ΠΆΠ΄Ρ‹ΠΉ кондСнсатор характСризуСтся ΠΏΡ€Π΅Π΄Π΅Π»ΡŒΠ½Ρ‹ΠΌ напряТСниСм , ΠΊΠΎΡ‚ΠΎΡ€ΠΎΠ΅ ΠΌΠΎΠΆΠ½ΠΎ ΠΏΡ€ΠΈΠΊΠ»Π°Π΄Ρ‹Π²Π°Ρ‚ΡŒ ΠΊ ΠΎΠ±ΠΊΠ»Π°Π΄ΠΊΠ°ΠΌ кондСнсатора, Π½Π΅ опасаясь Π΅Π³ΠΎ пробоя. ΠŸΡ€ΠΈ ΠΏΡ€Π΅Π²Ρ‹ΡˆΠ΅Π½ΠΈΠΈ этого напряТСния ΠΌΠ΅ΠΆΠ΄Ρƒ ΠΎΠ±ΠΊΠ»Π°Π΄ΠΊΠ°ΠΌΠΈ проскакиваСт искра, Π² Ρ€Π΅Π·ΡƒΠ»ΡŒΡ‚Π°Ρ‚Π΅ Ρ‡Π΅Π³ΠΎ Ρ€Π°Π·Ρ€ΡƒΡˆΠ°Π΅Ρ‚ΡΡ диэлСктрик ΠΈ кондСнсатор Π²Ρ‹Ρ…ΠΎΠ΄ΠΈΡ‚ ΠΈΠ· строя.

ЭквивалСнтная Ρ‘ΠΌΠΊΠΎΡΡ‚ΡŒ кондСнсаторов

ЭквивалСнтная Ρ‘ΠΌΠΊΠΎΡΡ‚ΡŒ кондСнсаторов при ΠΏΠΎΡΠ»Π΅Π΄ΠΎΠ²Π°Ρ‚Π΅Π»ΡŒΠ½ΠΎΠΌ соСдинСнии
ΠΠ»ΡŒΡ‚Π΅Ρ€Π½Π°Ρ‚ΠΈΠ²Π½ΠΎΠ΅ ΠΎΡ‚ΠΎΠ±Ρ€Π°ΠΆΠ΅Π½ΠΈΠ΅
ЭквивалСнтная Ρ‘ΠΌΠΊΠΎΡΡ‚ΡŒ кондСнсаторов при ΠΏΠ°Ρ€Π°Π»Π΅Π»ΡŒΠ½ΠΎΠΌ соСдинСнии
ΠΠ»ΡŒΡ‚Π΅Ρ€Π½Π°Ρ‚ΠΈΠ²Π½ΠΎΠ΅ ΠΎΡ‚ΠΎΠ±Ρ€Π°ΠΆΠ΅Π½ΠΈΠ΅


ΠŸΠΎΡΠ»Π΅Π΄ΠΎΠ²Π°Ρ‚Π΅Π»ΡŒΠ½ΠΎΠ΅ соСдинСниС


Написав Π±ΠΎΡ‚, расчСта размСрностСй БистСма Π΅Π΄ΠΈΠ½ΠΈΡ† измСрСния ΠΎΠ½Π»Π°ΠΉΠ½Β Ρ‚Π΅ΠΏΠ΅Ρ€ΡŒ Π½Π°Ρ‡Π½Π΅ΠΌ ΠΎΡΠ²Π°ΠΈΠ²Π°Ρ‚ΡŒ Ρ‚Π°ΠΊΡƒΡŽ ΠΌΠ½ΠΎΠ³ΠΎΠ³Ρ€Π°Π½Π½ΡƒΡŽ ΠΈ ΡΠ»ΠΎΠΆΠ½ΡƒΡŽ ΠΎΠ±Π»Π°ΡΡ‚ΡŒ ΠΊΠ°ΠΊ элСктротСхника.


И ΠΏΠ΅Ρ€Π²ΠΎΠ΅, Ρ‡Ρ‚ΠΎ Β Π½Π°ΠΌ пригодится, это расчСт эквивалСнтных  характСристик  основных элСктричСских элСмСнтов( Ρ‘ΠΌΠΊΠΎΡΡ‚ΡŒ, ΠΈΠ½Π΄ΡƒΠΊΡ‚ΠΈΠ²Π½ΠΎΡΡ‚ΡŒ, сопротивлСниС).


Π₯ΠΎΡ‚Π΅Π»ΠΎΡΡŒ Π±Ρ‹ Π½Π°ΠΏΠΎΠΌΠ½ΠΈΡ‚ΡŒ, Ρ‡Ρ‚ΠΎ ёмкости Π½Π΅ΠΊΠΎΡ‚ΠΎΡ€Ρ‹Ρ… Ρ‚ΠΈΠΏΠΎΠ²Ρ‹Ρ… конструкций ΠΌΡ‹ ΡƒΠΆΠ΅ ΡƒΠΌΠ΅Π΅ΠΌ Ρ€Π°ΡΡΡ‡ΠΈΡ‚Ρ‹Π²Π°Ρ‚ΡŒΒ ΠΠΌΠΊΠΎΡΡ‚ΡŒ кондСнсатора ΠΎΠ½Π»Π°ΠΉΠ½


Π‘Π°ΠΌΠΈ ΠΏΠΎ сСбС Ρ„ΠΎΡ€ΠΌΡƒΠ»Ρ‹ ΠΎΡ‡Π΅Π½ΡŒ просты, Π½ΠΎ нюанс  состоит Π² Ρ‚ΠΎΠΌ, ΠΊΠ°ΠΊ Π½Π°ΠΌ Ρ‚ΠΎΡ‡Π½ΠΎ ΠΏΠΎΡΡ‡ΠΈΡ‚Π°Ρ‚ΡŒ Β Ρ‘ΠΌΠΊΠΎΡΡ‚ΡŒ Π΄Π²ΡƒΡ… ΠΏΠΎΡΠ»Π΅Π΄ΠΎΠ²Π°Ρ‚Π΅Π»ΡŒΠ½ΠΎ соСдинСнных кондСнсаторов  Ссли ΠΎΠ΄ΠΈΠ½ ΠΈΠ· кондСнсаторов ΠΈΠΌΠ΅Π΅Ρ‚ Ρ‘ΠΌΠΊΠΎΡΡ‚ΡŒ 10 ΠΏΠ€, Π° Π²Ρ‚ΠΎΡ€ΠΎΠΉ 250Π½Π€. Π Π°Π·ΠΌΠ΅Ρ€Π½ΠΎΡΡ‚ΡŒ ΠΏΠΎΠΊΠ°Π·Ρ‹Π²Π°Π΅Ρ‚ Ρ‡Ρ‚ΠΎ ΠΎΠ½ΠΈ Ρ€Π°Π·Π»ΠΈΡ‡Π°ΡŽΡ‚ΡΡ Π² 1000 Ρ€Π°Π·.  МоТно ΠΊΠΎΠ½Π΅Ρ‡Π½ΠΎ всС пСрСвСсти Π² Π°Π±ΡΠΎΠ»ΡŽΡ‚Π½Ρ‹Π΅ значСния, Π½ΠΎ это Β ΠΏΡ€ΠΈ большом количСствС кондСнсаторов способствуСт возникновСнию ошибок.


Π˜Ρ‚Π°ΠΊ, ΠΏΠΎΡΠ»Π΅Π΄ΠΎΠ²Π°Ρ‚Π΅Π»ΡŒΠ½ΠΎΠ΅ соСдинСниС кондСнсаторов ΠΈΠΌΠ΅Π΅Ρ‚ ΡΠ»Π΅Π΄ΡƒΡŽΡ‰ΠΈΠΉ Π²ΠΈΠ΄


И Ρ„ΠΎΡ€ΠΌΡƒΠ»Π° расчСта эквивалСнтной ёмкости  выглядит Ρ‚Π°ΠΊ


Β 


ΠŸΠ°Ρ€Π°Π»Π΅Π»ΡŒΠ½ΠΎΠ΅ Π‘ΠžΠ•Π”Π˜ΠΠ•ΠΠ˜Π•


ΠŸΠΎΡΠ»Π΅Π΄ΠΎΠ²Π°Ρ‚Π΅Π»ΡŒΠ½ΠΎΠ΅ соСдинСниС Π½Π΅ΡΠΊΠΎΠ»ΡŒΠ½ΠΈΡ… кондСнсаторов выглядит Ρ‚Π°ΠΊ, ΠΊΠ°ΠΊ ΠΏΠΎΠΊΠ°Π·Π°Π½ΠΎ Π½Π° рисункС


А Ρ„ΠΎΡ€ΠΌΡƒΠ»Π°  становится Π΄ΠΎ бСзобразия простой ΠΈ наглядной


Бинтаксис


Он ΠΎΡ‡Π΅Π½ΡŒ прост 


calc_e  список СмкостСй с размСрностями Ρ‡Π΅Ρ€Π΅Π· Π·Π°ΠΏΡΡ‚ΡƒΡŽ.


Π’ ΠΎΡ‚Π²Π΅Ρ‚Π΅ ΠΌΡ‹ ΠΏΠΎΠ»ΡƒΡ‡ΠΈΠΌ эквививалСнтноС Π·Π½Π°Ρ‡Π΅Π½ΠΈΠ΅  ёмкости ΠΏΡ€ΠΈ ΠΏΠΎΡΠ»Π΅Π΄ΠΎΠ²Π°Ρ‚Π΅Π»ΡŒΠ½ΠΎΠΌ ΠΈ ΠΏΠ°Ρ€Π°Π»Π΅Π»ΡŒΠ½Ρ‹Ρ… соСдинСниях.


Π’Π°ΠΆΠ½ΠΎΠ΅ Π·Π°ΠΌΠ΅Ρ‡Π°Π½ΠΈΠ΅: размСрности Π½ΡƒΠΆΠ½ΠΎ ΠΏΠΈΡΠ°Ρ‚ΡŒ Π½Π° русском языкС. Для ΠΏΠΎΠ»ΡŒΠ·ΠΎΠ²Π°Ρ‚Π΅Π»Π΅ΠΉ сайта, Π½Π΅ Π·Π½Π°ΡŽΡ‰ΠΈΡ… русский язык, ΠΌΠΎΠΆΠ΅ΠΌ ΠΏΠΎ запросу Π΄ΠΎΠ±Π°Π²ΠΈΡ‚ΡŒ ΠΎΠ±Ρ€Π°Π±ΠΎΡ‚ΠΊΡƒ англоязычных Π½Π°ΠΈΠΌΠ΅Π½ΠΎΠ²Π°Π½ΠΈΠΉ приставок ΠΈ размСрностСй. Π­Ρ‚ΠΎ Π½Π΅ слоТно.


ΠŸΡ€ΠΈΠΌΠ΅Ρ€Ρ‹


Π Π°ΡΡΡ‡ΠΈΡ‚Π°Ρ‚ΡŒ Ρ‘ΠΌΠΊΠΎΡΡ‚ΡŒ Ρ‚Ρ€Π΅Ρ… кондСнсаторов Β ΡΠ»Π΅Π΄ΡƒΡŽΡ‰ΠΈΡ… Π½ΠΎΠΌΠΈΠ½Π°Π»ΠΎΠ²: 10 ΠΏΠ€, 0.2Π½Π€ ΠΈ 344ΠΌΠ€


Π’ запросС Ρ‚Π°ΠΊ ΠΈ пишСм calc_e 10ΠΏΠ€,0.2Π½Π€,344ΠΌΠ€


ΠžΡ‚Π²Π΅Ρ‚ Π½Π΅ заставит сСбя Π΄ΠΎΠ»Π³ΠΎ ΠΆΠ΄Π°Ρ‚ΡŒ ΠΈ выглядит Π²ΠΎΡ‚ Ρ‚Π°ΠΊ










ЭквивалСнтная Ρ‘ΠΌΠΊΠΎΡΡ‚ΡŒ кондСнсаторов ΠΏΡ€ΠΈ ΠΏΠ°Ρ€Π°Π»Π΅Π»ΡŒΠ½ΠΎΠΌ соСдинСнии


9.5238095235459ΠΏΠΈΠΊΠΎΡ„Π°Ρ€Π°Π΄


ΠΠ»ΡŒΡ‚Π΅Ρ€Π½Π°Ρ‚ΠΈΠ²Π½ΠΎΠ΅ ΠΎΡ‚ΠΎΠ±Ρ€Π°ΠΆΠ΅Π½ΠΈΠ΅

ЭквивалСнтная Ρ‘ΠΌΠΊΠΎΡΡ‚ΡŒ кондСнсаторов ΠΏΡ€ΠΈ ΠΏΠΎΡΠ»Π΅Π΄ΠΎΠ²Π°Ρ‚Π΅Π»ΡŒΠ½ΠΎΠΌ соСдинСнии


344. 00000021ΠΌΠΈΠ»ΠΈΡ„Π°Ρ€Π°Π΄


ΠΠ»ΡŒΡ‚Π΅Ρ€Π½Π°Ρ‚ΠΈΠ²Π½ΠΎΠ΅ ΠΎΡ‚ΠΎΠ±Ρ€Π°ΠΆΠ΅Π½ΠΈΠ΅


Π£Π΄Π°Ρ‡ΠΈ Π² расчСтах!


Β 

  • Π­ΠΊΠ²ΠΈΠ²Π°Π»Π΅Π½Ρ‚Π½ΠΎΠ΅ сопротивлСниС ΠΎΠ½Π»Π°ΠΉΠ½ >>

Π΅ΠΌΠΊΠΎΡΡ‚ΡŒ, напряТСниС, Π½Π°ΠΏΡ€ΡΠΆΠ΅Π½Π½ΠΎΡΡ‚ΡŒ ΠΈ ΠΏΡ€ΠΎΡ‡Π΅Π΅

Π’ этой ΡΡ‚Π°Ρ‚ΡŒΠ΅ ΠΌΡ‹ Π½Π°Ρ‡Π½Π΅ΠΌ Ρ€Π°Π·Π±ΠΈΡ€Π°Ρ‚ΡŒ кондСнсаторы β€œΠΏΠΎ косточкам”. ΠœΡ‹ ΡƒΠ·Π½Π°Π΅ΠΌ, Β ΠΊΠ°ΠΊ зависит напряТСниС Π½Π° кондСнсаторС ΠΎΡ‚ расстояния ΠΌΠ΅ΠΆΠ΄Ρƒ пластин, Π² Ρ‡Π΅ΠΌ ΠΎΡ‚Π»ΠΈΡ‡ΠΈΠ΅ повСдСния кондСнсатора Π² случаях, ΠΊΠΎΠ³Π΄Π° ΠΎΠ½ ΠΏΠΎΠ΄ΠΊΠ»ΡŽΡ‡Π΅Π½ ΠΊ источнику ΠΈ ΠΊΠΎΠ³Π΄Π° Π½Π΅Ρ‚. Π’ ΠΏΠΎΡΠ»Π΅Π΄ΡƒΡŽΡ‰ΠΈΡ… ΡΡ‚Π°Ρ‚ΡŒΡΡ… – ΠΏΡ€ΠΎΠ΄ΠΎΠ»ΠΆΠ΅Π½ΠΈΠ΅.

Π—Π°Π΄Π°Ρ‡Π° 1. Найти Π΅ΠΌΠΊΠΎΡΡ‚ΡŒ сфСричСского кондСнсатора, состоящСго ΠΈΠ· Π΄Π²ΡƒΡ… концСнтричСских сфСр радиусами ΠΌ ΠΈ ΠΌ. ΠŸΡ€ΠΎΡΡ‚Ρ€Π°Π½ΡΡ‚Π²ΠΎ ΠΌΠ΅ΠΆΠ΄Ρƒ сфСрами Π·Π°ΠΏΠΎΠ»Π½Π΅Π½ΠΎ маслом. Какого радиуса Π΄ΠΎΠ»ΠΆΠ΅Π½Β Π±Ρ‹Ρ‚ΡŒ ΠΈΠ·ΠΎΠ»ΠΈΡ€ΠΎΠ²Π°Π½Π½Ρ‹ΠΉ ΡˆΠ°Ρ€, Ρ‡Ρ‚ΠΎΠ±Ρ‹ ΠΎΠ½ ΠΈΠΌΠ΅Π» Π΅ΠΌΠΊΠΎΡΡ‚ΡŒ, Ρ€Π°Π²Π½ΡƒΡŽ Смкости Ρ‚Π°ΠΊΠΎΠ³ΠΎ кондСнсатора?

Как извСстно,

Β  Β 

Π—Π°ΠΏΠΈΡˆΠ΅ΠΌ ΠΏΠΎΡ‚Π΅Π½Ρ†ΠΈΠ°Π»Ρ‹ сфСр:

Β  Β 

Β  Β 

Π Π°Π·Π½ΠΎΡΡ‚ΡŒ ΠΏΠΎΡ‚Π΅Π½Ρ†ΠΈΠ°Π»ΠΎΠ²:

Β  Β 

Π’ΠΎΠ³Π΄Π° Π΅ΠΌΠΊΠΎΡΡ‚ΡŒ кондСнсатора Ρ€Π°Π²Π½Π° (диэлСктричСская ΠΏΡ€ΠΎΠ½ΠΈΡ†Π°Π΅ΠΌΠΎΡΡ‚ΡŒ масла Ρ€Π°Π²Π½Π° ):

Β  Β 

А радиус ΡˆΠ°Ρ€Π° Π±Ρ‹Π» Π±Ρ‹ Ρ€Π°Π²Π΅Π½

Β  Β 

ΠžΡ‚Π²Π΅Ρ‚: ΠΏΠ€, ΠΌ.
Π—Π°Π΄Π°Ρ‡Π° 2. Найти Π΅ΠΌΠΊΠΎΡΡ‚ΡŒ плоского кондСнсатора, состоящСго ΠΈΠ· Π΄Π²ΡƒΡ…Β ΠΊΡ€ΡƒΠ³Π»Ρ‹Ρ… пластин Π΄ΠΈΠ°ΠΌΠ΅Ρ‚Ρ€ΠΎΠΌ см, Ρ€Π°Π·Π΄Π΅Π»Π΅Π½Π½Ρ‹Ρ… ΠΏΠ°Ρ€Π°Ρ„ΠΈΠ½ΠΎΠ²ΠΎΠΉ прослойкой Ρ‚ΠΎΠ»Ρ‰ΠΈΠ½ΠΎΠΉ ΠΌΠΌ.

ДиэлСктричСская ΠΏΡ€ΠΎΠ½ΠΈΡ†Π°Π΅ΠΌΠΎΡΡ‚ΡŒ ΠΏΠ°Ρ€Π°Ρ„ΠΈΠ½Π° .

По Ρ„ΠΎΡ€ΠΌΡƒΠ»Π΅

Β  Β 

ΠžΡ‚Π²Π΅Ρ‚: 556 ΠΌΠΊΠ€

Π—Π°Π΄Π°Ρ‡Π° 3. ΠŸΠ»ΠΎΡ‰Π°Π΄ΡŒ ΠΊΠ°ΠΆΠ΄ΠΎΠΉ пластины плоского кондСнсатора см. На ΠΊΠ°ΠΊΠΎΠΌ расстоянии Π΄Ρ€ΡƒΠ³ ΠΎΡ‚ Π΄Ρ€ΡƒΠ³Π° Π½Π°Π΄ΠΎ Ρ€Π°ΡΠΏΠΎΠ»ΠΎΠΆΠΈΡ‚ΡŒ Π² Π²ΠΎΠ·Π΄ΡƒΡ…Π΅ пластины, Ρ‡Ρ‚ΠΎΠ±Ρ‹ Π΅ΠΌΠΊΠΎΡΡ‚ΡŒ кондСнсатора Π±Ρ‹Π»Π° ΠΏΠ€?

ДиэлСктричСская ΠΏΡ€ΠΎΠ½ΠΈΡ†Π°Π΅ΠΌΠΎΡΡ‚ΡŒ Π²ΠΎΠ·Π΄ΡƒΡ…Π° .

Из Ρ„ΠΎΡ€ΠΌΡƒΠ»Ρ‹

Β  Β 

Β«Π²Ρ‹Ρ‚Π°Ρ‰ΠΈΠΌΒ» :

Β  Β 

ΠžΡ‚Π²Π΅Ρ‚: 1 см

Π—Π°Π΄Π°Ρ‡Π° 4. РасстояниС ΠΌΠ΅ΠΆΠ΄Ρƒ ΠΎΠ±ΠΊΠ»Π°Π΄ΠΊΠ°ΠΌΠΈ плоского кондСнсатора ΡƒΠ²Π΅Π»ΠΈΡ‡ΠΈΠ²Π°ΡŽΡ‚. Как измСнится: Π°) ΡΠ»Π΅ΠΊΡ‚Ρ€ΠΎΠ΅ΠΌΠΊΠΎΡΡ‚ΡŒ кондСнсатора; Π±) Π½Π°ΠΏΡ€ΡΠΆΠ΅Π½Π½ΠΎΡΡ‚ΡŒ элСктричСского поля; Π²) напряТСниС? Π Π°ΡΡΠΌΠΎΡ‚Ρ€Π΅Ρ‚ΡŒ Π΄Π²Π° случая: 1) кондСнсатор заряТСн ΠΈ ΠΎΡ‚ΠΊΠ»ΡŽΡ‡Π΅Π½ ΠΎΡ‚ источника Ρ‚ΠΎΠΊΠ°; 2) кондСнсатор ΠΏΠΎΠ΄ΠΊΠ»ΡŽΡ‡Π΅Π½ ΠΊ источнику Ρ‚ΠΎΠΊΠ°.

Π—Π΄Π΅ΡΡŒ Π½Π΅ΠΎΠ±Ρ…ΠΎΠ΄ΠΈΠΌΠΎ Π·Π°ΠΏΠΎΠΌΠ½ΠΈΡ‚ΡŒ: Ссли кондСнсатор заряТСн ΠΈ послС этого ΠΎΡ‚ΠΊΠ»ΡŽΡ‡Π΅Π½, Ρ‚ΠΎ заряд Π½Π° Π½Π΅ΠΌ сохраняСтся. Π”Π΅ΠΉΡΡ‚Π²ΠΈΡ‚Π΅Π»ΡŒΠ½ΠΎ, ΠΊΡƒΠ΄Π° Π΅ΠΌΡƒ Π΄Π΅Π²Π°Ρ‚ΡŒΡΡ? А Ссли Π½Π°Ρ‡Π°Ρ‚ΡŒ Ρ‡Ρ‚ΠΎ-Π»ΠΈΠ±ΠΎ ΠΌΠ΅Π½ΡΡ‚ΡŒ, Ρ‚ΠΎ Π±ΡƒΠ΄ΡƒΡ‚ ΠΌΠ΅Π½ΡΡ‚ΡŒΡΡ Π΅ΠΌΠΊΠΎΡΡ‚ΡŒ ΠΈ напряТСниС.

Если ΠΆΠ΅ кондСнсатор ΠΏΠΎΠ΄ΠΊΠ»ΡŽΡ‡Π΅Π½ ΠΊ источнику, Ρ‚ΠΎ напряТСниС Π½Π° Π½Π΅ΠΌ постоянно, ΠΈ ΠΏΡ€ΠΈ Π»ΡŽΠ±Ρ‹Ρ… Π²ΠΌΠ΅ΡˆΠ°Ρ‚Π΅Π»ΡŒΡΡ‚Π²Π°Ρ… (Ρ€Π°Π·Π΄Π²ΠΈΠ½ΡƒΠ»ΠΈ пластины, Π²Π»ΠΎΠΆΠΈΠ»ΠΈ диэлСктрик) Π±ΡƒΠ΄Π΅Ρ‚ ΠΌΠ΅Π½ΡΡ‚ΡŒΡΡ Π΅ΠΌΠΊΠΎΡΡ‚ΡŒ ΠΈ заряд.

Π’ΠΎΠ³Π΄Π° Π² ΠΏΠ΅Ρ€Π²ΠΎΠΌ случаС (заряд постоянСн!): Ρ‚Π°ΠΊ ΠΊΠ°ΠΊ Π·Π°Π²ΠΈΡΠΈΠΌΠΎΡΡ‚ΡŒ Смкости ΠΎΡ‚ обратная, Ρ‚ΠΎ Π΅ΠΌΠΊΠΎΡΡ‚ΡŒ Π±ΡƒΠ΄Π΅Ρ‚ ΠΏΠ°Π΄Π°Ρ‚ΡŒ ΠΏΡ€ΠΈ ΡƒΠ²Π΅Π»ΠΈΡ‡Π΅Π½ΠΈΠΈ расстояния ΠΌΠ΅ΠΆΠ΄Ρƒ пластинами. ΠΠ°ΠΏΡ€ΡΠΆΠ΅Π½Π½ΠΎΡΡ‚ΡŒ – Π½ΠΈΠΊΠ°ΠΊ Π½Π΅ зависит ΠΎΡ‚ расстояния ΠΌΠ΅ΠΆΠ΄Ρƒ ΠΎΠ±ΠΊΠ»Π°Π΄ΠΊΠ°ΠΌΠΈ, ΠΎΠ½Π° Π½Π΅ измСнится; напряТСниС – увСличится, ΠΎΠ½ΠΎ ΠΎΡ‚ Π²Π΅Π»ΠΈΡ‡ΠΈΠ½Ρ‹ зависит прямо.

Π’ΠΎ Π²Ρ‚ΠΎΡ€ΠΎΠΌ случаС (напряТСниС постоянно): Π½Π°ΠΏΡ€ΡΠΆΠ΅Π½Π½ΠΎΡΡ‚ΡŒ поля ΡƒΠΌΠ΅Π½ΡŒΡˆΠΈΡ‚ΡΡ; Π΅ΠΌΠΊΠΎΡΡ‚ΡŒ ΡƒΠΌΠ΅Π½ΡŒΡˆΠΈΡ‚ΡΡ.

Π—Π°Π΄Π°Ρ‡Π° 5. Плоский кондСнсатор состоит ΠΈΠ· Π΄Π²ΡƒΡ… пластин, ΠΏΠ»ΠΎΡ‰Π°Π΄ΡŒΡŽΒ  см каТдая, располоТСнных Π½Π° расстоянии ΠΌΠΌ Π΄Ρ€ΡƒΠ³ ΠΎΡ‚ Π΄Ρ€ΡƒΠ³Π°, ΠΌΠ΅ΠΆΠ΄Ρƒ ΠΊΠΎΡ‚ΠΎΡ€Ρ‹ΠΌΠΈ находится слой ΡΠ»ΡŽΠ΄Ρ‹. Какой наибольший заряд ΠΌΠΎΠΆΠ½ΠΎ ΡΠΎΠΎΠ±Ρ‰ΠΈΡ‚ΡŒ кондСнсатору, Ссли допустимоС напряТСниС ΠΊΠ’?

ДиэлСктричСская ΠΏΡ€ΠΎΠ½ΠΈΡ†Π°Π΅ΠΌΠΎΡΡ‚ΡŒ ΡΠ»ΡŽΠ΄Ρ‹ .

Β  Β 

Β  Β 

ΠžΡ‚Π²Π΅Ρ‚: 1,59 мкКл

Π—Π°Π΄Π°Ρ‡Π° 6.  Плоский Π²ΠΎΠ·Π΄ΡƒΡˆΠ½Ρ‹ΠΉ кондСнсатор, расстояниС ΠΌΠ΅ΠΆΠ΄Ρƒ пластинами ΠΊΠΎΡ‚ΠΎΡ€ΠΎΠ³ΠΎ ΠΌΠΌ, заряТСн Π΄ΠΎ напряТСния Π’ ΠΈ ΠΎΡ‚ΠΊΠ»ΡŽΡ‡Π΅Π½ ΠΎΡ‚ источника. Каким Π±ΡƒΠ΄Π΅Ρ‚ напряТСниС ,  Ссли пластины Ρ€Π°Π·Π΄Π²ΠΈΠ½ΡƒΡ‚ΡŒ Π΄ΠΎ расстояния ΠΌΠΌ?

Если кондСнсатор заряТСн ΠΈ послС этого ΠΎΡ‚ΠΊΠ»ΡŽΡ‡Π΅Π½, Ρ‚ΠΎ заряд Π½Π° Π½Π΅ΠΌ сохраняСтся. Π’ΠΎΠ³Π΄Π°

Β  Β 

Β  Β 

Β  Β 

ΠžΡ‚Π²Π΅Ρ‚: Π’

Π—Π°Π΄Π°Ρ‡Π° 7. Π‘ ΠΊΠ°ΠΊΠΎΠΉ силой Π²Π·Π°ΠΈΠΌΠΎΠ΄Π΅ΠΉΡΡ‚Π²ΡƒΡŽΡ‚ пластины плоского Π²ΠΎΠ·Π΄ΡƒΡˆΠ½ΠΎΠ³ΠΎ кондСнсатора ΠΏΠ»ΠΎΡ‰Π°Π΄ΡŒΡŽ ΠΌ, Ссли напряТСниС Π½Π° пластинах Π’ ΠΈ расстояниС ΠΌΠ΅ΠΆΠ΄Ρƒ Π½ΠΈΠΌΠΈ ΠΌ?

Π‘ΠΈΠ»Π° взаимодСйствия пластин ΠΌΠΎΠΆΠ΅Ρ‚ Π±Ρ‹Ρ‚ΡŒ вычислСна ΠΊΠ°ΠΊ ΠΏΡ€ΠΎΠΈΠ·Π²Π΅Π΄Π΅Π½ΠΈΠ΅ заряда пластины Π½Π° Π½Π°ΠΏΡ€ΡΠΆΠ΅Π½Π½ΠΎΡΡ‚ΡŒ поля пластины: – Π΄Π΅Π»ΠΈΠΌ ΠΏΠΎΠΏΠΎΠ»Π°ΠΌ, ΠΏΠΎΡ‚ΠΎΠΌΡƒ Ρ‡Ρ‚ΠΎ Π½Π°ΠΏΡ€ΡΠΆΠ΅Π½Π½ΠΎΡΡ‚ΡŒ поля ΠΎΠ΄Π½ΠΎΠΉ пластины Π²Π΄Π²ΠΎΠ΅ мСньшС напряТСнности поля кондСнсатора – Ρ‚Π°ΠΌ пластин Π΄Π²Π΅ ΡˆΡ‚ΡƒΠΊΠΈ.

Β  Β 

Β  Β 

ΠžΡ‚Π²Π΅Ρ‚: мН.

Π•ΠΌΠΊΠΎΡΡ‚ΡŒ кондСнсатора Ρ„ΠΎΡ€ΠΌΡƒΠ»Π° Ρ‡Π΅Ρ€Π΅Π· напряТСниС ΠΈ заряд

По Π½Π°Π·Π½Π°Ρ‡Π΅Π½ΠΈΡŽ кондСнсатор ΠΌΠΎΠΆΠ½ΠΎ ΡΡ€Π°Π²Π½ΠΈΡ‚ΡŒ с Π±Π°Ρ‚Π°Ρ€Π΅ΠΉΠΊΠΎΠΉ. Но имССтся ΠΏΡ€ΠΈΠ½Ρ†ΠΈΠΏΠΈΠ°Π»ΡŒΠ½ΠΎΠ΅ ΠΎΡ‚Π»ΠΈΡ‡ΠΈΠ΅ Π² Ρ€Π°Π±ΠΎΡ‚Π΅ Π΄Π°Π½Π½Ρ‹Ρ… элСмСнтов. Π‘ΡƒΡ‰Π΅ΡΡ‚Π²ΡƒΡŽΡ‚ отличия Π² ΠΏΡ€Π΅Π΄Π΅Π»ΡŒΠ½ΠΎΠΉ Смкости ΠΈ скорости зарядки кондСнсатора ΠΈ Π±Π°Ρ‚Π°Ρ€Π΅ΠΉΠΊΠΈ.

Π€ΠΎΡ€ΠΌΡƒΠ»Π° заряда кондСнсатора

Π’Π΅Π»ΠΈΡ‡ΠΈΠ½Π° заряда кондСнсатора (q) связана с Π΅Π³ΠΎ Π΅ΠΌΠΊΠΎΡΡ‚ΡŒΡŽ (C) ΠΈ Ρ€Π°Π·Π½ΠΎΡΡ‚ΡŒΡŽ ΠΏΠΎΡ‚Π΅Π½Ρ†ΠΈΠ°Π»ΠΎΠ² (U) ΠΌΠ΅ΠΆΠ΄Ρƒ Π΅Π³ΠΎ ΠΎΠ±ΠΊΠ»Π°Π΄ΠΊΠ°ΠΌΠΈ ΠΊΠ°ΠΊ:

Π³Π΄Π΅ q – Π²Π΅Π»ΠΈΡ‡ΠΈΠ½Π° заряда ΠΎΠ΄Π½ΠΎΠΉ ΠΈΠ· ΠΎΠ±ΠΊΠ»Π°Π΄ΠΎΠΊ кондСнсатора, Π° – Ρ€Π°Π·Π½ΠΎΡΡ‚ΡŒ ΠΏΠΎΡ‚Π΅Π½Ρ†ΠΈΠ°Π»ΠΎΠ² ΠΌΠ΅ΠΆΠ΄Ρƒ Π΅Π³ΠΎ ΠΎΠ±ΠΊΠ»Π°Π΄ΠΊΠ°ΠΌΠΈ.

Π­Π»Π΅ΠΊΡ‚Ρ€ΠΎΠ΅ΠΌΠΊΠΎΡΡ‚ΡŒ кондСнсатора β€” это Π²Π΅Π»ΠΈΡ‡ΠΈΠ½Π°, которая зависит Ρ‚ΠΎ Ρ€Π°Π·ΠΌΠ΅Ρ€ΠΎΠ² ΠΈ устройства кондСнсатора.

Заряд Π½Π° пластинах плоского кондСнсатора Ρ€Π°Π²Π΅Π½:

Π³Π΄Π΅ – элСктричСская постоянная; – ΠΏΠ»ΠΎΡ‰Π°Π΄ΡŒ ΠΊΠ°ΠΆΠ΄ΠΎΠΉ (ΠΈΠ»ΠΈ наимСньшСй) пластины; – расстояниС ΠΌΠ΅ΠΆΠ΄Ρƒ пластинами; – диэлСктричСская ΠΏΡ€ΠΎΠ½ΠΈΡ†Π°Π΅ΠΌΠΎΡΡ‚ΡŒ диэлСктрика, ΠΊΠΎΡ‚ΠΎΡ€Ρ‹ΠΉ находится ΠΌΠ΅ΠΆΠ΄Ρƒ пластинами кондСнсатора.

Заряд Π½Π° ΠΎΠ±ΠΊΠ»Π°Π΄ΠΊΠ°Ρ… цилиндричСского кондСнсатора вычисляСтся ΠΏΡ€ΠΈ ΠΏΠΎΠΌΠΎΡ‰ΠΈ Ρ„ΠΎΡ€ΠΌΡƒΠ»Ρ‹:

Π³Π΄Π΅ l – высота Ρ†ΠΈΠ»ΠΈΠ½Π΄Ρ€ΠΎΠ²; – радиус внСшнСй ΠΎΠ±ΠΊΠ»Π°Π΄ΠΊΠΈ; – радиус Π²Π½ΡƒΡ‚Ρ€Π΅Π½Π½Π΅ΠΉ ΠΎΠ±ΠΊΠ»Π°Π΄ΠΊΠΈ.

Заряд Π½Π° ΠΎΠ±ΠΊΠ»Π°Π΄ΠΊΠ°Ρ… сфСричСского кондСнсатора Π½Π°ΠΉΠ΄Π΅ΠΌ ΠΊΠ°ΠΊ:

Π³Π΄Π΅ – радиусы ΠΎΠ±ΠΊΠ»Π°Π΄ΠΎΠΊ кондСнсатора.

Заряд кондСнсатора связан с энСргиСй поля (W) Π²Π½ΡƒΡ‚Ρ€ΠΈ Π½Π΅Π³ΠΎ:

Из Ρ„ΠΎΡ€ΠΌΡƒΠ»Ρ‹ (6) слСдуСт, Ρ‡Ρ‚ΠΎ заряд ΠΌΠΎΠΆΠ½ΠΎ Π²Ρ‹Ρ€Π°Π·ΠΈΡ‚ΡŒ ΠΊΠ°ΠΊ:

Рассмотрим ΠΏΠΎΡΠ»Π΅Π΄ΠΎΠ²Π°Ρ‚Π΅Π»ΡŒΠ½ΠΎΠ΅ соСдинСниС ΠΈΠ· N кондСнсаторов ( рис. 1).

Π—Π΄Π΅ΡΡŒ (рис.1) ΠΏΠΎΠ»ΠΎΠΆΠΈΡ‚Π΅Π»ΡŒΠ½Π°Ρ ΠΎΠ±ΠΊΠ»Π°Π΄ΠΊΠ° ΠΎΠ΄Π½ΠΎΠ³ΠΎ кондСнсатора соСдиняСтся с ΠΎΡ‚Ρ€ΠΈΡ†Π°Ρ‚Π΅Π»ΡŒΠ½ΠΎΠΉ ΠΎΠ±ΠΊΠ»Π°Π΄ΠΊΠΎΠΉ ΡΠ»Π΅Π΄ΡƒΡŽΡ‰Π΅Π³ΠΎ кондСнсатора. ΠŸΡ€ΠΈ Ρ‚Π°ΠΊΠΎΠΌ соСдинСнии, ΠΎΠ±ΠΊΠ»Π°Π΄ΠΊΠΈ сосСдних кондСнсаторов ΡΠΎΠ·Π΄Π°ΡŽΡ‚ Π΅Π΄ΠΈΠ½Ρ‹ΠΉ ΠΏΡ€ΠΎΠ²ΠΎΠ΄Π½ΠΈΠΊ. Π£ всСх кондСнсаторов, соСдинСнных ΠΏΠΎΡΠ»Π΅Π΄ΠΎΠ²Π°Ρ‚Π΅Π»ΡŒΠ½ΠΎ Π½Π° ΠΎΠ±ΠΊΠ»Π°Π΄ΠΊΠ°Ρ… ΠΈΠΌΠ΅ΡŽΡ‚ΡΡ Ρ€Π°Π²Π½Ρ‹Π΅ ΠΏΠΎ Π²Π΅Π»ΠΈΡ‡ΠΈΠ½Π΅ заряды.

ΠŸΡ€ΠΈ ΠΏΠ°Ρ€Π°Π»Π»Π΅Π»ΡŒΠ½ΠΎΠΌ соСдинСнии кондСнсаторов (рис.2), ΡΠΎΠ΅Π΄ΠΈΠ½ΡΡŽΡ‚ ΠΎΠ±ΠΊΠ»Π°Π΄ΠΊΠΈ, ΠΈΠΌΠ΅ΡŽΡ‰ΠΈΠ΅ заряды ΠΎΠ΄Π½ΠΎΠ³ΠΎ Π·Π½Π°ΠΊΠ°. Π‘ΡƒΠΌΠΌΠ°Ρ€Π½Ρ‹ΠΉ заряд соСдинСния (q) Ρ€Π°Π²Π΅Π½ суммС зарядов кондСнсаторов.

ΠŸΡ€ΠΈΠΌΠ΅Ρ€Ρ‹ Ρ€Π΅ΡˆΠ΅Π½ΠΈΡ Π·Π°Π΄Π°Ρ‡ ΠΏΠΎ Ρ‚Π΅ΠΌΠ΅ «Заряд кондСнсатора»

Π—Π°Π΄Π°Π½ΠΈΠ΅ΠšΠ°ΠΊΠΎΠ²Ρ‹ заряды Π½Π° ΠΎΠ±ΠΊΠ»Π°Π΄ΠΊΠ°Ρ… кондСнсаторов, Ссли ΠΎΠ½ΠΈ ΠΈΠΌΠ΅ΡŽΡ‚ Смкости Π€ ΠΈ Π€, соСдинСны ΠΏΠΎΡΠ»Π΅Π΄ΠΎΠ²Π°Ρ‚Π΅Π»ΡŒΠ½ΠΎ ΠΈ присоСдинСны ΠΊ Π±Π°Ρ‚Π°Ρ€Π΅Π΅ с Π­Π”Π‘ Ρ€Π°Π²Π½ΠΎΠΉ Π’ (рис.3)? Π§Π΅ΠΌΡƒ Ρ€Π°Π²Π΅Π½ суммарный заряд соСдинСния?
Π Π΅ΡˆΠ΅Π½ΠΈΠ΅Π Π°Π·Π½ΠΎΡΡ‚ΠΈ ΠΏΠΎΡ‚Π΅Π½Ρ†ΠΈΠ°Π»ΠΎΠ² Π½Π° ΠΎΠ±ΠΊΠ»Π°Π΄ΠΊΠ°Ρ… кондСнсаторов Π±ΡƒΠ΄ΡƒΡ‚ ΠΏΡ€ΠΈ Ρ‚Π°ΠΊΠΎΠΌ соСдинСнии Ρ€Π°Π²Π½Ρ‹:

Заряд Π½Π° ΠΏΠ΅Ρ€Π²ΠΎΠΌ кондСнсаторС ΠΏΡ€ΠΈ этом Ρ€Π°Π²Π΅Π½:

Заряд Π½Π° ΠΎΠ±ΠΊΠ»Π°Π΄ΠΊΠ°Ρ… Π²Ρ‚ΠΎΡ€ΠΎΠ³ΠΎ кондСнсатора:

Π‘ΡƒΠΌΠΌΠ°Ρ€Π½Ρ‹ΠΉ заряд систСмы ΠΌΠΎΠΆΠ½ΠΎ Π½Π°ΠΉΡ‚ΠΈ ΠΊΠ°ΠΊ:

Π’ΠΎΠ³Π΄Π° суммарный заряд Ρ€Π°Π²Π΅Π½:

ΠžΡ‚Π²Π΅Ρ‚ Кл; Кл; Кл
Π—Π°Π΄Π°Π½ΠΈΠ΅Π•ΠΌΠΊΠΎΡΡ‚ΡŒ пускового устройства элСктричСского двигатСля Ρ€Π°Π²Π½Π° C. Π­Π½Π΅Ρ€Π³ΠΈΠΈ ΠΈΠΌΠ΅ΡŽΡ‰Π΅ΠΉΡΡ Π² кондСнсаторС достаточно для Ρ‚ΠΎΠ³ΠΎ Ρ‡Ρ‚ΠΎΠ±Ρ‹ ΠΏΠΎΠ΄Π½ΡΡ‚ΡŒ Π³Ρ€ΡƒΠ· массы m Π½Π° высоту h. Π§Π΅ΠΌΡƒ Ρ€Π°Π²Π΅Π½ заряд кондСнсатора?
Π Π΅ΡˆΠ΅Π½ΠΈΠ΅ΠŸΡ€ΠΈ поднятии Π³Ρ€ΡƒΠ·Π° Π½Π° высоту h происходит ΠΏΠ΅Ρ€Π΅Ρ…ΠΎΠ΄ энСргии поля кондСнсатора () Π² ΠΏΠΎΡ‚Π΅Π½Ρ†ΠΈΠ°Π»ΡŒΠ½ΡƒΡŽ ΡΠ½Π΅Ρ€Π³ΠΈΡŽ Ρ‚Π΅Π»Π° (), поднятого Π½Π°Π΄ Π—Π΅ΠΌΠ»Π΅ΠΉ, поэтому запишСм:

Π­Π½Π΅Ρ€Π³ΠΈΡŽ Π½Π°ΠΉΠ΄Π΅ΠΌ ΠΊΠ°ΠΊ:

Π­Π½Π΅Ρ€Π³ΠΈΡŽ элСктричСского поля кондСнсатора Π±ΡƒΠ΄Π΅Ρ‚ ΡƒΠ΄ΠΎΠ±Π½Π΅Π΅ Π²Ρ‹Ρ€Π°Π·ΠΈΡ‚ΡŒ:

ΠŸΠΎΠ΄ΡΡ‚Π°Π²ΠΈΠΌ Π² Π²Ρ‹Ρ€Π°ΠΆΠ΅Π½ΠΈΠ΅ (2.1) ΠΏΡ€Π°Π²Ρ‹Π΅ части (2.2) ΠΈ (2.3), ΠΈΠΌΠ΅Π΅ΠΌ:

По Π½Π°Π·Π½Π°Ρ‡Π΅Π½ΠΈΡŽ кондСнсатор ΠΌΠΎΠΆΠ½ΠΎ ΡΡ€Π°Π²Π½ΠΈΡ‚ΡŒ с Π±Π°Ρ‚Π°Ρ€Π΅ΠΉΠΊΠΎΠΉ. Но имССтся ΠΏΡ€ΠΈΠ½Ρ†ΠΈΠΏΠΈΠ°Π»ΡŒΠ½ΠΎΠ΅ ΠΎΡ‚Π»ΠΈΡ‡ΠΈΠ΅ Π² Ρ€Π°Π±ΠΎΡ‚Π΅ Π΄Π°Π½Π½Ρ‹Ρ… элСмСнтов. Π‘ΡƒΡ‰Π΅ΡΡ‚Π²ΡƒΡŽΡ‚ отличия Π² ΠΏΡ€Π΅Π΄Π΅Π»ΡŒΠ½ΠΎΠΉ Смкости ΠΈ скорости зарядки кондСнсатора ΠΈ Π±Π°Ρ‚Π°Ρ€Π΅ΠΉΠΊΠΈ.

Π€ΠΎΡ€ΠΌΡƒΠ»Π° заряда кондСнсатора

Π’Π΅Π»ΠΈΡ‡ΠΈΠ½Π° заряда кондСнсатора (q) связана с Π΅Π³ΠΎ Π΅ΠΌΠΊΠΎΡΡ‚ΡŒΡŽ (C) ΠΈ Ρ€Π°Π·Π½ΠΎΡΡ‚ΡŒΡŽ ΠΏΠΎΡ‚Π΅Π½Ρ†ΠΈΠ°Π»ΠΎΠ² (U) ΠΌΠ΅ΠΆΠ΄Ρƒ Π΅Π³ΠΎ ΠΎΠ±ΠΊΠ»Π°Π΄ΠΊΠ°ΠΌΠΈ ΠΊΠ°ΠΊ:

Π³Π΄Π΅ q – Π²Π΅Π»ΠΈΡ‡ΠΈΠ½Π° заряда ΠΎΠ΄Π½ΠΎΠΉ ΠΈΠ· ΠΎΠ±ΠΊΠ»Π°Π΄ΠΎΠΊ кондСнсатора, Π° – Ρ€Π°Π·Π½ΠΎΡΡ‚ΡŒ ΠΏΠΎΡ‚Π΅Π½Ρ†ΠΈΠ°Π»ΠΎΠ² ΠΌΠ΅ΠΆΠ΄Ρƒ Π΅Π³ΠΎ ΠΎΠ±ΠΊΠ»Π°Π΄ΠΊΠ°ΠΌΠΈ.

Π­Π»Π΅ΠΊΡ‚Ρ€ΠΎΠ΅ΠΌΠΊΠΎΡΡ‚ΡŒ кондСнсатора β€” это Π²Π΅Π»ΠΈΡ‡ΠΈΠ½Π°, которая зависит Ρ‚ΠΎ Ρ€Π°Π·ΠΌΠ΅Ρ€ΠΎΠ² ΠΈ устройства кондСнсатора.

Заряд Π½Π° пластинах плоского кондСнсатора Ρ€Π°Π²Π΅Π½:

Π³Π΄Π΅ – элСктричСская постоянная; – ΠΏΠ»ΠΎΡ‰Π°Π΄ΡŒ ΠΊΠ°ΠΆΠ΄ΠΎΠΉ (ΠΈΠ»ΠΈ наимСньшСй) пластины; – расстояниС ΠΌΠ΅ΠΆΠ΄Ρƒ пластинами; – диэлСктричСская ΠΏΡ€ΠΎΠ½ΠΈΡ†Π°Π΅ΠΌΠΎΡΡ‚ΡŒ диэлСктрика, ΠΊΠΎΡ‚ΠΎΡ€Ρ‹ΠΉ находится ΠΌΠ΅ΠΆΠ΄Ρƒ пластинами кондСнсатора.

Заряд Π½Π° ΠΎΠ±ΠΊΠ»Π°Π΄ΠΊΠ°Ρ… цилиндричСского кондСнсатора вычисляСтся ΠΏΡ€ΠΈ ΠΏΠΎΠΌΠΎΡ‰ΠΈ Ρ„ΠΎΡ€ΠΌΡƒΠ»Ρ‹:

Π³Π΄Π΅ l – высота Ρ†ΠΈΠ»ΠΈΠ½Π΄Ρ€ΠΎΠ²; – радиус внСшнСй ΠΎΠ±ΠΊΠ»Π°Π΄ΠΊΠΈ; – радиус Π²Π½ΡƒΡ‚Ρ€Π΅Π½Π½Π΅ΠΉ ΠΎΠ±ΠΊΠ»Π°Π΄ΠΊΠΈ.

Заряд Π½Π° ΠΎΠ±ΠΊΠ»Π°Π΄ΠΊΠ°Ρ… сфСричСского кондСнсатора Π½Π°ΠΉΠ΄Π΅ΠΌ ΠΊΠ°ΠΊ:

Π³Π΄Π΅ – радиусы ΠΎΠ±ΠΊΠ»Π°Π΄ΠΎΠΊ кондСнсатора.

Заряд кондСнсатора связан с энСргиСй поля (W) Π²Π½ΡƒΡ‚Ρ€ΠΈ Π½Π΅Π³ΠΎ:

Из Ρ„ΠΎΡ€ΠΌΡƒΠ»Ρ‹ (6) слСдуСт, Ρ‡Ρ‚ΠΎ заряд ΠΌΠΎΠΆΠ½ΠΎ Π²Ρ‹Ρ€Π°Π·ΠΈΡ‚ΡŒ ΠΊΠ°ΠΊ:

Рассмотрим ΠΏΠΎΡΠ»Π΅Π΄ΠΎΠ²Π°Ρ‚Π΅Π»ΡŒΠ½ΠΎΠ΅ соСдинСниС ΠΈΠ· N кондСнсаторов ( рис. 1).

Π—Π΄Π΅ΡΡŒ (рис.1) ΠΏΠΎΠ»ΠΎΠΆΠΈΡ‚Π΅Π»ΡŒΠ½Π°Ρ ΠΎΠ±ΠΊΠ»Π°Π΄ΠΊΠ° ΠΎΠ΄Π½ΠΎΠ³ΠΎ кондСнсатора соСдиняСтся с ΠΎΡ‚Ρ€ΠΈΡ†Π°Ρ‚Π΅Π»ΡŒΠ½ΠΎΠΉ ΠΎΠ±ΠΊΠ»Π°Π΄ΠΊΠΎΠΉ ΡΠ»Π΅Π΄ΡƒΡŽΡ‰Π΅Π³ΠΎ кондСнсатора. ΠŸΡ€ΠΈ Ρ‚Π°ΠΊΠΎΠΌ соСдинСнии, ΠΎΠ±ΠΊΠ»Π°Π΄ΠΊΠΈ сосСдних кондСнсаторов ΡΠΎΠ·Π΄Π°ΡŽΡ‚ Π΅Π΄ΠΈΠ½Ρ‹ΠΉ ΠΏΡ€ΠΎΠ²ΠΎΠ΄Π½ΠΈΠΊ. Π£ всСх кондСнсаторов, соСдинСнных ΠΏΠΎΡΠ»Π΅Π΄ΠΎΠ²Π°Ρ‚Π΅Π»ΡŒΠ½ΠΎ Π½Π° ΠΎΠ±ΠΊΠ»Π°Π΄ΠΊΠ°Ρ… ΠΈΠΌΠ΅ΡŽΡ‚ΡΡ Ρ€Π°Π²Π½Ρ‹Π΅ ΠΏΠΎ Π²Π΅Π»ΠΈΡ‡ΠΈΠ½Π΅ заряды.

ΠŸΡ€ΠΈ ΠΏΠ°Ρ€Π°Π»Π»Π΅Π»ΡŒΠ½ΠΎΠΌ соСдинСнии кондСнсаторов (рис.2), ΡΠΎΠ΅Π΄ΠΈΠ½ΡΡŽΡ‚ ΠΎΠ±ΠΊΠ»Π°Π΄ΠΊΠΈ, ΠΈΠΌΠ΅ΡŽΡ‰ΠΈΠ΅ заряды ΠΎΠ΄Π½ΠΎΠ³ΠΎ Π·Π½Π°ΠΊΠ°. Π‘ΡƒΠΌΠΌΠ°Ρ€Π½Ρ‹ΠΉ заряд соСдинСния (q) Ρ€Π°Π²Π΅Π½ суммС зарядов кондСнсаторов.

ΠŸΡ€ΠΈΠΌΠ΅Ρ€Ρ‹ Ρ€Π΅ΡˆΠ΅Π½ΠΈΡ Π·Π°Π΄Π°Ρ‡ ΠΏΠΎ Ρ‚Π΅ΠΌΠ΅ «Заряд кондСнсатора»

Π—Π°Π΄Π°Π½ΠΈΠ΅ΠšΠ°ΠΊΠΎΠ²Ρ‹ заряды Π½Π° ΠΎΠ±ΠΊΠ»Π°Π΄ΠΊΠ°Ρ… кондСнсаторов, Ссли ΠΎΠ½ΠΈ ΠΈΠΌΠ΅ΡŽΡ‚ Смкости Π€ ΠΈ Π€, соСдинСны ΠΏΠΎΡΠ»Π΅Π΄ΠΎΠ²Π°Ρ‚Π΅Π»ΡŒΠ½ΠΎ ΠΈ присоСдинСны ΠΊ Π±Π°Ρ‚Π°Ρ€Π΅Π΅ с Π­Π”Π‘ Ρ€Π°Π²Π½ΠΎΠΉ Π’ (рис.3)? Π§Π΅ΠΌΡƒ Ρ€Π°Π²Π΅Π½ суммарный заряд соСдинСния?
Π Π΅ΡˆΠ΅Π½ΠΈΠ΅Π Π°Π·Π½ΠΎΡΡ‚ΠΈ ΠΏΠΎΡ‚Π΅Π½Ρ†ΠΈΠ°Π»ΠΎΠ² Π½Π° ΠΎΠ±ΠΊΠ»Π°Π΄ΠΊΠ°Ρ… кондСнсаторов Π±ΡƒΠ΄ΡƒΡ‚ ΠΏΡ€ΠΈ Ρ‚Π°ΠΊΠΎΠΌ соСдинСнии Ρ€Π°Π²Π½Ρ‹:

Заряд Π½Π° ΠΏΠ΅Ρ€Π²ΠΎΠΌ кондСнсаторС ΠΏΡ€ΠΈ этом Ρ€Π°Π²Π΅Π½:

Заряд Π½Π° ΠΎΠ±ΠΊΠ»Π°Π΄ΠΊΠ°Ρ… Π²Ρ‚ΠΎΡ€ΠΎΠ³ΠΎ кондСнсатора:

Π‘ΡƒΠΌΠΌΠ°Ρ€Π½Ρ‹ΠΉ заряд систСмы ΠΌΠΎΠΆΠ½ΠΎ Π½Π°ΠΉΡ‚ΠΈ ΠΊΠ°ΠΊ:

Π’ΠΎΠ³Π΄Π° суммарный заряд Ρ€Π°Π²Π΅Π½:

ΠžΡ‚Π²Π΅Ρ‚ Кл; Кл; Кл
Π—Π°Π΄Π°Π½ΠΈΠ΅Π•ΠΌΠΊΠΎΡΡ‚ΡŒ пускового устройства элСктричСского двигатСля Ρ€Π°Π²Π½Π° C. Π­Π½Π΅Ρ€Π³ΠΈΠΈ ΠΈΠΌΠ΅ΡŽΡ‰Π΅ΠΉΡΡ Π² кондСнсаторС достаточно для Ρ‚ΠΎΠ³ΠΎ Ρ‡Ρ‚ΠΎΠ±Ρ‹ ΠΏΠΎΠ΄Π½ΡΡ‚ΡŒ Π³Ρ€ΡƒΠ· массы m Π½Π° высоту h. Π§Π΅ΠΌΡƒ Ρ€Π°Π²Π΅Π½ заряд кондСнсатора?
Π Π΅ΡˆΠ΅Π½ΠΈΠ΅ΠŸΡ€ΠΈ поднятии Π³Ρ€ΡƒΠ·Π° Π½Π° высоту h происходит ΠΏΠ΅Ρ€Π΅Ρ…ΠΎΠ΄ энСргии поля кондСнсатора () Π² ΠΏΠΎΡ‚Π΅Π½Ρ†ΠΈΠ°Π»ΡŒΠ½ΡƒΡŽ ΡΠ½Π΅Ρ€Π³ΠΈΡŽ Ρ‚Π΅Π»Π° (), поднятого Π½Π°Π΄ Π—Π΅ΠΌΠ»Π΅ΠΉ, поэтому запишСм:

Π­Π½Π΅Ρ€Π³ΠΈΡŽ Π½Π°ΠΉΠ΄Π΅ΠΌ ΠΊΠ°ΠΊ:

Π­Π½Π΅Ρ€Π³ΠΈΡŽ элСктричСского поля кондСнсатора Π±ΡƒΠ΄Π΅Ρ‚ ΡƒΠ΄ΠΎΠ±Π½Π΅Π΅ Π²Ρ‹Ρ€Π°Π·ΠΈΡ‚ΡŒ:

ΠŸΠΎΠ΄ΡΡ‚Π°Π²ΠΈΠΌ Π² Π²Ρ‹Ρ€Π°ΠΆΠ΅Π½ΠΈΠ΅ (2.1) ΠΏΡ€Π°Π²Ρ‹Π΅ части (2.2) ΠΈ (2.3), ΠΈΠΌΠ΅Π΅ΠΌ:

Одним ΠΈΠ· Π²Π°ΠΆΠ½Ρ‹Ρ… элСмСнтов элСктричСской Ρ†Π΅ΠΏΠΈ являСтся кондСнсатор, Ρ„ΠΎΡ€ΠΌΡƒΠ»Ρ‹ для ΠΊΠΎΡ‚ΠΎΡ€ΠΎΠ³ΠΎ ΠΏΠΎΠ·Π²ΠΎΠ»ΡΡŽΡ‚ Ρ€Π°ΡΡΡ‡ΠΈΡ‚Π°Ρ‚ΡŒ ΠΈ ΠΏΠΎΠ΄ΠΎΠ±Ρ€Π°Ρ‚ΡŒ Π½Π°ΠΈΠ±ΠΎΠ»Π΅Π΅ подходящий Π²Π°Ρ€ΠΈΠ°Π½Ρ‚. Основная функция Π΄Π°Π½Π½ΠΎΠ³ΠΎ устройства Π·Π°ΠΊΠ»ΡŽΡ‡Π°Π΅Ρ‚ΡΡ Π² Π½Π°ΠΊΠΎΠΏΠ»Π΅Π½ΠΈΠΈ ΠΎΠΏΡ€Π΅Π΄Π΅Π»Π΅Π½Π½ΠΎΠ³ΠΎ количСства элСктроэнСргии. ΠŸΡ€ΠΎΡΡ‚Π΅ΠΉΡˆΠ°Ρ систСма Π²ΠΊΠ»ΡŽΡ‡Π°Π΅Ρ‚ Π² сСбя Π΄Π²Π° элСктрода ΠΈΠ»ΠΈ ΠΎΠ±ΠΊΠ»Π°Π΄ΠΊΠΈ, Ρ€Π°Π·Π΄Π΅Π»Π΅Π½Π½Ρ‹Π΅ ΠΌΠ΅ΠΆΠ΄Ρƒ собой диэлСктриком.

Π’ Ρ‡Π΅ΠΌ измСряСтся Π΅ΠΌΠΊΠΎΡΡ‚ΡŒ кондСнсатора

Одной ΠΈΠ· Π²Π°ΠΆΠ½Π΅ΠΉΡˆΠΈΡ… характСристик кондСнсатора являСтся Π΅Π³ΠΎ Π΅ΠΌΠΊΠΎΡΡ‚ΡŒ. Π”Π°Π½Π½Ρ‹ΠΉ ΠΏΠ°Ρ€Π°ΠΌΠ΅Ρ‚Ρ€ опрСдСляСтся количСством элСктроэнСргии, Π½Π°ΠΊΠ°ΠΏΠ»ΠΈΠ²Π°Π΅ΠΌΠΎΠΉ этим ΠΏΡ€ΠΈΠ±ΠΎΡ€ΠΎΠΌ. НакоплСниС происходит Π² Π²ΠΈΠ΄Π΅ элСктронов. Π˜Ρ… количСство, ΠΏΠΎΠΌΠ΅Ρ‰Π°ΡŽΡ‰Π΅Π΅ΡΡ Π² кондСнсаторС, опрСдСляСт Π²Π΅Π»ΠΈΡ‡ΠΈΠ½Ρƒ Смкости ΠΊΠΎΠ½ΠΊΡ€Π΅Ρ‚Π½ΠΎΠ³ΠΎ устройства.

Для измСрСния Смкости примСняСтся Π΅Π΄ΠΈΠ½ΠΈΡ†Π° – Ρ„Π°Ρ€Π°Π΄Π°. Π•ΠΌΠΊΠΎΡΡ‚ΡŒ кондСнсатора Π² 1 Ρ„Π°Ρ€Π°Π΄Ρƒ соотвСтствуСт элСктричСскому заряду Π² 1 ΠΊΡƒΠ»ΠΎΠ½, Π° Π½Π° ΠΎΠ±ΠΊΠ»Π°Π΄ΠΊΠ°Ρ… Ρ€Π°Π·Π½ΠΎΡΡ‚ΡŒ ΠΏΠΎΡ‚Π΅Π½Ρ†ΠΈΠ°Π»ΠΎΠ² Ρ€Π°Π²Π½Π° 1 Π²ΠΎΠ»ΡŒΡ‚Ρƒ. Π­Ρ‚Π° классичСская Ρ„ΠΎΡ€ΠΌΡƒΠ»ΠΈΡ€ΠΎΠ²ΠΊΠ° Π½Π΅ ΠΏΠΎΠ΄Ρ…ΠΎΠ΄ΠΈΡ‚ для практичСских расчСтов, ΠΏΠΎΡΠΊΠΎΠ»ΡŒΠΊΡƒ Π² кондСнсаторС ΡΠΎΠ±ΠΈΡ€Π°ΡŽΡ‚ΡΡ Π½Π΅ заряды, Π° элСктроны. Π•ΠΌΠΊΠΎΡΡ‚ΡŒ любого кондСнсатора находится Π² прямой зависимости ΠΎΡ‚ объСма элСктронов, способных Π½Π°ΠΊΠ°ΠΏΠ»ΠΈΠ²Π°Ρ‚ΡŒΡΡ ΠΏΡ€ΠΈ Π½ΠΎΡ€ΠΌΠ°Π»ΡŒΠ½ΠΎΠΌ Ρ€Π°Π±ΠΎΡ‡Π΅ΠΌ Ρ€Π΅ΠΆΠΈΠΌΠ΅. Для обозначСния Смкости всС Ρ€Π°Π²Π½ΠΎ ΠΈΡΠΏΠΎΠ»ΡŒΠ·ΡƒΠ΅Ρ‚ΡΡ Ρ„Π°Ρ€Π°Π΄Π°, Π° количСствСнныС ΠΏΠ°Ρ€Π°ΠΌΠ΅Ρ‚Ρ€Ρ‹ ΠΎΠΏΡ€Π΅Π΄Π΅Π»ΡΡŽΡ‚ΡΡ ΠΏΠΎ Ρ„ΠΎΡ€ΠΌΡƒΠ»Π΅: Π‘ = Q / U, Π³Π΄Π΅ Π‘ ΠΎΠ·Π½Π°Ρ‡Π°Π΅Ρ‚ Π΅ΠΌΠΊΠΎΡΡ‚ΡŒ, Q – заряд Π² ΠΊΡƒΠ»ΠΎΠ½Π°Ρ…, Π° U являСтся напряТСниСм. Π’Π°ΠΊΠΈΠΌ ΠΎΠ±Ρ€Π°Π·ΠΎΠΌ, просматриваСтся взаимная связь заряда ΠΈ напряТСния, ΠΎΠΊΠ°Π·Ρ‹Π²Π°ΡŽΡ‰ΠΈΡ… влияниС Π½Π° ΡΠΏΠΎΡΠΎΠ±Π½ΠΎΡΡ‚ΡŒ кондСнсатора ΠΊ накоплСнию ΠΈ ΡƒΠ΄Π΅Ρ€ΠΆΠ°Π½ΠΈΡŽ ΠΎΠΏΡ€Π΅Π΄Π΅Π»Π΅Π½Π½ΠΎΠ³ΠΎ количСства элСктричСства.

Для расчСтов Смкости плоского кондСнсатора ΠΈΡΠΏΠΎΠ»ΡŒΠ·ΡƒΠ΅Ρ‚ΡΡ Ρ„ΠΎΡ€ΠΌΡƒΠ»Π°:
Π² ΠΊΠΎΡ‚ΠΎΡ€ΠΎΠΉ Ξ΅ = 8,854187817 Ρ… 10 -12 Ρ„/ΠΌ прСдставляСт собой ΠΏΠΎΡΡ‚ΠΎΡΠ½Π½ΡƒΡŽ Π²Π΅Π»ΠΈΡ‡ΠΈΠ½Ρƒ. ΠŸΡ€ΠΎΡ‡ΠΈΠ΅ Π²Π΅Π»ΠΈΡ‡ΠΈΠ½Ρ‹: Ξ΅ – являСтся диэлСктричСской ΠΏΡ€ΠΎΠ½ΠΈΡ†Π°Π΅ΠΌΠΎΡΡ‚ΡŒΡŽ диэлСктрика, находящСгося ΠΌΠ΅ΠΆΠ΄Ρƒ ΠΎΠ±ΠΊΠ»Π°Π΄ΠΊΠ°ΠΌΠΈ, S – ΠΎΠ·Π½Π°Ρ‡Π°Π΅Ρ‚ ΠΏΠ»ΠΎΡ‰Π°Π΄ΡŒ ΠΎΠ±ΠΊΠ»Π°Π΄ΠΊΠΈ, Π° d – Π·Π°Π·ΠΎΡ€ ΠΌΠ΅ΠΆΠ΄Ρƒ ΠΎΠ±ΠΊΠ»Π°Π΄ΠΊΠ°ΠΌΠΈ.

Π€ΠΎΡ€ΠΌΡƒΠ»Π° энСргии кондСнсатора

Π‘ Π΅ΠΌΠΊΠΎΡΡ‚ΡŒΡŽ самым тСсным ΠΎΠ±Ρ€Π°Π·ΠΎΠΌ связана другая Π²Π΅Π»ΠΈΡ‡ΠΈΠ½Π°, извСстная ΠΊΠ°ΠΊ энСргия заряТСнного кондСнсатора. ПослС зарядки любого кондСнсатора, Π² Π½Π΅ΠΌ образуСтся ΠΎΠΏΡ€Π΅Π΄Π΅Π»Π΅Π½Π½ΠΎΠ΅ количСство энСргии, ΠΊΠΎΡ‚ΠΎΡ€ΠΎΠ΅ Π² дальнСйшСм выдСляСтся Π² процСссС разрядки. Π‘ этой ΠΏΠΎΡ‚Π΅Π½Ρ†ΠΈΠ°Π»ΡŒΠ½ΠΎΠΉ энСргиСй Π²ΡΡ‚ΡƒΠΏΠ°ΡŽΡ‚ Π²ΠΎ взаимодСйствиС ΠΎΠ±ΠΊΠ»Π°Π΄ΠΊΠΈ кондСнсатора. Π’ Π½ΠΈΡ… ΠΎΠ±Ρ€Π°Π·ΡƒΡŽΡ‚ΡΡ Ρ€Π°Π·Π½ΠΎΠΈΠΌΠ΅Π½Π½Ρ‹Π΅ заряды, ΠΏΡ€ΠΈΡ‚ΡΠ³ΠΈΠ²Π°ΡŽΡ‰ΠΈΠ΅ΡΡ Π΄Ρ€ΡƒΠ³ ΠΊ Π΄Ρ€ΡƒΠ³Ρƒ.

Π’ процСссС зарядки происходит расходованиС энСргии внСшнСго источника для раздСлСния зарядов с ΠΏΠΎΠ»ΠΎΠΆΠΈΡ‚Π΅Π»ΡŒΠ½Ρ‹ΠΌ ΠΈ ΠΎΡ‚Ρ€ΠΈΡ†Π°Ρ‚Π΅Π»ΡŒΠ½Ρ‹ΠΌ Π·Π½Π°Ρ‡Π΅Π½ΠΈΠ΅ΠΌ, ΠΊΠΎΡ‚ΠΎΡ€Ρ‹Π΅, Π·Π°Ρ‚Π΅ΠΌ Ρ€Π°ΡΠΏΠΎΠ»Π°Π³Π°ΡŽΡ‚ΡΡ Π½Π° ΠΎΠ±ΠΊΠ»Π°Π΄ΠΊΠ°Ρ… кондСнсатора. ΠŸΠΎΡΡ‚ΠΎΠΌΡƒ Π² соотвСтствии с Π·Π°ΠΊΠΎΠ½ΠΎΠΌ сохранСния энСргии, ΠΎΠ½Π° Π½Π΅ исчСзаСт бСсслСдно, Π° остаСтся Π²Π½ΡƒΡ‚Ρ€ΠΈ кондСнсатора Π² Π²ΠΈΠ΄Π΅ элСктричСского поля, сосрСдоточСнного ΠΌΠ΅ΠΆΠ΄Ρƒ пластинами. Π Π°Π·Π½ΠΎΠΈΠΌΠ΅Π½Π½Ρ‹Π΅ заряды ΠΎΠ±Ρ€Π°Π·ΡƒΡŽΡ‚ взаимодСйствиС ΠΈ ΠΏΠΎΡΠ»Π΅Π΄ΡƒΡŽΡ‰Π΅Π΅ притяТСниС ΠΎΠ±ΠΊΠ»Π°Π΄ΠΎΠΊ ΠΌΠ΅ΠΆΠ΄Ρƒ собой.

КаТдая пластина кондСнсатора ΠΏΠΎΠ΄ дСйствиСм заряда создаСт Π½Π°ΠΏΡ€ΡΠΆΠ΅Π½Π½ΠΎΡΡ‚ΡŒ элСктричСского поля, Ρ€Π°Π²Π½ΡƒΡŽ Π•/2. ΠžΠ±Ρ‰Π΅Π΅ ΠΏΠΎΠ»Π΅ Π±ΡƒΠ΄Π΅Ρ‚ ΡΠΊΠ»Π°Π΄Ρ‹Π²Π°Ρ‚ΡŒΡΡ ΠΈΠ· ΠΎΠ±ΠΎΠΈΡ… ΠΏΠΎΠ»Π΅ΠΉ, Π²ΠΎΠ·Π½ΠΈΠΊΠ°ΡŽΡ‰ΠΈΡ… Π² ΠΊΠ°ΠΆΠ΄ΠΎΠΉ ΠΎΠ±ΠΊΠ»Π°Π΄ΠΊΠ΅ с ΠΎΠ΄ΠΈΠ½Π°ΠΊΠΎΠ²Ρ‹ΠΌΠΈ зарядами, ΠΈΠΌΠ΅ΡŽΡ‰ΠΈΠΌΠΈ ΠΏΡ€ΠΎΡ‚ΠΈΠ²ΠΎΠΏΠΎΠ»ΠΎΠΆΠ½Ρ‹Π΅ значСния.

Π’Π°ΠΊΠΈΠΌ ΠΎΠ±Ρ€Π°Π·ΠΎΠΌ, энСргия кондСнсатора выраТаСтся Ρ„ΠΎΡ€ΠΌΡƒΠ»ΠΎΠΉ: W=q(E/2)d. Π’ свою ΠΎΡ‡Π΅Ρ€Π΅Π΄ΡŒ, напряТСниС выраТаСтся с ΠΏΠΎΠΌΠΎΡ‰ΡŒΡŽ понятий напряТСнности ΠΈ расстояния ΠΈ прСдставляСтся Π² Π²ΠΈΠ΄Π΅ Ρ„ΠΎΡ€ΠΌΡƒΠ»Ρ‹ U=Ed. Π­Ρ‚ΠΎ Π·Π½Π°Ρ‡Π΅Π½ΠΈΠ΅, подставлСнноС Π² ΠΏΠ΅Ρ€Π²ΡƒΡŽ Ρ„ΠΎΡ€ΠΌΡƒΠ»Ρƒ, ΠΎΡ‚ΠΎΠ±Ρ€Π°ΠΆΠ°Π΅Ρ‚ ΡΠ½Π΅Ρ€Π³ΠΈΡŽ кондСнсатора Π² Ρ‚Π°ΠΊΠΎΠΌ Π²ΠΈΠ΄Π΅: W=qU/2. Для получСния ΠΎΠΊΠΎΠ½Ρ‡Π°Ρ‚Π΅Π»ΡŒΠ½ΠΎΠ³ΠΎ Ρ€Π΅Π·ΡƒΠ»ΡŒΡ‚Π°Ρ‚Π° Π½Π΅ΠΎΠ±Ρ…ΠΎΠ΄ΠΈΠΌΠΎ ΠΈΡΠΏΠΎΠ»ΡŒΠ·ΠΎΠ²Π°Ρ‚ΡŒ ΠΎΠΏΡ€Π΅Π΄Π΅Π»Π΅Π½ΠΈΠ΅ Смкости: C=q/U, ΠΈ Π² ΠΊΠΎΠ½Ρ†Π΅ ΠΊΠΎΠ½Ρ†ΠΎΠ² энСргия заряТСнного кондСнсатора Π±ΡƒΠ΄Π΅Ρ‚ Π²Ρ‹Π³Π»ΡΠ΄Π΅Ρ‚ΡŒ ΡΠ»Π΅Π΄ΡƒΡŽΡ‰ΠΈΠΌ ΠΎΠ±Ρ€Π°Π·ΠΎΠΌ: Wэл = CU 2 /2.

Π€ΠΎΡ€ΠΌΡƒΠ»Π° заряда кондСнсатора

Для выполнСния зарядки, кондСнсатор Π΄ΠΎΠ»ΠΆΠ΅Π½ Π±Ρ‹Ρ‚ΡŒ ΠΏΠΎΠ΄ΠΊΠ»ΡŽΡ‡Π΅Π½ ΠΊ Ρ†Π΅ΠΏΠΈ постоянного Ρ‚ΠΎΠΊΠ°. Π‘ этой Ρ†Π΅Π»ΡŒΡŽ ΠΌΠΎΠΆΠ΅Ρ‚ ΠΈΡΠΏΠΎΠ»ΡŒΠ·ΠΎΠ²Π°Ρ‚ΡŒΡΡ Π³Π΅Π½Π΅Ρ€Π°Ρ‚ΠΎΡ€. Π£ ΠΊΠ°ΠΆΠ΄ΠΎΠ³ΠΎ Π³Π΅Π½Π΅Ρ€Π°Ρ‚ΠΎΡ€Π° имССтся Π²Π½ΡƒΡ‚Ρ€Π΅Π½Π½Π΅Π΅ сопротивлСниС. ΠŸΡ€ΠΈ Π·Π°ΠΌΡ‹ΠΊΠ°Π½ΠΈΠΈ Ρ†Π΅ΠΏΠΈ происходит зарядка кондСнсатора. ΠœΠ΅ΠΆΠ΄Ρƒ Π΅Π³ΠΎ ΠΎΠ±ΠΊΠ»Π°Π΄ΠΊΠ°ΠΌΠΈ появляСтся напряТСниС, Ρ€Π°Π²Π½ΠΎΠ΅ элСктродвиТущСй силС Π³Π΅Π½Π΅Ρ€Π°Ρ‚ΠΎΡ€Π°: Uc = E.

Обкладка, ΠΏΠΎΠ΄ΠΊΠ»ΡŽΡ‡Π΅Π½Π½Π°Ρ ΠΊ ΠΏΠΎΠ»ΠΎΠΆΠΈΡ‚Π΅Π»ΡŒΠ½ΠΎΠΌΡƒ ΠΏΠΎΠ»ΡŽΡΡƒ Π³Π΅Π½Π΅Ρ€Π°Ρ‚ΠΎΡ€Π°, заряТаСтся ΠΏΠΎΠ»ΠΎΠΆΠΈΡ‚Π΅Π»ΡŒΠ½ΠΎ (+q), Π° другая ΠΎΠ±ΠΊΠ»Π°Π΄ΠΊΠ° ΠΏΠΎΠ»ΡƒΡ‡Π°Π΅Ρ‚ Ρ€Π°Π²Π½ΠΎΠ·Π½Π°Ρ‡Π½Ρ‹ΠΉ заряд с ΠΎΡ‚Ρ€ΠΈΡ†Π°Ρ‚Π΅Π»ΡŒΠ½ΠΎΠΉ Π²Π΅Π»ΠΈΡ‡ΠΈΠ½ΠΎΠΉ (- q). Π’Π΅Π»ΠΈΡ‡ΠΈΠ½Π° заряда q находится Π² прямой ΠΏΡ€ΠΎΠΏΠΎΡ€Ρ†ΠΈΠΎΠ½Π°Π»ΡŒΠ½ΠΎΠΉ зависимости с Π΅ΠΌΠΊΠΎΡΡ‚ΡŒΡŽ кондСнсатора Π‘ ΠΈ напряТСниСм Π½Π° ΠΎΠ±ΠΊΠ»Π°Π΄ΠΊΠ°Ρ… Uc. Π­Ρ‚Π° Π·Π°Π²ΠΈΡΠΈΠΌΠΎΡΡ‚ΡŒ выраТаСтся Ρ„ΠΎΡ€ΠΌΡƒΠ»ΠΎΠΉ: q = C x Uc.

Π’ процСссС зарядки ΠΎΠ΄Π½Π° ΠΈΠ· ΠΎΠ±ΠΊΠ»Π°Π΄ΠΎΠΊ кондСнсатора ΠΏΡ€ΠΈΠΎΠ±Ρ€Π΅Ρ‚Π°Π΅Ρ‚, Π° другая тСряСт ΠΎΠΏΡ€Π΅Π΄Π΅Π»Π΅Π½Π½ΠΎΠ΅ количСство элСктронов. Они пСрСносятся ΠΏΠΎ внСшнСй Ρ†Π΅ΠΏΠΈ ΠΏΠΎΠ΄ влияниСм элСктродвиТущСй силы Π³Π΅Π½Π΅Ρ€Π°Ρ‚ΠΎΡ€Π°. Π’Π°ΠΊΠΎΠ΅ ΠΏΠ΅Ρ€Π΅ΠΌΠ΅Ρ‰Π΅Π½ΠΈΠ΅ являСтся элСктричСским Ρ‚ΠΎΠΊΠΎΠΌ, извСстным Π΅Ρ‰Π΅ ΠΊΠ°ΠΊ зарядный Смкостной Ρ‚ΠΎΠΊ (IΠ·Π°Ρ€).

Π’Π΅Ρ‡Π΅Π½ΠΈΠ΅ зарядного Ρ‚ΠΎΠΊΠ° Π² Ρ†Π΅ΠΏΠΈ происходит практичСски Π·Π° тысячныС Π΄ΠΎΠ»ΠΈ сСкунды, Π΄ΠΎ Ρ‚ΠΎΠ³ΠΎ ΠΌΠΎΠΌΠ΅Π½Ρ‚Π°, ΠΏΠΎΠΊΠ° напряТСниС кондСнсатора Π½Π΅ станСт Ρ€Π°Π²Π½Ρ‹ΠΌ элСктродвиТущСй силС Π³Π΅Π½Π΅Ρ€Π°Ρ‚ΠΎΡ€Π°. НапряТСниС увСличиваСтся ΠΏΠ»Π°Π²Π½ΠΎ, Π° ΠΏΠΎΡ‚ΠΎΠΌ постСпСнно замСдляСтся. Π”Π°Π»Π΅Π΅ Π·Π½Π°Ρ‡Π΅Π½ΠΈΠ΅ напряТСния кондСнсатора Π±ΡƒΠ΄Π΅Ρ‚ постоянным. Π’ΠΎ врСмя зарядки ΠΏΠΎ Ρ†Π΅ΠΏΠΈ Ρ‚Π΅Ρ‡Π΅Ρ‚ зарядный Ρ‚ΠΎΠΊ. Π’ самом Π½Π°Ρ‡Π°Π»Π΅ ΠΎΠ½ достигаСт максимальной Π²Π΅Π»ΠΈΡ‡ΠΈΠ½Ρ‹, Ρ‚Π°ΠΊ ΠΊΠ°ΠΊ напряТСниС кондСнсатора ΠΈΠΌΠ΅Π΅Ρ‚ Π½ΡƒΠ»Π΅Π²ΠΎΠ΅ Π·Π½Π°Ρ‡Π΅Π½ΠΈΠ΅. Богласно Π·Π°ΠΊΠΎΠ½Π° Ома IΠ·Π°Ρ€ = Π•/Ri, ΠΏΠΎΡΠΊΠΎΠ»ΡŒΠΊΡƒ ΠΊ ΡΠΎΠΏΡ€ΠΎΡ‚ΠΈΠ²Π»Π΅Π½ΠΈΡŽ Ri ΠΏΡ€ΠΈΠ»ΠΎΠΆΠ΅Π½Π° вся Π­Π”Π‘ Π³Π΅Π½Π΅Ρ€Π°Ρ‚ΠΎΡ€Π°.

Π€ΠΎΡ€ΠΌΡƒΠ»Π° Ρ‚ΠΎΠΊΠ° ΡƒΡ‚Π΅Ρ‡ΠΊΠΈ кондСнсатора

Π’ΠΎΠΊ ΡƒΡ‚Π΅Ρ‡ΠΊΠΈ кондСнсатора Π²ΠΏΠΎΠ»Π½Π΅ ΠΌΠΎΠΆΠ½ΠΎ ΡΡ€Π°Π²Π½ΠΈΡ‚ΡŒ с воздСйствиСм ΠΏΠΎΠ΄ΠΊΠ»ΡŽΡ‡Π΅Π½Π½ΠΎΠ³ΠΎ ΠΊ Π½Π΅ΠΌΡƒ рСзистора с ΠΊΠ°ΠΊΠΈΠΌ-Π»ΠΈΠ±ΠΎ сопротивлСниСм R. Π’ΠΎΠΊ ΡƒΡ‚Π΅Ρ‡ΠΊΠΈ тСсно связан с Ρ‚ΠΈΠΏΠΎΠΌ кондСнсатора ΠΈ качСством ΠΈΡΠΏΠΎΠ»ΡŒΠ·ΡƒΠ΅ΠΌΠΎΠ³ΠΎ диэлСктрика. ΠšΡ€ΠΎΠΌΠ΅ Ρ‚ΠΎΠ³ΠΎ, Π²Π°ΠΆΠ½Ρ‹ΠΌ Ρ„Π°ΠΊΡ‚ΠΎΡ€ΠΎΠΌ становится конструкция корпуса ΠΈ ΡΡ‚Π΅ΠΏΠ΅Π½ΡŒ Π΅Π³ΠΎ загрязнСнности.

НСкоторыС кондСнсаторы ΠΈΠΌΠ΅ΡŽΡ‚ Π½Π΅Π³Π΅Ρ€ΠΌΠ΅Ρ‚ΠΈΡ‡Π½Ρ‹ΠΉ корпус, Ρ‡Ρ‚ΠΎ ΠΏΡ€ΠΈΠ²ΠΎΠ΄ΠΈΡ‚ ΠΊ ΠΏΡ€ΠΎΠ½ΠΈΠΊΠ½ΠΎΠ²Π΅Π½ΠΈΡŽ Π²Π»Π°Π³ΠΈ ΠΈΠ· Π²ΠΎΠ·Π΄ΡƒΡ…Π° ΠΈ Π²ΠΎΠ·Ρ€Π°ΡΡ‚Π°Π½ΠΈΡŽ Ρ‚ΠΎΠΊΠ° ΡƒΡ‚Π΅Ρ‡ΠΊΠΈ. Π’ ΠΏΠ΅Ρ€Π²ΡƒΡŽ ΠΎΡ‡Π΅Ρ€Π΅Π΄ΡŒ это касаСтся устройств, Π³Π΄Π΅ Π² качСствС диэлСктрика использована промаслСнная Π±ΡƒΠΌΠ°Π³Π°. Π—Π½Π°Ρ‡ΠΈΡ‚Π΅Π»ΡŒΠ½Ρ‹Π΅ Ρ‚ΠΎΠΊΠΈ ΡƒΡ‚Π΅Ρ‡ΠΊΠΈ Π²ΠΎΠ·Π½ΠΈΠΊΠ°ΡŽΡ‚ ΠΈΠ·-Π·Π° сниТСния элСктричСского сопротивлСния изоляции. Π’ Ρ€Π΅Π·ΡƒΠ»ΡŒΡ‚Π°Ρ‚Π΅ Π½Π°Ρ€ΡƒΡˆΠ°Π΅Ρ‚ΡΡ основная функция кондСнсатора – ΡΠΏΠΎΡΠΎΠ±Π½ΠΎΡΡ‚ΡŒ ΠΏΠΎΠ»ΡƒΡ‡Π°Ρ‚ΡŒ ΠΈ ΡΠΎΡ…Ρ€Π°Π½ΡΡ‚ΡŒ заряд элСктричСского Ρ‚ΠΎΠΊΠ°.

Основная Ρ„ΠΎΡ€ΠΌΡƒΠ»Π° для расчСта выглядит ΡΠ»Π΅Π΄ΡƒΡŽΡ‰ΠΈΠΌ ΠΎΠ±Ρ€Π°Π·ΠΎΠΌ: IΡƒΡ‚ = U/Rd, Π³Π΄Π΅ IΡƒΡ‚, – это Ρ‚ΠΎΠΊ ΡƒΡ‚Π΅Ρ‡ΠΊΠΈ, U – напряТСниС, ΠΏΡ€ΠΈΠ»Π°Π³Π°Π΅ΠΌΠΎΠ΅ ΠΊ кондСнсатору, Π° Rd – сопротивлСниС изоляции.

Π—Π°Π΄Π°Ρ‡ΠΈ Π½Π° кондСнсаторы ΠΈ ΡΠ»Π΅ΠΊΡ‚Ρ€ΠΎΠ΅ΠΌΠΊΠΎΡΡ‚ΡŒ с Ρ€Π΅ΡˆΠ΅Π½ΠΈΡΠΌΠΈ

ΠšΠΎΠ½Π΄Π΅Π½ΡΠ°Ρ‚ΠΎΡ€ – Π΄Π΅Ρ‚Π°Π»ΡŒΠΊΠ°, Π±Π΅Π· ΠΊΠΎΡ‚ΠΎΡ€ΠΎΠΉ Π½Π΅ обойдСтся Ρ€Π°Π±ΠΎΡ‚Π° Π½ΠΈ ΠΎΠ΄Π½ΠΎΠ³ΠΎ элСктронного ΠΏΡ€ΠΈΠ±ΠΎΡ€Π°. Но ΠΏΡ€Π΅ΠΆΠ΄Π΅ Ρ‡Π΅ΠΌ Ρ€Π°Π·Π±ΠΈΡ€Π°Ρ‚ΡŒΡΡ с основами элСктроники, Π½ΡƒΠΆΠ½ΠΎ Π½Π°ΡƒΡ‡ΠΈΡ‚ΡŒΡΡ Ρ€Π΅ΡˆΠ°Ρ‚ΡŒ физичСскиС Π·Π°Π΄Π°Ρ‡ΠΈ Π½Π° кондСнсатор ΠΈ ΡΠ»Π΅ΠΊΡ‚Ρ€ΠΎΠ΅ΠΌΠΊΠΎΡΡ‚ΡŒ. ИмСнно этим ΠΌΡ‹ ΠΈ займСмся Π² сСгодняшнСй ΡΡ‚Π°Ρ‚ΡŒΠ΅, посвящСнной ΠΏΠΎΠ΄Ρ€ΠΎΠ±Π½ΠΎΠΌΡƒ Ρ€Π°Π·Π±ΠΎΡ€Ρƒ Ρ€Π΅ΡˆΠ΅Π½ΠΈΠΉ Π·Π°Π΄Π°Ρ‡.

ΠŸΠΎΠ΄ΠΏΠΈΡΡ‹Π²Π°ΠΉΡ‚Π΅ΡΡŒ Π½Π° наш Ρ‚Π΅Π»Π΅Π³Ρ€Π°ΠΌ: Ρ‚Π΅ΠΏΠ΅Ρ€ΡŒ ΠΏΠΎΠΌΠΈΠΌΠΎ ΠΏΠΎΠ»Π΅Π·Π½Ρ‹Ρ… и интСрСсных ΠΌΠ°Ρ‚Π΅Ρ€ΠΈΠ°Π»ΠΎΠ²Β Ρ‚Π°ΠΌ ΠΌΠΎΠΆΠ½ΠΎ Π½Π°ΠΉΡ‚ΠΈ скидки ΠΈ Π°ΠΊΡ†ΠΈΠΈ Π½Π° Π»ΡŽΠ±Ρ‹Π΅ Ρ€Π°Π±ΠΎΡ‚Ρ‹.

Π—Π°Π΄Π°Ρ‡ΠΈ Π½Π° кондСнсаторы ΠΈ ΡΠ»Π΅ΠΊΡ‚Ρ€ΠΎΠ΅ΠΌΠΊΠΎΡΡ‚ΡŒ с Ρ€Π΅ΡˆΠ΅Π½ΠΈΠ΅ΠΌ

Если Π²Ρ‹ Π½Π΅ Π·Π½Π°Π΅Ρ‚Π΅, ΠΊΠ°ΠΊ Ρ€Π΅ΡˆΠ°Ρ‚ΡŒ Π·Π°Π΄Π°Ρ‡ΠΈ с кондСнсаторами, сначала посмотритС Ρ‚Π΅ΠΎΡ€ΠΈΡŽ ΠΈ вспомнитС ΠΏΡ€ΠΎ памятку по Ρ€Π΅ΡˆΠ΅Π½ΠΈΡŽ Π·Π°Π΄Π°Ρ‡ ΠΏΠΎ Ρ„ΠΈΠ·ΠΈΠΊΠ΅ ΠΈ ΠΏΠΎΠ»Π΅Π·Π½Ρ‹Π΅ Ρ„ΠΎΡ€ΠΌΡƒΠ»Ρ‹.

Π—Π°Π΄Π°Ρ‡Π° β„–1 Π½Π° ΡΠ»Π΅ΠΊΡ‚Ρ€ΠΎΠ΅ΠΌΠΊΠΎΡΡ‚ΡŒ Π±Π°Ρ‚Π°Ρ€Π΅ΠΈ кондСнсаторов

УсловиС

Плоский кондСнсатор Π΅ΠΌΠΊΠΎΡΡ‚ΡŒΡŽ 16 ΠΌΠΊΠ€ Ρ€Π°Π·Ρ€Π΅Π·Π°ΡŽΡ‚ Π½Π° 4 Ρ€Π°Π²Π½Ρ‹Π΅ части вдоль плоскостСй, пСрпСндикулярных ΠΎΠ±ΠΊΠ»Π°Π΄ΠΊΠ°ΠΌ. ΠŸΠΎΠ»ΡƒΡ‡Π΅Π½Π½Ρ‹Π΅ кондСнсаторы ΡΠΎΠ΅Π΄ΠΈΠ½ΡΡŽΡ‚ ΠΏΠΎΡΠ»Π΅Π΄ΠΎΠ²Π°Ρ‚Π΅Π»ΡŒΠ½ΠΎ. Π§Π΅ΠΌΡƒ Ρ€Π°Π²Π½Π° Π΅ΠΌΠΊΠΎΡΡ‚ΡŒΒ Π±Π°Ρ‚aΡ€Π΅ΠΈ кондСнсаторов?

РСшСниС

Из условия слСдуСт, Ρ‡Ρ‚ΠΎ ΠΏΠ»ΠΎΡ‰Π°Π΄ΡŒ ΠΏΠΎΠ»ΡƒΡ‡ΠΈΠ²ΡˆΠΈΡ…ΡΡ кондСнсаторов Π² 4 Ρ€Π°Π·Π° мСньшС, Ρ‡Π΅ΠΌ Ρƒ исходного. Зная это, ΠΌΠΎΠΆΠ½ΠΎ Π½Π°ΠΉΡ‚ΠΈ Π΅ΠΌΠΊΠΎΡΡ‚ΡŒ ΠΊΠ°ΠΆΠ΄ΠΎΠ³ΠΎ ΠΏΠΎΠ»ΡƒΡ‡Π΅Π½Π½ΠΎΠ³ΠΎ кондСнсатора:

БоСдиняя 4 Ρ‚Π°ΠΊΠΈΡ… кондСнсатора ΠΏΠΎΡΠ»Π΅Π΄ΠΎΠ²Π°Ρ‚Π΅Π»ΡŒΠ½ΠΎ, ΠΏΠΎΠ»ΡƒΡ‡Π°Π΅ΠΌ:

ΠžΡ‚Π²Π΅Ρ‚: 1 ΠΌΠΊΠ€.

Π—Π°Π΄Π°Ρ‡Π° β„–2 Π½Π° ΡΠ½Π΅Ρ€Π³ΠΈΡŽ плоского кондСнсатора

УсловиС

Плоский кондСнсатор Π·Π°ΠΏΠΎΠ»Π½ΠΈΠ»ΠΈ диэлСктриком с диэлСктричСской ΠΏΡ€ΠΎΠ½ΠΈΡ†Π°Π΅ΠΌΠΎΡΡ‚ΡŒΡŽ, Ρ€Π°Π²Π½ΠΎΠΉ 2. ЭнСргия кондСнсатора Π±Π΅Π· диэлСктрика Ρ€Π°Π²Π½Π° 20 ΠΌΠΊΠ”ΠΆ. Π§Π΅ΠΌΡƒ Ρ€Π°Π²Π½Π° энСргия кондСнсатора послС заполнСния диэлСктриком? Π‘Ρ‡ΠΈΡ‚Π°Ρ‚ΡŒ, Ρ‡Ρ‚ΠΎ источник питания ΠΎΡ‚ΠΊΠ»ΡŽΡ‡Π΅Π½ ΠΎΡ‚ кондСнсатора.

РСшСниС

ЭнСргия кондСнсатора Π΄ΠΎ заполнСния диэлСктриком Ρ€Π°Π²Π½Π°:

ПослС заполнСния Π΅ΠΌΠΊΠΎΡΡ‚ΡŒ кондСнсатора измСнится:

ЭнСргия кондСнсатора послС заполнСния:

ΠžΡ‚Π²Π΅Ρ‚: 40 ΠΌΠΊΠ€.

Π—Π°Π΄Π°Ρ‡Π° β„–3 Π½Π° ΠΏΠΎΡΠ»Π΅Π΄ΠΎΠ²Π°Ρ‚Π΅Π»ΡŒΠ½ΠΎΠ΅ ΠΈ ΠΏΠ°Ρ€Π°Π»Π»Π΅Π»ΡŒΠ½ΠΎΠ΅ соСдинСниС кондСнсаторов

УсловиС

На рисункС ΠΈΠ·ΠΎΠ±Ρ€Π°ΠΆΠ΅Π½Π° батарСя кондСнсаторов. ΠšΠ°ΠΆΠ΄Ρ‹ΠΉ кондСнсатор ΠΈΠΌΠ΅Π΅Ρ‚ Π΅ΠΌΠΊΠΎΡΡ‚ΡŒ 1 ΠΌΠΊΠ€. НайдитС Π΅ΠΌΠΊΠΎΡΡ‚ΡŒ Π±Π°Ρ‚Π°Ρ€Π΅ΠΈ.

РСшСниС

Как Π²ΠΈΠ΄ΠΈΠΌ, Ρ‡Π°ΡΡ‚ΡŒ кондСнсаторов соСдинСна ΠΏΠ°Ρ€Π°Π»Π»Π΅Π»ΡŒΠ½ΠΎ, Π° Ρ‡Π°ΡΡ‚ΡŒ ΠΏΠΎΡΠ»Π΅Π΄ΠΎΠ²Π°Ρ‚Π΅Π»ΡŒΠ½ΠΎ. Π­Ρ‚ΠΎ Ρ‚ΠΈΠΏΠΈΡ‡Π½Ρ‹ΠΉ ΠΏΡ€ΠΈΠΌΠ΅Ρ€ смСшанного соСдинСния кондСнсаторов. Алгоритм Ρ€Π΅ΡˆΠ΅Π½ΠΈΡΒ Π·Π°Π΄Π°Ρ‡ ΠΏΡ€ΠΈ смСшанном соСдинСнии кондСнсаторов сводится ΠΊ Ρ‚ΠΎΠΌΡƒ, Ρ‡Ρ‚ΠΎΠ±Ρ‹ ΡƒΠΏΡ€ΠΎΡΡ‚ΠΈΡ‚ΡŒ схСму ΠΈ свСсти всС Ρ‚ΠΎΠ»ΡŒΠΊΠΎ ΠΊ ΠΏΠ°Ρ€Π°Π»Π»Π΅Π»ΡŒΠ½ΠΎΠΌΡƒ ΠΈΠ»ΠΈ ΠΏΠΎΡΠ»Π΅Π΄ΠΎΠ²Π°Ρ‚Π΅Π»ΡŒΠ½ΠΎΠΌΡƒ соСдинСнию.

ΠšΠΎΠ½Π΄Π΅Π½ΡΠ°Ρ‚ΠΎΡ€Ρ‹ 3 ΠΈ 4 соСдинСны ΠΏΠ°Ρ€Π°Π»Π»Π΅Π»ΡŒΠ½ΠΎ. Бкладывая ΠΈΡ… Π΅ΠΌΠΊΠΎΡΡ‚ΡŒ, ΠΏΠΎΠ»ΡƒΡ‡Π°Π΅ΠΌ Π² ΠΈΡ‚ΠΎΠ³Π΅ ΠΏΠΎΡΠ»Π΅Π΄ΠΎΠ²Π°Ρ‚Π΅Π»ΡŒΠ½ΠΎΠ΅ соСдинСниС Ρ‡Π΅Ρ‚Ρ‹Ρ€Π΅Ρ… кондСнсаторов: 1, 2, 5 ΠΈ 3-4. Для ΠΏΠ°Ρ€Π°Π»Π»Π΅Π»ΡŒΠ½ΠΎΠ³ΠΎ соСдинСния:

Для ΠΏΠΎΡΠ»Π΅Π΄ΠΎΠ²Π°Ρ‚Π΅Π»ΡŒΠ½ΠΎΠ³ΠΎ соСдинСния:

ΠžΡ‚Π²Π΅Ρ‚: 0,285 ΠΌΠΊΠ€.

Π—Π°Π΄Π°Ρ‡Π° β„–4 Π½Π° ΠΏΡ€ΠΎΠ»Π΅Ρ‚ частицы Π² кондСнсаторС

Заряд кондСнсатора Ρ€Π°Π²Π΅Π½ 0,3 нКл, Π° Π΅ΠΌΠΊΠΎΡΡ‚ΡŒ – 10 ΠΏΠ€. ΠšΠ°ΠΊΡƒΡŽ ΡΠΊΠΎΡ€ΠΎΡΡ‚ΡŒ ΠΏΡ€ΠΈΠΎΠ±Ρ€Π΅Ρ‚Π΅Ρ‚ элСктрон, пролСтая Π² кондСнсаторС ΠΎΡ‚ ΠΎΠ΄Π½ΠΎΠΉ пластины ΠΊ Π΄Ρ€ΡƒΠ³ΠΎΠΉ. 7 ΠΌ/с.

Π—Π°Π΄Π°Ρ‡Π° β„–5Β Π½Π° вычислСниС энСргии элСктричСского поля кондСнсатора

УсловиС

ΠšΠΎΠ½Π΄Π΅Π½ΡΠ°Ρ‚ΠΎΡ€ ΠΏΠΎΠ΄ΠΊΠ»ΡŽΡ‡Π΅Π½ ΠΊ источнику постоянного напряТСния U=1 ΠΊΠ’. Π•ΠΌΠΊΠΎΡΡ‚ΡŒ кондСнсатора Ρ€Π°Π²Π½Π° 5 ΠΏΠ€. Как ΠΈΠ·ΠΌΠ΅Π½ΡΡ‚ΡŒΡΡ заряд Π½Π° ΠΎΠ±ΠΊΠ»Π°Π΄ΠΊΠ°Ρ… кондСнсатора ΠΈ Π΅Π³ΠΎ энСргия, Ссли расстояниС ΠΌΠ΅ΠΆΠ΄Ρƒ ΠΎΠ±ΠΊΠ»Π°Π΄ΠΊΠ°ΠΌΠΈ ΡƒΠΌΠ΅Π½ΡŒΡˆΠΈΡ‚ΡŒ Π² Ρ‚Ρ€ΠΈ Ρ€Π°Π·Π°.

РСшСниС

Заряд кондСнсатора Ρ€Π°Π²Π΅Π½:

ИзмСнСниС заряда Π±ΡƒΠ΄Π΅Ρ‚ Ρ€Π°Π²Π½ΠΎ:

ИзмСнСниС энСргии:

ΠžΡ‚Π²Π΅Ρ‚: 5 ΠΌΠΊΠ”ΠΆ.

Вопросы Π½Π° Ρ‚Π΅ΠΌΡƒ Β«ΠšΠΎΠ½Π΄Π΅Π½ΡΠ°Ρ‚ΠΎΡ€ ΠΈ ΡΠ»Π΅ΠΊΡ‚Ρ€ΠΎΠ΅ΠΌΠΊΠΎΡΡ‚ΡŒΒ»

Вопрос 1. Π§Ρ‚ΠΎ Ρ‚Π°ΠΊΠΎΠ΅ кондСнсатор?

ΠžΡ‚Π²Π΅Ρ‚. ΠšΠΎΠ½Π΄Π΅Π½ΡΠ°Ρ‚ΠΎΡ€ – устройство, ΠΈΠΌΠ΅ΡŽΡ‰Π΅Π΅ Π΄Π²Π° полюса ΠΈ ΠΏΡ€Π΅Π΄Π½Π°Π·Π½Π°Ρ‡Π΅Π½Π½ΠΎΠ΅ для накоплСния элСктричСского заряда.

ΠŸΡ€ΠΎΡΡ‚Π΅ΠΉΡˆΠΈΠΉ Ρ‚ΠΈΠΏ кондСнсатора – плоский Π²ΠΎΠ·Π΄ΡƒΡˆΠ½Ρ‹ΠΉ кондСнсатор. Он состоит ΠΈΠ· Π΄Π²ΡƒΡ… пластин (ΠΎΠ±ΠΊΠ»Π°Π΄ΠΎΠΊ), ΠΈΠΌΠ΅ΡŽΡ‰ΠΈΡ… Ρ€Π°Π·Π½Ρ‹Π΅ заряды ΠΈ Ρ€Π°Π·Π΄Π΅Π»Π΅Π½Π½Ρ‹Ρ… Π²ΠΎΠ·Π΄ΡƒΡ…ΠΎΠΌ. Π’ зависимости ΠΎΡ‚ диэлСктрика, Ρ€Π°Π·Π΄Π΅Π»ΡΡŽΡ‰Π΅Π³ΠΎ ΠΎΠ±ΠΊΠ»Π°Π΄ΠΊΠΈ, Ρ€Π°Π·Π΄Π΅Π»ΡΡŽΡ‚:

  • Π²ΠΎΠ·Π΄ΡƒΡˆΠ½Ρ‹Π΅ кондСнсаторы;
  • Π±ΡƒΠΌΠ°ΠΆΠ½Ρ‹Π΅ кондСнсаторы;
  • ΡΠ»ΡŽΠ΄ΡΠ½Ρ‹Π΅ ΠΈ Π΄Ρ€ΡƒΠ³ΠΈΠ΅ кондСнсаторы.

Основная Ρ€ΠΎΠ»ΡŒ кондСнсатора Π² элСктронных ΠΏΡ€ΠΈΠ±ΠΎΡ€Π°Ρ… – Π½Π°ΠΊΠ°ΠΏΠ»ΠΈΠ²Π°Ρ‚ΡŒ заряд, Π° ΠΏΠΎΡ‚ΠΎΠΌ ΠΏΠ΅Ρ€Π΅Π΄Π°Π²Π°Ρ‚ΡŒ Π΅Π³ΠΎ дальшС Π² Ρ†Π΅ΠΏΡŒ.

Вопрос 2. Π§Ρ‚ΠΎ Ρ‚Π°ΠΊΠΎΠ΅ ΡΠ»Π΅ΠΊΡ‚Ρ€ΠΎΠ΅ΠΌΠΊΠΎΡΡ‚ΡŒ?

ΠžΡ‚Π²Π΅Ρ‚. Π­Π»Π΅ΠΊΡ‚Ρ€ΠΎΠ΅ΠΌΠΊΠΎΡΡ‚ΡŒ – скалярная физичСская Π²Π΅Π»ΠΈΡ‡ΠΈΠ½Π°, Ρ…Π°Ρ€Π°ΠΊΡ‚Π΅Ρ€ΠΈΠ·ΡƒΡŽΡ‰Π°Ρ ΡΠΏΠΎΡΠΎΠ±Π½ΠΎΡΡ‚ΡŒ Π½Π°ΠΊΠ°ΠΏΠ»ΠΈΠ²Π°Ρ‚ΡŒ элСктричСский заряд. Π’ систСмС БИ измСряСтся Π² Π€Π°Ρ€Π°Π΄Π°Ρ….

Вопрос 3. КакиС Π΅ΡΡ‚ΡŒ способы соСдинСния кондСнсаторов?

ΠžΡ‚Π²Π΅Ρ‚. ΠšΠΎΠ½Π΄Π΅Π½ΡΠ°Ρ‚ΠΎΡ€Ρ‹ ΠΌΠΎΠΆΠ½ΠΎ ΡΠΎΠ΅Π΄ΠΈΠ½ΠΈΡ‚ΡŒ ΠΏΠΎΡΠ»Π΅Π΄ΠΎΠ²Π°Ρ‚Π΅Π»ΡŒΠ½ΠΎ ΠΈ ΠΏΠ°Ρ€Π°Π»Π»Π΅Π»ΡŒΠ½ΠΎ.

ΠŸΡ€ΠΈ ΠΏΠ°Ρ€Π°Π»Π»Π΅Π»ΡŒΠ½ΠΎΠΌ соСдинСнии Π΅ΠΌΠΊΠΎΡΡ‚ΡŒ Ρ†Π΅ΠΏΠΈ Ρ€Π°Π²Π½Π° суммС СмкостСй ΠΎΡ‚Π΄Π΅Π»ΡŒΠ½Ρ‹Ρ… кондСнсаторов.

ΠŸΡ€ΠΈ ΠΏΠΎΡΠ»Π΅Π΄ΠΎΠ²Π°Ρ‚Π΅Π»ΡŒΠ½ΠΎΠΌ соСдинСнии Π²Π΅Π»ΠΈΡ‡ΠΈΠ½Π°, обратная ΠΎΠ±Ρ‰Π΅ΠΉ Смкости, Ρ€Π°Π²Π½Π° суммС ΠΎΠ±Ρ€Π°Ρ‚Π½Ρ‹Ρ… СмкостСй ΠΊΠ°ΠΆΠ΄ΠΎΠ³ΠΎ кондСнсатора.

Вопрос 4. Π§Ρ‚ΠΎ Ρ‚Π°ΠΊΠΎΠ΅ ΠΊΠΎΠ»Π΅Π±Π°Ρ‚Π΅Π»ΡŒΠ½Ρ‹ΠΉ ΠΊΠΎΠ½Ρ‚ΡƒΡ€?

ΠžΡ‚Π²Π΅Ρ‚. Π­Ρ‚ΠΎ ΠΏΡ€ΠΎΡΡ‚Π΅ΠΉΡˆΠ°Ρ элСктричСская Ρ†Π΅ΠΏΡŒ, состоящая ΠΈΠ· кондСнсатора, ΠΊΠ°Ρ‚ΡƒΡˆΠΊΠΈ индуктивности ΠΈ источника Ρ‚ΠΎΠΊΠ°. Π’ ΠΊΠΎΠ»Π΅Π±Π°Ρ‚Π΅Π»ΡŒΠ½ΠΎΠΌ ΠΊΠΎΠ½Ρ‚ΡƒΡ€Π΅ происходят свободныС элСктромагнитныС колСбания: энСргия кондСнсатора ΠΏΠ΅Ρ€Π΅Ρ…ΠΎΠ΄ΠΈΡ‚ Π² ΡΠ½Π΅Ρ€Π³ΠΈΡŽ ΠΊΠ°Ρ‚ΡƒΡˆΠΊΠΈ, ΠΈ Π½Π°ΠΎΠ±ΠΎΡ€ΠΎΡ‚. Β 

Вопрос 5. Π§Ρ‚ΠΎ происходит ΠΏΡ€ΠΈ ΠΎΡ‚ΠΊΠ»ΡŽΡ‡Π΅Π½ΠΈΠΈ источника питания, ΠΊ ΠΊΠΎΡ‚ΠΎΡ€ΠΎΠΌΡƒ ΠΏΠΎΠ΄ΠΊΠ»ΡŽΡ‡Π΅Π½ кондСнсатор Π² Ρ†Π΅ΠΏΠΈ?

ΠžΡ‚Π²Π΅Ρ‚. Π’ этот ΠΌΠΎΠΌΠ΅Π½Ρ‚ кондСнсатор Π½Π°Ρ‡ΠΈΠ½Π°Π΅Ρ‚ Ρ€Π°Π·Ρ€ΡΠΆΠ°Ρ‚ΡŒΡΡ,  отдавая Π½Π°ΠΊΠΎΠΏΠ»Π΅Π½Π½Ρ‹ΠΉ заряд Π΄Ρ€ΡƒΠ³ΠΈΠΌ элСмСнтам Ρ†Π΅ΠΏΠΈ.

ΠœΡ‹ Π½Π΅ ΠΏΠΎΠ½Π°ΡΡ‹Π»ΡˆΠΊΠ΅ Π·Π½Π°Π΅ΠΌ, Ρ‡Ρ‚ΠΎ от слоТных Π·Π°Π΄Π°Ρ‡ Π½Π° кондСнсаторы ΠΌΠΎΠ·Π³ΠΈ Π±ΡƒΠΊΠ²Π°Π»ΡŒΠ½ΠΎ плавятся. Если ваш ΠΌΠΎΠ·Π³ устал ΠΎΡ‚ постоянного Ρ€Π΅ΡˆΠ΅Π½ΠΈΡ Π·Π°Π΄Π°Ρ‡ ΠΏΠΎ Ρ„ΠΈΠ·ΠΈΠΊΠ΅ ΠΈ Π΄Ρ€ΡƒΠ³ΠΈΡ… Π·Π°Π΄Π°Π½ΠΈΠΉ, ΠΎΠ±Ρ€Π°Ρ‰Π°ΠΉΡ‚Π΅ΡΡŒ Π² ΠΏΡ€ΠΎΡ„Π΅ΡΡΠΈΠΎΠ½Π°Π»ΡŒΠ½Ρ‹ΠΉ ΠΎΠ±Ρ€Π°Π·ΠΎΠ²Π°Ρ‚Π΅Π»ΡŒΠ½Ρ‹ΠΉ сСрвис Π·Π° ΠΊΠΎΠ½ΡΡƒΠ»ΡŒΡ‚Π°Ρ†ΠΈΠ΅ΠΉ ΠΈ ΠΏΠΎΠ΄Π΄Π΅Ρ€ΠΆΠΊΠΎΠΉ Π² любоС врСмя. Π£ нас Π΅ΡΡ‚ΡŒ Ρ€Π΅ΡˆΠ΅Π½ΠΈΠ΅ для Π²Π°ΡˆΠΈΡ… ΠΏΡ€ΠΎΠ±Π»Π΅ΠΌ с ΡƒΡ‡Π΅Π±ΠΎΠΉ!

Как Π½Π°ΠΉΡ‚ΠΈ Π΅ΠΌΠΊΠΎΡΡ‚ΡŒ кондСнсатора

Для Ρ‚ΠΎΠ³ΠΎ Ρ‡Ρ‚ΠΎΠ±Ρ‹ Π·Π½Π°Ρ‚ΡŒ, ΠΌΠΎΠΆΠ½ΠΎ Π»ΠΈ ΠΈΡΠΏΠΎΠ»ΡŒΠ·ΠΎΠ²Π°Ρ‚ΡŒ Π² Ρ‚ΠΎΠΌ ΠΈΠ»ΠΈ ΠΈΠ½ΠΎΠΌ мСстС схСмы кондСнсатор, слСдуСт ΠΎΠΏΡ€Π΅Π΄Π΅Π»ΠΈΡ‚ΡŒ Π΅Π³ΠΎ . Бпособ нахоТдСния этого ΠΏΠ°Ρ€Π°ΠΌΠ΅Ρ‚Ρ€Π° зависит ΠΎΡ‚ Ρ‚ΠΎΠ³ΠΎ, ΠΊΠ°ΠΊΠΈΠΌ ΠΎΠ±Ρ€Π°Π·ΠΎΠΌ ΠΎΠ½ ΠΎΠ±ΠΎΠ·Π½Π°Ρ‡Π΅Π½ Π½Π° кондСнсаторС ΠΈ ΠΎΠ±ΠΎΠ·Π½Π°Ρ‡Π΅Π½ Π»ΠΈ Π²ΠΎΠΎΠ±Ρ‰Π΅.Π’Π°ΠΌ понадобится

На ΠΊΡ€ΡƒΠΏΠ½Ρ‹Ρ… кондСнсаторах Π΅ΠΌΠΊΠΎΡΡ‚ΡŒ ΠΎΠ±Ρ‹Ρ‡Π½ΠΎ ΠΎΠ±ΠΎΠ·Π½Π°Ρ‡Π΅Π½Π° ΠΎΡ‚ΠΊΡ€Ρ‹Ρ‚Ρ‹ΠΌ тСкстом: 0,25 ΠΌΠΊΠ€ ΠΈΠ»ΠΈ 15 uF. Π’ этом случаС, способ Π΅Π΅ опрСдСлСния Ρ‚Ρ€ΠΈΠ²ΠΈΠ°Π»Π΅Π½.

На ΠΌΠ΅Π½Π΅Π΅ ΠΊΡ€ΡƒΠΏΠ½Ρ‹Ρ… кондСнсаторах (Π² Ρ‚ΠΎΠΌ числС, SMD) Π΅ΠΌΠΊΠΎΡΡ‚ΡŒ обозначаСтся двумя ΠΈΠ»ΠΈ трСмя Ρ†ΠΈΡ„Ρ€Π°ΠΌΠΈ. Π’ ΠΏΠ΅Ρ€Π²ΠΎΠΌ случаС, ΠΎΠ½Π° ΠΎΠ±ΠΎΠ·Π½Π°Ρ‡Π΅Π½Π° Π² ΠΏΠΈΠΊΠΎΡ„Π°Ρ€Π°Π΄Π°Ρ…. Π’ΠΎ Π²Ρ‚ΠΎΡ€ΠΎΠΌ случаС, ΠΏΠ΅Ρ€Π²Ρ‹Π΅ Π΄Π²Π΅ Ρ†ΠΈΡ„Ρ€Ρ‹ ΠΎΠ·Π½Π°Ρ‡Π°ΡŽΡ‚ Π΅ΠΌΠΊΠΎΡΡ‚ΡŒ, Π° Ρ‚Ρ€Π΅Ρ‚ΡŒΡ — Π² ΠΊΠ°ΠΊΠΈΡ… Π΅Π΄ΠΈΠ½ΠΈΡ†Π°Ρ… ΠΎΠ½Π° Π²Ρ‹Ρ€Π°ΠΆΠ΅Π½Π°:1 — дСсятки ΠΏΠΈΠΊΠΎΡ„Π°Ρ€Π°Π΄;
2 — сотни ΠΏΠΈΠΊΠΎΡ„Π°Ρ€Π°Π΄;
3 — Π½Π°Π½ΠΎΡ„Π°Ρ€Π°Π΄Ρ‹;
4 — дСсятки Π½Π°Π½ΠΎΡ„Π°Ρ€Π°Π΄;
5 — дСсятыС Π΄ΠΎΠ»ΠΈ ΠΌΠΈΠΊΡ€ΠΎΡ„Π°Ρ€Π°Π΄Ρ‹.

БущСствуСт Ρ‚Π°ΠΊΠΆΠ΅ систСма обозначСния Смкости, ΠΈΡΠΏΠΎΠ»ΡŒΠ·ΡƒΡŽΡ‰Π°Ρ сочСтания латинских Π±ΡƒΠΊΠ² ΠΈ Ρ†ΠΈΡ„Ρ€. Π‘ΡƒΠΊΠ²Ρ‹ ΠΎΠ±ΠΎΠ·Π½Π°Ρ‡Π°ΡŽΡ‚ ΡΠ»Π΅Π΄ΡƒΡŽΡ‰ΠΈΠ΅ Ρ†ΠΈΡ„Ρ€Ρ‹:A — 10;
B — 11;
C — 12;
D — 13;
E — 15;
F — 16;
G — 18;
H — 20;
J — 22;
K — 24;
L — 27;
M — 30;
N — 33;
P — 36;
Q — 39;
R — 43;
S — 47;
T — 51;
U — 56;
V — 62;
W — 68;
X — 75;
Y — 82;
Z — 91.ΠŸΠΎΠ»ΡƒΡ‡Π΅Π½Π½ΠΎΠ΅ число слСдуСт ΡƒΠΌΠ½ΠΎΠΆΠΈΡ‚ΡŒ Π½Π° число 10, ΠΏΡ€Π΅Π΄Π²Π°Ρ€ΠΈΡ‚Π΅Π»ΡŒΠ½ΠΎ Π²ΠΎΠ·Π²Π΅Π΄Π΅Π½Π½ΠΎΠ΅ Π² ΡΡ‚Π΅ΠΏΠ΅Π½ΡŒ, Ρ€Π°Π²Π½ΡƒΡŽ Ρ†ΠΈΡ„Ρ€Π΅, ΡΠ»Π΅Π΄ΡƒΡŽΡ‰Π΅ΠΉ послС Π±ΡƒΠΊΠ²Ρ‹. Π Π΅Π·ΡƒΠ»ΡŒΡ‚Π°Ρ‚ Π±ΡƒΠ΄Π΅Ρ‚ Π²Ρ‹Ρ€Π°ΠΆΠ΅Π½ Π² ΠΏΠΈΠΊΠΎΡ„Π°Ρ€Π°Π΄Π°Ρ….

Π’ΡΡ‚Ρ€Π΅Ρ‡Π°ΡŽΡ‚ΡΡ кондСнсаторы, Π΅ΠΌΠΊΠΎΡΡ‚ΡŒ Π½Π° ΠΊΠΎΡ‚ΠΎΡ€Ρ‹Ρ… Π½Π΅ ΠΎΠ±ΠΎΠ·Π½Π°Ρ‡Π΅Π½Π° Π²ΠΎΠΎΠ±Ρ‰Π΅. Π’Ρ‹ навСрняка встрСчали ΠΈΡ…, Π² частности, Π² стартСрах Π»Π°ΠΌΠΏ Π΄Π½Π΅Π²Π½ΠΎΠ³ΠΎ свСта. Π’ этом случаС, ΠΈΠ·ΠΌΠ΅Ρ€ΠΈΡ‚ΡŒ Π΅ΠΌΠΊΠΎΡΡ‚ΡŒ ΠΌΠΎΠΆΠ½ΠΎ Ρ‚ΠΎΠ»ΡŒΠΊΠΎ ΡΠΏΠ΅Ρ†ΠΈΠ°Π»ΡŒΠ½Ρ‹ΠΌ ΠΏΡ€ΠΈΠ±ΠΎΡ€ΠΎΠΌ. Они Π±Ρ‹Π²Π°ΡŽΡ‚ Ρ†ΠΈΡ„Ρ€ΠΎΠ²Ρ‹ΠΌΠΈ ΠΈ мостовыми.Π’ любом случаС, Ссли кондСнсатор впаян Π² Ρ‚ΠΎ ΠΈΠ»ΠΈ ΠΈΠ½ΠΎΠ΅ устройство, Π΅Π³ΠΎ слСдуСт ΠΎΠ±Π΅ΡΡ‚ΠΎΡ‡ΠΈΡ‚ΡŒ, Ρ€Π°Π·Ρ€ΡΠ΄ΠΈΡ‚ΡŒ Π² Π½Π΅ΠΌ кондСнсаторы Ρ„ΠΈΠ»ΡŒΡ‚Ρ€Π° ΠΈ сам кондСнсатор, Π΅ΠΌΠΊΠΎΡΡ‚ΡŒ ΠΊΠΎΡ‚ΠΎΡ€ΠΎΠ³ΠΎ слСдуСт ΠΈΠ·ΠΌΠ΅Ρ€ΠΈΡ‚ΡŒ, ΠΈ лишь послС этого Π²Ρ‹ΠΏΠ°ΡΡ‚ΡŒ Π΅Π³ΠΎ. Π—Π°Ρ‚Π΅ΠΌ Π΅Π³ΠΎ Π½Π΅ΠΎΠ±Ρ…ΠΎΠ΄ΠΈΠΌΠΎ ΠΏΠΎΠ΄ΠΊΠ»ΡŽΡ‡ΠΈΡ‚ΡŒ ΠΊ ΠΏΡ€ΠΈΠ±ΠΎΡ€Ρƒ.На Ρ†ΠΈΡ„Ρ€ΠΎΠ²ΠΎΠΌ ΠΈΠ·ΠΌΠ΅Ρ€ΠΈΡ‚Π΅Π»Π΅ сначала Π²Ρ‹Π±ΠΈΡ€Π°ΡŽΡ‚ самый Π³Ρ€ΡƒΠ±Ρ‹ΠΉ ΠΏΡ€Π΅Π΄Π΅Π», Π·Π°Ρ‚Π΅ΠΌ ΠΏΠ΅Ρ€Π΅ΠΊΠ»ΡŽΡ‡Π°ΡŽΡ‚ Π΅Π³ΠΎ Π΄ΠΎ Ρ‚Π΅Ρ… ΠΏΠΎΡ€, ΠΏΠΎΠΊΠ° ΠΎΠ½ Π½Π΅ ΠΏΠΎΠΊΠ°ΠΆΠ΅Ρ‚ ΠΏΠ΅Ρ€Π΅Π³Ρ€ΡƒΠ·ΠΊΡƒ. ПослС этого ΠΏΠ΅Ρ€Π΅ΠΊΠ»ΡŽΡ‡Π°Ρ‚Π΅Π»ΡŒ пСрСводят Π½Π° ΠΎΠ΄ΠΈΠ½ ΠΏΡ€Π΅Π΄Π΅Π» Π½Π°Π·Π°Π΄ ΠΈ Ρ‡ΠΈΡ‚Π°ΡŽΡ‚ показания, Π° ΠΏΠΎ полоТСнию ΠΏΠ΅Ρ€Π΅ΠΊΠ»ΡŽΡ‡Π°Ρ‚Π΅Π»Ρ ΠΎΠΏΡ€Π΅Π΄Π΅Π»ΡΡŽΡ‚, Π² ΠΊΠ°ΠΊΠΈΡ… Π΅Π΄ΠΈΠ½ΠΈΡ†Π°Ρ… ΠΎΠ½ΠΈ Π²Ρ‹Ρ€Π°ΠΆΠ΅Π½Ρ‹.На мостовом ΠΈΠ·ΠΌΠ΅Ρ€ΠΈΡ‚Π΅Π»Π΅, ΠΏΠΎΡΠ»Π΅Π΄ΠΎΠ²Π°Ρ‚Π΅Π»ΡŒΠ½ΠΎ ΠΏΠ΅Ρ€Π΅ΠΊΠ»ΡŽΡ‡Π°Ρ ΠΏΡ€Π΅Π΄Π΅Π»Ρ‹, Π½Π° ΠΊΠ°ΠΆΠ΄ΠΎΠΌ ΠΈΠ· Π½ΠΈΡ… ΠΏΡ€ΠΎΠΊΡ€ΡƒΡ‡ΠΈΠ²Π°ΡŽΡ‚ рСгулятор ΠΈΠ· ΠΎΠ΄Π½ΠΎΠ³ΠΎ ΠΊΠΎΠ½Ρ†Π° ΡˆΠΊΠ°Π»Ρ‹ Π² Π΄Ρ€ΡƒΠ³ΠΎΠΉ, ΠΏΠΎΠΊΠ° Π·Π²ΡƒΠΊ ΠΈΠ· Π΄ΠΈΠ½Π°ΠΌΠΈΠΊΠ° Π½Π΅ исчСзнСт. Π”ΠΎΠ±ΠΈΠ²ΡˆΠΈΡΡŒ исчСзновСния Π·Π²ΡƒΠΊΠ°, ΠΏΠΎ шкалС рСгулятора ΡΡ‡ΠΈΡ‚Ρ‹Π²Π°ΡŽΡ‚ Ρ€Π΅Π·ΡƒΠ»ΡŒΡ‚Π°Ρ‚, Π° Π΅Π΄ΠΈΠ½ΠΈΡ†Ρ‹, Π² ΠΊΠΎΡ‚ΠΎΡ€Ρ‹Ρ… ΠΎΠ½ Π²Ρ‹Ρ€Π°ΠΆΠ΅Π½, Ρ‚Π°ΠΊΠΆΠ΅ ΠΎΠΏΡ€Π΅Π΄Π΅Π»ΡΡŽΡ‚ ΠΏΠΎ полоТСнию ΠΏΠ΅Ρ€Π΅ΠΊΠ»ΡŽΡ‡Π°Ρ‚Π΅Π»Ρ. Π—Π°Ρ‚Π΅ΠΌ кондСнсатор ΡƒΡΡ‚Π°Π½Π°Π²Π»ΠΈΠ²Π°ΡŽΡ‚ ΠΎΠ±Ρ€Π°Ρ‚Π½ΠΎ Π² устройство.

ΠšΠΎΠ½Π΄Π΅Π½ΡΠ°Ρ‚ΠΎΡ€Ρ‹ ΠΈ диэлСктрики | Π€ΠΈΠ·ΠΈΠΊΠ° II

Π¦Π΅Π»ΠΈ обучСния

К ΠΊΠΎΠ½Ρ†Ρƒ этого Ρ€Π°Π·Π΄Π΅Π»Π° Π²Ρ‹ смоТСтС:

  • ΠžΠΏΠΈΡˆΠΈΡ‚Π΅ дСйствиС кондСнсатора ΠΈ ΠΎΠΏΡ€Π΅Π΄Π΅Π»ΠΈΡ‚Π΅ Π΅ΠΌΠΊΠΎΡΡ‚ΡŒ.
  • ΠžΠ±ΡŠΡΡΠ½ΠΈΡ‚Π΅, ΠΏΠΎΡ‡Π΅ΠΌΡƒ кондСнсаторы с ΠΏΠ°Ρ€Π°Π»Π»Π΅Π»ΡŒΠ½Ρ‹ΠΌΠΈ пластинами ΠΈ ΠΈΡ… Смкости.
  • ΠžΠ±ΡΡƒΠ΄ΠΈΡ‚Π΅ процСсс увСличСния Смкости диэлСктрика.
  • ΠžΠΏΡ€Π΅Π΄Π΅Π»ΠΈΡ‚Π΅ Π΅ΠΌΠΊΠΎΡΡ‚ΡŒ с ΡƒΡ‡Π΅Ρ‚ΠΎΠΌ заряда ΠΈ напряТСния.

ΠšΠΎΠ½Π΄Π΅Π½ΡΠ°Ρ‚ΠΎΡ€ — это устройство для хранСния элСктричСского заряда.ΠšΠΎΠ½Π΄Π΅Π½ΡΠ°Ρ‚ΠΎΡ€Ρ‹ ΠΈΠΌΠ΅ΡŽΡ‚ Ρ€Π°Π·Π»ΠΈΡ‡Π½Ρ‹Π΅ примСнСния: ΠΎΡ‚ Ρ„ΠΈΠ»ΡŒΡ‚Ρ€Π°Ρ†ΠΈΠΈ статичСского элСктричСства ΠΏΡ€ΠΈ радиосигналС Π΄ΠΎ накоплСния энСргии Π² дСфибрилляторах сСрдца. ΠžΠ±Ρ‹Ρ‡Π½ΠΎ Π² ΠΏΡ€ΠΎΠΌΡ‹ΡˆΠ»Π΅Π½Π½Ρ‹Ρ… кондСнсаторах Π΄Π²Π΅ токопроводящиС части располоТСны Π±Π»ΠΈΠ·ΠΊΠΎ Π΄Ρ€ΡƒΠ³ ΠΊ Π΄Ρ€ΡƒΠ³Ρƒ, Π½ΠΎ Π½Π΅ ΡΠΎΠΏΡ€ΠΈΠΊΠ°ΡΠ°ΡŽΡ‚ΡΡ, ΠΊΠ°ΠΊ, Π½Π°ΠΏΡ€ΠΈΠΌΠ΅Ρ€, Π½Π° рисункС 1. (Π’ Π±ΠΎΠ»ΡŒΡˆΠΈΠ½ΡΡ‚Π²Π΅ случаСв ΠΌΠ΅ΠΆΠ΄Ρƒ двумя пластинами ΠΈΡΠΏΠΎΠ»ΡŒΠ·ΡƒΠ΅Ρ‚ΡΡ изолятор для обСспСчСния раздСлСния — см. ΠžΠ±ΡΡƒΠΆΠ΄Π΅Π½ΠΈΠ΅ диэлСктриков Π½ΠΈΠΆΠ΅). ΠšΠ»Π΅ΠΌΠΌΡ‹ Π±Π°Ρ‚Π°Ρ€Π΅ΠΈ ΠΏΠΎΠ΄ΠΊΠ»ΡŽΡ‡Π΅Π½Ρ‹ ΠΊ ΠΏΠ΅Ρ€Π²ΠΎΠ½Π°Ρ‡Π°Π»ΡŒΠ½ΠΎ нСзаряТСнному кондСнсатору, Ρ€Π°Π²Π½Ρ‹Π΅ количСства ΠΏΠΎΠ»ΠΎΠΆΠΈΡ‚Π΅Π»ΡŒΠ½ΠΎΠ³ΠΎ ΠΈ ΠΎΡ‚Ρ€ΠΈΡ†Π°Ρ‚Π΅Π»ΡŒΠ½ΠΎΠ³ΠΎ заряда, + Q ΠΈ — Q , Ρ€Π°Π·Π΄Π΅Π»Π΅Π½Ρ‹ Π½Π° Π΅Π³ΠΎ Π΄Π²Π΅ пластины. ΠšΠΎΠ½Π΄Π΅Π½ΡΠ°Ρ‚ΠΎΡ€ Π² Ρ†Π΅Π»ΠΎΠΌ остаСтся Π½Π΅ΠΉΡ‚Ρ€Π°Π»ΡŒΠ½Ρ‹ΠΌ, Π½ΠΎ Π² этом случаС ΠΌΡ‹ Π½Π°Π·Ρ‹Π²Π°Π΅ΠΌ Π΅Π³ΠΎ хранящим заряд Q .

Рис. 1. Оба кондСнсатора, ΠΏΠΎΠΊΠ°Π·Π°Π½Π½Ρ‹Π΅ здСсь, Π±Ρ‹Π»ΠΈ ΠΈΠ·Π½Π°Ρ‡Π°Π»ΡŒΠ½ΠΎ разряТСны ΠΏΠ΅Ρ€Π΅Π΄ ΠΏΠΎΠ΄ΠΊΠ»ΡŽΡ‡Π΅Π½ΠΈΠ΅ΠΌ ΠΊ Π±Π°Ρ‚Π°Ρ€Π΅Π΅. Π’Π΅ΠΏΠ΅Ρ€ΡŒ Ρƒ Π½ΠΈΡ… Ρ€Π°Π·Π΄Π΅Π»Π΅Π½Ρ‹ заряды + Q ΠΈ — Q Π½Π° Π΄Π²ΡƒΡ… ΠΏΠΎΠ»ΠΎΠ²ΠΈΠ½Π°Ρ…. (Π°) ΠšΠΎΠ½Π΄Π΅Π½ΡΠ°Ρ‚ΠΎΡ€ с ΠΏΠ°Ρ€Π°Π»Π»Π΅Π»ΡŒΠ½Ρ‹ΠΌΠΈ пластинами. (b) Π‘ΠΊΡ€ΡƒΡ‡Π΅Π½Π½Ρ‹ΠΉ кондСнсатор с изоляционным ΠΌΠ°Ρ‚Π΅Ρ€ΠΈΠ°Π»ΠΎΠΌ ΠΌΠ΅ΠΆΠ΄Ρƒ двумя проводящими листами.

ΠšΠΎΠ½Π΄Π΅Π½ΡΠ°Ρ‚ΠΎΡ€

ΠšΠΎΠ½Π΄Π΅Π½ΡΠ°Ρ‚ΠΎΡ€ — это устройство для хранСния элСктричСского заряда.

ΠšΠΎΠ»ΠΈΡ‡Π΅ΡΡ‚Π²ΠΎ заряда Q , ΠΊΠΎΡ‚ΠΎΡ€Ρ‹ΠΉ ΠΌΠΎΠΆΠ΅Ρ‚ Ρ…Ρ€Π°Π½ΠΈΡ‚ΡŒ кондСнсатор , зависит ΠΎΡ‚ Π΄Π²ΡƒΡ… основных Ρ„Π°ΠΊΡ‚ΠΎΡ€ΠΎΠ² — ΠΏΡ€ΠΈΠ»ΠΎΠΆΠ΅Π½Π½ΠΎΠ³ΠΎ напряТСния ΠΈ физичСских характСристик кондСнсатора, Ρ‚Π°ΠΊΠΈΡ… ΠΊΠ°ΠΊ Π΅Π³ΠΎ Ρ€Π°Π·ΠΌΠ΅Ρ€.

ΠšΠΎΠ»ΠΈΡ‡Π΅ΡΡ‚Π²ΠΎ заряда

Q кондСнсатор ΠΌΠΎΠΆΠ΅Ρ‚ Ρ…Ρ€Π°Π½ΠΈΡ‚ΡŒ

ΠšΠΎΠ»ΠΈΡ‡Π΅ΡΡ‚Π²ΠΎ заряда Q , ΠΊΠΎΡ‚ΠΎΡ€Ρ‹ΠΉ ΠΌΠΎΠΆΠ΅Ρ‚ Ρ…Ρ€Π°Π½ΠΈΡ‚ΡŒ кондСнсатор , зависит ΠΎΡ‚ Π΄Π²ΡƒΡ… основных Ρ„Π°ΠΊΡ‚ΠΎΡ€ΠΎΠ² — ΠΏΡ€ΠΈΠ»ΠΎΠΆΠ΅Π½Π½ΠΎΠ³ΠΎ напряТСния ΠΈ физичСских характСристик кондСнсатора, Ρ‚Π°ΠΊΠΈΡ… ΠΊΠ°ΠΊ Π΅Π³ΠΎ Ρ€Π°Π·ΠΌΠ΅Ρ€.

Рис. 2. Π›ΠΈΠ½ΠΈΠΈ элСктричСского поля Π² этом кондСнсаторС с ΠΏΠ°Ρ€Π°Π»Π»Π΅Π»ΡŒΠ½Ρ‹ΠΌΠΈ пластинами, ΠΊΠ°ΠΊ всСгда, Π½Π°Ρ‡ΠΈΠ½Π°ΡŽΡ‚ΡΡ с ΠΏΠΎΠ»ΠΎΠΆΠΈΡ‚Π΅Π»ΡŒΠ½Ρ‹Ρ… зарядов ΠΈ Π·Π°ΠΊΠ°Π½Ρ‡ΠΈΠ²Π°ΡŽΡ‚ΡΡ ΠΎΡ‚Ρ€ΠΈΡ†Π°Ρ‚Π΅Π»ΡŒΠ½Ρ‹ΠΌΠΈ.ΠŸΠΎΡΠΊΠΎΠ»ΡŒΠΊΡƒ Π½Π°ΠΏΡ€ΡΠΆΠ΅Π½Π½ΠΎΡΡ‚ΡŒ элСктричСского поля ΠΏΡ€ΠΎΠΏΠΎΡ€Ρ†ΠΈΠΎΠ½Π°Π»ΡŒΠ½Π° плотности силовых Π»ΠΈΠ½ΠΈΠΉ, ΠΎΠ½Π° Ρ‚Π°ΠΊΠΆΠ΅ ΠΏΡ€ΠΎΠΏΠΎΡ€Ρ†ΠΈΠΎΠ½Π°Π»ΡŒΠ½Π° количСству заряда Π½Π° кондСнсаторС.

БистСма, состоящая ΠΈΠ· Π΄Π²ΡƒΡ… ΠΈΠ΄Π΅Π½Ρ‚ΠΈΡ‡Π½Ρ‹Ρ… ΠΏΠ°Ρ€Π°Π»Π»Π΅Π»ΡŒΠ½Ρ‹Ρ… проводящих пластин, Ρ€Π°Π·Π΄Π΅Π»Π΅Π½Π½Ρ‹Ρ… расстояниСм, ΠΊΠ°ΠΊ ΠΏΠΎΠΊΠ°Π·Π°Π½ΠΎ Π½Π° рисункС 2, называСтся кондСнсатором с ΠΏΠ°Ρ€Π°Π»Π»Π΅Π»ΡŒΠ½Ρ‹ΠΌΠΈ пластинами . Π›Π΅Π³ΠΊΠΎ ΡƒΠ²ΠΈΠ΄Π΅Ρ‚ΡŒ взаимосвязь ΠΌΠ΅ΠΆΠ΄Ρƒ напряТСниСм ΠΈ Π½Π°ΠΊΠΎΠΏΠ»Π΅Π½Π½Ρ‹ΠΌ зарядом для кондСнсатора с ΠΏΠ°Ρ€Π°Π»Π»Π΅Π»ΡŒΠ½Ρ‹ΠΌΠΈ пластинами, ΠΊΠ°ΠΊ ΠΏΠΎΠΊΠ°Π·Π°Π½ΠΎ Π½Π° рисункС 2. КаТдая линия элСктричСского поля начинаСтся с ΠΎΡ‚Π΄Π΅Π»ΡŒΠ½ΠΎΠ³ΠΎ ΠΏΠΎΠ»ΠΎΠΆΠΈΡ‚Π΅Π»ΡŒΠ½ΠΎΠ³ΠΎ заряда ΠΈ заканчиваСтся ΠΎΡ‚Ρ€ΠΈΡ†Π°Ρ‚Π΅Π»ΡŒΠ½Ρ‹ΠΌ, Ρ‚Π°ΠΊ Ρ‡Ρ‚ΠΎ поля Π±ΡƒΠ΄Π΅Ρ‚ большС. Π»ΠΈΠ½ΠΈΠΉ, Ссли большС заряда.(РисованиС ΠΎΠ΄Π½ΠΎΠΉ силовой Π»ΠΈΠ½ΠΈΠΈ Π½Π° ΠΊΠ°ΠΆΠ΄Ρ‹ΠΉ заряд — это Ρ‚ΠΎΠ»ΡŒΠΊΠΎ удобство. ΠœΡ‹ ΠΌΠΎΠΆΠ΅ΠΌ Π½Π°Ρ€ΠΈΡΠΎΠ²Π°Ρ‚ΡŒ ΠΌΠ½ΠΎΠ³ΠΎ силовых Π»ΠΈΠ½ΠΈΠΉ для ΠΊΠ°ΠΆΠ΄ΠΎΠ³ΠΎ заряда, Π½ΠΎ ΠΈΡ… ΠΎΠ±Ρ‰Π΅Π΅ количСство ΠΏΡ€ΠΎΠΏΠΎΡ€Ρ†ΠΈΠΎΠ½Π°Π»ΡŒΠ½ΠΎ количСству зарядов. ) Π’Π°ΠΊΠΈΠΌ ΠΎΠ±Ρ€Π°Π·ΠΎΠΌ, Π½Π°ΠΏΡ€ΡΠΆΠ΅Π½Π½ΠΎΡΡ‚ΡŒ элСктричСского поля прямо ΠΏΡ€ΠΎΠΏΠΎΡ€Ρ†ΠΈΠΎΠ½Π°Π»ΡŒΠ½Π° Ом. .

ПолС ΠΏΡ€ΠΎΠΏΠΎΡ€Ρ†ΠΈΠΎΠ½Π°Π»ΡŒΠ½ΠΎ Π½Π°Ρ‡ΠΈΡΠ»Π΅Π½ΠΈΡŽ:

E ∝ Q ,

, Π³Π΄Π΅ символ ∝ ΠΎΠ·Π½Π°Ρ‡Π°Π΅Ρ‚ Β«ΠΏΡ€ΠΎΠΏΠΎΡ€Ρ†ΠΈΠΎΠ½Π°Π»ΡŒΠ½ΠΎΒ». Из обсуТдСния Π² ΡΡ‚Π°Ρ‚ΡŒΠ΅ «ЭлСктричСский ΠΏΠΎΡ‚Π΅Π½Ρ†ΠΈΠ°Π» Π² ΠΎΠ΄Π½ΠΎΡ€ΠΎΠ΄Π½ΠΎΠΌ элСктричСском ΠΏΠΎΠ»Π΅Β» ΠΌΡ‹ Π·Π½Π°Π΅ΠΌ, Ρ‡Ρ‚ΠΎ напряТСниС Π½Π° ΠΏΠ°Ρ€Π°Π»Π»Π΅Π»ΡŒΠ½Ρ‹Ρ… пластинах Ρ€Π°Π²Π½ΠΎ

.

V = Ed .

Π’Π°ΠΊΠΈΠΌ ΠΎΠ±Ρ€Π°Π·ΠΎΠΌ, V ∝ E . ΠžΡ‚ΡΡŽΠ΄Π° слСдуСт, Ρ‡Ρ‚ΠΎ V ∝ Q , ΠΈ, Π½Π°ΠΎΠ±ΠΎΡ€ΠΎΡ‚,

Q ∝ V .

Π’ Ρ†Π΅Π»ΠΎΠΌ это Π²Π΅Ρ€Π½ΠΎ: Ρ‡Π΅ΠΌ большС напряТСниС, ΠΏΡ€ΠΈΠ»ΠΎΠΆΠ΅Π½Π½ΠΎΠ΅ ΠΊ Π»ΡŽΠ±ΠΎΠΌΡƒ кондСнсатору, Ρ‚Π΅ΠΌ большС Π² Π½Π΅ΠΌ хранится заряд.

Π Π°Π·Π»ΠΈΡ‡Π½Ρ‹Π΅ кондСнсаторы Π±ΡƒΠ΄ΡƒΡ‚ Π½Π°ΠΊΠ°ΠΏΠ»ΠΈΠ²Π°Ρ‚ΡŒ Ρ€Π°Π·Π½ΠΎΠ΅ количСство заряда для ΠΎΠ΄Π½ΠΎΠ³ΠΎ ΠΈ Ρ‚ΠΎΠ³ΠΎ ΠΆΠ΅ ΠΏΡ€ΠΈΠ»ΠΎΠΆΠ΅Π½Π½ΠΎΠ³ΠΎ напряТСния Π² зависимости ΠΎΡ‚ ΠΈΡ… физичСских характСристик. ΠœΡ‹ опрСдСляСм ΠΈΡ… Π΅ΠΌΠΊΠΎΡΡ‚ΡŒ C ΠΊΠ°ΠΊ Ρ‚Π°ΠΊΡƒΡŽ, Ρ‡Ρ‚ΠΎ заряд Q , хранящийся Π² кондСнсаторС, ΠΏΡ€ΠΎΠΏΠΎΡ€Ρ†ΠΈΠΎΠ½Π°Π»Π΅Π½ C .Заряд, Π½Π°ΠΊΠΎΠΏΠ»Π΅Π½Π½Ρ‹ΠΉ Π² кондСнсаторС, Ρ€Π°Π²Π΅Π½

.

Q = CV .

Π­Ρ‚ΠΎ ΡƒΡ€Π°Π²Π½Π΅Π½ΠΈΠ΅ Π²Ρ‹Ρ€Π°ΠΆΠ°Π΅Ρ‚ Π΄Π²Π° основных Ρ„Π°ΠΊΡ‚ΠΎΡ€Π°, Π²Π»ΠΈΡΡŽΡ‰ΠΈΡ… Π½Π° количСство Π½Π°ΠΊΠΎΠΏΠ»Π΅Π½Π½ΠΎΠ³ΠΎ заряда. Π­Ρ‚ΠΈΠΌΠΈ Ρ„Π°ΠΊΡ‚ΠΎΡ€Π°ΠΌΠΈ ΡΠ²Π»ΡΡŽΡ‚ΡΡ физичСскиС характСристики кондСнсатора C ΠΈ напряТСниС Π’ . ИзмСнив ΡƒΡ€Π°Π²Π½Π΅Π½ΠΈΠ΅, ΠΌΡ‹ Π²ΠΈΠ΄ΠΈΠΌ, Ρ‡Ρ‚ΠΎ Π΅ΠΌΠΊΠΎΡΡ‚ΡŒ C — это количСство заряда, сохранСнного Π½Π° Π²ΠΎΠ»ΡŒΡ‚, ΠΈΠ»ΠΈ

[латСкс] C = \ frac {Q} {V} \\ [/ latex].

Π•ΠΌΠΊΠΎΡΡ‚ΡŒ

Π•ΠΌΠΊΠΎΡΡ‚ΡŒ C — это Π²Π΅Π»ΠΈΡ‡ΠΈΠ½Π° Π½Π°ΠΊΠΎΠΏΠ»Π΅Π½Π½ΠΎΠ³ΠΎ заряда Π½Π° Π²ΠΎΠ»ΡŒΡ‚, ΠΈΠ»ΠΈ

[латСкс] C = \ frac {Q} {V} \\ [/ latex]

Π•Π΄ΠΈΠ½ΠΈΡ†Π° измСрСния Смкости — Ρ„Π°Ρ€Π°Π΄ (Π€), названная Π² Ρ‡Π΅ΡΡ‚ΡŒ Майкла ЀарадСя (1791–1867), английского ΡƒΡ‡Π΅Π½ΠΎΠ³ΠΎ, внСсшСго Π²ΠΊΠ»Π°Π΄ Π² области элСктромагнСтизма ΠΈ элСктрохимии. ΠŸΠΎΡΠΊΠΎΠ»ΡŒΠΊΡƒ Π΅ΠΌΠΊΠΎΡΡ‚ΡŒ — это заряд Π½Π° Π΅Π΄ΠΈΠ½ΠΈΡ†Ρƒ напряТСния, ΠΌΡ‹ Π²ΠΈΠ΄ΠΈΠΌ, Ρ‡Ρ‚ΠΎ Ρ„Π°Ρ€Π°Π΄ — это ΠΊΡƒΠ»ΠΎΠ½ Π½Π° Π²ΠΎΠ»ΡŒΡ‚, ΠΈΠ»ΠΈ

.

[латСкс] 1 \ text {F} = \ frac {1 \ text {C}} {1 \ text {V}} \\ [/ latex].

ΠšΠΎΠ½Π΄Π΅Π½ΡΠ°Ρ‚ΠΎΡ€ Π΅ΠΌΠΊΠΎΡΡ‚ΡŒΡŽ 1 Ρ„Π°Ρ€Π°Π΄ ΠΌΠΎΠΆΠ΅Ρ‚ Ρ…Ρ€Π°Π½ΠΈΡ‚ΡŒ 1 ΠΊΡƒΠ»ΠΎΠ½ (ΠΎΡ‡Π΅Π½ΡŒ большоС количСство заряда) ΠΏΡ€ΠΈ ΠΏΠΎΠ΄Π°Ρ‡Π΅ всСго 1 Π²ΠΎΠ»ΡŒΡ‚. Π’Π°ΠΊΠΈΠΌ ΠΎΠ±Ρ€Π°Π·ΠΎΠΌ, ΠΎΠ΄Π½Π° Ρ„Π°Ρ€Π°Π΄Π° — это ΠΎΡ‡Π΅Π½ΡŒ большая Π΅ΠΌΠΊΠΎΡΡ‚ΡŒ. Π’ΠΈΠΏΠΈΡ‡Π½Ρ‹ΠΉ Π΄ΠΈΠ°ΠΏΠ°Π·ΠΎΠ½ кондСнсаторов составляСт ΠΎΡ‚ Π΄ΠΎΠ»Π΅ΠΉ ΠΏΠΈΠΊΠΎΡ„Π°Ρ€Π°Π΄Π° (1 ΠΏΠ€ = 10 βˆ’12 Π€) Π΄ΠΎ ΠΌΠΈΠ»Π»ΠΈΡ„Π°Ρ€Π°Π΄ΠΎΠ² (1 ΠΌΠ€ = 10 βˆ’3 Π€).

На рисункС 3 ΠΏΠΎΠΊΠ°Π·Π°Π½Ρ‹ Π½Π΅ΠΊΠΎΡ‚ΠΎΡ€Ρ‹Π΅ распространСнныС кондСнсаторы. ΠšΠΎΠ½Π΄Π΅Π½ΡΠ°Ρ‚ΠΎΡ€Ρ‹ Π² основном ΠΈΠ·Π³ΠΎΡ‚Π°Π²Π»ΠΈΠ²Π°ΡŽΡ‚ΡΡ ΠΈΠ· ΠΊΠ΅Ρ€Π°ΠΌΠΈΠΊΠΈ, стСкла ΠΈΠ»ΠΈ пластика, Π² зависимости ΠΎΡ‚ назначСния ΠΈ Ρ€Π°Π·ΠΌΠ΅Ρ€Π°. Как обсуТдаСтся Π½ΠΈΠΆΠ΅, Π² ΠΈΡ… конструкции ΠΎΠ±Ρ‹Ρ‡Π½ΠΎ ΠΈΡΠΏΠΎΠ»ΡŒΠ·ΡƒΡŽΡ‚ΡΡ изоляционныС ΠΌΠ°Ρ‚Π΅Ρ€ΠΈΠ°Π»Ρ‹, Π½Π°Π·Ρ‹Π²Π°Π΅ΠΌΡ‹Π΅ диэлСктриками.

Рисунок 3. НСкоторыС Ρ‚ΠΈΠΏΠΈΡ‡Π½Ρ‹Π΅ кондСнсаторы. Π Π°Π·ΠΌΠ΅Ρ€ ΠΈ Π·Π½Π°Ρ‡Π΅Π½ΠΈΠ΅ Смкости Π½Π΅ ΠΎΠ±ΡΠ·Π°Ρ‚Π΅Π»ΡŒΠ½ΠΎ связаны. (Π˜ΡΡ‚ΠΎΡ‡Π½ΠΈΠΊ: Windell Oskay)

ΠšΠΎΠ½Π΄Π΅Π½ΡΠ°Ρ‚ΠΎΡ€ с ΠΏΠ°Ρ€Π°Π»Π»Π΅Π»ΡŒΠ½ΠΎΠΉ пластиной

Рис. 4. ΠšΠΎΠ½Π΄Π΅Π½ΡΠ°Ρ‚ΠΎΡ€ с ΠΏΠ°Ρ€Π°Π»Π»Π΅Π»ΡŒΠ½Ρ‹ΠΌΠΈ пластинами, Ρ€Π°Π·Π΄Π΅Π»Π΅Π½Π½Ρ‹Π΅ пластинами Π½Π° расстояниС d. КаТдая пластина ΠΈΠΌΠ΅Π΅Ρ‚ ΠΏΠ»ΠΎΡ‰Π°Π΄ΡŒ A.

ΠšΠΎΠ½Π΄Π΅Π½ΡΠ°Ρ‚ΠΎΡ€ с ΠΏΠ°Ρ€Π°Π»Π»Π΅Π»ΡŒΠ½Ρ‹ΠΌΠΈ пластинами, ΠΏΠΎΠΊΠ°Π·Π°Π½Π½Ρ‹ΠΉ Π½Π° рис. 4, ΠΈΠΌΠ΅Π΅Ρ‚ Π΄Π²Π΅ ΠΈΠ΄Π΅Π½Ρ‚ΠΈΡ‡Π½Ρ‹Π΅ проводящиС пластины, каТдая ΠΈΠ· ΠΊΠΎΡ‚ΠΎΡ€Ρ‹Ρ… ΠΈΠΌΠ΅Π΅Ρ‚ ΠΏΠ»ΠΎΡ‰Π°Π΄ΡŒ повСрхности A, , Ρ€Π°Π·Π΄Π΅Π»Π΅Π½Π½Ρ‹Ρ… расстояниСм d (Π±Π΅Π· ΠΌΠ°Ρ‚Π΅Ρ€ΠΈΠ°Π»Π° ΠΌΠ΅ΠΆΠ΄Ρƒ пластинами). Когда Π½Π° кондСнсатор подаСтся напряТСниС Π’, , ΠΎΠ½ сохраняСт заряд Q , ΠΊΠ°ΠΊ ΠΏΠΎΠΊΠ°Π·Π°Π½ΠΎ. ΠœΡ‹ ΠΌΠΎΠΆΠ΅ΠΌ ΡƒΠ²ΠΈΠ΄Π΅Ρ‚ΡŒ, ΠΊΠ°ΠΊ Π΅Π³ΠΎ Π΅ΠΌΠΊΠΎΡΡ‚ΡŒ зависит ΠΎΡ‚ A ΠΈ d , рассмотрСв характСристики кулоновской силы. ΠœΡ‹ Π·Π½Π°Π΅ΠΌ, Ρ‡Ρ‚ΠΎ ΠΎΠ΄ΠΈΠ½Π°ΠΊΠΎΠ²Ρ‹Π΅ заряды ΠΎΡ‚Ρ‚Π°Π»ΠΊΠΈΠ²Π°ΡŽΡ‚ΡΡ, Π² ΠΎΡ‚Π»ΠΈΡ‡ΠΈΠ΅ ΠΎΡ‚ зарядов ΠΏΡ€ΠΈΡ‚ΡΠ³ΠΈΠ²Π°ΡŽΡ‚ΡΡ, ΠΈ сила ΠΌΠ΅ΠΆΠ΄Ρƒ зарядами ΡƒΠΌΠ΅Π½ΡŒΡˆΠ°Π΅Ρ‚ΡΡ с расстояниСм. ΠŸΠΎΡΡ‚ΠΎΠΌΡƒ каТСтся Π²ΠΏΠΎΠ»Π½Π΅ Ρ€Π°Π·ΡƒΠΌΠ½Ρ‹ΠΌ, Ρ‡Ρ‚ΠΎ Ρ‡Π΅ΠΌ большС пластины, Ρ‚Π΅ΠΌ большС заряда ΠΎΠ½ΠΈ ΠΌΠΎΠ³ΡƒΡ‚ Ρ…Ρ€Π°Π½ΠΈΡ‚ΡŒ, ΠΏΠΎΡ‚ΠΎΠΌΡƒ Ρ‡Ρ‚ΠΎ заряды ΠΌΠΎΠ³ΡƒΡ‚ Ρ€Π°ΡΡ…ΠΎΠ΄ΠΈΡ‚ΡŒΡΡ большС. Π’Π°ΠΊΠΈΠΌ ΠΎΠ±Ρ€Π°Π·ΠΎΠΌ, C Π΄ΠΎΠ»ΠΆΠ΅Π½ Π±Ρ‹Ρ‚ΡŒ большС для большСго A .Π’ΠΎΡ‡Π½ΠΎ Ρ‚Π°ΠΊ ΠΆΠ΅, Ρ‡Π΅ΠΌ Π±Π»ΠΈΠΆΠ΅ пластины Π΄Ρ€ΡƒΠ³ ΠΊ Π΄Ρ€ΡƒΠ³Ρƒ, Ρ‚Π΅ΠΌ большС Π½Π° Π½ΠΈΡ… притяТСния ΠΏΡ€ΠΎΡ‚ΠΈΠ²ΠΎΠΏΠΎΠ»ΠΎΠΆΠ½Ρ‹Ρ… зарядов. Π—Π½Π°Ρ‡ΠΈΡ‚, C Π΄ΠΎΠ»ΠΆΠ½ΠΎ Π±Ρ‹Ρ‚ΡŒ большС для мСньшСго d .

МоТно ΠΏΠΎΠΊΠ°Π·Π°Ρ‚ΡŒ, Ρ‡Ρ‚ΠΎ для кондСнсатора с ΠΏΠ°Ρ€Π°Π»Π»Π΅Π»ΡŒΠ½Ρ‹ΠΌΠΈ пластинами Π΅ΡΡ‚ΡŒ Ρ‚ΠΎΠ»ΡŒΠΊΠΎ Π΄Π²Π° Ρ„Π°ΠΊΡ‚ΠΎΡ€Π° ( A, ΠΈ d ), ΠΊΠΎΡ‚ΠΎΡ€Ρ‹Π΅ Π²Π»ΠΈΡΡŽΡ‚ Π½Π° Π΅Π³ΠΎ Π΅ΠΌΠΊΠΎΡΡ‚ΡŒ C . Π•ΠΌΠΊΠΎΡΡ‚ΡŒ кондСнсатора с ΠΏΠ°Ρ€Π°Π»Π»Π΅Π»ΡŒΠ½Ρ‹ΠΌΠΈ пластинами Π² Ρ„ΠΎΡ€ΠΌΠ΅ уравнСния Ρ€Π°Π²Π½Π°

.

[латСкс] C = \ epsilon_ {o} \ frac {A} {d} \\ [/ latex].

Π•ΠΌΠΊΠΎΡΡ‚ΡŒ ΠΏΠ°Ρ€Π°Π»Π»Π΅Π»ΡŒΠ½ΠΎΠ³ΠΎ пластинчатого кондСнсатора

[латСкс] C = \ epsilon_ {o} \ frac {A} {d} \\ [/ latex]

A — это ΠΏΠ»ΠΎΡ‰Π°Π΄ΡŒ ΠΎΠ΄Π½ΠΎΠΉ пластины Π² ΠΊΠ²Π°Π΄Ρ€Π°Ρ‚Π½Ρ‹Ρ… ΠΌΠ΅Ρ‚Ρ€Π°Ρ…, Π° d — это расстояниС ΠΌΠ΅ΠΆΠ΄Ρƒ пластинами Π² ΠΌΠ΅Ρ‚Ρ€Π°Ρ….ΠšΠΎΠ½ΡΡ‚Π°Π½Ρ‚Π° Ξ΅ 0 — диэлСктричСская ΠΏΡ€ΠΎΠ½ΠΈΡ†Π°Π΅ΠΌΠΎΡΡ‚ΡŒ свободного пространства; Π΅Π³ΠΎ числовоС Π·Π½Π°Ρ‡Π΅Π½ΠΈΠ΅ Π² Π΅Π΄ΠΈΠ½ΠΈΡ†Π°Ρ… БИ составляСт Ξ΅ 0 = 8,85 Γ— 10 βˆ’12 Π€ / ΠΌ. Π•Π΄ΠΈΠ½ΠΈΡ†Ρ‹ измСрСния Π€ / ΠΌ эквивалСнтны C 2 / Н Β· ΠΌ 2 . НСбольшоС числовоС Π·Π½Π°Ρ‡Π΅Π½ΠΈΠ΅ Ξ΅ 0 связано с большим Ρ€Π°Π·ΠΌΠ΅Ρ€ΠΎΠΌ Ρ„Π°Ρ€Π°Π΄Π°. ΠšΠΎΠ½Π΄Π΅Π½ΡΠ°Ρ‚ΠΎΡ€ с ΠΏΠ°Ρ€Π°Π»Π»Π΅Π»ΡŒΠ½Ρ‹ΠΌΠΈ пластинами Π΄ΠΎΠ»ΠΆΠ΅Π½ ΠΈΠΌΠ΅Ρ‚ΡŒ Π±ΠΎΠ»ΡŒΡˆΡƒΡŽ ΠΏΠ»ΠΎΡ‰Π°Π΄ΡŒ, Ρ‡Ρ‚ΠΎΠ±Ρ‹ Π΅Π³ΠΎ Π΅ΠΌΠΊΠΎΡΡ‚ΡŒ ΠΏΡ€ΠΈΠ±Π»ΠΈΠΆΠ°Π»Π°ΡΡŒ ΠΊ Ρ„Π°Ρ€Π°Π΄Ρƒ. (ΠžΠ±Ρ€Π°Ρ‚ΠΈΡ‚Π΅ Π²Π½ΠΈΠΌΠ°Π½ΠΈΠ΅, Ρ‡Ρ‚ΠΎ ΠΏΡ€ΠΈΠ²Π΅Π΄Π΅Π½Π½ΠΎΠ΅ Π²Ρ‹ΡˆΠ΅ ΡƒΡ€Π°Π²Π½Π΅Π½ΠΈΠ΅ Π΄Π΅ΠΉΡΡ‚Π²ΠΈΡ‚Π΅Π»ΡŒΠ½ΠΎ, ΠΊΠΎΠ³Π΄Π° ΠΏΠ°Ρ€Π°Π»Π»Π΅Π»ΡŒΠ½Ρ‹Π΅ пластины Ρ€Π°Π·Π΄Π΅Π»Π΅Π½Ρ‹ Π²ΠΎΠ·Π΄ΡƒΡ…ΠΎΠΌ ΠΈΠ»ΠΈ свободным пространством.Когда ΠΌΠ΅ΠΆΠ΄Ρƒ пластинами помСщаСтся Π΄Ρ€ΡƒΠ³ΠΎΠΉ ΠΌΠ°Ρ‚Π΅Ρ€ΠΈΠ°Π», ΡƒΡ€Π°Π²Π½Π΅Π½ΠΈΠ΅ измСняСтся, ΠΊΠ°ΠΊ обсуТдаСтся Π½ΠΈΠΆΠ΅.)

ΠŸΡ€ΠΈΠΌΠ΅Ρ€ 1.

Π•ΠΌΠΊΠΎΡΡ‚ΡŒ ΠΈ заряд, Π½Π°ΠΊΠΎΠΏΠ»Π΅Π½Π½Ρ‹ΠΉ Π² ΠΏΠ°Ρ€Π°Π»Π»Π΅Π»ΡŒΠ½ΠΎΠΌ пластинчатом кондСнсаторС

  1. Какова Π΅ΠΌΠΊΠΎΡΡ‚ΡŒ кондСнсатора с ΠΏΠ°Ρ€Π°Π»Π»Π΅Π»ΡŒΠ½Ρ‹ΠΌΠΈ пластинами, каТдая ΠΈΠ· ΠΊΠΎΡ‚ΠΎΡ€Ρ‹Ρ… ΠΈΠΌΠ΅Π΅Ρ‚ ΠΏΠ»ΠΎΡ‰Π°Π΄ΡŒ 1,00 ΠΌ 2 , Ρ€Π°Π·Π΄Π΅Π»Π΅Π½Π½Ρ‹Ρ… расстояниСм 1,00 ΠΌΠΌ?
  2. Какой заряд хранится Π² этом кондСнсаторС, Ссли Π½Π° Π½Π΅Π³ΠΎ ΠΏΡ€ΠΈΠ»ΠΎΠΆΠ΅Π½ΠΎ напряТСниС 3,00 Γ— 10 3 Π’?
БтратСгия

ΠžΠΏΡ€Π΅Π΄Π΅Π»Π΅Π½ΠΈΠ΅ Смкости C прСдставляСт собой прямоС ΠΏΡ€ΠΈΠ»ΠΎΠΆΠ΅Π½ΠΈΠ΅ уравнСния [латСкс] C = \ epsilon_ {o} \ frac {A} {d} \\ [/ latex].{-9} \ text {F} = 8.85 \ text {nF} \ end {array} \\ [/ latex]

ΠžΠ±ΡΡƒΠΆΠ΄Π΅Π½ΠΈΠ΅ части 1

Π­Ρ‚ΠΎ нСбольшоС Π·Π½Π°Ρ‡Π΅Π½ΠΈΠ΅ Смкости ΡƒΠΊΠ°Π·Ρ‹Π²Π°Π΅Ρ‚ Π½Π° Ρ‚ΠΎ, насколько слоТно ΠΈΠ·Π³ΠΎΡ‚ΠΎΠ²ΠΈΡ‚ΡŒ устройство с большой Π΅ΠΌΠΊΠΎΡΡ‚ΡŒΡŽ. ΠŸΠΎΠΌΠΎΠ³Π°ΡŽΡ‚ ΡΠΏΠ΅Ρ†ΠΈΠ°Π»ΡŒΠ½Ρ‹Π΅ ΠΏΡ€ΠΈΠ΅ΠΌΡ‹, Π½Π°ΠΏΡ€ΠΈΠΌΠ΅Ρ€, использованиС Ρ‚ΠΎΠ½ΠΊΠΈΡ… ΠΏΠ»Π΅Π½ΠΎΠΊ ΠΎΡ‡Π΅Π½ΡŒ большой ΠΏΠ»ΠΎΡ‰Π°Π΄ΠΈ, располоТСнных Π±Π»ΠΈΠ·ΠΊΠΎ Π΄Ρ€ΡƒΠ³ ΠΊ Π΄Ρ€ΡƒΠ³Ρƒ.

РСшСниС для Части 2

Заряд любого кондСнсатора опрСдСляСтся ΡƒΡ€Π°Π²Π½Π΅Π½ΠΈΠ΅ΠΌ Q = CV . Π’Π²ΠΎΠ΄ извСстных Π·Π½Π°Ρ‡Π΅Π½ΠΈΠΉ Π² это ΡƒΡ€Π°Π²Π½Π΅Π½ΠΈΠ΅ Π΄Π°Π΅Ρ‚

[латСкс] \ begin {array} {lll} Q & = & CV = \ left (8. {3} \ text {V} \ right) \\\ text {} & = & 26.6 \ mu \ text {C} \ end {array} \\ [/ latex]

ΠžΠ±ΡΡƒΠΆΠ΄Π΅Π½ΠΈΠ΅ части 2

Π­Ρ‚ΠΎΡ‚ заряд лишь Π½Π΅ΠΌΠ½ΠΎΠ³ΠΎ большС, Ρ‡Π΅ΠΌ ΠΏΡ€ΠΈ ΠΎΠ±Ρ‹Ρ‡Π½ΠΎΠΌ статичСском элСктричСствС. ΠŸΠΎΡΠΊΠΎΠ»ΡŒΠΊΡƒ Π²ΠΎΠ·Π΄ΡƒΡ… разрываСтся ΠΏΡ€ΠΈ ΠΏΡ€ΠΈΠΌΠ΅Ρ€Π½ΠΎ 3,00 Γ— 10 6 Π’ / ΠΌ, Π½Π° этом кондСнсаторС Π½Π΅ ΠΌΠΎΠΆΠ΅Ρ‚ Π±Ρ‹Ρ‚ΡŒ Π½Π°ΠΊΠΎΠΏΠ»Π΅Π½ΠΎ большС заряда Π·Π° счСт увСличСния напряТСния.

Π”Ρ€ΡƒΠ³ΠΎΠΉ интСрСсный биологичСский ΠΏΡ€ΠΈΠΌΠ΅Ρ€, связанный с элСктричСским ΠΏΠΎΡ‚Π΅Π½Ρ†ΠΈΠ°Π»ΠΎΠΌ, ΠΎΠ±Π½Π°Ρ€ΡƒΠΆΠ΅Π½ Π² плазматичСской ΠΌΠ΅ΠΌΠ±Ρ€Π°Π½Π΅ ΠΊΠ»Π΅Ρ‚ΠΊΠΈ. ΠœΠ΅ΠΌΠ±Ρ€Π°Π½Π° отдСляСт ΠΊΠ»Π΅Ρ‚ΠΊΡƒ ΠΎΡ‚ ΠΎΠΊΡ€ΡƒΠΆΠ°ΡŽΡ‰Π΅ΠΉ срСды, Π° Ρ‚Π°ΠΊΠΆΠ΅ позволяСт ΠΈΠΎΠ½Π°ΠΌ Π²Ρ‹Π±ΠΎΡ€ΠΎΡ‡Π½ΠΎ Π²Ρ…ΠΎΠ΄ΠΈΡ‚ΡŒ ΠΈ Π²Ρ‹Ρ…ΠΎΠ΄ΠΈΡ‚ΡŒ ΠΈΠ· ΠΊΠ»Π΅Ρ‚ΠΊΠΈ.БущСствуСт Ρ€Π°Π·Π½ΠΎΡΡ‚ΡŒ ΠΏΠΎΡ‚Π΅Π½Ρ†ΠΈΠ°Π»ΠΎΠ² Π½Π° ΠΌΠ΅ΠΌΠ±Ρ€Π°Π½Π΅ ΠΎΠΊΠΎΠ»ΠΎ –70 ΠΌΠ’. Π­Ρ‚ΠΎ связано с ΠΏΡ€Π΅ΠΎΠ±Π»Π°Π΄Π°Π½ΠΈΠ΅ΠΌ Π² ΠΊΠ»Π΅Ρ‚ΠΊΠ΅ ΠΎΡ‚Ρ€ΠΈΡ†Π°Ρ‚Π΅Π»ΡŒΠ½ΠΎ заряТСнных ΠΈΠΎΠ½ΠΎΠ² ΠΈ ΠΏΡ€Π΅ΠΎΠ±Π»Π°Π΄Π°Π½ΠΈΠ΅ΠΌ ΠΏΠΎΠ»ΠΎΠΆΠΈΡ‚Π΅Π»ΡŒΠ½ΠΎ заряТСнных ΠΈΠΎΠ½ΠΎΠ² натрия (Na + ) снаруТи. ВсС мСняСтся ΠΏΡ€ΠΈ стимуляции Π½Π΅Ρ€Π²Π½ΠΎΠΉ ΠΊΠ»Π΅Ρ‚ΠΊΠΈ. Π˜ΠΎΠ½Ρ‹ Na + проходят Ρ‡Π΅Ρ€Π΅Π· ΠΌΠ΅ΠΌΠ±Ρ€Π°Π½Ρƒ Π² ΠΊΠ»Π΅Ρ‚ΠΊΡƒ, создавая ΠΏΠΎΠ»ΠΎΠΆΠΈΡ‚Π΅Π»ΡŒΠ½Ρ‹ΠΉ ΠΌΠ΅ΠΌΠ±Ρ€Π°Π½Π½Ρ‹ΠΉ ΠΏΠΎΡ‚Π΅Π½Ρ†ΠΈΠ°Π» — Π½Π΅Ρ€Π²Π½Ρ‹ΠΉ сигнал. ΠšΠ»Π΅Ρ‚ΠΎΡ‡Π½Π°Ρ ΠΌΠ΅ΠΌΠ±Ρ€Π°Π½Π° ΠΈΠΌΠ΅Π΅Ρ‚ Ρ‚ΠΎΠ»Ρ‰ΠΈΠ½Ρƒ ΠΎΡ‚ 7 Π΄ΠΎ 10 Π½ΠΌ. {6} \ text {V / m} \\ [/ latex]

Π­Ρ‚ΠΎΠ³ΠΎ элСктричСского поля достаточно, Ρ‡Ρ‚ΠΎΠ±Ρ‹ Π²Ρ‹Π·Π²Π°Ρ‚ΡŒ ΠΏΡ€ΠΎΠ±ΠΎΠΉ Π² Π²ΠΎΠ·Π΄ΡƒΡ…Π΅.

ДиэлСктрик

ΠŸΡ€Π΅Π΄Ρ‹Π΄ΡƒΡ‰ΠΈΠΉ ΠΏΡ€ΠΈΠΌΠ΅Ρ€ ΠΏΠΎΠ΄Ρ‡Π΅Ρ€ΠΊΠΈΠ²Π°Π΅Ρ‚ ΡΠ»ΠΎΠΆΠ½ΠΎΡΡ‚ΡŒ сохранСния большого количСства заряда Π² кондСнсаторах. Если d ΡΠ΄Π΅Π»Π°Ρ‚ΡŒ мСньшС, Ρ‡Ρ‚ΠΎΠ±Ρ‹ ΠΏΠΎΠ»ΡƒΡ‡ΠΈΡ‚ΡŒ Π±ΠΎΠ»ΡŒΡˆΡƒΡŽ Π΅ΠΌΠΊΠΎΡΡ‚ΡŒ, Ρ‚ΠΎ максимальноС напряТСниС Π΄ΠΎΠ»ΠΆΠ½ΠΎ Π±Ρ‹Ρ‚ΡŒ ΡƒΠΌΠ΅Π½ΡŒΡˆΠ΅Π½ΠΎ ΠΏΡ€ΠΎΠΏΠΎΡ€Ρ†ΠΈΠΎΠ½Π°Π»ΡŒΠ½ΠΎ, Ρ‡Ρ‚ΠΎΠ±Ρ‹ ΠΈΠ·Π±Π΅ΠΆΠ°Ρ‚ΡŒ пробоя (ΠΏΠΎΡΠΊΠΎΠ»ΡŒΠΊΡƒ [латСкс] E = \ frac {V} {d} \\ [/ latex]). Π’Π°ΠΆΠ½Ρ‹ΠΌ Ρ€Π΅ΡˆΠ΅Π½ΠΈΠ΅ΠΌ этой ΠΏΡ€ΠΎΠ±Π»Π΅ΠΌΡ‹ являСтся Ρ€Π°Π·ΠΌΠ΅Ρ‰Π΅Π½ΠΈΠ΅ изоляционного ΠΌΠ°Ρ‚Π΅Ρ€ΠΈΠ°Π»Π°, Π½Π°Π·Ρ‹Π²Π°Π΅ΠΌΠΎΠ³ΠΎ диэлСктриком , ΠΌΠ΅ΠΆΠ΄Ρƒ пластинами кондСнсатора ΠΈ обСспСчСниС минимально Π²ΠΎΠ·ΠΌΠΎΠΆΠ½ΠΎΠ³ΠΎ Ρ€Π°Π·ΠΌΠ΅Ρ€Π° d .Мало Ρ‚ΠΎΠ³ΠΎ, Ρ‡Ρ‚ΠΎ мСньший d ΡƒΠ²Π΅Π»ΠΈΡ‡ΠΈΠ²Π°Π΅Ρ‚ Π΅ΠΌΠΊΠΎΡΡ‚ΡŒ, Π½ΠΎ ΠΌΠ½ΠΎΠ³ΠΈΠ΅ изоляторы ΠΌΠΎΠ³ΡƒΡ‚ Π²Ρ‹Π΄Π΅Ρ€ΠΆΠΈΠ²Π°Ρ‚ΡŒ Π±ΠΎΠ»Π΅Π΅ ΡΠΈΠ»ΡŒΠ½Ρ‹Π΅ элСктричСскиС поля, Ρ‡Π΅ΠΌ Π²ΠΎΠ·Π΄ΡƒΡ…, ΠΏΡ€Π΅ΠΆΠ΄Π΅ Ρ‡Π΅ΠΌ ΠΎΠ½ΠΈ ΡΠ»ΠΎΠΌΠ°ΡŽΡ‚ΡΡ.

Π•ΡΡ‚ΡŒ Π΅Ρ‰Π΅ ΠΎΠ΄Π½ΠΎ прСимущСство использования диэлСктрика Π² кондСнсаторС. Π’ зависимости ΠΎΡ‚ ΠΈΡΠΏΠΎΠ»ΡŒΠ·ΡƒΠ΅ΠΌΠΎΠ³ΠΎ ΠΌΠ°Ρ‚Π΅Ρ€ΠΈΠ°Π»Π° Π΅ΠΌΠΊΠΎΡΡ‚ΡŒ большС, Ρ‡Π΅ΠΌ заданная ΡƒΡ€Π°Π²Π½Π΅Π½ΠΈΠ΅ΠΌ [латСкс] C = \ kappa \ epsilon_ {0} \ frac {A} {d} \\ [/ latex], Π½Π° коэффициСнт ΞΊ , Π½Π°Π·Ρ‹Π²Π°Π΅ΠΌΡ‹ΠΉ диэлСктричСская ΠΏΡ€ΠΎΠ½ΠΈΡ†Π°Π΅ΠΌΠΎΡΡ‚ΡŒ . ΠšΠΎΠ½Π΄Π΅Π½ΡΠ°Ρ‚ΠΎΡ€ с ΠΏΠ°Ρ€Π°Π»Π»Π΅Π»ΡŒΠ½Ρ‹ΠΌΠΈ пластинами с диэлСктриком ΠΌΠ΅ΠΆΠ΄Ρƒ пластинами ΠΈΠΌΠ΅Π΅Ρ‚ Π΅ΠΌΠΊΠΎΡΡ‚ΡŒ, ΠΎΠΏΡ€Π΅Π΄Π΅Π»ΡΠ΅ΠΌΡƒΡŽ Π²Ρ‹Ρ€Π°ΠΆΠ΅Π½ΠΈΠ΅ΠΌ [латСкс] C = \ kappa \ epsilon_ {0} \ frac {A} {d} \\ [/ latex] (кондСнсатор с ΠΏΠ°Ρ€Π°Π»Π»Π΅Π»ΡŒΠ½Ρ‹ΠΌΠΈ пластинами с диэлСктриком).

ЗначСния диэлСктричСской проницаСмости ΞΊ для Ρ€Π°Π·Π»ΠΈΡ‡Π½Ρ‹Ρ… ΠΌΠ°Ρ‚Π΅Ρ€ΠΈΠ°Π»ΠΎΠ² ΠΏΡ€ΠΈΠ²Π΅Π΄Π΅Π½Ρ‹ Π² Ρ‚Π°Π±Π»ΠΈΡ†Π΅ 1. ΠžΠ±Ρ€Π°Ρ‚ΠΈΡ‚Π΅ Π²Π½ΠΈΠΌΠ°Π½ΠΈΠ΅, Ρ‡Ρ‚ΠΎ ΞΊ для Π²Π°ΠΊΡƒΡƒΠΌΠ° Ρ€Π°Π²Π½ΠΎ 1, ΠΈ поэтому ΠΏΡ€ΠΈΠ²Π΅Π΄Π΅Π½Π½ΠΎΠ΅ Π²Ρ‹ΡˆΠ΅ ΡƒΡ€Π°Π²Π½Π΅Π½ΠΈΠ΅ справСдливо ΠΈ Π² этом случаС. Если ΠΈΡΠΏΠΎΠ»ΡŒΠ·ΠΎΠ²Π°Ρ‚ΡŒ диэлСктрик, Π½Π°ΠΏΡ€ΠΈΠΌΠ΅Ρ€, помСстив Ρ‚Π΅Ρ„Π»ΠΎΠ½ ΠΌΠ΅ΠΆΠ΄Ρƒ пластинами кондСнсатора Π² ΠΏΡ€ΠΈΠΌΠ΅Ρ€Π΅ 1, Ρ‚ΠΎ Π΅ΠΌΠΊΠΎΡΡ‚ΡŒ Π±ΡƒΠ΄Π΅Ρ‚ большС Π² ΞΊ Ρ€Π°Π·, Ρ‡Ρ‚ΠΎ для Ρ‚Π΅Ρ„Π»ΠΎΠ½Π° составляСт 2,1.

ЭкспСримСнт Π½Π° вынос: созданиС кондСнсатора

Насколько большой кондСнсатор ΠΌΠΎΠΆΠ½ΠΎ ΡΠ΄Π΅Π»Π°Ρ‚ΡŒ с ΠΏΠΎΠΌΠΎΡ‰ΡŒΡŽ ΠΎΠ±Π΅Ρ€Ρ‚ΠΊΠΈ ΠΎΡ‚ ΠΆΠ΅Π²Π°Ρ‚Π΅Π»ΡŒΠ½ΠΎΠΉ Ρ€Π΅Π·ΠΈΠ½ΠΊΠΈ? ΠŸΠ»Π°ΡΡ‚ΠΈΠ½Ρ‹ Π±ΡƒΠ΄ΡƒΡ‚ ΠΈΠ· алюминиСвой Ρ„ΠΎΠ»ΡŒΠ³ΠΈ, Π° Ρ€Π°Π·Π΄Π΅Π»ΠΈΡ‚Π΅Π»ΡŒ (диэлСктрик) ΠΌΠ΅ΠΆΠ΄Ρƒ Π½ΠΈΠΌΠΈ — ΠΈΠ· Π±ΡƒΠΌΠ°Π³ΠΈ.

Π’Π°Π±Π»ΠΈΡ†Π° 1. ДиэлСктричСская ΠΏΡ€ΠΎΠ½ΠΈΡ†Π°Π΅ΠΌΠΎΡΡ‚ΡŒ ΠΈ диэлСктричСская ΠΏΡ€ΠΎΡ‡Π½ΠΎΡΡ‚ΡŒ для Ρ€Π°Π·Π»ΠΈΡ‡Π½Ρ‹Ρ… ΠΌΠ°Ρ‚Π΅Ρ€ΠΈΠ°Π»ΠΎΠ² ΠΏΡ€ΠΈ 20ΒΊC
ΠœΠ°Ρ‚Π΅Ρ€ΠΈΠ°Π» ДиэлСктричСская ΠΏΡ€ΠΎΠ½ΠΈΡ†Π°Π΅ΠΌΠΎΡΡ‚ΡŒ ΞΊ ЭлСктричСская ΠΏΡ€ΠΎΡ‡Π½ΠΎΡΡ‚ΡŒ (Π’ / ΠΌ)
Π’Π°ΠΊΡƒΡƒΠΌ 1,00000–
Π’ΠΎΠ·Π΄ΡƒΡ… 1.00059 3 Γ— 10 6
Π‘Π°ΠΊΠ΅Π»ΠΈΡ‚ 4,9 24 Γ— 10 6
ΠŸΠ»Π°Π²Π»Π΅Π½Ρ‹ΠΉ ΠΊΠ²Π°Ρ€Ρ† 3.78 8 Γ— 10 6
НСопрСновый ΠΊΠ°ΡƒΡ‡ΡƒΠΊ 6,7 12 Γ— 10 6
НСйлон 3,4 14 Γ— 10 6
Π‘ΡƒΠΌΠ°Π³Π° 3,7 16 Γ— 10 6
ΠŸΠΎΠ»ΠΈΡΡ‚ΠΈΡ€ΠΎΠ» 2,56 24 Γ— 10 6
Π‘Ρ‚Π΅ΠΊΠ»ΠΎ Pyrex 5,6 14 Γ— 10 6
ΠšΡ€Π΅ΠΌΠ½ΠΈΠ΅Π²ΠΎΠ΅ масло 2. 5 15 Γ— 10 6
Ρ‚ΠΈΡ‚Π°Π½Π°Ρ‚ стронция 233 8 Γ— 10 6
Π’Π΅Ρ„Π»ΠΎΠ½ 2,1 60 Γ— 10 6
Π’ΠΎΠ΄Π° 80–

ΠžΠ±Ρ€Π°Ρ‚ΠΈΡ‚Π΅ Π²Π½ΠΈΠΌΠ°Π½ΠΈΠ΅, Ρ‡Ρ‚ΠΎ диэлСктричСская ΠΏΡ€ΠΎΠ½ΠΈΡ†Π°Π΅ΠΌΠΎΡΡ‚ΡŒ Π²ΠΎΠ·Π΄ΡƒΡ…Π° ΠΎΡ‡Π΅Π½ΡŒ Π±Π»ΠΈΠ·ΠΊΠ° ΠΊ 1, Ρ‚Π°ΠΊ Ρ‡Ρ‚ΠΎ кондСнсаторы с Π²ΠΎΠ·Π΄ΡƒΡˆΠ½Ρ‹ΠΌ Π½Π°ΠΏΠΎΠ»Π½Π΅Π½ΠΈΠ΅ΠΌ Π΄Π΅ΠΉΡΡ‚Π²ΡƒΡŽΡ‚ Ρ‚Π°ΠΊ ΠΆΠ΅, ΠΊΠ°ΠΊ кондСнсаторы с Π²Π°ΠΊΡƒΡƒΠΌΠΎΠΌ ΠΌΠ΅ΠΆΠ΄Ρƒ пластинами , Π·Π° ΠΈΡΠΊΠ»ΡŽΡ‡Π΅Π½ΠΈΠ΅ΠΌ , Ρ‡Ρ‚ΠΎ Π²ΠΎΠ·Π΄ΡƒΡ… ΠΌΠΎΠΆΠ΅Ρ‚ ΡΡ‚Π°Ρ‚ΡŒ проводящим, Ссли Π½Π°ΠΏΡ€ΡΠΆΠ΅Π½Π½ΠΎΡΡ‚ΡŒ элСктричСского поля становится Ρ€Π°Π²Π½ΠΎΠΉ. слишком большой.(Напомним, Ρ‡Ρ‚ΠΎ [латСкс] E = \ frac {V} {d} \\ [/ latex] для кондСнсатора с ΠΏΠ°Ρ€Π°Π»Π»Π΅Π»ΡŒΠ½Ρ‹ΠΌΠΈ пластинами.) Π’ Ρ‚Π°Π±Π»ΠΈΡ†Π΅ 1 Ρ‚Π°ΠΊΠΆΠ΅ ΠΏΠΎΠΊΠ°Π·Π°Π½Ρ‹ ΠΌΠ°ΠΊΡΠΈΠΌΠ°Π»ΡŒΠ½Ρ‹Π΅ напряТСнности элСктричСского поля Π² Π’ / ΠΌ, ΠΊΠΎΡ‚ΠΎΡ€Ρ‹Π΅ Π½Π°Π·Ρ‹Π²Π°ΡŽΡ‚ΡΡ диэлСктричСской ΠΏΡ€ΠΎΡ‡Π½ΠΎΡΡ‚ΡŒΡŽ , для Π½Π΅ΡΠΊΠΎΠ»ΡŒΠΊΠΈΡ… ΠΌΠ°Ρ‚Π΅Ρ€ΠΈΠ°Π»ΠΎΠ². Π­Ρ‚ΠΎ поля, Π½Π°Π΄ ΠΊΠΎΡ‚ΠΎΡ€Ρ‹ΠΌΠΈ ΠΌΠ°Ρ‚Π΅Ρ€ΠΈΠ°Π» Π½Π°Ρ‡ΠΈΠ½Π°Π΅Ρ‚ Ρ€Π°Π·Ρ€ΡƒΡˆΠ°Ρ‚ΡŒΡΡ ΠΈ ΠΏΡ€ΠΎΠ²ΠΎΠ΄ΠΈΡ‚ΡŒ. ДиэлСктричСская ΠΏΡ€ΠΎΡ‡Π½ΠΎΡΡ‚ΡŒ Π½Π°ΠΊΠ»Π°Π΄Ρ‹Π²Π°Π΅Ρ‚ ΠΎΠ³Ρ€Π°Π½ΠΈΡ‡Π΅Π½ΠΈΠ΅ Π½Π° напряТСниС, ΠΊΠΎΡ‚ΠΎΡ€ΠΎΠ΅ ΠΌΠΎΠΆΠ΅Ρ‚ Π±Ρ‹Ρ‚ΡŒ ΠΏΡ€ΠΈΠ»ΠΎΠΆΠ΅Π½ΠΎ для Π΄Π°Π½Π½ΠΎΠ³ΠΎ расстояния ΠΌΠ΅ΠΆΠ΄Ρƒ пластинами. 6 \ text {V / m} \ right) \ left ( 1.{-3} \ text {m} \ right) \\\ text {} & = & 3000 \ text {V} \ end {array} \\ [/ latex]

Однако ΠΏΡ€Π΅Π΄Π΅Π» для расстояния Π² 1,00 ΠΌΠΌ, Π·Π°ΠΏΠΎΠ»Π½Π΅Π½Π½ΠΎΠ³ΠΎ Ρ‚Π΅Ρ„Π»ΠΎΠ½ΠΎΠΌ, составляСт 60 000 Π’, ΠΏΠΎΡΠΊΠΎΠ»ΡŒΠΊΡƒ элСктричСская ΠΏΡ€ΠΎΡ‡Π½ΠΎΡΡ‚ΡŒ Ρ‚Π΅Ρ„Π»ΠΎΠ½Π° составляСт 60 Γ— 10 6 Π’ / ΠΌ. Π’Π°ΠΊΠΈΠΌ ΠΎΠ±Ρ€Π°Π·ΠΎΠΌ, Ρ‚ΠΎΡ‚ ΠΆΠ΅ кондСнсатор, Π·Π°ΠΏΠΎΠ»Π½Π΅Π½Π½Ρ‹ΠΉ Ρ‚Π΅Ρ„Π»ΠΎΠ½ΠΎΠΌ, ΠΈΠΌΠ΅Π΅Ρ‚ Π±ΠΎΠ»ΡŒΡˆΡƒΡŽ Π΅ΠΌΠΊΠΎΡΡ‚ΡŒ ΠΈ ΠΌΠΎΠΆΠ΅Ρ‚ ΠΏΠΎΠ΄Π²Π΅Ρ€Π³Π°Ρ‚ΡŒΡΡ Π³ΠΎΡ€Π°Π·Π΄ΠΎ Π±ΠΎΠ»ΡŒΡˆΠ΅ΠΌΡƒ Π½Π°ΠΏΡ€ΡΠΆΠ΅Π½ΠΈΡŽ. Π˜ΡΠΏΠΎΠ»ΡŒΠ·ΡƒΡ Π΅ΠΌΠΊΠΎΡΡ‚ΡŒ, ΠΊΠΎΡ‚ΠΎΡ€ΡƒΡŽ ΠΌΡ‹ рассчитали Π² ΠΏΡ€ΠΈΠ²Π΅Π΄Π΅Π½Π½ΠΎΠΌ Π²Ρ‹ΡˆΠ΅ ΠΏΡ€ΠΈΠΌΠ΅Ρ€Π΅ для кондСнсатора с ΠΏΠ°Ρ€Π°Π»Π»Π΅Π»ΡŒΠ½Ρ‹ΠΌΠΈ пластинами, Π·Π°ΠΏΠΎΠ»Π½Π΅Π½Π½ΠΎΠ³ΠΎ Π²ΠΎΠ·Π΄ΡƒΡ…ΠΎΠΌ, ΠΌΡ‹ ΠΎΠ±Π½Π°Ρ€ΡƒΠΆΠΈΠ»ΠΈ, Ρ‡Ρ‚ΠΎ кондСнсатор с Ρ‚Π΅Ρ„Π»ΠΎΠ½ΠΎΠ²Ρ‹ΠΌ Π·Π°ΠΏΠΎΠ»Π½Π΅Π½ΠΈΠ΅ΠΌ ΠΌΠΎΠΆΠ΅Ρ‚ Ρ…Ρ€Π°Π½ΠΈΡ‚ΡŒ ΠΌΠ°ΠΊΡΠΈΠΌΠ°Π»ΡŒΠ½Ρ‹ΠΉ заряд

[латСкс] \ begin {array} {lll} Q & = & CV \\\ text {} & = & \ kappa {C} _ {\ text {air}} V \\\ text {} & = & (2.4 \ text {V}) \\\ text {} & = & 1.1 \ text {mC} \ end {array} \\ [/ latex]

Π­Ρ‚ΠΎ Π² 42 Ρ€Π°Π·Π° большС заряда Ρ‚ΠΎΠ³ΠΎ ΠΆΠ΅ кондСнсатора, Π·Π°ΠΏΠΎΠ»Π½Π΅Π½Π½ΠΎΠ³ΠΎ Π²ΠΎΠ·Π΄ΡƒΡ…ΠΎΠΌ.

ДиэлСктричСская ΠΏΡ€ΠΎΡ‡Π½ΠΎΡΡ‚ΡŒ

Максимальная Π½Π°ΠΏΡ€ΡΠΆΠ΅Π½Π½ΠΎΡΡ‚ΡŒ элСктричСского поля, ΠΏΡ€ΠΈ ΠΏΡ€Π΅Π²Ρ‹ΡˆΠ΅Π½ΠΈΠΈ ΠΊΠΎΡ‚ΠΎΡ€ΠΎΠΉ изоляционный ΠΌΠ°Ρ‚Π΅Ρ€ΠΈΠ°Π» Π½Π°Ρ‡ΠΈΠ½Π°Π΅Ρ‚ Ρ€Π°Π·Ρ€ΡƒΡˆΠ°Ρ‚ΡŒΡΡ ΠΈ становится ΠΏΡ€ΠΎΠ²ΠΎΠ΄Π½ΠΈΠΊΠΎΠΌ, называСтся Π΅Π³ΠΎ диэлСктричСской ΠΏΡ€ΠΎΡ‡Π½ΠΎΡΡ‚ΡŒΡŽ.

ΠœΠΈΠΊΡ€ΠΎΡΠΊΠΎΠΏΠΈΡ‡Π΅ΡΠΊΠΈ, ΠΊΠ°ΠΊ диэлСктрик ΡƒΠ²Π΅Π»ΠΈΡ‡ΠΈΠ²Π°Π΅Ρ‚ Π΅ΠΌΠΊΠΎΡΡ‚ΡŒ? Π—Π° это ΠΎΡ‚Π²Π΅Ρ‡Π°Π΅Ρ‚ поляризация изолятора. Π§Π΅ΠΌ Π»Π΅Π³Ρ‡Π΅ ΠΎΠ½ поляризуСтся, Ρ‚Π΅ΠΌ большС Π΅Π³ΠΎ диэлСктричСская ΠΏΡ€ΠΎΠ½ΠΈΡ†Π°Π΅ΠΌΠΎΡΡ‚ΡŒ ΞΊ .Π’ΠΎΠ΄Π°, Π½Π°ΠΏΡ€ΠΈΠΌΠ΅Ρ€, прСдставляСт собой ΠΏΠΎΠ»ΡΡ€Π½ΡƒΡŽ ΠΌΠΎΠ»Π΅ΠΊΡƒΠ»Ρƒ , ΠΏΠΎΡ‚ΠΎΠΌΡƒ Ρ‡Ρ‚ΠΎ ΠΎΠ΄ΠΈΠ½ ΠΊΠΎΠ½Π΅Ρ† ΠΌΠΎΠ»Π΅ΠΊΡƒΠ»Ρ‹ ΠΈΠΌΠ΅Π΅Ρ‚ нСбольшой ΠΏΠΎΠ»ΠΎΠΆΠΈΡ‚Π΅Π»ΡŒΠ½Ρ‹ΠΉ заряд, Π° Π΄Ρ€ΡƒΠ³ΠΎΠΉ ΠΊΠΎΠ½Π΅Ρ† ΠΈΠΌΠ΅Π΅Ρ‚ нСбольшой ΠΎΡ‚Ρ€ΠΈΡ†Π°Ρ‚Π΅Π»ΡŒΠ½Ρ‹ΠΉ заряд. ΠŸΠΎΠ»ΡΡ€Π½ΠΎΡΡ‚ΡŒ Π²ΠΎΠ΄Ρ‹ обуславливаСт Π΅Π΅ ΠΎΡ‚Π½ΠΎΡΠΈΡ‚Π΅Π»ΡŒΠ½ΠΎ Π±ΠΎΠ»ΡŒΡˆΡƒΡŽ Π΄ΠΈΡΠ»Π΅ΠΊΡ‚Ρ€ΠΈΡ‡Π΅ΡΠΊΡƒΡŽ ΠΏΡ€ΠΎΠ½ΠΈΡ†Π°Π΅ΠΌΠΎΡΡ‚ΡŒ, Ρ€Π°Π²Π½ΡƒΡŽ 80. Π­Ρ„Ρ„Π΅ΠΊΡ‚ поляризации Π»ΡƒΡ‡ΡˆΠ΅ всСго ΠΌΠΎΠΆΠ½ΠΎ ΠΎΠ±ΡŠΡΡΠ½ΠΈΡ‚ΡŒ с Ρ‚ΠΎΡ‡ΠΊΠΈ зрСния характСристик кулоновской силы. На рис. 5 схСматично ΠΏΠΎΠΊΠ°Π·Π°Π½ΠΎ Ρ€Π°Π·Π΄Π΅Π»Π΅Π½ΠΈΠ΅ зарядов Π² ΠΌΠΎΠ»Π΅ΠΊΡƒΠ»Π°Ρ… диэлСктричСского ΠΌΠ°Ρ‚Π΅Ρ€ΠΈΠ°Π»Π°, ΠΏΠΎΠΌΠ΅Ρ‰Π΅Π½Π½Ρ‹Ρ… ΠΌΠ΅ΠΆΠ΄Ρƒ заряТСнными пластинами кондСнсатора. ΠšΡƒΠ»ΠΎΠ½ΠΎΠ²ΡΠΊΠ°Ρ сила ΠΌΠ΅ΠΆΠ΄Ρƒ блиТайшими ΠΊΠΎΠ½Ρ†Π°ΠΌΠΈ ΠΌΠΎΠ»Π΅ΠΊΡƒΠ» ΠΈ зарядом Π½Π° пластинах притягиваСт ΠΈ ΠΎΡ‡Π΅Π½ΡŒ сильна, ΠΏΠΎΡΠΊΠΎΠ»ΡŒΠΊΡƒ ΠΎΠ½ΠΈ располоТСны ΠΎΡ‡Π΅Π½ΡŒ Π±Π»ΠΈΠ·ΠΊΠΎ Π΄Ρ€ΡƒΠ³ ΠΊ Π΄Ρ€ΡƒΠ³Ρƒ.Π­Ρ‚ΠΎ притягиваСт большС заряда ΠΊ пластинам, Ρ‡Π΅ΠΌ Ссли Π±Ρ‹ пространство Π±Ρ‹Π»ΠΎ пустым, Π° ΠΏΡ€ΠΎΡ‚ΠΈΠ²ΠΎΠΏΠΎΠ»ΠΎΠΆΠ½Ρ‹Π΅ заряды Π½Π°Ρ…ΠΎΠ΄ΠΈΠ»ΠΈΡΡŒ Π½Π° расстоянии d ΠΎΡ‚.

Рис. 5. (a) ΠœΠΎΠ»Π΅ΠΊΡƒΠ»Ρ‹ изоляционного ΠΌΠ°Ρ‚Π΅Ρ€ΠΈΠ°Π»Π° ΠΌΠ΅ΠΆΠ΄Ρƒ пластинами кондСнсатора поляризованы заряТСнными пластинами. Π­Ρ‚ΠΎ создаСт слой ΠΏΡ€ΠΎΡ‚ΠΈΠ²ΠΎΠΏΠΎΠ»ΠΎΠΆΠ½ΠΎΠ³ΠΎ заряда Π½Π° повСрхности диэлСктрика, ΠΊΠΎΡ‚ΠΎΡ€Ρ‹ΠΉ притягиваСт большС заряда ΠΊ пластинС, увСличивая Π΅Π΅ Π΅ΠΌΠΊΠΎΡΡ‚ΡŒ. (Π±) ДиэлСктрик сниТаСт Π½Π°ΠΏΡ€ΡΠΆΠ΅Π½Π½ΠΎΡΡ‚ΡŒ элСктричСского поля Π²Π½ΡƒΡ‚Ρ€ΠΈ кондСнсатора, Ρ‡Ρ‚ΠΎ ΠΏΡ€ΠΈΠ²ΠΎΠ΄ΠΈΡ‚ ΠΊ ΡƒΠΌΠ΅Π½ΡŒΡˆΠ΅Π½ΠΈΡŽ напряТСния ΠΌΠ΅ΠΆΠ΄Ρƒ пластинами ΠΏΡ€ΠΈ ΠΎΠ΄ΠΈΠ½Π°ΠΊΠΎΠ²ΠΎΠΌ зарядС.ΠšΠΎΠ½Π΄Π΅Π½ΡΠ°Ρ‚ΠΎΡ€ сохраняСт Ρ‚ΠΎΡ‚ ΠΆΠ΅ заряд ΠΏΡ€ΠΈ мСньшСм напряТСнии, Ρ‡Ρ‚ΠΎ ΠΎΠ·Π½Π°Ρ‡Π°Π΅Ρ‚, Ρ‡Ρ‚ΠΎ ΠΎΠ½ ΠΈΠΌΠ΅Π΅Ρ‚ Π±ΠΎΠ»ΡŒΡˆΡƒΡŽ Π΅ΠΌΠΊΠΎΡΡ‚ΡŒ ΠΈΠ·-Π·Π° диэлСктрика.

Π”Ρ€ΡƒΠ³ΠΎΠΉ способ ΠΏΠΎΠ½ΡΡ‚ΡŒ, ΠΊΠ°ΠΊ диэлСктрик ΡƒΠ²Π΅Π»ΠΈΡ‡ΠΈΠ²Π°Π΅Ρ‚ Π΅ΠΌΠΊΠΎΡΡ‚ΡŒ, — это Ρ€Π°ΡΡΠΌΠΎΡ‚Ρ€Π΅Ρ‚ΡŒ Π΅Π³ΠΎ влияниС Π½Π° элСктричСскоС ΠΏΠΎΠ»Π΅ Π²Π½ΡƒΡ‚Ρ€ΠΈ кондСнсатора. На рис. 5 (Π±) ΠΏΠΎΠΊΠ°Π·Π°Π½Ρ‹ силовыС Π»ΠΈΠ½ΠΈΠΈ элСктричСского поля с установлСнным диэлСктриком. ΠŸΠΎΡΠΊΠΎΠ»ΡŒΠΊΡƒ силовыС Π»ΠΈΠ½ΠΈΠΈ Π·Π°ΠΊΠ°Π½Ρ‡ΠΈΠ²Π°ΡŽΡ‚ΡΡ зарядами Π² диэлСктрикС, ΠΈΡ… мСньшС, ΠΈΠ΄ΡƒΡ‰ΠΈΡ… ΠΎΡ‚ ΠΎΠ΄Π½ΠΎΠΉ стороны кондСнсатора ΠΊ Π΄Ρ€ΡƒΠ³ΠΎΠΉ. Π’Π°ΠΊΠΈΠΌ ΠΎΠ±Ρ€Π°Π·ΠΎΠΌ, Π½Π°ΠΏΡ€ΡΠΆΠ΅Π½Π½ΠΎΡΡ‚ΡŒ элСктричСского поля мСньшС, Ρ‡Π΅ΠΌ Ссли Π±Ρ‹ ΠΌΠ΅ΠΆΠ΄Ρƒ пластинами Π±Ρ‹Π» Π²Π°ΠΊΡƒΡƒΠΌ, Π΄Π°ΠΆΠ΅ Ссли Π±Ρ‹ Π½Π° пластинах Π±Ρ‹Π» ΠΎΠ΄ΠΈΠ½Π°ΠΊΠΎΠ²Ρ‹ΠΉ заряд. НапряТСниС ΠΌΠ΅ΠΆΠ΄Ρƒ пластинами составляСт Π’, = Π•Π΄, , поэтому ΠΎΠ½ΠΎ Ρ‚ΠΎΠΆΠ΅ сниТаСтся Π·Π° счСт диэлСктрика. Π’Π°ΠΊΠΈΠΌ ΠΎΠ±Ρ€Π°Π·ΠΎΠΌ Π΅ΡΡ‚ΡŒ мСньшСС напряТСниС Π’, для Ρ‚ΠΎΠ³ΠΎ ΠΆΠ΅ заряда Q ; ΠΏΠΎΡΠΊΠΎΠ»ΡŒΠΊΡƒ [латСкс] C = \ frac {Q} {V} \\ [/ latex], Π΅ΠΌΠΊΠΎΡΡ‚ΡŒ C, большС.

ДиэлСктричСская постоянная ΠΎΠ±Ρ‹Ρ‡Π½ΠΎ опрСдСляСтся ΠΊΠ°ΠΊ [латСкс] \ kappa = \ frac {E_0} {E} \\ [/ latex], ΠΈΠ»ΠΈ ΠΎΡ‚Π½ΠΎΡˆΠ΅Π½ΠΈΠ΅ элСктричСского поля Π² Π²Π°ΠΊΡƒΡƒΠΌΠ΅ ΠΊ элСктричСскому полю Π² диэлСктричСском ΠΌΠ°Ρ‚Π΅Ρ€ΠΈΠ°Π»Π΅, ΠΈ Π² ΠΊΠΎΠ½Π΅Ρ‡Π½ΠΎΠΌ ΠΈΡ‚ΠΎΠ³Π΅ связанных с ΠΏΠΎΠ»ΡΡ€ΠΈΠ·ΡƒΠ΅ΠΌΠΎΡΡ‚ΡŒΡŽ ΠΌΠ°Ρ‚Π΅Ρ€ΠΈΠ°Π»Π°.

Π’Π΅Π»ΠΈΠΊΠΈΠ΅ ΠΈ ΠΌΠ°Π»Ρ‹Π΅ Π²Π΅Ρ‰ΠΈ: субмикроскопичСскоС происхоТдСниС поляризации

ΠŸΠΎΠ»ΡΡ€ΠΈΠ·Π°Ρ†ΠΈΡ — это Ρ€Π°Π·Π΄Π΅Π»Π΅Π½ΠΈΠ΅ зарядов Π² Π°Ρ‚ΠΎΠΌΠ΅ ΠΈΠ»ΠΈ ΠΌΠΎΠ»Π΅ΠΊΡƒΠ»Π΅. Как ΡƒΠΆΠ΅ ΠΎΡ‚ΠΌΠ΅Ρ‡Π°Π»ΠΎΡΡŒ, планСтарная модСль Π°Ρ‚ΠΎΠΌΠ° описываСт Π΅Π³ΠΎ ΠΊΠ°ΠΊ ΠΈΠΌΠ΅ΡŽΡ‰Π΅Π΅ ΠΏΠΎΠ»ΠΎΠΆΠΈΡ‚Π΅Π»ΡŒΠ½ΠΎΠ΅ ядро, Π²Ρ€Π°Ρ‰Π°ΡŽΡ‰Π΅Π΅ΡΡ Π²ΠΎΠΊΡ€ΡƒΠ³ ΠΎΡ‚Ρ€ΠΈΡ†Π°Ρ‚Π΅Π»ΡŒΠ½Ρ‹Ρ… элСктронов, Π°Π½Π°Π»ΠΎΠ³ΠΈΡ‡Π½ΠΎ ΠΏΠ»Π°Π½Π΅Ρ‚Π°ΠΌ, Π²Ρ€Π°Ρ‰Π°ΡŽΡ‰ΠΈΠΌΡΡ Π²ΠΎΠΊΡ€ΡƒΠ³ Π‘ΠΎΠ»Π½Ρ†Π°. Π₯отя эта модСль Π½Π΅ совсСм Ρ‚ΠΎΡ‡Π½Π°, ΠΎΠ½Π° ΠΎΡ‡Π΅Π½ΡŒ ΠΏΠΎΠ»Π΅Π·Π½Π° для объяснСния ΡˆΠΈΡ€ΠΎΠΊΠΎΠ³ΠΎ ΠΊΡ€ΡƒΠ³Π° явлСний ΠΈ Π±ΡƒΠ΄Π΅Ρ‚ ΡƒΡ‚ΠΎΡ‡Π½Π΅Π½Π° Π² Π΄Ρ€ΡƒΠ³ΠΈΡ… мСстах, Π½Π°ΠΏΡ€ΠΈΠΌΠ΅Ρ€, Π² Π°Ρ‚ΠΎΠΌΠ½ΠΎΠΉ Ρ„ΠΈΠ·ΠΈΠΊΠ΅. БубмикроскопичСскоС происхоТдСниС поляризации ΠΌΠΎΠΆΠ½ΠΎ ΡΠΌΠΎΠ΄Π΅Π»ΠΈΡ€ΠΎΠ²Π°Ρ‚ΡŒ, ΠΊΠ°ΠΊ ΠΏΠΎΠΊΠ°Π·Π°Π½ΠΎ Π½Π° рисункС 6.

Рис. 6. Π₯удоТСствСнноС прСдставлСниС ΠΎ поляризованном Π°Ρ‚ΠΎΠΌΠ΅. ΠžΡ€Π±ΠΈΡ‚Ρ‹ элСктронов Π²ΠΎΠΊΡ€ΡƒΠ³ ядра Π½Π΅ΠΌΠ½ΠΎΠ³ΠΎ смСщСны внСшними зарядами (ΠΏΠΎΠΊΠ°Π·Π°Π½Ρ‹ Π² ΠΏΡ€Π΅ΡƒΠ²Π΅Π»ΠΈΡ‡Π΅Π½ΠΈΠΈ). ΠŸΠΎΠ»ΡƒΡ‡Π°ΡŽΡ‰Π΅Π΅ΡΡ Ρ€Π°Π·Π΄Π΅Π»Π΅Π½ΠΈΠ΅ зарядов Π²Π½ΡƒΡ‚Ρ€ΠΈ Π°Ρ‚ΠΎΠΌΠ° ΠΎΠ·Π½Π°Ρ‡Π°Π΅Ρ‚, Ρ‡Ρ‚ΠΎ ΠΎΠ½ поляризован. ΠžΠ±Ρ€Π°Ρ‚ΠΈΡ‚Π΅ Π²Π½ΠΈΠΌΠ°Π½ΠΈΠ΅, Ρ‡Ρ‚ΠΎ Π½Π΅ΠΏΠΎΡ…ΠΎΠΆΠΈΠΉ заряд Ρ‚Π΅ΠΏΠ΅Ρ€ΡŒ Π±Π»ΠΈΠΆΠ΅ ΠΊ внСшним зарядам, вызывая ΠΏΠΎΠ»ΡΡ€ΠΈΠ·Π°Ρ†ΠΈΡŽ.

Π’ Π°Ρ‚ΠΎΠΌΠ½ΠΎΠΉ Ρ„ΠΈΠ·ΠΈΠΊΠ΅ ΠΌΡ‹ ΠΎΠ±Π½Π°Ρ€ΡƒΠΆΠΈΠΌ, Ρ‡Ρ‚ΠΎ ΠΎΡ€Π±ΠΈΡ‚Ρ‹ элСктронов Π±ΠΎΠ»Π΅Π΅ ΠΏΡ€Π°Π²ΠΈΠ»ΡŒΠ½ΠΎ Ρ€Π°ΡΡΠΌΠ°Ρ‚Ρ€ΠΈΠ²Π°Ρ‚ΡŒ ΠΊΠ°ΠΊ элСктронныС ΠΎΠ±Π»Π°ΠΊΠ° с ΠΏΠ»ΠΎΡ‚Π½ΠΎΡΡ‚ΡŒΡŽ ΠΎΠ±Π»Π°ΠΊΠ°, связанной с Π²Π΅Ρ€ΠΎΡΡ‚Π½ΠΎΡΡ‚ΡŒΡŽ нахоТдСния элСктрона Π² этом мСстС (Π² ΠΎΡ‚Π»ΠΈΡ‡ΠΈΠ΅ ΠΎΡ‚ ΠΎΠΏΡ€Π΅Π΄Π΅Π»Π΅Π½Π½Ρ‹Ρ… мСстополоТСний ΠΈ ΠΏΡƒΡ‚Π΅ΠΉ ΠΏΠ»Π°Π½Π΅Ρ‚ Π½Π° ΠΈΡ… ΠΎΡ€Π±ΠΈΡ‚Π°Ρ…). Π²ΠΎΠΊΡ€ΡƒΠ³ Π‘ΠΎΠ»Π½Ρ†Π°).Π­Ρ‚ΠΎ ΠΎΠ±Π»Π°ΠΊΠΎ сдвигаСтся кулоновской силой, Ρ‚Π°ΠΊ Ρ‡Ρ‚ΠΎ Π² срСднСм Π°Ρ‚ΠΎΠΌ ΠΈΠΌΠ΅Π΅Ρ‚ Ρ€Π°Π·Π΄Π΅Π»Π΅Π½Π½Ρ‹ΠΉ заряд. Π₯отя Π°Ρ‚ΠΎΠΌ остаСтся Π½Π΅ΠΉΡ‚Ρ€Π°Π»ΡŒΠ½Ρ‹ΠΌ, Ρ‚Π΅ΠΏΠ΅Ρ€ΡŒ ΠΎΠ½ ΠΌΠΎΠΆΠ΅Ρ‚ Π±Ρ‹Ρ‚ΡŒ источником кулоновской силы, ΠΏΠΎΡΠΊΠΎΠ»ΡŒΠΊΡƒ заряд, поднСсСнный ΠΊ Π°Ρ‚ΠΎΠΌΡƒ, Π±ΡƒΠ΄Π΅Ρ‚ Π±Π»ΠΈΠΆΠ΅ ΠΊ ΠΎΠ΄Π½ΠΎΠΌΡƒ Ρ‚ΠΈΠΏΡƒ заряда, Ρ‡Π΅ΠΌ ΠΊ Π΄Ρ€ΡƒΠ³ΠΎΠΌΡƒ.

НСкоторым ΠΌΠΎΠ»Π΅ΠΊΡƒΠ»Π°ΠΌ, Π½Π°ΠΏΡ€ΠΈΠΌΠ΅Ρ€ ΠΌΠΎΠ»Π΅ΠΊΡƒΠ»Π°ΠΌ Π²ΠΎΠ΄Ρ‹, присущС Ρ€Π°Π·Π΄Π΅Π»Π΅Π½ΠΈΠ΅ зарядов, поэтому ΠΎΠ½ΠΈ Π½Π°Π·Ρ‹Π²Π°ΡŽΡ‚ΡΡ полярными ΠΌΠΎΠ»Π΅ΠΊΡƒΠ»Π°ΠΌΠΈ. На рисункС 7 ΠΏΠΎΠΊΠ°Π·Π°Π½ΠΎ Ρ€Π°Π·Π΄Π΅Π»Π΅Π½ΠΈΠ΅ зарядов Π² ΠΌΠΎΠ»Π΅ΠΊΡƒΠ»Π΅ Π²ΠΎΠ΄Ρ‹, которая ΠΈΠΌΠ΅Π΅Ρ‚ Π΄Π²Π° Π°Ρ‚ΠΎΠΌΠ° Π²ΠΎΠ΄ΠΎΡ€ΠΎΠ΄Π° ΠΈ ΠΎΠ΄ΠΈΠ½ Π°Ρ‚ΠΎΠΌ кислорода (H 2 O).ΠœΠΎΠ»Π΅ΠΊΡƒΠ»Π° Π²ΠΎΠ΄Ρ‹ нСсиммСтрична — Π°Ρ‚ΠΎΠΌΡ‹ Π²ΠΎΠ΄ΠΎΡ€ΠΎΠ΄Π° ΠΎΡ‚Ρ‚Π°Π»ΠΊΠΈΠ²Π°ΡŽΡ‚ΡΡ Π² ΠΎΠ΄Π½Ρƒ сторону, придавая ΠΌΠΎΠ»Π΅ΠΊΡƒΠ»Π΅ Ρ„ΠΎΡ€ΠΌΡƒ Π±ΡƒΠΌΠ΅Ρ€Π°Π½Π³Π°. Π­Π»Π΅ΠΊΡ‚Ρ€ΠΎΠ½Ρ‹ Π² ΠΌΠΎΠ»Π΅ΠΊΡƒΠ»Π΅ Π²ΠΎΠ΄Ρ‹ большС сконцСнтрированы Π²ΠΎΠΊΡ€ΡƒΠ³ Π±ΠΎΠ»Π΅Π΅ заряТСнного ядра кислорода, Ρ‡Π΅ΠΌ Π²ΠΎΠΊΡ€ΡƒΠ³ ядСр Π²ΠΎΠ΄ΠΎΡ€ΠΎΠ΄Π°. Π­Ρ‚ΠΎ Π΄Π΅Π»Π°Π΅Ρ‚ кислородный ΠΊΠΎΠ½Π΅Ρ† ΠΌΠΎΠ»Π΅ΠΊΡƒΠ»Ρ‹ слСгка ΠΎΡ‚Ρ€ΠΈΡ†Π°Ρ‚Π΅Π»ΡŒΠ½Ρ‹ΠΌ, Π° Π²ΠΎΠ΄ΠΎΡ€ΠΎΠ΄Π½Ρ‹ΠΉ ΠΊΠΎΠ½Π΅Ρ† слСгка ΠΏΠΎΠ»ΠΎΠΆΠΈΡ‚Π΅Π»ΡŒΠ½Ρ‹ΠΌ. БобствСнноС Ρ€Π°Π·Π΄Π΅Π»Π΅Π½ΠΈΠ΅ зарядов Π² полярных ΠΌΠΎΠ»Π΅ΠΊΡƒΠ»Π°Ρ… ΠΎΠ±Π»Π΅Π³Ρ‡Π°Π΅Ρ‚ ΠΈΡ… Π²Ρ‹Ρ€Π°Π²Π½ΠΈΠ²Π°Π½ΠΈΠ΅ с внСшними полями ΠΈ зарядами. Π‘Π»Π΅Π΄ΠΎΠ²Π°Ρ‚Π΅Π»ΡŒΠ½ΠΎ, полярныС ΠΌΠΎΠ»Π΅ΠΊΡƒΠ»Ρ‹ ΠΏΡ€ΠΎΡΠ²Π»ΡΡŽΡ‚ Π±ΠΎΠ»Π΅Π΅ ΡΠΈΠ»ΡŒΠ½Ρ‹Π΅ поляризационныС эффСкты ΠΈ ΠΈΠΌΠ΅ΡŽΡ‚ Π±ΠΎΠ»Π΅Π΅ высокиС диэлСктричСскиС проницаСмости.Π’Π΅, ΠΊΡ‚ΠΎ ΠΈΠ·ΡƒΡ‡Π°Π΅Ρ‚ Ρ…ΠΈΠΌΠΈΡŽ, ΠΎΠ±Π½Π°Ρ€ΡƒΠΆΠ°Ρ‚, Ρ‡Ρ‚ΠΎ полярная ΠΏΡ€ΠΈΡ€ΠΎΠ΄Π° Π²ΠΎΠ΄Ρ‹ ΠΈΠΌΠ΅Π΅Ρ‚ мноТСство эффСктов. НапримСр, ΠΌΠΎΠ»Π΅ΠΊΡƒΠ»Ρ‹ Π²ΠΎΠ΄Ρ‹ ΡΠΎΠ±ΠΈΡ€Π°ΡŽΡ‚ ΠΈΠΎΠ½Ρ‹ Π½Π°ΠΌΠ½ΠΎΠ³ΠΎ эффСктивнСС, ΠΏΠΎΡ‚ΠΎΠΌΡƒ Ρ‡Ρ‚ΠΎ Ρƒ Π½ΠΈΡ… Π΅ΡΡ‚ΡŒ элСктричСскоС ΠΏΠΎΠ»Π΅ ΠΈ Ρ€Π°Π·Π΄Π΅Π»Π΅Π½ΠΈΠ΅ зарядов, Ρ‡Ρ‚ΠΎΠ±Ρ‹ ΠΏΡ€ΠΈΡ‚ΡΠ³ΠΈΠ²Π°Ρ‚ΡŒ заряды ΠΎΠ±ΠΎΠΈΡ… Π·Π½Π°ΠΊΠΎΠ². ΠšΡ€ΠΎΠΌΠ΅ Ρ‚ΠΎΠ³ΠΎ, ΠΊΠ°ΠΊ ΡƒΠΊΠ°Π·Ρ‹Π²Π°Π»ΠΎΡΡŒ Π² ΠΏΡ€Π΅Π΄Ρ‹Π΄ΡƒΡ‰Π΅ΠΉ Π³Π»Π°Π²Π΅, полярная Π²ΠΎΠ΄Π° обСспСчиваСт Π·Π°Ρ‰ΠΈΡ‚Ρƒ ΠΈΠ»ΠΈ экранированиС элСктричСских ΠΏΠΎΠ»Π΅ΠΉ Π² сильно заряТСнных ΠΌΠΎΠ»Π΅ΠΊΡƒΠ»Π°Ρ…, ΠΏΡ€Π΅Π΄ΡΡ‚Π°Π²Π»ΡΡŽΡ‰ΠΈΡ… интСрСс Π² биологичСских систСмах.

Рис. 7. Π₯удоТСствСнная концСпция ΠΌΠΎΠ»Π΅ΠΊΡƒΠ»Ρ‹ Π²ΠΎΠ΄Ρ‹. БущСствуСт Π²Π½ΡƒΡ‚Ρ€Π΅Π½Π½Π΅Π΅ Ρ€Π°Π·Π΄Π΅Π»Π΅Π½ΠΈΠ΅ зарядов, поэтому Π²ΠΎΠ΄Π° — полярная ΠΌΠΎΠ»Π΅ΠΊΡƒΠ»Π°.Π­Π»Π΅ΠΊΡ‚Ρ€ΠΎΠ½Ρ‹ Π² ΠΌΠΎΠ»Π΅ΠΊΡƒΠ»Π΅ ΠΏΡ€ΠΈΡ‚ΡΠ³ΠΈΠ²Π°ΡŽΡ‚ΡΡ ΠΊ ядру кислорода ΠΈ ΠΎΡΡ‚Π°Π²Π»ΡΡŽΡ‚ ΠΈΠ·Π±Ρ‹Ρ‚ΠΎΠΊ ΠΏΠΎΠ»ΠΎΠΆΠΈΡ‚Π΅Π»ΡŒΠ½ΠΎΠ³ΠΎ заряда ΠΎΠΊΠΎΠ»ΠΎ Π΄Π²ΡƒΡ… ядСр Π²ΠΎΠ΄ΠΎΡ€ΠΎΠ΄Π°. (ΠžΠ±Ρ€Π°Ρ‚ΠΈΡ‚Π΅ Π²Π½ΠΈΠΌΠ°Π½ΠΈΠ΅, Ρ‡Ρ‚ΠΎ схСма справа являСтся ΠΏΡ€ΠΈΠ±Π»ΠΈΠ·ΠΈΡ‚Π΅Π»ΡŒΠ½ΠΎΠΉ ΠΈΠ»Π»ΡŽΡΡ‚Ρ€Π°Ρ†ΠΈΠ΅ΠΉ распрСдСлСния элСктронов Π² ΠΌΠΎΠ»Π΅ΠΊΡƒΠ»Π΅ Π²ΠΎΠ΄Ρ‹. На Π½Π΅ΠΉ Π½Π΅ ΠΏΠΎΠΊΠ°Π·Π°Π½ΠΎ фактичСскоС количСство ΠΏΡ€ΠΎΡ‚ΠΎΠ½ΠΎΠ² ΠΈ элСктронов, ΡƒΡ‡Π°ΡΡ‚Π²ΡƒΡŽΡ‰ΠΈΡ… Π² структурС.)

ИсслСдования PhET: лаборатория кондСнсаторов

Π£Π·Π½Π°ΠΉΡ‚Π΅, ΠΊΠ°ΠΊ Ρ€Π°Π±ΠΎΡ‚Π°Π΅Ρ‚ кондСнсатор! Π˜Π·ΠΌΠ΅Π½ΠΈΡ‚Π΅ Ρ€Π°Π·ΠΌΠ΅Ρ€ пластин ΠΈ Π΄ΠΎΠ±Π°Π²ΡŒΡ‚Π΅ диэлСктрик, Ρ‡Ρ‚ΠΎΠ±Ρ‹ ΡƒΠ²ΠΈΠ΄Π΅Ρ‚ΡŒ влияниС Π½Π° Π΅ΠΌΠΊΠΎΡΡ‚ΡŒ.Π˜Π·ΠΌΠ΅Π½ΠΈΡ‚Π΅ напряТСниС ΠΈ посмотритС, ΠΊΠ°ΠΊ Π½Π° пластинах Π½Π°ΠΊΠ°ΠΏΠ»ΠΈΠ²Π°ΡŽΡ‚ΡΡ заряды. ΠΠ°Π±Π»ΡŽΠ΄Π°ΠΉΡ‚Π΅ Π·Π° элСктричСским ΠΏΠΎΠ»Π΅ΠΌ Π² кондСнсаторС. Π˜Π·ΠΌΠ΅Ρ€ΡŒΡ‚Π΅ напряТСниС ΠΈ элСктричСскоС ΠΏΠΎΠ»Π΅.

Π©Π΅Π»ΠΊΠ½ΠΈΡ‚Π΅, Ρ‡Ρ‚ΠΎΠ±Ρ‹ Π·Π°Π³Ρ€ΡƒΠ·ΠΈΡ‚ΡŒ ΡΠΈΠΌΡƒΠ»ΡΡ†ΠΈΡŽ. Π—Π°ΠΏΡƒΡΠΊΠ°Ρ‚ΡŒ Π½Π° Java.

Π‘Π²ΠΎΠ΄ΠΊΠ° Ρ€Π°Π·Π΄Π΅Π»Π°

  • ΠšΠΎΠ½Π΄Π΅Π½ΡΠ°Ρ‚ΠΎΡ€ — это устройство для хранСния заряда.
  • ΠšΠΎΠ»ΠΈΡ‡Π΅ΡΡ‚Π²ΠΎ заряда Q , ΠΊΠΎΡ‚ΠΎΡ€ΠΎΠ΅ ΠΌΠΎΠΆΠ΅Ρ‚ ΡΠΎΡ…Ρ€Π°Π½ΠΈΡ‚ΡŒ кондСнсатор, зависит ΠΎΡ‚ Π΄Π²ΡƒΡ… основных Ρ„Π°ΠΊΡ‚ΠΎΡ€ΠΎΠ² — ΠΏΡ€ΠΈΠ»ΠΎΠΆΠ΅Π½Π½ΠΎΠ³ΠΎ напряТСния ΠΈ физичСских характСристик кондСнсатора, Ρ‚Π°ΠΊΠΈΡ… ΠΊΠ°ΠΊ Π΅Π³ΠΎ Ρ€Π°Π·ΠΌΠ΅Ρ€.
  • Π•ΠΌΠΊΠΎΡΡ‚ΡŒ C, — это Π²Π΅Π»ΠΈΡ‡ΠΈΠ½Π° заряда, Π½Π°ΠΊΠΎΠΏΠ»Π΅Π½Π½ΠΎΠ³ΠΎ Π½Π° Π²ΠΎΠ»ΡŒΡ‚, ΠΈΠ»ΠΈ [латСкс] C = \ frac {Q} {V} \\ [/ latex].
  • Π•ΠΌΠΊΠΎΡΡ‚ΡŒ кондСнсатора с ΠΏΠ°Ρ€Π°Π»Π»Π΅Π»ΡŒΠ½Ρ‹ΠΌΠΈ пластинами составляСт [латСкс] C = {\ epsilon} _ {0} \ frac {A} {d} \\ [/ latex], ΠΊΠΎΠ³Π΄Π° пластины Ρ€Π°Π·Π΄Π΅Π»Π΅Π½Ρ‹ Π²ΠΎΠ·Π΄ΡƒΡ…ΠΎΠΌ ΠΈΠ»ΠΈ свободным пространством. [latex] {\ epsilon} _ {\ text {0}} [/ latex] называСтся диэлСктричСской ΠΏΡ€ΠΎΠ½ΠΈΡ†Π°Π΅ΠΌΠΎΡΡ‚ΡŒΡŽ свободного пространства.
  • ΠšΠΎΠ½Π΄Π΅Π½ΡΠ°Ρ‚ΠΎΡ€ с ΠΏΠ°Ρ€Π°Π»Π»Π΅Π»ΡŒΠ½Ρ‹ΠΌΠΈ пластинами с диэлСктриком ΠΌΠ΅ΠΆΠ΄Ρƒ пластинами ΠΈΠΌΠ΅Π΅Ρ‚ Π΅ΠΌΠΊΠΎΡΡ‚ΡŒ, ΠΎΠΏΡ€Π΅Π΄Π΅Π»ΡΠ΅ΠΌΡƒΡŽ Π²Ρ‹Ρ€Π°ΠΆΠ΅Π½ΠΈΠ΅ΠΌ [латСкс] C = \ kappa \ epsilon_ {0} \ frac {A} {d} \\ [/ latex], Π³Π΄Π΅ ΞΊ — диэлСктрик. постоянная ΠΌΠ°Ρ‚Π΅Ρ€ΠΈΠ°Π»Π°.
  • Максимальная Π½Π°ΠΏΡ€ΡΠΆΠ΅Π½Π½ΠΎΡΡ‚ΡŒ элСктричСского поля, ΠΏΡ€ΠΈ ΠΏΡ€Π΅Π²Ρ‹ΡˆΠ΅Π½ΠΈΠΈ ΠΊΠΎΡ‚ΠΎΡ€ΠΎΠΉ ΠΈΠ·ΠΎΠ»ΠΈΡ€ΡƒΡŽΡ‰ΠΈΠΉ ΠΌΠ°Ρ‚Π΅Ρ€ΠΈΠ°Π» Π½Π°Ρ‡ΠΈΠ½Π°Π΅Ρ‚ Ρ€Π°Π·Ρ€ΡƒΡˆΠ°Ρ‚ΡŒΡΡ ΠΈ становится ΠΏΡ€ΠΎΠ²ΠΎΠ΄Π½ΠΈΠΊΠΎΠΌ, называСтся элСктричСской ΠΏΡ€ΠΎΡ‡Π½ΠΎΡΡ‚ΡŒΡŽ.

ΠšΠΎΠ½Ρ†Π΅ΠΏΡ‚ΡƒΠ°Π»ΡŒΠ½Ρ‹Π΅ вопросы

  1. Зависит Π»ΠΈ Π΅ΠΌΠΊΠΎΡΡ‚ΡŒ устройства ΠΎΡ‚ ΠΏΡ€ΠΈΠ»ΠΎΠΆΠ΅Π½Π½ΠΎΠ³ΠΎ напряТСния? А ΠΊΠ°ΠΊ насчСт хранящСгося Π² Π½Π΅ΠΌ заряда?
  2. Π˜ΡΠΏΠΎΠ»ΡŒΠ·ΡƒΠΉΡ‚Π΅ характСристики кулоновской силы, Ρ‡Ρ‚ΠΎΠ±Ρ‹ ΠΎΠ±ΡŠΡΡΠ½ΠΈΡ‚ΡŒ, ΠΏΠΎΡ‡Π΅ΠΌΡƒ Π΅ΠΌΠΊΠΎΡΡ‚ΡŒ Π΄ΠΎΠ»ΠΆΠ½Π° Π±Ρ‹Ρ‚ΡŒ ΠΏΡ€ΠΎΠΏΠΎΡ€Ρ†ΠΈΠΎΠ½Π°Π»ΡŒΠ½Π° ΠΏΠ»ΠΎΡ‰Π°Π΄ΠΈ пластины кондСнсатора. Π’ΠΎΡ‡Π½ΠΎ Ρ‚Π°ΠΊ ΠΆΠ΅ ΠΎΠ±ΡŠΡΡΠ½ΠΈΡ‚Π΅, ΠΏΠΎΡ‡Π΅ΠΌΡƒ Π΅ΠΌΠΊΠΎΡΡ‚ΡŒ Π΄ΠΎΠ»ΠΆΠ½Π° Π±Ρ‹Ρ‚ΡŒ ΠΎΠ±Ρ€Π°Ρ‚Π½ΠΎ ΠΏΡ€ΠΎΠΏΠΎΡ€Ρ†ΠΈΠΎΠ½Π°Π»ΡŒΠ½Π° Ρ€Π°ΡΡΡ‚ΠΎΡΠ½ΠΈΡŽ ΠΌΠ΅ΠΆΠ΄Ρƒ пластинами.
  3. ΠžΠ±ΡŠΡΡΠ½ΠΈΡ‚Π΅ ΠΏΡ€ΠΈΡ‡ΠΈΠ½Ρƒ, ΠΏΠΎ ΠΊΠΎΡ‚ΠΎΡ€ΠΎΠΉ диэлСктричСский ΠΌΠ°Ρ‚Π΅Ρ€ΠΈΠ°Π» ΡƒΠ²Π΅Π»ΠΈΡ‡ΠΈΠ²Π°Π΅Ρ‚ Π΅ΠΌΠΊΠΎΡΡ‚ΡŒ ΠΏΠΎ ΡΡ€Π°Π²Π½Π΅Π½ΠΈΡŽ с Ρ‚Π΅ΠΌ, Ρ‡Ρ‚ΠΎ Π±Ρ‹Π»ΠΎ Π±Ρ‹ с Π²ΠΎΠ·Π΄ΡƒΡ…ΠΎΠΌ ΠΌΠ΅ΠΆΠ΄Ρƒ пластинами кондСнсатора.Какова нСзависимая ΠΏΡ€ΠΈΡ‡ΠΈΠ½Π° Ρ‚ΠΎΠ³ΠΎ, Ρ‡Ρ‚ΠΎ диэлСктричСский ΠΌΠ°Ρ‚Π΅Ρ€ΠΈΠ°Π» Ρ‚Π°ΠΊΠΆΠ΅ позволяСт ΠΏΡ€ΠΈΠ»ΠΎΠΆΠΈΡ‚ΡŒ большСС напряТСниС ΠΊ кондСнсатору? (Π’Π°ΠΊΠΈΠΌ ΠΎΠ±Ρ€Π°Π·ΠΎΠΌ, диэлСктрик ΡƒΠ²Π΅Π»ΠΈΡ‡ΠΈΠ²Π°Π΅Ρ‚ C ΠΈ допускаСт Π±ΠΎΠ»Π΅Π΅ V .)
  4. Как полярный Ρ…Π°Ρ€Π°ΠΊΡ‚Π΅Ρ€ ΠΌΠΎΠ»Π΅ΠΊΡƒΠ» Π²ΠΎΠ΄Ρ‹ ΠΏΠΎΠΌΠΎΠ³Π°Π΅Ρ‚ ΠΎΠ±ΡŠΡΡΠ½ΠΈΡ‚ΡŒ ΠΎΡ‚Π½ΠΎΡΠΈΡ‚Π΅Π»ΡŒΠ½ΠΎ Π±ΠΎΠ»ΡŒΡˆΡƒΡŽ Π΄ΠΈΡΠ»Π΅ΠΊΡ‚Ρ€ΠΈΡ‡Π΅ΡΠΊΡƒΡŽ ΠΏΡ€ΠΎΠ½ΠΈΡ†Π°Π΅ΠΌΠΎΡΡ‚ΡŒ Π²ΠΎΠ΄Ρ‹? (Π‘ΠΌ. Рисунок 7.)
  5. Π˜ΡΠΊΡ€Ρ‹ Π²ΠΎΠ·Π½ΠΈΠΊΠ°ΡŽΡ‚ ΠΌΠ΅ΠΆΠ΄Ρƒ пластинами Π·Π°ΠΏΠΎΠ»Π½Π΅Π½Π½ΠΎΠ³ΠΎ Π²ΠΎΠ·Π΄ΡƒΡ…ΠΎΠΌ кондСнсатора ΠΏΡ€ΠΈ Π±ΠΎΠ»Π΅Π΅ Π½ΠΈΠ·ΠΊΠΎΠΌ напряТСнии, ΠΊΠΎΠ³Π΄Π° Π²ΠΎΠ·Π΄ΡƒΡ… Π²Π»Π°ΠΆΠ½Ρ‹ΠΉ, Ρ‡Π΅ΠΌ ΠΊΠΎΠ³Π΄Π° сухой. ΠžΠ±ΡŠΡΡΠ½ΠΈΡ‚Π΅ ΠΏΠΎΡ‡Π΅ΠΌΡƒ, учитывая полярный Ρ…Π°Ρ€Π°ΠΊΡ‚Π΅Ρ€ ΠΌΠΎΠ»Π΅ΠΊΡƒΠ» Π²ΠΎΠ΄Ρ‹.
  6. Π’ΠΎΠ΄Π° ΠΈΠΌΠ΅Π΅Ρ‚ Π±ΠΎΠ»ΡŒΡˆΡƒΡŽ Π΄ΠΈΡΠ»Π΅ΠΊΡ‚Ρ€ΠΈΡ‡Π΅ΡΠΊΡƒΡŽ ΠΏΡ€ΠΎΠ½ΠΈΡ†Π°Π΅ΠΌΠΎΡΡ‚ΡŒ, Π½ΠΎ Ρ€Π΅Π΄ΠΊΠΎ ΠΈΡΠΏΠΎΠ»ΡŒΠ·ΡƒΠ΅Ρ‚ΡΡ Π² кондСнсаторах. ΠžΠ±ΡŠΡΡΠ½ΠΈΡ‚ΡŒ, ΠΏΠΎΡ‡Π΅ΠΌΡƒ.
  7. ΠœΠ΅ΠΌΠ±Ρ€Π°Π½Ρ‹ Π² ΠΆΠΈΠ²Ρ‹Ρ… ΠΊΠ»Π΅Ρ‚ΠΊΠ°Ρ…, Π² Ρ‚ΠΎΠΌ числС Π² чСловСчСских, Ρ…Π°Ρ€Π°ΠΊΡ‚Π΅Ρ€ΠΈΠ·ΡƒΡŽΡ‚ΡΡ Ρ€Π°Π·Π΄Π΅Π»Π΅Π½ΠΈΠ΅ΠΌ заряда Ρ‡Π΅Ρ€Π΅Π· ΠΌΠ΅ΠΌΠ±Ρ€Π°Π½Ρƒ. Π’Π°ΠΊΠΈΠΌ ΠΎΠ±Ρ€Π°Π·ΠΎΠΌ, ΠΌΠ΅ΠΌΠ±Ρ€Π°Π½Ρ‹ ΠΏΡ€Π΅Π΄ΡΡ‚Π°Π²Π»ΡΡŽΡ‚ собой заряТСнныС кондСнсаторы, Π²Π°ΠΆΠ½Ρ‹Π΅ Ρ„ΡƒΠ½ΠΊΡ†ΠΈΠΈ ΠΊΠΎΡ‚ΠΎΡ€Ρ‹Ρ… связаны с Ρ€Π°Π·Π½ΠΎΡΡ‚ΡŒΡŽ ΠΏΠΎΡ‚Π΅Π½Ρ†ΠΈΠ°Π»ΠΎΠ² Π½Π° ΠΌΠ΅ΠΌΠ±Ρ€Π°Π½Π΅. ВрСбуСтся Π»ΠΈ энСргия для раздСлСния этих зарядов Π² ΠΆΠΈΠ²Ρ‹Ρ… ΠΌΠ΅ΠΌΠ±Ρ€Π°Π½Π°Ρ…, ΠΈ Ссли Π΄Π°, Ρ‚ΠΎ являСтся Π»ΠΈ Π΅Π΅ источником ΠΌΠ΅Ρ‚Π°Π±ΠΎΠ»ΠΈΠ·ΠΌ ΠΏΠΈΡ‰Π΅Π²ΠΎΠΉ энСргии ΠΈΠ»ΠΈ ΠΊΠ°ΠΊΠΈΠΌ-Π»ΠΈΠ±ΠΎ Π΄Ρ€ΡƒΠ³ΠΈΠΌ источником?

Рисунок 8.ΠŸΠΎΠ»ΡƒΠΏΡ€ΠΎΠ½ΠΈΡ†Π°Π΅ΠΌΠ°Ρ ΠΌΠ΅ΠΌΠ±Ρ€Π°Π½Π° ΠΊΠ»Π΅Ρ‚ΠΊΠΈ ΠΈΠΌΠ΅Π΅Ρ‚ Ρ€Π°Π·Π½ΡƒΡŽ ΠΊΠΎΠ½Ρ†Π΅Π½Ρ‚Ρ€Π°Ρ†ΠΈΡŽ ΠΈΠΎΠ½ΠΎΠ² Π²Π½ΡƒΡ‚Ρ€ΠΈ ΠΈ снаруТи. Диффузия ΠΏΠ΅Ρ€Π΅ΠΌΠ΅Ρ‰Π°Π΅Ρ‚ ΠΈΠΎΠ½Ρ‹ K + (ΠΊΠ°Π»ΠΈΠΉ) ΠΈ Cl (Ρ…Π»ΠΎΡ€ΠΈΠ΄) Π² ΠΏΠΎΠΊΠ°Π·Π°Π½Π½Ρ‹Ρ… направлСниях, ΠΏΠΎΠΊΠ° кулоновская сила Π½Π΅ остановит дальнСйший пСрСнос. Π­Ρ‚ΠΎ ΠΏΡ€ΠΈΠ²ΠΎΠ΄ΠΈΡ‚ ΠΊ слою ΠΏΠΎΠ»ΠΎΠΆΠΈΡ‚Π΅Π»ΡŒΠ½ΠΎΠ³ΠΎ заряда снаруТи, слою ΠΎΡ‚Ρ€ΠΈΡ†Π°Ρ‚Π΅Π»ΡŒΠ½ΠΎΠ³ΠΎ заряда Π²Π½ΡƒΡ‚Ρ€ΠΈ ΠΈ, ΡΠ»Π΅Π΄ΠΎΠ²Π°Ρ‚Π΅Π»ΡŒΠ½ΠΎ, ΠΊ Π½Π°ΠΏΡ€ΡΠΆΠ΅Π½ΠΈΡŽ Π½Π° ΠΊΠ»Π΅Ρ‚ΠΎΡ‡Π½ΠΎΠΉ ΠΌΠ΅ΠΌΠ±Ρ€Π°Π½Π΅. ΠœΠ΅ΠΌΠ±Ρ€Π°Π½Π° ΠΎΠ±Ρ‹Ρ‡Π½ΠΎ Π½Π΅ΠΏΡ€ΠΎΠ½ΠΈΡ†Π°Π΅ΠΌΠ° для Na + (ΠΈΠΎΠ½Ρ‹ натрия).

Π—Π°Π΄Π°Ρ‡ΠΈ ΠΈ упраТнСния

  1. Какой заряд хранится Π² кондСнсаторС 180 ΠΌΠΊΠ€, ΠΊΠΎΠ³Π΄Π° ΠΊ Π½Π΅ΠΌΡƒ ΠΏΡ€ΠΈΠ»ΠΎΠΆΠ΅Π½ΠΎ 120 Π’?
  2. НайдитС Π½Π°ΠΊΠΎΠΏΠ»Π΅Π½Π½Ρ‹ΠΉ заряд, ΠΊΠΎΠ³Π΄Π° 5.На кондСнсатор Π΅ΠΌΠΊΠΎΡΡ‚ΡŒΡŽ 8,00 ΠΏΠ€ подаСтся 50 Π’.
  3. Какой заряд хранится Π² кондСнсаторС Π² ΠΏΡ€ΠΈΠΌΠ΅Ρ€Π΅ 1?
  4. ВычислитС напряТСниС, ΠΏΡ€ΠΈΠ»ΠΎΠΆΠ΅Π½Π½ΠΎΠ΅ ΠΊ кондСнсатору 2,00 ΠΌΠΊΠ€, ΠΊΠΎΠ³Π΄Π° ΠΎΠ½ ΠΈΠΌΠ΅Π΅Ρ‚ заряд 3,10 мкКл.
  5. КакоС напряТСниС Π½Π΅ΠΎΠ±Ρ…ΠΎΠ΄ΠΈΠΌΠΎ ΠΏΠΎΠ΄Π°Ρ‚ΡŒ Π½Π° кондСнсатор Π΅ΠΌΠΊΠΎΡΡ‚ΡŒΡŽ 8,00 Π½Π€ для накоплСния заряда 0,160 мкКл?
  6. Какая Π΅ΠΌΠΊΠΎΡΡ‚ΡŒ Π½Π΅ΠΎΠ±Ρ…ΠΎΠ΄ΠΈΠΌΠ° для хранСния 3,00 мкКл заряда ΠΏΡ€ΠΈ напряТСнии 120 Π’?
  7. Какая Π΅ΠΌΠΊΠΎΡΡ‚ΡŒ Ρ‚Π΅Ρ€ΠΌΠΈΠ½Π°Π»Π° большого Π³Π΅Π½Π΅Ρ€Π°Ρ‚ΠΎΡ€Π° Π’Π°Π½ Π΄Π΅ Π“Ρ€Π°Π°Ρ„Π°, учитывая, Ρ‡Ρ‚ΠΎ ΠΎΠ½ Ρ…Ρ€Π°Π½ΠΈΡ‚ 8?00 мкКл заряда ΠΏΡ€ΠΈ напряТСнии 12,0 ΠœΠ’?
  8. НайдитС Π΅ΠΌΠΊΠΎΡΡ‚ΡŒ кондСнсатора с ΠΏΠ°Ρ€Π°Π»Π»Π΅Π»ΡŒΠ½Ρ‹ΠΌΠΈ пластинами, ΠΏΠ»ΠΎΡ‰Π°Π΄ΡŒ пластин ΠΊΠΎΡ‚ΠΎΡ€ΠΎΠ³ΠΎ составляСт 5,00 ΠΌ 2 , Ρ€Π°Π·Π΄Π΅Π»Π΅Π½Π½Ρ‹Ρ… слоСм Ρ‚Π΅Ρ„Π»ΠΎΠ½Π° 0,100 ΠΌΠΌ.
  9. (a) Какова Π΅ΠΌΠΊΠΎΡΡ‚ΡŒ кондСнсатора с ΠΏΠ°Ρ€Π°Π»Π»Π΅Π»ΡŒΠ½Ρ‹ΠΌΠΈ пластинами, ΠΏΠ»ΠΎΡ‰Π°Π΄ΡŒ пластин ΠΊΠΎΡ‚ΠΎΡ€ΠΎΠ³ΠΎ составляСт 1,50 ΠΌ 2 , Ρ€Π°Π·Π΄Π΅Π»Π΅Π½Π½Ρ‹Ρ… 0,0200 ΠΌΠΌ Π½Π΅ΠΎΠΏΡ€Π΅Π½ΠΎΠ²ΠΎΠΉ Ρ€Π΅Π·ΠΈΠ½Ρ‹? (b) Какой заряд ΠΎΠ½ Π΄Π΅Ρ€ΠΆΠΈΡ‚, ΠΊΠΎΠ³Π΄Π° ΠΊ Π½Π΅ΠΌΡƒ ΠΏΡ€ΠΈΠ»ΠΎΠΆΠ΅Π½ΠΎ 9,00 Π’?
  10. ΠšΠΎΠΌΠΏΠ»Π΅ΠΊΡΠ½Ρ‹Π΅ ΠΊΠΎΠ½Ρ†Π΅ΠΏΡ†ΠΈΠΈ. Π¨ΡƒΡ‚Π½ΠΈΠΊ ΠΏΠΎΠ΄Π°Π΅Ρ‚ 450 Π’ Π½Π° 80.ΠšΠΎΠ½Π΄Π΅Π½ΡΠ°Ρ‚ΠΎΡ€ 0 ΠΌΠΊΠ€, Π° Π·Π°Ρ‚Π΅ΠΌ бросаСт Π΅Π³ΠΎ Π½ΠΈΡ‡Π΅Π³ΠΎ Π½Π΅ ΠΏΠΎΠ΄ΠΎΠ·Ρ€Π΅Π²Π°ΡŽΡ‰Π΅ΠΉ ΠΆΠ΅Ρ€Ρ‚Π²Π΅. ΠŸΠ°Π»Π΅Ρ† ΠΏΠΎΡΡ‚Ρ€Π°Π΄Π°Π²ΡˆΠ΅Π³ΠΎ ΠΎΠ±Π³ΠΎΡ€Π΅Π» ΠΎΡ‚ разряда кондСнсатора Ρ‡Π΅Ρ€Π΅Π· 0,200 Π³ мяса. КакоС ΠΏΠΎΠ²Ρ‹ΡˆΠ΅Π½ΠΈΠ΅ Ρ‚Π΅ΠΌΠΏΠ΅Ρ€Π°Ρ‚ΡƒΡ€Ρ‹ мяса? Π Π°Π·ΡƒΠΌΠ½ΠΎ Π»ΠΈ ΠΏΡ€Π΅Π΄ΠΏΠΎΠ»Π°Π³Π°Ρ‚ΡŒ отсутствиС измСнСния Ρ„Π°Π·Ρ‹?
  11. НСобоснованныС Ρ€Π΅Π·ΡƒΠ»ΡŒΡ‚Π°Ρ‚Ρ‹. (a) ΠšΠΎΠ½Π΄Π΅Π½ΡΠ°Ρ‚ΠΎΡ€ с ΠΏΠ°Ρ€Π°Π»Π»Π΅Π»ΡŒΠ½Ρ‹ΠΌΠΈ пластинами ΠΈΠΌΠ΅Π΅Ρ‚ ΠΏΠ»ΠΎΡ‰Π°Π΄ΡŒ пластин 4,00 ΠΌ 2 , Ρ€Π°Π·Π΄Π΅Π»Π΅Π½Π½Ρ‹Ρ… Π½Π΅ΠΉΠ»ΠΎΠ½ΠΎΠΌ Ρ‚ΠΎΠ»Ρ‰ΠΈΠ½ΠΎΠΉ 0,0100 ΠΌΠΌ, ΠΈ Π½Π°ΠΊΠ°ΠΏΠ»ΠΈΠ²Π°Π΅Ρ‚ 0,170 Кл заряда. КакоС ΠΏΡ€ΠΈΠ»ΠΎΠΆΠ΅Π½Π½ΠΎΠ΅ напряТСниС? Π±) Π§Ρ‚ΠΎ Π½Π΅Ρ€Π°Π·ΡƒΠΌΠ½ΠΎΠ³ΠΎ Π² этом Ρ€Π΅Π·ΡƒΠ»ΡŒΡ‚Π°Ρ‚Π΅? (c) КакиС прСдполоТСния ΡΠ²Π»ΡΡŽΡ‚ΡΡ отвСтствСнными ΠΈΠ»ΠΈ ΠΏΡ€ΠΎΡ‚ΠΈΠ²ΠΎΡ€Π΅Ρ‡ΠΈΠ²Ρ‹ΠΌΠΈ?

Глоссарий

кондСнсатор: устройство, Π½Π°ΠΊΠ°ΠΏΠ»ΠΈΠ²Π°ΡŽΡ‰Π΅Π΅ элСктричСский заряд

Π΅ΠΌΠΊΠΎΡΡ‚ΡŒ: количСство заряда Π½Π° Π΅Π΄ΠΈΠ½ΠΈΡ†Ρƒ Π²ΠΎΠ»ΡŒΡ‚

диэлСктрик: изоляционный ΠΌΠ°Ρ‚Π΅Ρ€ΠΈΠ°Π»

диэлСктричСская ΠΏΡ€ΠΎΡ‡Π½ΠΎΡΡ‚ΡŒ: максимальноС элСктричСскоС ΠΏΠΎΠ»Π΅, Π²Ρ‹ΡˆΠ΅ ΠΊΠΎΡ‚ΠΎΡ€ΠΎΠ³ΠΎ ΠΈΠ·ΠΎΠ»ΠΈΡ€ΡƒΡŽΡ‰ΠΈΠΉ ΠΌΠ°Ρ‚Π΅Ρ€ΠΈΠ°Π» Π½Π°Ρ‡ΠΈΠ½Π°Π΅Ρ‚ Ρ€Π°Π·Ρ€ΡƒΡˆΠ°Ρ‚ΡŒΡΡ ΠΈ ΠΏΡ€ΠΎΠ²ΠΎΠ΄ΠΈΡ‚ΡŒ

ΠšΠΎΠ½Π΄Π΅Π½ΡΠ°Ρ‚ΠΎΡ€ с ΠΏΠ°Ρ€Π°Π»Π»Π΅Π»ΡŒΠ½Ρ‹ΠΌΠΈ пластинами: Π΄Π²Π΅ ΠΈΠ΄Π΅Π½Ρ‚ΠΈΡ‡Π½Ρ‹Π΅ проводящиС пластины, Ρ€Π°Π·Π΄Π΅Π»Π΅Π½Π½Ρ‹Π΅ расстояниСм

полярная ΠΌΠΎΠ»Π΅ΠΊΡƒΠ»Π°: ΠΌΠΎΠ»Π΅ΠΊΡƒΠ»Π° с Π²Π½ΡƒΡ‚Ρ€Π΅Π½Π½ΠΈΠΌ Ρ€Π°Π·Π΄Π΅Π»Π΅Π½ΠΈΠ΅ΠΌ заряда

Π˜Π·Π±Ρ€Π°Π½Π½Ρ‹Π΅ Ρ€Π΅ΡˆΠ΅Π½ΠΈΡ ΠΏΡ€ΠΎΠ±Π»Π΅ΠΌ ΠΈ упраТнСния

1. 21,6 мК

3. 80.0 ΠΌΠ‘

5. 20,0 ΠΊΠ’

7. 667 ΠΏΠ€

9. (Π°) 4,4 ΠΌΠΊΠ€; (Π±) 4.0 Γ— 10 βˆ’5 C

11. (Π°) 14,2 ΠΊΠ’; (b) НапряТСниС Π½Π΅ΠΎΠΏΡ€Π°Π²Π΄Π°Π½Π½ΠΎ Π²Π΅Π»ΠΈΠΊΠΎ, Π±ΠΎΠ»Π΅Π΅ Ρ‡Π΅ΠΌ Π² 100 Ρ€Π°Π· ΠΏΡ€Π΅Π²Ρ‹ΡˆΠ°Π΅Ρ‚ напряТСниС пробоя Π½Π΅ΠΉΠ»ΠΎΠ½Π°; (c) ΠŸΡ€Π΅Π΄ΠΏΠΎΠ»Π°Π³Π°Π΅ΠΌΡ‹ΠΉ заряд Π½Π΅ΠΎΠΏΡ€Π°Π²Π΄Π°Π½Π½ΠΎ Π²Π΅Π»ΠΈΠΊ ΠΈ Π½Π΅ ΠΌΠΎΠΆΠ΅Ρ‚ Ρ…Ρ€Π°Π½ΠΈΡ‚ΡŒΡΡ Π² кондСнсаторС Ρ‚Π°ΠΊΠΈΡ… Ρ€Π°Π·ΠΌΠ΅Ρ€ΠΎΠ².

ΠšΠΎΠ½Π΄Π΅Π½ΡΠ°Ρ‚ΠΎΡ€Ρ‹ ΠΈ Π΅ΠΌΠΊΠΎΡΡ‚ΡŒ — УнивСрситСтская Ρ„ΠΈΠ·ΠΈΠΊΠ°, Ρ‚ΠΎΠΌ 2

Π¦Π΅Π»ΠΈ обучСния

К ΠΊΠΎΠ½Ρ†Ρƒ этого Ρ€Π°Π·Π΄Π΅Π»Π° Π²Ρ‹ смоТСтС:

  • ΠžΠ±ΡŠΡΡΠ½ΠΈΡ‚Π΅ понятиС кондСнсатора ΠΈ Π΅Π³ΠΎ Смкости
  • ΠžΠΏΠΈΡˆΠΈΡ‚Π΅, ΠΊΠ°ΠΊ ΠΎΡ†Π΅Π½ΠΈΡ‚ΡŒ Π΅ΠΌΠΊΠΎΡΡ‚ΡŒ систСмы ΠΏΡ€ΠΎΠ²ΠΎΠ΄ΠΎΠ²

ΠšΠΎΠ½Π΄Π΅Π½ΡΠ°Ρ‚ΠΎΡ€ — это устройство, ΠΈΡΠΏΠΎΠ»ΡŒΠ·ΡƒΠ΅ΠΌΠΎΠ΅ для хранСния элСктричСского заряда ΠΈ элСктричСской энСргии.Он состоит ΠΊΠ°ΠΊ ΠΌΠΈΠ½ΠΈΠΌΡƒΠΌ ΠΈΠ· Π΄Π²ΡƒΡ… элСктричСских ΠΏΡ€ΠΎΠ²ΠΎΠ΄Π½ΠΈΠΊΠΎΠ², Ρ€Π°Π·Π΄Π΅Π»Π΅Π½Π½Ρ‹Ρ… расстояниСм. (ΠžΠ±Ρ€Π°Ρ‚ΠΈΡ‚Π΅ Π²Π½ΠΈΠΌΠ°Π½ΠΈΠ΅, Ρ‡Ρ‚ΠΎ Ρ‚Π°ΠΊΠΈΠ΅ элСктричСскиС ΠΏΡ€ΠΎΠ²ΠΎΠ΄Π½ΠΈΠΊΠΈ ΠΈΠ½ΠΎΠ³Π΄Π° Π½Π°Π·Ρ‹Π²Π°ΡŽΡ‚ «элСктродами», Π½ΠΎ, Ρ‚ΠΎΡ‡Π½Π΅Π΅, ΠΎΠ½ΠΈ Β«ΠΎΠ±ΠΊΠ»Π°Π΄ΠΊΠΈ кондСнсатора».) ΠŸΡ€ΠΎΡΡ‚Ρ€Π°Π½ΡΡ‚Π²ΠΎ ΠΌΠ΅ΠΆΠ΄Ρƒ кондСнсаторами ΠΌΠΎΠΆΠ΅Ρ‚ Π±Ρ‹Ρ‚ΡŒ просто Π²Π°ΠΊΡƒΡƒΠΌΠΎΠΌ, ΠΈ Π² этом случаС кондСнсатор Π±ΡƒΠ΄Π΅Ρ‚ извСстСн ΠΊΠ°ΠΊ Β«Π’Π°ΠΊΡƒΡƒΠΌΠ½Ρ‹ΠΉ кондСнсатор». Однако пространство ΠΎΠ±Ρ‹Ρ‡Π½ΠΎ Π·Π°ΠΏΠΎΠ»Π½Π΅Π½ΠΎ изоляционным ΠΌΠ°Ρ‚Π΅Ρ€ΠΈΠ°Π»ΠΎΠΌ, извСстным ΠΊΠ°ΠΊ диэлСктрик. (Π’Ρ‹ ΡƒΠ·Π½Π°Π΅Ρ‚Π΅ большС ΠΎ диэлСктриках Π² Ρ€Π°Π·Π΄Π΅Π»Π°Ρ…, посвящСнных диэлСктрикам, Π΄Π°Π»Π΅Π΅ Π² этой Π³Π»Π°Π²Π΅.) ОбъСм кондСнсатора опрСдСляСтся свойством, Π½Π°Π·Ρ‹Π²Π°Π΅ΠΌΡ‹ΠΌ Π΅ΠΌΠΊΠΎΡΡ‚ΡŒΡŽ , , ΠΎ ΠΊΠΎΡ‚ΠΎΡ€ΠΎΠΌ Π²Ρ‹ ΡƒΠ·Π½Π°Π΅Ρ‚Π΅ большС Ρ‡ΡƒΡ‚ΡŒ ΠΏΠΎΠ·ΠΆΠ΅ Π² этом Ρ€Π°Π·Π΄Π΅Π»Π΅.

ΠšΠΎΠ½Π΄Π΅Π½ΡΠ°Ρ‚ΠΎΡ€Ρ‹

ΠΌΠΎΠ³ΡƒΡ‚ ΠΏΡ€ΠΈΠΌΠ΅Π½ΡΡ‚ΡŒΡΡ Π² самых Ρ€Π°Π·Π½Ρ‹Ρ… областях — ΠΎΡ‚ Ρ„ΠΈΠ»ΡŒΡ‚Ρ€Π°Ρ†ΠΈΠΈ статичСского элСктричСства ΠΎΡ‚ Ρ€Π°Π΄ΠΈΠΎΠΏΡ€ΠΈΠ΅ΠΌΠ° Π΄ΠΎ накоплСния энСргии Π² дСфибрилляторах сСрдца. Как ΠΏΡ€Π°Π²ΠΈΠ»ΠΎ, Ρƒ ΠΏΡ€ΠΎΠΌΡ‹ΡˆΠ»Π΅Π½Π½Ρ‹Ρ… кондСнсаторов Π΄Π²Π΅ проводящиС части располоТСны Π±Π»ΠΈΠ·ΠΊΠΎ Π΄Ρ€ΡƒΠ³ ΠΊ Π΄Ρ€ΡƒΠ³Ρƒ, Π½ΠΎ Π½Π΅ ΡΠΎΠΏΡ€ΠΈΠΊΠ°ΡΠ°ΡŽΡ‚ΡΡ, ΠΊΠ°ΠΊ ΠΏΠΎΠΊΠ°Π·Π°Π½ΠΎ Π½Π° (Рисунок). Π’ Π±ΠΎΠ»ΡŒΡˆΠΈΠ½ΡΡ‚Π²Π΅ случаСв ΠΌΠ΅ΠΆΠ΄Ρƒ двумя пластинами ΠΈΡΠΏΠΎΠ»ΡŒΠ·ΡƒΠ΅Ρ‚ΡΡ диэлСктрик. Когда ΠΊΠ»Π΅ΠΌΠΌΡ‹ аккумулятора ΠΏΠΎΠ΄ΠΊΠ»ΡŽΡ‡Π΅Π½Ρ‹ ΠΊ ΠΈΠ·Π½Π°Ρ‡Π°Π»ΡŒΠ½ΠΎ нСзаряТСнному кондСнсатору, ΠΏΠΎΡ‚Π΅Π½Ρ†ΠΈΠ°Π» аккумулятора ΠΏΠ΅Ρ€Π΅ΠΌΠ΅Ρ‰Π°Π΅Ρ‚ нСбольшой заряд Π²Π΅Π»ΠΈΡ‡ΠΈΠ½ΠΎΠΉ Q с ΠΏΠΎΠ»ΠΎΠΆΠΈΡ‚Π΅Π»ΡŒΠ½ΠΎΠΉ пластины Π½Π° ΠΎΡ‚Ρ€ΠΈΡ†Π°Ρ‚Π΅Π»ΡŒΠ½ΡƒΡŽ.ΠšΠΎΠ½Π΄Π΅Π½ΡΠ°Ρ‚ΠΎΡ€ Π² Ρ†Π΅Π»ΠΎΠΌ остаСтся Π½Π΅ΠΉΡ‚Ρ€Π°Π»ΡŒΠ½Ρ‹ΠΌ, Π½ΠΎ заряТаСтся ΠΈ находится Π½Π° ΠΏΡ€ΠΎΡ‚ΠΈΠ²ΠΎΠΏΠΎΠ»ΠΎΠΆΠ½Ρ‹Ρ… пластинах.

Оба кондСнсатора, ΠΏΠΎΠΊΠ°Π·Π°Π½Π½Ρ‹Π΅ здСсь, Π±Ρ‹Π»ΠΈ ΠΈΠ·Π½Π°Ρ‡Π°Π»ΡŒΠ½ΠΎ разряТСны ΠΏΠ΅Ρ€Π΅Π΄ ΠΏΠΎΠ΄ΠΊΠ»ΡŽΡ‡Π΅Π½ΠΈΠ΅ΠΌ ΠΊ Π±Π°Ρ‚Π°Ρ€Π΅Π΅. Π’Π΅ΠΏΠ΅Ρ€ΡŒ Ρƒ Π½ΠΈΡ… Π΅ΡΡ‚ΡŒ заряды ΠΈ (соотвСтствСнно) Π½Π° своих Ρ‚Π°Ρ€Π΅Π»ΠΊΠ°Ρ…. (a) ΠšΠΎΠ½Π΄Π΅Π½ΡΠ°Ρ‚ΠΎΡ€ с ΠΏΠ°Ρ€Π°Π»Π»Π΅Π»ΡŒΠ½Ρ‹ΠΌΠΈ пластинами состоит ΠΈΠ· Π΄Π²ΡƒΡ… пластин ΠΏΡ€ΠΎΡ‚ΠΈΠ²ΠΎΠΏΠΎΠ»ΠΎΠΆΠ½ΠΎΠ³ΠΎ заряда с ΠΏΠ»ΠΎΡ‰Π°Π΄ΡŒΡŽ A, , Ρ€Π°Π·Π΄Π΅Π»Π΅Π½Π½Ρ‹ΠΌΠΈ расстояниСм d . (b) ΠšΠ°Ρ‚Π°Π½Ρ‹ΠΉ кондСнсатор ΠΈΠΌΠ΅Π΅Ρ‚ диэлСктричСский ΠΌΠ°Ρ‚Π΅Ρ€ΠΈΠ°Π» ΠΌΠ΅ΠΆΠ΄Ρƒ двумя проводящими листами (пластинами).

БистСма, состоящая ΠΈΠ· Π΄Π²ΡƒΡ… ΠΈΠ΄Π΅Π½Ρ‚ΠΈΡ‡Π½Ρ‹Ρ… ΠΏΠ°Ρ€Π°Π»Π»Π΅Π»ΡŒΠ½ΠΎ проводящих пластин, Ρ€Π°Π·Π΄Π΅Π»Π΅Π½Π½Ρ‹Ρ… расстояниСм, называСтся кондСнсатором с ΠΏΠ°Ρ€Π°Π»Π»Π΅Π»ΡŒΠ½Ρ‹ΠΌΠΈ пластинами ((Рисунок)).Π’Π΅Π»ΠΈΡ‡ΠΈΠ½Π° элСктричСского поля Π² пространствС ΠΌΠ΅ΠΆΠ΄Ρƒ ΠΏΠ°Ρ€Π°Π»Π»Π΅Π»ΡŒΠ½Ρ‹ΠΌΠΈ пластинами составляСт, Π³Π΄Π΅ ΠΎΠ±ΠΎΠ·Π½Π°Ρ‡Π°Π΅Ρ‚ ΠΏΠΎΠ²Π΅Ρ€Ρ…Π½ΠΎΡΡ‚Π½ΡƒΡŽ ΠΏΠ»ΠΎΡ‚Π½ΠΎΡΡ‚ΡŒ заряда Π½Π° ΠΎΠ΄Π½ΠΎΠΉ пластинС (Π½Π°ΠΏΠΎΠΌΠ½ΠΈΠΌ, Ρ‡Ρ‚ΠΎ это заряд Q Π½Π° ΠΏΠ»ΠΎΡ‰Π°Π΄ΡŒ повСрхности A ). Π’Π°ΠΊΠΈΠΌ ΠΎΠ±Ρ€Π°Π·ΠΎΠΌ, Π²Π΅Π»ΠΈΡ‡ΠΈΠ½Π° поля прямо ΠΏΡ€ΠΎΠΏΠΎΡ€Ρ†ΠΈΠΎΠ½Π°Π»ΡŒΠ½Π° Q .

Π Π°Π·Π΄Π΅Π»Π΅Π½ΠΈΠ΅ зарядов Π² кондСнсаторС ΠΏΠΎΠΊΠ°Π·Ρ‹Π²Π°Π΅Ρ‚, Ρ‡Ρ‚ΠΎ заряды ΠΎΡΡ‚Π°ΡŽΡ‚ΡΡ Π½Π° повСрхности ΠΎΠ±ΠΊΠ»Π°Π΄ΠΎΠΊ кондСнсатора. Π›ΠΈΠ½ΠΈΠΈ элСктричСского поля Π² кондСнсаторС с ΠΏΠ°Ρ€Π°Π»Π»Π΅Π»ΡŒΠ½Ρ‹ΠΌΠΈ пластинами Π½Π°Ρ‡ΠΈΠ½Π°ΡŽΡ‚ΡΡ с ΠΏΠΎΠ»ΠΎΠΆΠΈΡ‚Π΅Π»ΡŒΠ½Ρ‹Ρ… зарядов ΠΈ Π·Π°ΠΊΠ°Π½Ρ‡ΠΈΠ²Π°ΡŽΡ‚ΡΡ ΠΎΡ‚Ρ€ΠΈΡ†Π°Ρ‚Π΅Π»ΡŒΠ½Ρ‹ΠΌΠΈ зарядами.Π’Π΅Π»ΠΈΡ‡ΠΈΠ½Π° элСктричСского поля Π² пространствС ΠΌΠ΅ΠΆΠ΄Ρƒ пластинами прямо ΠΏΡ€ΠΎΠΏΠΎΡ€Ρ†ΠΈΠΎΠ½Π°Π»ΡŒΠ½Π° количСству заряда Π½Π° кондСнсаторС.

ΠšΠΎΠ½Π΄Π΅Π½ΡΠ°Ρ‚ΠΎΡ€Ρ‹ с Ρ€Π°Π·Π½Ρ‹ΠΌΠΈ физичСскими характСристиками (Ρ‚Π°ΠΊΠΈΠΌΠΈ ΠΊΠ°ΠΊ Ρ„ΠΎΡ€ΠΌΠ° ΠΈ Ρ€Π°Π·ΠΌΠ΅Ρ€ пластин) Π½Π°ΠΊΠ°ΠΏΠ»ΠΈΠ²Π°ΡŽΡ‚ Ρ€Π°Π·Π½ΠΎΠ΅ количСство заряда ΠΏΡ€ΠΈ ΠΎΠ΄ΠΈΠ½Π°ΠΊΠΎΠ²ΠΎΠΌ ΠΏΡ€ΠΈΠ»ΠΎΠΆΠ΅Π½Π½ΠΎΠΌ напряТСнии Π’ Π½Π° своих пластинах. Π•ΠΌΠΊΠΎΡΡ‚ΡŒ C кондСнсатора опрСдСляСтся ΠΊΠ°ΠΊ ΠΎΡ‚Π½ΠΎΡˆΠ΅Π½ΠΈΠ΅ максимального заряда Q , ΠΊΠΎΡ‚ΠΎΡ€Ρ‹ΠΉ ΠΌΠΎΠΆΠ΅Ρ‚ Ρ…Ρ€Π°Π½ΠΈΡ‚ΡŒΡΡ Π² кондСнсаторС, ΠΊ ΠΏΡ€ΠΈΠ»ΠΎΠΆΠ΅Π½Π½ΠΎΠΌΡƒ Π½Π°ΠΏΡ€ΡΠΆΠ΅Π½ΠΈΡŽ Π’ Π½Π° Π΅Π³ΠΎ пластинах.Π”Ρ€ΡƒΠ³ΠΈΠΌΠΈ словами, Π΅ΠΌΠΊΠΎΡΡ‚ΡŒ — это наибольшая Π²Π΅Π»ΠΈΡ‡ΠΈΠ½Π° заряда Π½Π° Π²ΠΎΠ»ΡŒΡ‚, которая ΠΌΠΎΠΆΠ΅Ρ‚ Ρ…Ρ€Π°Π½ΠΈΡ‚ΡŒΡΡ Π½Π° устройствС:

Π•Π΄ΠΈΠ½ΠΈΡ†Π° измСрСния Смкости Π² систСмС БИ — Ρ„Π°Ρ€Π°Π΄ (Π€), названная Π² Ρ‡Π΅ΡΡ‚ΡŒ Майкла ЀарадСя (1791–1867). ΠŸΠΎΡΠΊΠΎΠ»ΡŒΠΊΡƒ Π΅ΠΌΠΊΠΎΡΡ‚ΡŒ — это заряд Π½Π° Π΅Π΄ΠΈΠ½ΠΈΡ†Ρƒ напряТСния, ΠΎΠ΄ΠΈΠ½ Ρ„Π°Ρ€Π°Π΄ Ρ€Π°Π²Π΅Π½ ΠΎΠ΄Π½ΠΎΠΌΡƒ ΠΊΡƒΠ»ΠΎΠ½Ρƒ Π½Π° ΠΎΠ΄ΠΈΠ½ Π²ΠΎΠ»ΡŒΡ‚, ΠΈΠ»ΠΈ

.

По ΠΎΠΏΡ€Π΅Π΄Π΅Π»Π΅Π½ΠΈΡŽ, кондСнсатор Π΅ΠΌΠΊΠΎΡΡ‚ΡŒΡŽ 1,0 ΠΌΠΊΠ€ ΠΌΠΎΠΆΠ΅Ρ‚ ΡΠΎΡ…Ρ€Π°Π½ΡΡ‚ΡŒ заряд 1,0 К (ΠΎΡ‡Π΅Π½ΡŒ большой заряд), ΠΊΠΎΠ³Π΄Π° Ρ€Π°Π·Π½ΠΎΡΡ‚ΡŒ ΠΏΠΎΡ‚Π΅Π½Ρ†ΠΈΠ°Π»ΠΎΠ² ΠΌΠ΅ΠΆΠ΄Ρƒ Π΅Π³ΠΎ пластинами составляСт всСго 1,0 Π’. Π‘Π»Π΅Π΄ΠΎΠ²Π°Ρ‚Π΅Π»ΡŒΠ½ΠΎ, ΠΎΠ΄ΠΈΠ½ Ρ„Π°Ρ€Π°Π΄ — это ΠΎΡ‡Π΅Π½ΡŒ большая Π΅ΠΌΠΊΠΎΡΡ‚ΡŒ.Π’ΠΈΠΏΠΈΡ‡Π½Ρ‹Π΅ значСния Смкости находятся Π² Π΄ΠΈΠ°ΠΏΠ°Π·ΠΎΠ½Π΅ ΠΎΡ‚ ΠΏΠΈΠΊΠΎΡ„Π°Ρ€Π°Π΄ Π΄ΠΎ ΠΌΠΈΠ»Π»ΠΈΡ„Π°Ρ€Π°Π΄, Π²ΠΊΠ»ΡŽΡ‡Π°Ρ ΠΌΠΈΠΊΡ€ΠΎΡ„Π°Ρ€Π°Π΄Ρ‹ (). ΠšΠΎΠ½Π΄Π΅Π½ΡΠ°Ρ‚ΠΎΡ€Ρ‹ ΠΌΠΎΠ³ΡƒΡ‚ Π±Ρ‹Ρ‚ΡŒ ΠΈΠ·Π³ΠΎΡ‚ΠΎΠ²Π»Π΅Π½Ρ‹ Ρ€Π°Π·Π»ΠΈΡ‡Π½Ρ‹Ρ… Ρ„ΠΎΡ€ΠΌ ΠΈ Ρ€Π°Π·ΠΌΠ΅Ρ€ΠΎΠ² ((рисунок)).

Π­Ρ‚ΠΎ Π½Π΅ΠΊΠΎΡ‚ΠΎΡ€Ρ‹Π΅ Ρ‚ΠΈΠΏΠΈΡ‡Π½Ρ‹Π΅ кондСнсаторы, ΠΈΡΠΏΠΎΠ»ΡŒΠ·ΡƒΠ΅ΠΌΡ‹Π΅ Π² элСктронных устройствах. Π Π°Π·ΠΌΠ΅Ρ€ кондСнсатора Π½Π΅ ΠΎΠ±ΡΠ·Π°Ρ‚Π΅Π»ΡŒΠ½ΠΎ зависит ΠΎΡ‚ Π΅Π³ΠΎ Смкости. (Π˜ΡΡ‚ΠΎΡ‡Π½ΠΈΠΊ: Windell Oskay)

РасчСт Смкости

ΠœΡ‹ ΠΌΠΎΠΆΠ΅ΠΌ Ρ€Π°ΡΡΡ‡ΠΈΡ‚Π°Ρ‚ΡŒ Π΅ΠΌΠΊΠΎΡΡ‚ΡŒ ΠΏΠ°Ρ€Ρ‹ ΠΏΡ€ΠΎΠ²ΠΎΠ΄ΠΎΠ² с ΠΏΠΎΠΌΠΎΡ‰ΡŒΡŽ ΡΠ»Π΅Π΄ΡƒΡŽΡ‰Π΅Π³ΠΎ стандартного ΠΏΠΎΠ΄Ρ…ΠΎΠ΄Π°.

БтратСгия Ρ€Π΅ΡˆΠ΅Π½ΠΈΡ ΠΏΡ€ΠΎΠ±Π»Π΅ΠΌ: расчСт Смкости

  1. ΠŸΡ€Π΅Π΄ΠΏΠΎΠ»ΠΎΠΆΠΈΠΌ, Ρ‡Ρ‚ΠΎ кондСнсатор ΠΈΠΌΠ΅Π΅Ρ‚ заряд Q .
  2. ΠžΠΏΡ€Π΅Π΄Π΅Π»ΠΈΡ‚Π΅ элСктричСскоС ΠΏΠΎΠ»Π΅ ΠΌΠ΅ΠΆΠ΄Ρƒ ΠΏΡ€ΠΎΠ²ΠΎΠ΄Π½ΠΈΠΊΠ°ΠΌΠΈ. Если симмСтрия присутствуСт Π² располоТСнии ΠΏΡ€ΠΎΠ²ΠΎΠ΄Π½ΠΈΠΊΠΎΠ², Π²Ρ‹ ΠΌΠΎΠΆΠ΅Ρ‚Π΅ ΠΈΡΠΏΠΎΠ»ΡŒΠ·ΠΎΠ²Π°Ρ‚ΡŒ Π·Π°ΠΊΠΎΠ½ Гаусса для этого расчСта.
  3. НайдитС Ρ€Π°Π·Π½ΠΎΡΡ‚ΡŒ ΠΏΠΎΡ‚Π΅Π½Ρ†ΠΈΠ°Π»ΠΎΠ² ΠΌΠ΅ΠΆΠ΄Ρƒ ΠΏΡ€ΠΎΠ²ΠΎΠ΄Π½ΠΈΠΊΠ°ΠΌΠΈ ΠΈΠ·

    , Π³Π΄Π΅ ΠΏΡƒΡ‚ΡŒ ΠΈΠ½Ρ‚Π΅Π³Ρ€Π°Ρ†ΠΈΠΈ Π²Π΅Π΄Π΅Ρ‚ ΠΎΡ‚ ΠΎΠ΄Π½ΠΎΠ³ΠΎ ΠΏΡ€ΠΎΠ²ΠΎΠ΄Π½ΠΈΠΊΠ° ΠΊ Π΄Ρ€ΡƒΠ³ΠΎΠΌΡƒ. Π’ΠΎΠ³Π΄Π° Π²Π΅Π»ΠΈΡ‡ΠΈΠ½Π° разности ΠΏΠΎΡ‚Π΅Π½Ρ†ΠΈΠ°Π»ΠΎΠ² Ρ€Π°Π²Π½Π°.

  4. Если извСстно Π’, , ΠΎΠΏΡ€Π΅Π΄Π΅Π»ΠΈΡ‚Π΅ Π΅ΠΌΠΊΠΎΡΡ‚ΡŒ нСпосрСдствСнно ΠΈΠ· (Рисунок).

Π§Ρ‚ΠΎΠ±Ρ‹ ΠΏΠΎΠΊΠ°Π·Π°Ρ‚ΡŒ, ΠΊΠ°ΠΊ Ρ€Π°Π±ΠΎΡ‚Π°Π΅Ρ‚ эта ΠΏΡ€ΠΎΡ†Π΅Π΄ΡƒΡ€Π°, ΠΌΡ‹ Ρ‚Π΅ΠΏΠ΅Ρ€ΡŒ вычисляСм Смкости ΠΏΠ°Ρ€Π°Π»Π»Π΅Π»ΡŒΠ½Ρ‹Ρ… пластин, сфСричСских ΠΈ цилиндричСских кондСнсаторов. Π’ΠΎ всСх случаях ΠΌΡ‹ ΠΏΡ€Π΅Π΄ΠΏΠΎΠ»Π°Π³Π°Π΅ΠΌ Π²Π°ΠΊΡƒΡƒΠΌΠ½Ρ‹Π΅ кондСнсаторы (пустыС кондСнсаторы) Π±Π΅Π· диэлСктричСского вСщСства Π² пространствС ΠΌΠ΅ΠΆΠ΄Ρƒ ΠΏΡ€ΠΎΠ²ΠΎΠ΄Π½ΠΈΠΊΠ°ΠΌΠΈ.

ΠšΠΎΠ½Π΄Π΅Π½ΡΠ°Ρ‚ΠΎΡ€ с ΠΏΠ°Ρ€Π°Π»Π»Π΅Π»ΡŒΠ½Ρ‹ΠΌΠΈ пластинами

ΠšΠΎΠ½Π΄Π΅Π½ΡΠ°Ρ‚ΠΎΡ€ с ΠΏΠ°Ρ€Π°Π»Π»Π΅Π»ΡŒΠ½Ρ‹ΠΌΠΈ пластинами ((Рисунок)) ΠΈΠΌΠ΅Π΅Ρ‚ Π΄Π²Π΅ ΠΈΠ΄Π΅Π½Ρ‚ΠΈΡ‡Π½Ρ‹Π΅ проводящиС пластины, каТдая ΠΈΠ· ΠΊΠΎΡ‚ΠΎΡ€Ρ‹Ρ… ΠΈΠΌΠ΅Π΅Ρ‚ ΠΏΠ»ΠΎΡ‰Π°Π΄ΡŒ повСрхности A , Ρ€Π°Π·Π΄Π΅Π»Π΅Π½Π½Ρ‹Ρ… расстояниСм d .Когда Π½Π° кондСнсатор подаСтся напряТСниС Π’, , ΠΎΠ½ сохраняСт заряд Q , ΠΊΠ°ΠΊ ΠΏΠΎΠΊΠ°Π·Π°Π½ΠΎ. ΠœΡ‹ ΠΌΠΎΠΆΠ΅ΠΌ ΡƒΠ²ΠΈΠ΄Π΅Ρ‚ΡŒ, ΠΊΠ°ΠΊ Π΅Π³ΠΎ Π΅ΠΌΠΊΠΎΡΡ‚ΡŒ ΠΌΠΎΠΆΠ΅Ρ‚ Π·Π°Π²ΠΈΡΠ΅Ρ‚ΡŒ ΠΎΡ‚ A ΠΈ d , рассматривая характСристики кулоновской силы. ΠœΡ‹ Π·Π½Π°Π΅ΠΌ, Ρ‡Ρ‚ΠΎ сила ΠΌΠ΅ΠΆΠ΄Ρƒ зарядами увСличиваСтся с ΡƒΠ²Π΅Π»ΠΈΡ‡Π΅Π½ΠΈΠ΅ΠΌ заряда ΠΈ ΡƒΠΌΠ΅Π½ΡŒΡˆΠ°Π΅Ρ‚ΡΡ с расстояниСм ΠΌΠ΅ΠΆΠ΄Ρƒ Π½ΠΈΠΌΠΈ. Π‘Π»Π΅Π΄ΡƒΠ΅Ρ‚ ΠΎΠΆΠΈΠ΄Π°Ρ‚ΡŒ, Ρ‡Ρ‚ΠΎ Ρ‡Π΅ΠΌ большС пластины, Ρ‚Π΅ΠΌ большС заряда ΠΎΠ½ΠΈ ΠΌΠΎΠ³ΡƒΡ‚ Ρ…Ρ€Π°Π½ΠΈΡ‚ΡŒ. Π’Π°ΠΊΠΈΠΌ ΠΎΠ±Ρ€Π°Π·ΠΎΠΌ, C Π΄ΠΎΠ»ΠΆΠ½ΠΎ Π±Ρ‹Ρ‚ΡŒ большС для большСго значСния A .Π’ΠΎΡ‡Π½ΠΎ Ρ‚Π°ΠΊ ΠΆΠ΅, Ρ‡Π΅ΠΌ Π±Π»ΠΈΠΆΠ΅ пластины Π΄Ρ€ΡƒΠ³ ΠΊ Π΄Ρ€ΡƒΠ³Ρƒ, Ρ‚Π΅ΠΌ большС Π½Π° Π½ΠΈΡ… притяТСния ΠΏΡ€ΠΎΡ‚ΠΈΠ²ΠΎΠΏΠΎΠ»ΠΎΠΆΠ½Ρ‹Ρ… зарядов. Π‘Π»Π΅Π΄ΠΎΠ²Π°Ρ‚Π΅Π»ΡŒΠ½ΠΎ, C Π΄ΠΎΠ»ΠΆΠ½ΠΎ Π±Ρ‹Ρ‚ΡŒ большС для мСньшСго d .

Π’ кондСнсаторС с ΠΏΠ°Ρ€Π°Π»Π»Π΅Π»ΡŒΠ½Ρ‹ΠΌΠΈ пластинами с пластинами, разнСсСнными Π½Π° расстояниС d , каТдая пластина ΠΈΠΌΠ΅Π΅Ρ‚ ΠΎΠ΄ΠΈΠ½Π°ΠΊΠΎΠ²ΡƒΡŽ ΠΏΠ»ΠΎΡ‰Π°Π΄ΡŒ повСрхности A .

ΠžΠΏΡ€Π΅Π΄Π΅Π»ΠΈΠΌ ΠΏΠΎΠ²Π΅Ρ€Ρ…Π½ΠΎΡΡ‚Π½ΡƒΡŽ ΠΏΠ»ΠΎΡ‚Π½ΠΎΡΡ‚ΡŒ заряда Π½Π° пластинах ΠΊΠ°ΠΊ

Из ΠΏΡ€Π΅Π΄Ρ‹Π΄ΡƒΡ‰ΠΈΡ… Π³Π»Π°Π² ΠΌΡ‹ Π·Π½Π°Π΅ΠΌ, Ρ‡Ρ‚ΠΎ, ΠΊΠΎΠ³Π΄Π° d ΠΌΠ°Π»ΠΎ, элСктричСскоС ΠΏΠΎΠ»Π΅ ΠΌΠ΅ΠΆΠ΄Ρƒ пластинами довольно ΠΎΠ΄Π½ΠΎΡ€ΠΎΠ΄Π½ΠΎ (Π±Π΅Π· ΡƒΡ‡Π΅Ρ‚Π° ΠΊΡ€Π°Π΅Π²Ρ‹Ρ… эффСктов) ΠΈ Ρ‡Ρ‚ΠΎ Π΅Π³ΠΎ Π²Π΅Π»ΠΈΡ‡ΠΈΠ½Π° опрСдСляСтся ΠΊΠ°ΠΊ

, Π³Π΄Π΅ константа — это диэлСктричСская ΠΏΡ€ΠΎΠ½ΠΈΡ†Π°Π΅ΠΌΠΎΡΡ‚ΡŒ свободного пространства, Π΅Π΄ΠΈΠ½ΠΈΡ†Π° измСрСния Π€ / ΠΌ Π² систСмС БИ эквивалСнтна Ρ‚Π°ΠΊ ΠΊΠ°ΠΊ элСктричСскоС ΠΏΠΎΠ»Π΅ ΠΌΠ΅ΠΆΠ΄Ρƒ пластинами ΠΎΠ΄Π½ΠΎΡ€ΠΎΠ΄Π½ΠΎ, Ρ€Π°Π·Π½ΠΎΡΡ‚ΡŒ ΠΏΠΎΡ‚Π΅Π½Ρ†ΠΈΠ°Π»ΠΎΠ² ΠΌΠ΅ΠΆΠ΄Ρƒ пластинами Ρ€Π°Π²Π½Π°

.

Π‘Π»Π΅Π΄ΠΎΠ²Π°Ρ‚Π΅Π»ΡŒΠ½ΠΎ (рисунок) Π΄Π°Π΅Ρ‚ Π΅ΠΌΠΊΠΎΡΡ‚ΡŒ кондСнсатора с ΠΏΠ°Ρ€Π°Π»Π»Π΅Π»ΡŒΠ½Ρ‹ΠΌΠΈ пластинами ΠΊΠ°ΠΊ

ΠžΠ±Ρ€Π°Ρ‚ΠΈΡ‚Π΅ Π²Π½ΠΈΠΌΠ°Π½ΠΈΠ΅ Π½Π° это ΡƒΡ€Π°Π²Π½Π΅Π½ΠΈΠ΅, Ρ‡Ρ‚ΠΎ Π΅ΠΌΠΊΠΎΡΡ‚ΡŒ являСтся Ρ„ΡƒΠ½ΠΊΡ†ΠΈΠ΅ΠΉ Ρ‚ΠΎΠ»ΡŒΠΊΠΎ Π³Π΅ΠΎΠΌΠ΅Ρ‚Ρ€ΠΈΠΈ ΠΈ Ρ‚ΠΎΠ³ΠΎ, ΠΊΠ°ΠΊΠΎΠΉ ΠΌΠ°Ρ‚Π΅Ρ€ΠΈΠ°Π» заполняСт пространство ΠΌΠ΅ΠΆΠ΄Ρƒ пластинами (Π² Π΄Π°Π½Π½ΠΎΠΌ случаС Π²Π°ΠΊΡƒΡƒΠΌ) этого кондСнсатора.ЀактичСски, это Π²Π΅Ρ€Π½ΠΎ Π½Π΅ Ρ‚ΠΎΠ»ΡŒΠΊΠΎ для кондСнсатора с ΠΏΠ°Ρ€Π°Π»Π»Π΅Π»ΡŒΠ½Ρ‹ΠΌΠΈ пластинами, Π½ΠΎ ΠΈ для всСх кондСнсаторов: Π΅ΠΌΠΊΠΎΡΡ‚ΡŒ Π½Π΅ зависит ΠΎΡ‚ Q ΠΈΠ»ΠΈ Π’ . Если заряд измСняСтся, ΠΏΠΎΡ‚Π΅Π½Ρ†ΠΈΠ°Π» измСняСтся соотвСтствСнно, Ρ‚Π°ΠΊ Ρ‡Ρ‚ΠΎ Q / V остаСтся постоянным.

Π•ΠΌΠΊΠΎΡΡ‚ΡŒ ΠΈ заряд, Π½Π°ΠΊΠΎΠΏΠ»Π΅Π½Π½Ρ‹ΠΉ Π² кондСнсаторС с ΠΏΠ°Ρ€Π°Π»Π»Π΅Π»ΡŒΠ½Ρ‹ΠΌΠΈ пластинами (a) Какова Π΅ΠΌΠΊΠΎΡΡ‚ΡŒ пустого кондСнсатора с ΠΏΠ°Ρ€Π°Π»Π»Π΅Π»ΡŒΠ½Ρ‹ΠΌΠΈ пластинами с мСталличСскими пластинами, ΠΏΠ»ΠΎΡ‰Π°Π΄ΡŒ ΠΊΠ°ΠΆΠ΄ΠΎΠΉ ΠΈΠ· ΠΊΠΎΡ‚ΠΎΡ€Ρ‹Ρ… составляСт 1,00 ΠΌΠΌ? (Π±) Бколько заряда хранится Π² этом кондСнсаторС, Ссли ΠΊ Π½Π΅ΠΌΡƒ ΠΏΡ€ΠΈΠ»ΠΎΠΆΠ΅Π½ΠΎ напряТСниС?

БтратСгия ΠžΠΏΡ€Π΅Π΄Π΅Π»Π΅Π½ΠΈΠ΅ Смкости C прСдставляСт собой простоС ΠΏΡ€ΠΈΠ»ΠΎΠΆΠ΅Π½ΠΈΠ΅ (рисунок). Найдя C , ΠΌΡ‹ смоТСм Π½Π°ΠΉΡ‚ΠΈ Π½Π°ΠΊΠΎΠΏΠ»Π΅Π½Π½Ρ‹ΠΉ заряд, ΠΈΡΠΏΠΎΠ»ΡŒΠ·ΡƒΡ (Рисунок).

РСшСниС

  1. Π’Π²ΠΎΠ΄ Π΄Π°Π½Π½Ρ‹Ρ… Π·Π½Π°Ρ‡Π΅Π½ΠΈΠΉ Π² (рисунок) Π΄Π°Π΅Ρ‚

    Π­Ρ‚ΠΎ нСбольшоС Π·Π½Π°Ρ‡Π΅Π½ΠΈΠ΅ Смкости ΡƒΠΊΠ°Π·Ρ‹Π²Π°Π΅Ρ‚ Π½Π° Ρ‚ΠΎ, насколько слоТно ΠΈΠ·Π³ΠΎΡ‚ΠΎΠ²ΠΈΡ‚ΡŒ устройство с большой Π΅ΠΌΠΊΠΎΡΡ‚ΡŒΡŽ.

  2. Π˜Π½Π²Π΅Ρ€Ρ‚ΠΈΡ€ΠΎΠ²Π°Π½ΠΈΠ΅ (рисунок) ΠΈ Π²Π²ΠΎΠ΄ извСстных Π·Π½Π°Ρ‡Π΅Π½ΠΈΠΉ Π² это ΡƒΡ€Π°Π²Π½Π΅Π½ΠΈΠ΅ Π΄Π°Π΅Ρ‚

Π—Π½Π°Ρ‡Π΅Π½ΠΈΠ΅ Π­Ρ‚ΠΎΡ‚ заряд лишь Π½Π΅ΠΌΠ½ΠΎΠ³ΠΎ большС, Ρ‡Π΅ΠΌ Π² Ρ‚ΠΈΠΏΠΈΡ‡Π½Ρ‹Ρ… прилоТСниях статичСского элСктричСства.ΠŸΠΎΡΠΊΠΎΠ»ΡŒΠΊΡƒ Π²ΠΎΠ·Π΄ΡƒΡ… Ρ€Π°Π·Ρ€ΡƒΡˆΠ°Π΅Ρ‚ΡΡ (становится проводящим) ΠΏΡ€ΠΈ напряТСнности элСктричСского поля ΠΎΠΊΠΎΠ»ΠΎ 3,0 ΠœΠ’ / ΠΌ, Π½Π° этом кондСнсаторС большС Π½Π΅ ΠΌΠΎΠΆΠ΅Ρ‚ Ρ…Ρ€Π°Π½ΠΈΡ‚ΡŒΡΡ заряд ΠΏΡ€ΠΈ ΡƒΠ²Π΅Π»ΠΈΡ‡Π΅Π½ΠΈΠΈ напряТСния.

A 1-F кондСнсатор с ΠΏΠ°Ρ€Π°Π»Π»Π΅Π»ΡŒΠ½Ρ‹ΠΌΠΈ пластинами. ΠŸΡ€Π΅Π΄ΠΏΠΎΠ»ΠΎΠΆΠΈΠΌ, Π²Ρ‹ Ρ…ΠΎΡ‚ΠΈΡ‚Π΅ ΡΠΊΠΎΠ½ΡΡ‚Ρ€ΡƒΠΈΡ€ΠΎΠ²Π°Ρ‚ΡŒ кондСнсатор с ΠΏΠ°Ρ€Π°Π»Π»Π΅Π»ΡŒΠ½Ρ‹ΠΌΠΈ пластинами Π΅ΠΌΠΊΠΎΡΡ‚ΡŒΡŽ 1,0 F. ΠšΠ°ΠΊΡƒΡŽ ΠΏΠ»ΠΎΡ‰Π°Π΄ΡŒ Π½Π΅ΠΎΠ±Ρ…ΠΎΠ΄ΠΈΠΌΠΎ ΠΈΡΠΏΠΎΠ»ΡŒΠ·ΠΎΠ²Π°Ρ‚ΡŒ для ΠΊΠ°ΠΆΠ΄ΠΎΠΉ пластины, Ссли пластины Ρ€Π°Π·Π΄Π΅Π»Π΅Π½Ρ‹ Π½Π° 1,0 ΠΌΠΌ?

ΠŸΠ΅Ρ€Π΅ΡΡ‚Π°Π²Π»ΡΡ Ρ€Π΅ΡˆΠ΅Π½ΠΈΠ΅ (рисунок), ΠΏΠΎΠ»ΡƒΡ‡Π°Π΅ΠΌ

КаТдая квадратная пластина Π΄ΠΎΠ»ΠΆΠ½Π° Π±Ρ‹Ρ‚ΡŒ 10 ΠΊΠΌ Π² ΠΏΠΎΠΏΠ΅Ρ€Π΅Ρ‡Π½ΠΈΠΊΠ΅. РаньшС Π±Ρ‹Π»ΠΎ ΠΎΠ±Ρ‹Ρ‡Π½Ρ‹ΠΌ Ρ€ΠΎΠ·Ρ‹Π³Ρ€Ρ‹ΡˆΠ΅ΠΌ — ΠΏΠΎΠΏΡ€ΠΎΡΠΈΡ‚ΡŒ студСнта ΠΏΠΎΠΉΡ‚ΠΈ Π² склад Π»Π°Π±ΠΎΡ€Π°Ρ‚ΠΎΡ€ΠΈΠΈ ΠΈ ΠΏΠΎΠΏΡ€ΠΎΡΠΈΡ‚ΡŒ кондСнсатор с ΠΏΠ°Ρ€Π°Π»Π»Π΅Π»ΡŒΠ½Ρ‹ΠΌΠΈ пластинами 1F, ΠΏΠΎΠΊΠ° ΠΎΠ±ΡΠ»ΡƒΠΆΠΈΠ²Π°ΡŽΡ‰ΠΈΠΉ пСрсонал Π½Π΅ устанСт ΠΎΡ‚ ΡˆΡƒΡ‚ΠΎΠΊ.

ΠŸΡ€ΠΎΠ²Π΅Ρ€ΡŒΡ‚Π΅ своС ΠΏΠΎΠ½ΠΈΠΌΠ°Π½ΠΈΠ΅ Π•ΠΌΠΊΠΎΡΡ‚ΡŒ кондСнсатора с ΠΏΠ°Ρ€Π°Π»Π»Π΅Π»ΡŒΠ½Ρ‹ΠΌΠΈ пластинами составляСт 2,0 ΠΏΠ€. Если ΠΏΠ»ΠΎΡ‰Π°Π΄ΡŒ ΠΊΠ°ΠΆΠ΄ΠΎΠΉ пластины Ρ€Π°Π²Π½Π°, ΠΊΠ°ΠΊΠΎΠ²ΠΎ расстояниС ΠΌΠ΅ΠΆΠ΄Ρƒ пластинами?

ΠŸΡ€ΠΎΠ²Π΅Ρ€ΡŒΡ‚Π΅ своС ΠΏΠΎΠ½ΠΈΠΌΠ°Π½ΠΈΠ΅ Π£Π±Π΅Π΄ΠΈΡ‚Π΅ΡΡŒ, Ρ‡Ρ‚ΠΎ Ρƒ вас ΠΎΠ΄ΠΈΠ½Π°ΠΊΠΎΠ²Ρ‹Π΅ физичСскиС Π΅Π΄ΠΈΠ½ΠΈΡ†Ρ‹.

ЦилиндричСский кондСнсатор

ЦилиндричСский кондСнсатор состоит ΠΈΠ· Π΄Π²ΡƒΡ… концСнтричСских проводящих Ρ†ΠΈΠ»ΠΈΠ½Π΄Ρ€ΠΎΠ² ((Рисунок)). Π’Π½ΡƒΡ‚Ρ€Π΅Π½Π½ΠΈΠΉ Ρ†ΠΈΠ»ΠΈΠ½Π΄Ρ€ радиуса ΠΌΠΎΠΆΠ΅Ρ‚ Π±Ρ‹Ρ‚ΡŒ Π»ΠΈΠ±ΠΎ ΠΎΠ±ΠΎΠ»ΠΎΡ‡ΠΊΠΎΠΉ, Π»ΠΈΠ±ΠΎ ΠΏΠΎΠ»Π½ΠΎΡΡ‚ΡŒΡŽ Ρ‚Π²Π΅Ρ€Π΄Ρ‹ΠΌ.Π’Π½Π΅ΡˆΠ½ΠΈΠΉ Ρ†ΠΈΠ»ΠΈΠ½Π΄Ρ€ прСдставляСт собой ΠΎΠ±ΠΎΠ»ΠΎΡ‡ΠΊΡƒ Π²Π½ΡƒΡ‚Ρ€Π΅Π½Π½Π΅Π³ΠΎ радиуса. ΠœΡ‹ ΠΏΡ€Π΅Π΄ΠΏΠΎΠ»Π°Π³Π°Π΅ΠΌ, Ρ‡Ρ‚ΠΎ Π΄Π»ΠΈΠ½Π° ΠΊΠ°ΠΆΠ΄ΠΎΠ³ΠΎ Ρ†ΠΈΠ»ΠΈΠ½Π΄Ρ€Π° составляСт l ΠΈ Ρ‡Ρ‚ΠΎ ΠΈΠ·Π±Ρ‹Ρ‚ΠΎΡ‡Π½Ρ‹Π΅ заряды ΠΈ находятся Π½Π° Π²Π½ΡƒΡ‚Ρ€Π΅Π½Π½Π΅ΠΌ ΠΈ внСшнСм Ρ†ΠΈΠ»ΠΈΠ½Π΄Ρ€Π°Ρ… соотвСтствСнно.

ЦилиндричСский кондСнсатор состоит ΠΈΠ· Π΄Π²ΡƒΡ… концСнтричСских проводящих Ρ†ΠΈΠ»ΠΈΠ½Π΄Ρ€ΠΎΠ². Π—Π΄Π΅ΡΡŒ заряд Π½Π° внСшнСй повСрхности Π²Π½ΡƒΡ‚Ρ€Π΅Π½Π½Π΅Π³ΠΎ Ρ†ΠΈΠ»ΠΈΠ½Π΄Ρ€Π° ΠΏΠΎΠ»ΠΎΠΆΠΈΡ‚Π΅Π»ΡŒΠ½Ρ‹ΠΉ (ΠΎΠ±ΠΎΠ·Π½Π°Ρ‡Π΅Π½ Π·Π½Π°Ρ‡ΠΊΠΎΠΌ), Π° заряд Π½Π° Π²Π½ΡƒΡ‚Ρ€Π΅Π½Π½Π΅ΠΉ повСрхности внСшнСго Ρ†ΠΈΠ»ΠΈΠ½Π΄Ρ€Π° ΠΎΡ‚Ρ€ΠΈΡ†Π°Ρ‚Π΅Π»ΡŒΠ½Ρ‹ΠΉ (ΠΎΠ±ΠΎΠ·Π½Π°Ρ‡Π΅Π½ Π·Π½Π°Ρ‡ΠΊΠΎΠΌ).

Π‘Π΅Π· ΡƒΡ‡Π΅Ρ‚Π° ΠΊΡ€Π°Π΅Π²Ρ‹Ρ… эффСктов элСктричСскоС ΠΏΠΎΠ»Π΅ ΠΌΠ΅ΠΆΠ΄Ρƒ ΠΏΡ€ΠΎΠ²ΠΎΠ΄Π½ΠΈΠΊΠ°ΠΌΠΈ Π½Π°ΠΏΡ€Π°Π²Π»Π΅Π½ΠΎ Ρ€Π°Π΄ΠΈΠ°Π»ΡŒΠ½ΠΎ Π½Π°Ρ€ΡƒΠΆΡƒ ΠΎΡ‚ ΠΎΠ±Ρ‰Π΅ΠΉ оси Ρ†ΠΈΠ»ΠΈΠ½Π΄Ρ€ΠΎΠ².Π˜ΡΠΏΠΎΠ»ΡŒΠ·ΡƒΡ гауссову ΠΏΠΎΠ²Π΅Ρ€Ρ…Π½ΠΎΡΡ‚ΡŒ, ΠΏΠΎΠΊΠ°Π·Π°Π½Π½ΡƒΡŽ Π½Π° (Рисунок), ΠΌΡ‹ ΠΈΠΌΠ΅Π΅ΠΌ

Π‘Π»Π΅Π΄ΠΎΠ²Π°Ρ‚Π΅Π»ΡŒΠ½ΠΎ, элСктричСскоС ΠΏΠΎΠ»Π΅ ΠΌΠ΅ΠΆΠ΄Ρƒ Ρ†ΠΈΠ»ΠΈΠ½Π΄Ρ€Π°ΠΌΠΈ Ρ€Π°Π²Π½ΠΎ

Π—Π΄Π΅ΡΡŒ — Π΅Π΄ΠΈΠ½ΠΈΡ‡Π½Ρ‹ΠΉ Ρ€Π°Π΄ΠΈΠ°Π»ΡŒΠ½Ρ‹ΠΉ Π²Π΅ΠΊΡ‚ΠΎΡ€ ΠΏΠΎ радиусу Ρ†ΠΈΠ»ΠΈΠ½Π΄Ρ€Π°. МоТно ΠΏΠΎΠ΄ΡΡ‚Π°Π²ΠΈΡ‚ΡŒ Π² (рисунок) ΠΈ Π½Π°ΠΉΡ‚ΠΈ Ρ€Π°Π·Π½ΠΎΡΡ‚ΡŒ ΠΏΠΎΡ‚Π΅Π½Ρ†ΠΈΠ°Π»ΠΎΠ² ΠΌΠ΅ΠΆΠ΄Ρƒ Ρ†ΠΈΠ»ΠΈΠ½Π΄Ρ€Π°ΠΌΠΈ:

Π’Π°ΠΊΠΈΠΌ ΠΎΠ±Ρ€Π°Π·ΠΎΠΌ, Π΅ΠΌΠΊΠΎΡΡ‚ΡŒ цилиндричСского кондСнсатора

Как ΠΈ Π² Π΄Ρ€ΡƒΠ³ΠΈΡ… случаях, эта Π΅ΠΌΠΊΠΎΡΡ‚ΡŒ зависит Ρ‚ΠΎΠ»ΡŒΠΊΠΎ ΠΎΡ‚ Π³Π΅ΠΎΠΌΠ΅Ρ‚Ρ€ΠΈΠΈ располоТСния ΠΏΡ€ΠΎΠ²ΠΎΠ΄Π½ΠΈΠΊΠΎΠ². Π’Π°ΠΆΠ½Ρ‹ΠΌ ΠΏΡ€ΠΈΠΌΠ΅Π½Π΅Π½ΠΈΠ΅ΠΌ (рисунок) являСтся ΠΎΠΏΡ€Π΅Π΄Π΅Π»Π΅Π½ΠΈΠ΅ Смкости Π½Π° Π΅Π΄ΠΈΠ½ΠΈΡ†Ρƒ Π΄Π»ΠΈΠ½Ρ‹ коаксиального кабСля , ΠΊΠΎΡ‚ΠΎΡ€Ρ‹ΠΉ ΠΎΠ±Ρ‹Ρ‡Π½ΠΎ ΠΈΡΠΏΠΎΠ»ΡŒΠ·ΡƒΠ΅Ρ‚ΡΡ для ΠΏΠ΅Ρ€Π΅Π΄Π°Ρ‡ΠΈ ΠΈΠ·ΠΌΠ΅Π½ΡΡŽΡ‰ΠΈΡ…ΡΡ Π²ΠΎ Π²Ρ€Π΅ΠΌΠ΅Π½ΠΈ элСктричСских сигналов. ΠšΠΎΠ°ΠΊΡΠΈΠ°Π»ΡŒΠ½Ρ‹ΠΉ кабСль состоит ΠΈΠ· Π΄Π²ΡƒΡ… концСнтричСских цилиндричСских ΠΏΡ€ΠΎΠ²ΠΎΠ΄Π½ΠΈΠΊΠΎΠ², Ρ€Π°Π·Π΄Π΅Π»Π΅Π½Π½Ρ‹Ρ… изоляционным ΠΌΠ°Ρ‚Π΅Ρ€ΠΈΠ°Π»ΠΎΠΌ. (Π—Π΄Π΅ΡΡŒ ΠΌΡ‹ ΠΏΡ€Π΅Π΄ΠΏΠΎΠ»Π°Π³Π°Π΅ΠΌ Π½Π°Π»ΠΈΡ‡ΠΈΠ΅ Π²Π°ΠΊΡƒΡƒΠΌΠ° ΠΌΠ΅ΠΆΠ΄Ρƒ ΠΏΡ€ΠΎΠ²ΠΎΠ΄Π½ΠΈΠΊΠ°ΠΌΠΈ, Π½ΠΎ Ρ„ΠΈΠ·ΠΈΠΊΠ° практичСски такая ΠΆΠ΅, ΠΊΠΎΠ³Π΄Π° пространство ΠΌΠ΅ΠΆΠ΄Ρƒ ΠΏΡ€ΠΎΠ²ΠΎΠ΄Π½ΠΈΠΊΠ°ΠΌΠΈ Π·Π°ΠΏΠΎΠ»Π½Π΅Π½ΠΎ диэлСктриком.) Π­Ρ‚Π° конфигурация Π·Π°Ρ‰ΠΈΡ‰Π°Π΅Ρ‚ элСктричСский сигнал, Ρ€Π°ΡΠΏΡ€ΠΎΡΡ‚Ρ€Π°Π½ΡΡŽΡ‰ΠΈΠΉΡΡ ΠΏΠΎ Π²Π½ΡƒΡ‚Ρ€Π΅Π½Π½Π΅ΠΌΡƒ ΠΏΡ€ΠΎΠ²ΠΎΠ΄Π½ΠΈΠΊΡƒ, ΠΎΡ‚ ΠΏΠ°Ρ€Π°Π·ΠΈΡ‚Π½Ρ‹Ρ… элСктричСских ΠΏΠΎΠ»Π΅ΠΉ, Π²Π½Π΅ΡˆΠ½ΠΈΡ… ΠΏΠΎ ΠΎΡ‚Π½ΠΎΡˆΠ΅Π½ΠΈΡŽ ΠΊ ΠΏΡ€ΠΎΠ²ΠΎΠ΄Π½ΠΈΠΊΡƒ. кабСль. Π’ΠΎΠΊ Ρ‚Π΅Ρ‡Π΅Ρ‚ Π² ΠΏΡ€ΠΎΡ‚ΠΈΠ²ΠΎΠΏΠΎΠ»ΠΎΠΆΠ½Ρ‹Ρ… направлСниях Π²ΠΎ Π²Π½ΡƒΡ‚Ρ€Π΅Π½Π½Π΅ΠΌ ΠΈ внСшнСм ΠΏΡ€ΠΎΠ²ΠΎΠ΄Π½ΠΈΠΊΠ°Ρ…, ΠΏΡ€ΠΈ этом внСшний ΠΏΡ€ΠΎΠ²ΠΎΠ΄Π½ΠΈΠΊ ΠΎΠ±Ρ‹Ρ‡Π½ΠΎ Π·Π°Π·Π΅ΠΌΠ»Π΅Π½.Π’Π΅ΠΏΠ΅Ρ€ΡŒ, ΠΊΠ°ΠΊ ΠΏΠΎΠΊΠ°Π·Π°Π½ΠΎ Π½Π° рисункС, Π΅ΠΌΠΊΠΎΡΡ‚ΡŒ коаксиального кабСля Π½Π° Π΅Π΄ΠΈΠ½ΠΈΡ†Ρƒ Π΄Π»ΠΈΠ½Ρ‹ Ρ€Π°Π²Π½Π°

.

Π’ практичСских прилоТСниях Π²Π°ΠΆΠ½ΠΎ Π²Ρ‹Π±ΠΈΡ€Π°Ρ‚ΡŒ ΠΊΠΎΠ½ΠΊΡ€Π΅Ρ‚Π½Ρ‹Π΅ значСния C / l . Π­Ρ‚ΠΎ ΠΌΠΎΠΆΠ΅Ρ‚ Π±Ρ‹Ρ‚ΡŒ достигнуто Π·Π° счСт ΡΠΎΠΎΡ‚Π²Π΅Ρ‚ΡΡ‚Π²ΡƒΡŽΡ‰Π΅Π³ΠΎ Π²Ρ‹Π±ΠΎΡ€Π° радиусов ΠΏΡ€ΠΎΠ²ΠΎΠ΄Π½ΠΈΠΊΠΎΠ² ΠΈ изоляционного ΠΌΠ°Ρ‚Π΅Ρ€ΠΈΠ°Π»Π° ΠΌΠ΅ΠΆΠ΄Ρƒ Π½ΠΈΠΌΠΈ.

ΠŸΡ€ΠΎΠ²Π΅Ρ€ΡŒΡ‚Π΅ своС ΠΏΠΎΠ½ΠΈΠΌΠ°Π½ΠΈΠ΅ Когда цилиндричСский кондСнсатор ΠΏΠΎΠ»ΡƒΡ‡Π°Π΅Ρ‚ заряд 0,500 нКл, ΠΌΠ΅ΠΆΠ΄Ρƒ Ρ†ΠΈΠ»ΠΈΠ½Π΄Ρ€Π°ΠΌΠΈ измСряСтся Ρ€Π°Π·Π½ΠΎΡΡ‚ΡŒ ΠΏΠΎΡ‚Π΅Π½Ρ†ΠΈΠ°Π»ΠΎΠ² 20,0 Π’. Π°) Какова Π΅ΠΌΠΊΠΎΡΡ‚ΡŒ этой систСмы? (b) Если Π΄Π»ΠΈΠ½Π° Ρ†ΠΈΠ»ΠΈΠ½Π΄Ρ€ΠΎΠ² составляСт 1,0 ΠΌ, ΠΊΠ°ΠΊΠΎΠ²ΠΎ ΠΎΡ‚Π½ΠΎΡˆΠ΅Π½ΠΈΠ΅ ΠΈΡ… радиусов?

НСсколько Ρ‚ΠΈΠΏΠΎΠ² практичСских кондСнсаторов ΠΏΠΎΠΊΠ°Π·Π°Π½Ρ‹ Π½Π° (Рисунок). ΠžΠ±Ρ‹Ρ‡Π½Ρ‹Π΅ кондСнсаторы часто состоят ΠΈΠ· Π΄Π²ΡƒΡ… Π½Π΅Π±ΠΎΠ»ΡŒΡˆΠΈΡ… кусочков мСталличСской Ρ„ΠΎΠ»ΡŒΠ³ΠΈ, Ρ€Π°Π·Π΄Π΅Π»Π΅Π½Π½Ρ‹Ρ… двумя нСбольшими кусочками изоляции (см. (Рисунок) (b)). ΠœΠ΅Ρ‚Π°Π»Π»ΠΈΡ‡Π΅ΡΠΊΠ°Ρ Ρ„ΠΎΠ»ΡŒΠ³Π° ΠΈ изоляция ΠΏΠΎΠΊΡ€Ρ‹Ρ‚Ρ‹ Π·Π°Ρ‰ΠΈΡ‚Π½Ρ‹ΠΌ ΠΏΠΎΠΊΡ€Ρ‹Ρ‚ΠΈΠ΅ΠΌ, Π° Π΄Π²Π° мСталличСских Π²Ρ‹Π²ΠΎΠ΄Π° ΠΈΡΠΏΠΎΠ»ΡŒΠ·ΡƒΡŽΡ‚ΡΡ для ΠΏΠΎΠ΄ΠΊΠ»ΡŽΡ‡Π΅Π½ΠΈΡ Ρ„ΠΎΠ»ΡŒΠ³ΠΈ ΠΊ внСшнСй Ρ†Π΅ΠΏΠΈ. НСкоторыС распространСнныС изоляционныС ΠΌΠ°Ρ‚Π΅Ρ€ΠΈΠ°Π»Ρ‹ — это слюда, ΠΊΠ΅Ρ€Π°ΠΌΠΈΠΊΠ°, Π±ΡƒΠΌΠ°Π³Π° ΠΈ Π°Π½Ρ‚ΠΈΠΏΡ€ΠΈΠ³Π°Ρ€Π½ΠΎΠ΅ ΠΏΠΎΠΊΡ€Ρ‹Ρ‚ΠΈΠ΅ Teflon β„’.

Π”Ρ€ΡƒΠ³ΠΎΠΉ популярный Ρ‚ΠΈΠΏ кондСнсатора — элСктролитичСский кондСнсатор. Он состоит ΠΈΠ· окислСнного ΠΌΠ΅Ρ‚Π°Π»Π»Π° Π² проводящСй пастС. ΠžΡΠ½ΠΎΠ²Π½Ρ‹ΠΌ прСимущСством элСктролитичСского кондСнсатора являСтся Π΅Π³ΠΎ высокая Π΅ΠΌΠΊΠΎΡΡ‚ΡŒ ΠΏΠΎ ΡΡ€Π°Π²Π½Π΅Π½ΠΈΡŽ с Π΄Ρ€ΡƒΠ³ΠΈΠΌΠΈ распространСнными Ρ‚ΠΈΠΏΠ°ΠΌΠΈ кондСнсаторов. НапримСр, Π΅ΠΌΠΊΠΎΡΡ‚ΡŒ ΠΎΠ΄Π½ΠΎΠ³ΠΎ Ρ‚ΠΈΠΏΠ° алюминиСвого элСктролитичСского кондСнсатора ΠΌΠΎΠΆΠ΅Ρ‚ Π΄ΠΎΡΡ‚ΠΈΠ³Π°Ρ‚ΡŒ 1,0 F. Однако Π²Ρ‹ Π΄ΠΎΠ»ΠΆΠ½Ρ‹ Π±Ρ‹Ρ‚ΡŒ остороТны ΠΏΡ€ΠΈ использовании элСктролитичСского кондСнсатора Π² Ρ†Π΅ΠΏΠΈ, ΠΏΠΎΡ‚ΠΎΠΌΡƒ Ρ‡Ρ‚ΠΎ ΠΎΠ½ Ρ€Π°Π±ΠΎΡ‚Π°Π΅Ρ‚ ΠΏΡ€Π°Π²ΠΈΠ»ΡŒΠ½ΠΎ Ρ‚ΠΎΠ»ΡŒΠΊΠΎ Ρ‚ΠΎΠ³Π΄Π°, ΠΊΠΎΠ³Π΄Π° мСталличСская Ρ„ΠΎΠ»ΡŒΠ³Π° находится ΠΏΠΎΠ΄ Π±ΠΎΠ»Π΅Π΅ высоким ΠΏΠΎΡ‚Π΅Π½Ρ†ΠΈΠ°Π»ΠΎΠΌ, Ρ‡Π΅ΠΌ проводящая паста. Когда Π²ΠΎΠ·Π½ΠΈΠΊΠ°Π΅Ρ‚ обратная поляризация, элСктролитичСскоС дСйствиС Ρ€Π°Π·Ρ€ΡƒΡˆΠ°Π΅Ρ‚ ΠΎΠΊΡΠΈΠ΄Π½ΡƒΡŽ ΠΏΠ»Π΅Π½ΠΊΡƒ. Π­Ρ‚ΠΎΡ‚ Ρ‚ΠΈΠΏ кондСнсатора Π½Π΅ ΠΌΠΎΠΆΠ΅Ρ‚ Π±Ρ‹Ρ‚ΡŒ ΠΏΠΎΠ΄ΠΊΠ»ΡŽΡ‡Π΅Π½ ΠΊ источнику ΠΏΠ΅Ρ€Π΅ΠΌΠ΅Π½Π½ΠΎΠ³ΠΎ Ρ‚ΠΎΠΊΠ°, ΠΏΠΎΡ‚ΠΎΠΌΡƒ Ρ‡Ρ‚ΠΎ Π² ΠΏΠΎΠ»ΠΎΠ²ΠΈΠ½Π΅ случаСв ΠΏΠ΅Ρ€Π΅ΠΌΠ΅Π½Π½ΠΎΠ΅ напряТСниС Π±ΡƒΠ΄Π΅Ρ‚ ΠΈΠΌΠ΅Ρ‚ΡŒ Π½Π΅ΠΏΡ€Π°Π²ΠΈΠ»ΡŒΠ½ΡƒΡŽ ΠΏΠΎΠ»ΡΡ€Π½ΠΎΡΡ‚ΡŒ, ΠΏΠΎΡΠΊΠΎΠ»ΡŒΠΊΡƒ ΠΏΠ΅Ρ€Π΅ΠΌΠ΅Π½Π½Ρ‹ΠΉ Ρ‚ΠΎΠΊ мСняСт свою ΠΏΠΎΠ»ΡΡ€Π½ΠΎΡΡ‚ΡŒ (см. Π‘Ρ…Π΅ΠΌΡ‹ ΠΏΠ΅Ρ€Π΅ΠΌΠ΅Π½Π½ΠΎΠ³ΠΎ Ρ‚ΠΎΠΊΠ° Π² цСпях ΠΏΠ΅Ρ€Π΅ΠΌΠ΅Π½Π½ΠΎΠ³ΠΎ Ρ‚ΠΎΠΊΠ°).

Π’ΠΎΠ·Π΄ΡƒΡˆΠ½Ρ‹ΠΉ кондСнсатор ΠΏΠ΅Ρ€Π΅ΠΌΠ΅Π½Π½ΠΎΠΉ Смкости ((Рисунок)) ΠΈΠΌΠ΅Π΅Ρ‚ Π΄Π²Π° Π½Π°Π±ΠΎΡ€Π° ΠΏΠ°Ρ€Π°Π»Π»Π΅Π»ΡŒΠ½Ρ‹Ρ… пластин. Один Π½Π°Π±ΠΎΡ€ пластин Π·Π°ΠΊΡ€Π΅ΠΏΠ»Π΅Π½ (ΠΎΠ±ΠΎΠ·Π½Π°Ρ‡Π΅Π½ ΠΊΠ°ΠΊ «статор»), Π° Π΄Ρ€ΡƒΠ³ΠΎΠΉ Π½Π°Π±ΠΎΡ€ пластин ΠΏΡ€ΠΈΠΊΡ€Π΅ΠΏΠ»Π΅Π½ ΠΊ Π²Π°Π»Ρƒ, ΠΊΠΎΡ‚ΠΎΡ€Ρ‹ΠΉ ΠΌΠΎΠΆΠ΅Ρ‚ Π²Ρ€Π°Ρ‰Π°Ρ‚ΡŒΡΡ (обозначаСтся ΠΊΠ°ΠΊ Β«Ρ€ΠΎΡ‚ΠΎΡ€Β»).ΠŸΠΎΠ²ΠΎΡ€Π°Ρ‡ΠΈΠ²Π°Ρ Π²Π°Π», ΠΌΠΎΠΆΠ½ΠΎ ΠΈΠ·ΠΌΠ΅Π½ΡΡ‚ΡŒ ΠΏΠ»ΠΎΡ‰Π°Π΄ΡŒ ΠΏΠΎΠΏΠ΅Ρ€Π΅Ρ‡Π½ΠΎΠ³ΠΎ сСчСния Π² ΠΏΠ΅Ρ€Π΅ΠΊΡ€Ρ‹Ρ‚ΠΈΠΈ пластин; ΡΠ»Π΅Π΄ΠΎΠ²Π°Ρ‚Π΅Π»ΡŒΠ½ΠΎ, Π΅ΠΌΠΊΠΎΡΡ‚ΡŒ этой систСмы ΠΌΠΎΠΆΠ΅Ρ‚ Π±Ρ‹Ρ‚ΡŒ настроСна Π½Π° ΠΆΠ΅Π»Π°Π΅ΠΌΠΎΠ΅ Π·Π½Π°Ρ‡Π΅Π½ΠΈΠ΅. Настройка кондСнсатора Π½Π°Ρ…ΠΎΠ΄ΠΈΡ‚ ΠΏΡ€ΠΈΠΌΠ΅Π½Π΅Π½ΠΈΠ΅ Π² любом Ρ‚ΠΈΠΏΠ΅ Ρ€Π°Π΄ΠΈΠΎΠΏΠ΅Ρ€Π΅Π΄Π°Ρ‡ΠΈ ΠΈ ΠΏΡ€ΠΈ ΠΏΡ€ΠΈΠ΅ΠΌΠ΅ радиосигналов ΠΎΡ‚ элСктронных устройств. ΠšΠ°ΠΆΠ΄Ρ‹ΠΉ Ρ€Π°Π·, ΠΊΠΎΠ³Π΄Π° Π²Ρ‹ настраиваСтС Π°Π²Ρ‚ΠΎΠΌΠΎΠ±ΠΈΠ»ΡŒΠ½ΠΎΠ΅ Ρ€Π°Π΄ΠΈΠΎ Π½Π° Π»ΡŽΠ±ΠΈΠΌΡƒΡŽ Ρ€Π°Π΄ΠΈΠΎΡΡ‚Π°Π½Ρ†ΠΈΡŽ, Π΄ΡƒΠΌΠ°ΠΉΡ‚Π΅ ΠΎΠ± Смкости.

Π’ ΠΏΠ΅Ρ€Π΅ΠΌΠ΅Π½Π½ΠΎΠΌ Π²ΠΎΠ·Π΄ΡƒΡˆΠ½ΠΎΠΌ кондСнсаторС Π΅ΠΌΠΊΠΎΡΡ‚ΡŒ ΠΌΠΎΠΆΠ½ΠΎ Π½Π°ΡΡ‚Ρ€Π°ΠΈΠ²Π°Ρ‚ΡŒ, измСняя ΡΡ„Ρ„Π΅ΠΊΡ‚ΠΈΠ²Π½ΡƒΡŽ ΠΏΠ»ΠΎΡ‰Π°Π΄ΡŒ пластин. (ΠΊΡ€Π΅Π΄ΠΈΡ‚: модификация Ρ€Π°Π±ΠΎΡ‚Ρ‹ Π ΠΎΠ±Π±ΠΈ Π‘ΠΏΡ€ΠΎΡƒΠ»)

ΠžΠ±ΠΎΠ·Π½Π°Ρ‡Π΅Π½ΠΈΡ, ΠΏΠΎΠΊΠ°Π·Π°Π½Π½Ρ‹Π΅ Π½Π° (Рисунок), ΠΏΡ€Π΅Π΄ΡΡ‚Π°Π²Π»ΡΡŽΡ‚ собой схСмы Ρ€Π°Π·Π»ΠΈΡ‡Π½Ρ‹Ρ… Ρ‚ΠΈΠΏΠΎΠ² кондСнсаторов.ΠžΠ±Ρ‹Ρ‡Π½ΠΎ ΠΌΡ‹ ΠΈΡΠΏΠΎΠ»ΡŒΠ·ΡƒΠ΅ΠΌ символ, ΠΏΠΎΠΊΠ°Π·Π°Π½Π½Ρ‹ΠΉ Π½Π° (Рисунок) (Π°). Π‘ΠΈΠΌΠ²ΠΎΠ» Π½Π° (Рисунок) (c) прСдставляСт кондСнсатор ΠΏΠ΅Ρ€Π΅ΠΌΠ΅Π½Π½ΠΎΠΉ Смкости. ΠžΠ±Ρ€Π°Ρ‚ΠΈΡ‚Π΅ Π²Π½ΠΈΠΌΠ°Π½ΠΈΠ΅ Π½Π° сходство этих символов с симмСтриСй кондСнсатора с ΠΏΠ°Ρ€Π°Π»Π»Π΅Π»ΡŒΠ½Ρ‹ΠΌΠΈ пластинами. ЭлСктролитичСский кондСнсатор частично прСдставлСн символом (Рисунок) (b), Π³Π΄Π΅ изогнутая пластина ΠΎΠ±ΠΎΠ·Π½Π°Ρ‡Π°Π΅Ρ‚ ΠΎΡ‚Ρ€ΠΈΡ†Π°Ρ‚Π΅Π»ΡŒΠ½Ρ‹ΠΉ Π²Ρ‹Π²ΠΎΠ΄.

Π—Π΄Π΅ΡΡŒ ΠΏΠΎΠΊΠ°Π·Π°Π½Ρ‹ Ρ‚Ρ€ΠΈ Ρ€Π°Π·Π»ΠΈΡ‡Π½Ρ‹Ρ… схСмных прСдставлСния кондСнсаторов. Π‘ΠΈΠΌΠ²ΠΎΠ» Π² (Π°) являСтся Π½Π°ΠΈΠ±ΠΎΠ»Π΅Π΅ часто ΠΈΡΠΏΠΎΠ»ΡŒΠ·ΡƒΠ΅ΠΌΡ‹ΠΌ. Π‘ΠΈΠΌΠ²ΠΎΠ» Π² (b) прСдставляСт собой элСктролитичСский кондСнсатор.Π‘ΠΈΠΌΠ²ΠΎΠ» Π² (c) прСдставляСт кондСнсатор ΠΏΠ΅Ρ€Π΅ΠΌΠ΅Π½Π½ΠΎΠΉ Смкости.

Π˜Π½Ρ‚Π΅Ρ€Π΅ΡΠ½Ρ‹ΠΉ ΠΏΡ€ΠΈΠΊΠ»Π°Π΄Π½ΠΎΠΉ ΠΏΡ€ΠΈΠΌΠ΅Ρ€ ΠΌΠΎΠ΄Π΅Π»ΠΈ кондСнсатора взят ΠΈΠ· ΠΊΠ»Π΅Ρ‚ΠΎΡ‡Π½ΠΎΠΉ Π±ΠΈΠΎΠ»ΠΎΠ³ΠΈΠΈ ΠΈ ΠΈΠΌΠ΅Π΅Ρ‚ Π΄Π΅Π»ΠΎ с элСктричСским ΠΏΠΎΡ‚Π΅Π½Ρ†ΠΈΠ°Π»ΠΎΠΌ Π² плазматичСской ΠΌΠ΅ΠΌΠ±Ρ€Π°Π½Π΅ ΠΆΠΈΠ²ΠΎΠΉ ΠΊΠ»Π΅Ρ‚ΠΊΠΈ ((рисунок)). ΠšΠ»Π΅Ρ‚ΠΎΡ‡Π½Ρ‹Π΅ ΠΌΠ΅ΠΌΠ±Ρ€Π°Π½Ρ‹ ΠΎΡ‚Π΄Π΅Π»ΡΡŽΡ‚ ΠΊΠ»Π΅Ρ‚ΠΊΠΈ ΠΎΡ‚ ΠΈΡ… окруТСния, Π½ΠΎ ΠΏΠΎΠ·Π²ΠΎΠ»ΡΡŽΡ‚ Π½Π΅ΠΊΠΎΡ‚ΠΎΡ€Ρ‹ΠΌ ΠΎΡ‚ΠΎΠ±Ρ€Π°Π½Π½Ρ‹ΠΌ ΠΈΠΎΠ½Π°ΠΌ ΠΏΡ€ΠΎΡ…ΠΎΠ΄ΠΈΡ‚ΡŒ Π²Π½ΡƒΡ‚Ρ€ΡŒ ΠΈΠ»ΠΈ ΠΈΠ· ΠΊΠ»Π΅Ρ‚ΠΊΠΈ. Π Π°Π·Π½ΠΎΡΡ‚ΡŒ ΠΏΠΎΡ‚Π΅Π½Ρ†ΠΈΠ°Π»ΠΎΠ² Π½Π° ΠΌΠ΅ΠΌΠ±Ρ€Π°Π½Π΅ составляСт ΠΎΠΊΠΎΠ»ΠΎ 70 ΠΌΠ’. ΠšΠ»Π΅Ρ‚ΠΎΡ‡Π½Π°Ρ ΠΌΠ΅ΠΌΠ±Ρ€Π°Π½Π° ΠΌΠΎΠΆΠ΅Ρ‚ ΠΈΠΌΠ΅Ρ‚ΡŒ Ρ‚ΠΎΠ»Ρ‰ΠΈΠ½Ρƒ ΠΎΡ‚ 7 Π΄ΠΎ 10 Π½ΠΌ. Рассматривая ΠΊΠ»Π΅Ρ‚ΠΎΡ‡Π½ΡƒΡŽ ΠΌΠ΅ΠΌΠ±Ρ€Π°Π½Ρƒ ΠΊΠ°ΠΊ кондСнсатор Π½Π°Π½ΠΎΡ€Π°Π·ΠΌΠ΅Ρ€ΠΎΠ², ΠΎΡ†Π΅Π½ΠΊΠ° наимСньшСй напряТСнности элСктричСского поля Π½Π° Π΅Π΅ «пластинах» Π΄Π°Π΅Ρ‚ Π·Π½Π°Ρ‡Π΅Π½ΠΈΠ΅.

Π­Ρ‚ΠΎΠΉ Π²Π΅Π»ΠΈΡ‡ΠΈΠ½Ρ‹ элСктричСского поля достаточно, Ρ‡Ρ‚ΠΎΠ±Ρ‹ Π²Ρ‹Π·Π²Π°Ρ‚ΡŒ ΡΠ»Π΅ΠΊΡ‚Ρ€ΠΈΡ‡Π΅ΡΠΊΡƒΡŽ искру Π² Π²ΠΎΠ·Π΄ΡƒΡ…Π΅.

ΠŸΠΎΠ»ΡƒΠΏΡ€ΠΎΠ½ΠΈΡ†Π°Π΅ΠΌΠ°Ρ ΠΌΠ΅ΠΌΠ±Ρ€Π°Π½Π° биологичСской ΠΊΠ»Π΅Ρ‚ΠΊΠΈ ΠΈΠΌΠ΅Π΅Ρ‚ Ρ€Π°Π·Π½Ρ‹Π΅ ΠΊΠΎΠ½Ρ†Π΅Π½Ρ‚Ρ€Π°Ρ†ΠΈΠΈ ΠΈΠΎΠ½ΠΎΠ² Π½Π° Π²Π½ΡƒΡ‚Ρ€Π΅Π½Π½Π΅ΠΉ повСрхности, Ρ‡Π΅ΠΌ Π½Π° внСшнСй. Диффузия ΠΏΠ΅Ρ€Π΅ΠΌΠ΅Ρ‰Π°Π΅Ρ‚ ΠΈΠΎΠ½Ρ‹ (калия) ΠΈ (Ρ…Π»ΠΎΡ€ΠΈΠ΄Π°) Π² ΠΏΠΎΠΊΠ°Π·Π°Π½Π½Ρ‹Ρ… направлСниях, ΠΏΠΎΠΊΠ° кулоновская сила Π½Π΅ остановит Π΄Π°Π»ΡŒΠ½Π΅ΠΉΡˆΡƒΡŽ ΠΏΠ΅Ρ€Π΅Π΄Π°Ρ‡Ρƒ. Π’Π°ΠΊΠΈΠΌ ΠΎΠ±Ρ€Π°Π·ΠΎΠΌ, внСшняя ΠΏΠΎΠ²Π΅Ρ€Ρ…Π½ΠΎΡΡ‚ΡŒ ΠΌΠ΅ΠΌΠ±Ρ€Π°Π½Ρ‹ ΠΏΡ€ΠΈΠΎΠ±Ρ€Π΅Ρ‚Π°Π΅Ρ‚ ΠΏΠΎΠ»ΠΎΠΆΠΈΡ‚Π΅Π»ΡŒΠ½Ρ‹ΠΉ заряд, Π° Π΅Π΅ внутрСнняя ΠΏΠΎΠ²Π΅Ρ€Ρ…Π½ΠΎΡΡ‚ΡŒ ΠΏΡ€ΠΈΠΎΠ±Ρ€Π΅Ρ‚Π°Π΅Ρ‚ ΠΎΡ‚Ρ€ΠΈΡ†Π°Ρ‚Π΅Π»ΡŒΠ½Ρ‹ΠΉ заряд, создавая Ρ€Π°Π·Π½ΠΎΡΡ‚ΡŒ ΠΏΠΎΡ‚Π΅Π½Ρ†ΠΈΠ°Π»ΠΎΠ² Π½Π° ΠΌΠ΅ΠΌΠ±Ρ€Π°Π½Π΅.ΠœΠ΅ΠΌΠ±Ρ€Π°Π½Π° ΠΎΠ±Ρ‹Ρ‡Π½ΠΎ Π½Π΅ΠΏΡ€ΠΎΠ½ΠΈΡ†Π°Π΅ΠΌΠ° для Na + (ΠΈΠΎΠ½ΠΎΠ² натрия).

Π‘Π²ΠΎΠ΄ΠΊΠ°

  • ΠšΠΎΠ½Π΄Π΅Π½ΡΠ°Ρ‚ΠΎΡ€ — это устройство, ΠΊΠΎΡ‚ΠΎΡ€ΠΎΠ΅ Π½Π°ΠΊΠ°ΠΏΠ»ΠΈΠ²Π°Π΅Ρ‚ элСктричСский заряд ΠΈ ΡΠ»Π΅ΠΊΡ‚Ρ€ΠΈΡ‡Π΅ΡΠΊΡƒΡŽ ΡΠ½Π΅Ρ€Π³ΠΈΡŽ. ΠšΠΎΠ»ΠΈΡ‡Π΅ΡΡ‚Π²ΠΎ заряда, ΠΊΠΎΡ‚ΠΎΡ€ΠΎΠ΅ ΠΌΠΎΠΆΠ΅Ρ‚ Ρ…Ρ€Π°Π½ΠΈΡ‚ΡŒ Π²Π°ΠΊΡƒΡƒΠΌΠ½Ρ‹ΠΉ кондСнсатор, зависит ΠΎΡ‚ Π΄Π²ΡƒΡ… основных Ρ„Π°ΠΊΡ‚ΠΎΡ€ΠΎΠ²: ΠΏΡ€ΠΈΠ»ΠΎΠΆΠ΅Π½Π½ΠΎΠ³ΠΎ напряТСния ΠΈ физичСских характСристик кондСнсатора, Ρ‚Π°ΠΊΠΈΡ… ΠΊΠ°ΠΊ Π΅Π³ΠΎ Ρ€Π°Π·ΠΌΠ΅Ρ€ ΠΈ гСомСтрия.
  • Π•ΠΌΠΊΠΎΡΡ‚ΡŒ кондСнсатора — это ΠΏΠ°Ρ€Π°ΠΌΠ΅Ρ‚Ρ€, ΠΊΠΎΡ‚ΠΎΡ€Ρ‹ΠΉ Π³ΠΎΠ²ΠΎΡ€ΠΈΡ‚ Π½Π°ΠΌ, сколько заряда ΠΌΠΎΠΆΠ΅Ρ‚ Ρ…Ρ€Π°Π½ΠΈΡ‚ΡŒΡΡ Π² кондСнсаторС Π½Π° Π΅Π΄ΠΈΠ½ΠΈΡ†Ρƒ разности ΠΏΠΎΡ‚Π΅Π½Ρ†ΠΈΠ°Π»ΠΎΠ² ΠΌΠ΅ΠΆΠ΄Ρƒ Π΅Π³ΠΎ пластинами.Π•ΠΌΠΊΠΎΡΡ‚ΡŒ систСмы ΠΏΡ€ΠΎΠ²ΠΎΠ΄Π½ΠΈΠΊΠΎΠ² зависит Ρ‚ΠΎΠ»ΡŒΠΊΠΎ ΠΎΡ‚ Π³Π΅ΠΎΠΌΠ΅Ρ‚Ρ€ΠΈΠΈ ΠΈΡ… располоТСния ΠΈ физичСских свойств изоляционного ΠΌΠ°Ρ‚Π΅Ρ€ΠΈΠ°Π»Π°, Π·Π°ΠΏΠΎΠ»Π½ΡΡŽΡ‰Π΅Π³ΠΎ пространство ΠΌΠ΅ΠΆΠ΄Ρƒ ΠΏΡ€ΠΎΠ²ΠΎΠ΄Π½ΠΈΠΊΠ°ΠΌΠΈ. Π•Π΄ΠΈΠ½ΠΈΡ†Π΅ΠΉ измСрСния Смкости являСтся Ρ„Π°Ρ€Π°Π΄, Π³Π΄Π΅

ΠšΠΎΠ½Ρ†Π΅ΠΏΡ‚ΡƒΠ°Π»ΡŒΠ½Ρ‹Π΅ вопросы

Зависит Π»ΠΈ Π΅ΠΌΠΊΠΎΡΡ‚ΡŒ устройства ΠΎΡ‚ ΠΏΡ€ΠΈΠ»ΠΎΠΆΠ΅Π½Π½ΠΎΠ³ΠΎ напряТСния? Зависит Π»ΠΈ Π΅ΠΌΠΊΠΎΡΡ‚ΡŒ устройства ΠΎΡ‚ заряда, находящСгося Π½Π° Π½Π΅ΠΌ?

НС ΠΌΠΎΠ³Π»ΠΈ Π±Ρ‹ Π²Ρ‹ Ρ€Π°Π·ΠΌΠ΅ΡΡ‚ΠΈΡ‚ΡŒ пластины кондСнсатора с ΠΏΠ°Ρ€Π°Π»Π»Π΅Π»ΡŒΠ½Ρ‹ΠΌΠΈ пластинами Π±Π»ΠΈΠΆΠ΅ Π΄Ρ€ΡƒΠ³ ΠΊ Π΄Ρ€ΡƒΠ³Ρƒ ΠΈΠ»ΠΈ дальшС Π΄Ρ€ΡƒΠ³ ΠΎΡ‚ Π΄Ρ€ΡƒΠ³Π°, Ρ‡Ρ‚ΠΎΠ±Ρ‹ ΡƒΠ²Π΅Π»ΠΈΡ‡ΠΈΡ‚ΡŒ ΠΈΡ… Π΅ΠΌΠΊΠΎΡΡ‚ΡŒ?

Π—Π½Π°Ρ‡Π΅Π½ΠΈΠ΅ Смкости Ρ€Π°Π²Π½ΠΎ Π½ΡƒΠ»ΡŽ, Ссли пластины Π½Π΅ заряТСны. ΠŸΡ€Π°Π²Π΄Π° ΠΈΠ»ΠΈ лоТь?

Если пластины кондСнсатора ΠΈΠΌΠ΅ΡŽΡ‚ Ρ€Π°Π·Π½Ρ‹Π΅ ΠΏΠ»ΠΎΡ‰Π°Π΄ΠΈ, ΠΏΠΎΠ»ΡƒΡ‡Π°Ρ‚ Π»ΠΈ ΠΎΠ½ΠΈ ΠΎΠ΄ΠΈΠ½Π°ΠΊΠΎΠ²Ρ‹ΠΉ заряд, ΠΊΠΎΠ³Π΄Π° кондСнсатор ΠΏΠΎΠ΄ΠΊΠ»ΡŽΡ‡Π΅Π½ ΠΊ Π±Π°Ρ‚Π°Ρ€Π΅Π΅?

Зависит Π»ΠΈ Π΅ΠΌΠΊΠΎΡΡ‚ΡŒ сфСричСского кондСнсатора ΠΎΡ‚ Ρ‚ΠΎΠ³ΠΎ, какая сфСра заряТСна ΠΏΠΎΠ»ΠΎΠΆΠΈΡ‚Π΅Π»ΡŒΠ½ΠΎ ΠΈΠ»ΠΈ ΠΎΡ‚Ρ€ΠΈΡ†Π°Ρ‚Π΅Π»ΡŒΠ½ΠΎ?

ΠŸΡ€ΠΎΠ±Π»Π΅ΠΌΡ‹

Какой заряд сохраняСтся Π² кондСнсаторС, ΠΊΠΎΠ³Π΄Π° ΠΊ Π½Π΅ΠΌΡƒ ΠΏΡ€ΠΈΠ»ΠΎΠΆΠ΅Π½ΠΎ 120,0 Π’?

НайдитС заряд, Π½Π°ΠΊΠΎΠΏΠ»Π΅Π½Π½Ρ‹ΠΉ ΠΏΡ€ΠΈ ΠΏΡ€ΠΈΠ»ΠΎΠΆΠ΅Π½ΠΈΠΈ 5,50 Π’ ΠΊ кондСнсатору 8,00 ΠΏΠ€.

РассчитайтС напряТСниС, ΠΏΡ€ΠΈΠ»ΠΎΠΆΠ΅Π½Π½ΠΎΠ΅ ΠΊ кондСнсатору, ΠΊΠΎΠ³Π΄Π° ΠΎΠ½ сохраняСт заряд.

КакоС напряТСниС Π½Π΅ΠΎΠ±Ρ…ΠΎΠ΄ΠΈΠΌΠΎ ΠΏΠΎΠ΄Π°Ρ‚ΡŒ Π½Π° кондСнсатор Π΅ΠΌΠΊΠΎΡΡ‚ΡŒΡŽ 8,00 Π½Π€, Ρ‡Ρ‚ΠΎΠ±Ρ‹ Π½Π°ΠΊΠΎΠΏΠΈΡ‚ΡŒ заряд 0,160 мкКл?

Какая Π΅ΠΌΠΊΠΎΡΡ‚ΡŒ Π½Π΅ΠΎΠ±Ρ…ΠΎΠ΄ΠΈΠΌΠ° для хранСния заряда ΠΏΡ€ΠΈ напряТСнии 120 Π’?

Какова Π΅ΠΌΠΊΠΎΡΡ‚ΡŒ Π²Ρ‹Π²ΠΎΠ΄Π° большого Π³Π΅Π½Π΅Ρ€Π°Ρ‚ΠΎΡ€Π° Π’Π°Π½-Π΄Π΅-Π“Ρ€Π°Π°Ρ„Π°, Ссли ΠΎΠ½ Ρ…Ρ€Π°Π½ΠΈΡ‚ 8,00 мКл заряда ΠΏΡ€ΠΈ напряТСнии 12,0 ΠœΠ’?

ΠŸΠ»Π°ΡΡ‚ΠΈΠ½Ρ‹ пустого плоского кондСнсатора Π΅ΠΌΠΊΠΎΡΡ‚ΡŒΡŽ 5,0 ΠΏΠ€ находятся Π½Π° расстоянии 2,0 ΠΌΠΌ Π΄Ρ€ΡƒΠ³ ΠΎΡ‚ Π΄Ρ€ΡƒΠ³Π°. Какова ΠΏΠ»ΠΎΡ‰Π°Π΄ΡŒ ΠΊΠ°ΠΆΠ΄ΠΎΠΉ пластины?

А 60.Π’Π°ΠΊΡƒΡƒΠΌΠ½Ρ‹ΠΉ кондСнсатор 0 ΠΏΠ€ ΠΈΠΌΠ΅Π΅Ρ‚ ΠΏΠ»ΠΎΡΠΊΡƒΡŽ ΠΎΠ±ΠΊΠ»Π°Π΄ΠΊΡƒ. КакоС расстояниС ΠΌΠ΅ΠΆΠ΄Ρƒ Π΅Π³ΠΎ пластинами?

Набор ΠΏΠ°Ρ€Π°Π»Π»Π΅Π»ΡŒΠ½Ρ‹Ρ… пластин ΠΈΠΌΠ΅Π΅Ρ‚ Π΅ΠΌΠΊΠΎΡΡ‚ΡŒ. Какой заряд Π½ΡƒΠΆΠ½ΠΎ Π΄ΠΎΠ±Π°Π²ΠΈΡ‚ΡŒ ΠΊ пластинам, Ρ‡Ρ‚ΠΎΠ±Ρ‹ Ρ€Π°Π·Π½ΠΎΡΡ‚ΡŒ ΠΏΠΎΡ‚Π΅Π½Ρ†ΠΈΠ°Π»ΠΎΠ² ΠΌΠ΅ΠΆΠ΄Ρƒ Π½ΠΈΠΌΠΈ ΡƒΠ²Π΅Π»ΠΈΡ‡ΠΈΠ»Π°ΡΡŒ Π½Π° 100 Π’?

Π‘Ρ‡ΠΈΡ‚Π°ΠΉΡ‚Π΅ Π—Π΅ΠΌΠ»ΡŽ сфСричСским ΠΏΡ€ΠΎΠ²ΠΎΠ΄Π½ΠΈΠΊΠΎΠΌ радиусом 6400 ΠΊΠΌ ΠΈ вычислитС Π΅Π΅ Π΅ΠΌΠΊΠΎΡΡ‚ΡŒ.

Если Π΅ΠΌΠΊΠΎΡΡ‚ΡŒ Π½Π° Π΅Π΄ΠΈΠ½ΠΈΡ†Ρƒ Π΄Π»ΠΈΠ½Ρ‹ цилиндричСского кондСнсатора составляСт 20 ΠΏΠ€ / ΠΌ, ΠΊΠ°ΠΊΠΎΠ²ΠΎ ΠΎΡ‚Π½ΠΎΡˆΠ΅Π½ΠΈΠ΅ радиусов Π΄Π²ΡƒΡ… Ρ†ΠΈΠ»ΠΈΠ½Π΄Ρ€ΠΎΠ²?

ΠŸΡƒΡΡ‚ΠΎΠΉ кондСнсатор с ΠΏΠ°Ρ€Π°Π»Π»Π΅Π»ΡŒΠ½Ρ‹ΠΌΠΈ пластинами ΠΈΠΌΠ΅Π΅Ρ‚ Π΅ΠΌΠΊΠΎΡΡ‚ΡŒ.Бколько заряда Π΄ΠΎΠ»ΠΆΠ½ΠΎ ΡΡ‚Π΅Ρ‡ΡŒ с пластин, ΠΏΡ€Π΅ΠΆΠ΄Π΅ Ρ‡Π΅ΠΌ напряТСниС Π½Π° Π½ΠΈΡ… снизится Π½Π° 100 Π’?

Глоссарий

Π΅ΠΌΠΊΠΎΡΡ‚ΡŒ
количСство заряда Π½Π° Π΅Π΄ΠΈΠ½ΠΈΡ†Ρƒ Π²ΠΎΠ»ΡŒΡ‚
кондСнсатор
Устройство для хранСния элСктричСского заряда ΠΈ элСктричСской энСргии
диэлСктрик
изоляционный ΠΌΠ°Ρ‚Π΅Ρ€ΠΈΠ°Π», ΠΈΡΠΏΠΎΠ»ΡŒΠ·ΡƒΠ΅ΠΌΡ‹ΠΉ для заполнСния пространства ΠΌΠ΅ΠΆΠ΄Ρƒ двумя пластинами
кондСнсатор с ΠΏΠ°Ρ€Π°Π»Π»Π΅Π»ΡŒΠ½Ρ‹ΠΌΠΈ пластинами
БистСма Π΄Π²ΡƒΡ… ΠΎΠ΄ΠΈΠ½Π°ΠΊΠΎΠ²Ρ‹Ρ… ΠΏΠ°Ρ€Π°Π»Π»Π΅Π»ΡŒΠ½Ρ‹Ρ… проводящих пластин, Ρ€Π°Π·Π΄Π΅Π»Π΅Π½Π½Ρ‹Ρ… расстояниСм

Как ΠΈΠ·ΠΌΠ΅Ρ€ΠΈΡ‚ΡŒ Π΅ΠΌΠΊΠΎΡΡ‚ΡŒ с ΠΏΠΎΠΌΠΎΡ‰ΡŒΡŽ Ρ†ΠΈΡ„Ρ€ΠΎΠ²ΠΎΠ³ΠΎ ΠΌΡƒΠ»ΡŒΡ‚ΠΈΠΌΠ΅Ρ‚Ρ€Π°

ΠœΡƒΠ»ΡŒΡ‚ΠΈΠΌΠ΅Ρ‚Ρ€ опрСдСляСт Π΅ΠΌΠΊΠΎΡΡ‚ΡŒ, заряТая кондСнсатор извСстным Ρ‚ΠΎΠΊΠΎΠΌ, измСряя Ρ€Π΅Π·ΡƒΠ»ΡŒΡ‚ΠΈΡ€ΡƒΡŽΡ‰Π΅Π΅ напряТСниС ΠΈ Π·Π°Ρ‚Π΅ΠΌ вычисляя Π΅ΠΌΠΊΠΎΡΡ‚ΡŒ.

ΠŸΡ€Π΅Π΄ΡƒΠΏΡ€Π΅ΠΆΠ΄Π΅Π½ΠΈΠ΅: Π₯ΠΎΡ€ΠΎΡˆΠΈΠΉ кондСнсатор сохраняСт элСктричСский заряд ΠΈ ΠΌΠΎΠΆΠ΅Ρ‚ ΠΎΡΡ‚Π°Π²Π°Ρ‚ΡŒΡΡ ΠΏΠΎΠ΄ напряТСниСм послС ΠΎΡ‚ΠΊΠ»ΡŽΡ‡Π΅Π½ΠΈΡ питания. ΠŸΠ΅Ρ€Π΅Π΄ Ρ‚Π΅ΠΌ, ΠΊΠ°ΠΊ Π΄ΠΎΡ‚Ρ€ΠΎΠ½ΡƒΡ‚ΡŒΡΡ Π΄ΠΎ Π½Π΅Π³ΠΎ ΠΈΠ»ΠΈ провСсти ΠΈΠ·ΠΌΠ΅Ρ€Π΅Π½ΠΈΠ΅, Π°) Π²Ρ‹ΠΊΠ»ΡŽΡ‡ΠΈΡ‚Π΅ всС ΠΏΠΈΡ‚Π°Π½ΠΈΠ΅, Π±) ΠΈΡΠΏΠΎΠ»ΡŒΠ·ΡƒΠΉΡ‚Π΅ ΠΌΡƒΠ»ΡŒΡ‚ΠΈΠΌΠ΅Ρ‚Ρ€, Ρ‡Ρ‚ΠΎΠ±Ρ‹ ΡƒΠ±Π΅Π΄ΠΈΡ‚ΡŒΡΡ, Ρ‡Ρ‚ΠΎ ΠΏΠΈΡ‚Π°Π½ΠΈΠ΅ ΠΎΡ‚ΠΊΠ»ΡŽΡ‡Π΅Π½ΠΎ, ΠΈ Π²) остороТно разрядитС кондСнсатор, ΠΏΠΎΠ΄ΠΊΠ»ΡŽΡ‡ΠΈΠ² рСзистор ΠΊ Π΅Π³ΠΎ ΠΏΡ€ΠΎΠ²ΠΎΠ΄Π°ΠΌ (ΠΊΠ°ΠΊ ΡƒΠΊΠ°Π·Π°Π½ΠΎ Π² ΡΠ»Π΅Π΄ΡƒΡŽΡ‰Π΅ΠΌ Π°Π±Π·Π°Ρ†Π΅). ΠžΠ±ΡΠ·Π°Ρ‚Π΅Π»ΡŒΠ½ΠΎ ΠΈΡΠΏΠΎΠ»ΡŒΠ·ΡƒΠΉΡ‚Π΅ ΡΠΎΠΎΡ‚Π²Π΅Ρ‚ΡΡ‚Π²ΡƒΡŽΡ‰ΠΈΠ΅ срСдства ΠΈΠ½Π΄ΠΈΠ²ΠΈΠ΄ΡƒΠ°Π»ΡŒΠ½ΠΎΠΉ Π·Π°Ρ‰ΠΈΡ‚Ρ‹.

Для бСзопасной разрядки кондСнсатора: ПослС ΠΎΡ‚ΠΊΠ»ΡŽΡ‡Π΅Π½ΠΈΡ питания ΠΏΠΎΠ΄ΠΊΠ»ΡŽΡ‡ΠΈΡ‚Π΅ 5-Π²Π°Ρ‚Ρ‚Π½Ρ‹ΠΉ рСзистор 20 000 Ом ΠΊ ΠΊΠ»Π΅ΠΌΠΌΠ°ΠΌ кондСнсатора Π½Π° ΠΏΡΡ‚ΡŒ сСкунд.Π˜ΡΠΏΠΎΠ»ΡŒΠ·ΡƒΠΉΡ‚Π΅ ΠΌΡƒΠ»ΡŒΡ‚ΠΈΠΌΠ΅Ρ‚Ρ€, Ρ‡Ρ‚ΠΎΠ±Ρ‹ ΡƒΠ±Π΅Π΄ΠΈΡ‚ΡŒΡΡ, Ρ‡Ρ‚ΠΎ кондСнсатор ΠΏΠΎΠ»Π½ΠΎΡΡ‚ΡŒΡŽ разряТСн.

  1. Π˜ΡΠΏΠΎΠ»ΡŒΠ·ΡƒΠΉΡ‚Π΅ Ρ†ΠΈΡ„Ρ€ΠΎΠ²ΠΎΠΉ ΠΌΡƒΠ»ΡŒΡ‚ΠΈΠΌΠ΅Ρ‚Ρ€ (DMM), Ρ‡Ρ‚ΠΎΠ±Ρ‹ ΡƒΠ±Π΅Π΄ΠΈΡ‚ΡŒΡΡ, Ρ‡Ρ‚ΠΎ всС ΠΏΠΈΡ‚Π°Π½ΠΈΠ΅ Ρ†Π΅ΠΏΠΈ ΠΎΡ‚ΠΊΠ»ΡŽΡ‡Π΅Π½ΠΎ. Если кондСнсатор ΠΈΡΠΏΠΎΠ»ΡŒΠ·ΡƒΠ΅Ρ‚ΡΡ Π² Ρ†Π΅ΠΏΠΈ ΠΏΠ΅Ρ€Π΅ΠΌΠ΅Π½Π½ΠΎΠ³ΠΎ Ρ‚ΠΎΠΊΠ°, настройтС ΠΌΡƒΠ»ΡŒΡ‚ΠΈΠΌΠ΅Ρ‚Ρ€ Π½Π° ΠΈΠ·ΠΌΠ΅Ρ€Π΅Π½ΠΈΠ΅ ΠΏΠ΅Ρ€Π΅ΠΌΠ΅Π½Π½ΠΎΠ³ΠΎ напряТСния. Если ΠΎΠ½ ΠΈΡΠΏΠΎΠ»ΡŒΠ·ΡƒΠ΅Ρ‚ΡΡ Π² Ρ†Π΅ΠΏΠΈ постоянного Ρ‚ΠΎΠΊΠ°, настройтС Ρ†ΠΈΡ„Ρ€ΠΎΠ²ΠΎΠΉ ΠΌΡƒΠ»ΡŒΡ‚ΠΈΠΌΠ΅Ρ‚Ρ€ Π½Π° ΠΈΠ·ΠΌΠ΅Ρ€Π΅Π½ΠΈΠ΅ постоянного напряТСния.
  2. ΠžΡΠΌΠΎΡ‚Ρ€ΠΈΡ‚Π΅ кондСнсатор. Если ΡƒΡ‚Π΅Ρ‡ΠΊΠΈ, Ρ‚Ρ€Π΅Ρ‰ΠΈΠ½Ρ‹, вздутия ΠΈΠ»ΠΈ Π΄Ρ€ΡƒΠ³ΠΈΠ΅ ΠΏΡ€ΠΈΠ·Π½Π°ΠΊΠΈ износа ΠΎΡ‡Π΅Π²ΠΈΠ΄Π½Ρ‹, Π·Π°ΠΌΠ΅Π½ΠΈΡ‚Π΅ кондСнсатор.
  3. ΠŸΠΎΠ²Π΅Ρ€Π½ΠΈΡ‚Π΅ Ρ†ΠΈΡ„Π΅Ρ€Π±Π»Π°Ρ‚ Π² Ρ€Π΅ΠΆΠΈΠΌ измСрСния Смкости.Π‘ΠΈΠΌΠ²ΠΎΠ» часто раздСляСт Ρ‚ΠΎΡ‡ΠΊΡƒ Π½Π° Ρ†ΠΈΡ„Π΅Ρ€Π±Π»Π°Ρ‚Π΅ с Π΄Ρ€ΡƒΠ³ΠΎΠΉ Ρ„ΡƒΠ½ΠΊΡ†ΠΈΠ΅ΠΉ. Помимо Ρ€Π΅Π³ΡƒΠ»ΠΈΡ€ΠΎΠ²ΠΊΠΈ ΡˆΠΊΠ°Π»Ρ‹, для Π°ΠΊΡ‚ΠΈΠ²Π°Ρ†ΠΈΠΈ измСрСния ΠΎΠ±Ρ‹Ρ‡Π½ΠΎ трСбуСтся Π½Π°ΠΆΠ°Ρ‚ΡŒ Ρ„ΡƒΠ½ΠΊΡ†ΠΈΠΎΠ½Π°Π»ΡŒΠ½ΡƒΡŽ ΠΊΠ½ΠΎΠΏΠΊΡƒ. Π—Π° инструкциями ΠΎΠ±Ρ€Π°Ρ‚ΠΈΡ‚Π΅ΡΡŒ ΠΊ руководству ΠΏΠΎΠ»ΡŒΠ·ΠΎΠ²Π°Ρ‚Π΅Π»Ρ ΠΌΡƒΠ»ΡŒΡ‚ΠΈΠΌΠ΅Ρ‚Ρ€Π°.
  4. 4. Для ΠΏΡ€Π°Π²ΠΈΠ»ΡŒΠ½ΠΎΠ³ΠΎ измСрСния Π½Π΅ΠΎΠ±Ρ…ΠΎΠ΄ΠΈΠΌΠΎ ΡƒΠ΄Π°Π»ΠΈΡ‚ΡŒ кондСнсатор ΠΈΠ· Ρ†Π΅ΠΏΠΈ. РазрядитС кондСнсатор, ΠΊΠ°ΠΊ описано Π² ΠΏΡ€Π΅Π΄ΡƒΠΏΡ€Π΅ΠΆΠ΄Π΅Π½ΠΈΠΈ Π²Ρ‹ΡˆΠ΅.

    ΠŸΡ€ΠΈΠΌΠ΅Ρ‡Π°Π½ΠΈΠ΅: НСкоторыС ΠΌΡƒΠ»ΡŒΡ‚ΠΈΠΌΠ΅Ρ‚Ρ€Ρ‹ ΠΏΠΎΠ΄Π΄Π΅Ρ€ΠΆΠΈΠ²Π°ΡŽΡ‚ ΠΎΡ‚Π½ΠΎΡΠΈΡ‚Π΅Π»ΡŒΠ½Ρ‹ΠΉ (REL) Ρ€Π΅ΠΆΠΈΠΌ. ΠŸΡ€ΠΈ ΠΈΠ·ΠΌΠ΅Ρ€Π΅Π½ΠΈΠΈ ΠΌΠ°Π»Ρ‹Ρ… Π·Π½Π°Ρ‡Π΅Π½ΠΈΠΉ Смкости ΠΌΠΎΠΆΠ½ΠΎ ΠΈΡΠΏΠΎΠ»ΡŒΠ·ΠΎΠ²Π°Ρ‚ΡŒ ΠΎΡ‚Π½ΠΎΡΠΈΡ‚Π΅Π»ΡŒΠ½Ρ‹ΠΉ Ρ€Π΅ΠΆΠΈΠΌ для удалСния Смкости ΠΈΠ·ΠΌΠ΅Ρ€ΠΈΡ‚Π΅Π»ΡŒΠ½Ρ‹Ρ… ΠΏΡ€ΠΎΠ²ΠΎΠ΄ΠΎΠ².Π§Ρ‚ΠΎΠ±Ρ‹ пСрСвСсти ΠΌΡƒΠ»ΡŒΡ‚ΠΈΠΌΠ΅Ρ‚Ρ€ Π² ΠΎΡ‚Π½ΠΎΡΠΈΡ‚Π΅Π»ΡŒΠ½Ρ‹ΠΉ Ρ€Π΅ΠΆΠΈΠΌ измСрСния Смкости, ΠΎΡΡ‚Π°Π²ΡŒΡ‚Π΅ ΠΈΠ·ΠΌΠ΅Ρ€ΠΈΡ‚Π΅Π»ΡŒΠ½Ρ‹Π΅ ΠΏΡ€ΠΎΠ²ΠΎΠ΄Π° ΠΎΡ‚ΠΊΡ€Ρ‹Ρ‚Ρ‹ΠΌΠΈ ΠΈ Π½Π°ΠΆΠΌΠΈΡ‚Π΅ ΠΊΠ½ΠΎΠΏΠΊΡƒ REL. Π­Ρ‚ΠΎ удаляСт Π·Π½Π°Ρ‡Π΅Π½ΠΈΠ΅ остаточной Смкости ΠΈΠ·ΠΌΠ΅Ρ€ΠΈΡ‚Π΅Π»ΡŒΠ½Ρ‹Ρ… ΠΏΡ€ΠΎΠ²ΠΎΠ΄ΠΎΠ².

  5. ΠŸΠΎΠ΄ΠΊΠ»ΡŽΡ‡ΠΈΡ‚Π΅ ΠΈΠ·ΠΌΠ΅Ρ€ΠΈΡ‚Π΅Π»ΡŒΠ½Ρ‹Π΅ ΠΏΡ€ΠΎΠ²ΠΎΠ΄Π° ΠΊ ΠΊΠ»Π΅ΠΌΠΌΠ°ΠΌ кондСнсатора. ΠžΡΡ‚Π°Π²ΡŒΡ‚Π΅ ΠΈΠ·ΠΌΠ΅Ρ€ΠΈΡ‚Π΅Π»ΡŒΠ½Ρ‹Π΅ ΠΏΡ€ΠΎΠ²ΠΎΠ΄Π° ΠΏΠΎΠ΄ΠΊΠ»ΡŽΡ‡Π΅Π½Π½Ρ‹ΠΌΠΈ Π½Π° нСсколько сСкунд, Ρ‡Ρ‚ΠΎΠ±Ρ‹ ΠΌΡƒΠ»ΡŒΡ‚ΠΈΠΌΠ΅Ρ‚Ρ€ автоматичСски Π²Ρ‹Π±Ρ€Π°Π» ΠΏΡ€Π°Π²ΠΈΠ»ΡŒΠ½Ρ‹ΠΉ Π΄ΠΈΠ°ΠΏΠ°Π·ΠΎΠ½.
  6. Π‘Ρ‡ΠΈΡ‚Π°ΠΉΡ‚Π΅ ΠΎΡ‚ΠΎΠ±Ρ€Π°ΠΆΠ°Π΅ΠΌΠΎΠ΅ ΠΈΠ·ΠΌΠ΅Ρ€Π΅Π½ΠΈΠ΅. Если Π·Π½Π°Ρ‡Π΅Π½ΠΈΠ΅ Смкости находится Π² ΠΏΡ€Π΅Π΄Π΅Π»Π°Ρ… Π΄ΠΈΠ°ΠΏΠ°Π·ΠΎΠ½Π° измСрСния, ΠΌΡƒΠ»ΡŒΡ‚ΠΈΠΌΠ΅Ρ‚Ρ€ ΠΎΡ‚ΠΎΠ±Ρ€Π°Π·ΠΈΡ‚ Π·Π½Π°Ρ‡Π΅Π½ΠΈΠ΅ кондСнсатора.Он Π±ΡƒΠ΄Π΅Ρ‚ ΠΎΡ‚ΠΎΠ±Ρ€Π°ΠΆΠ°Ρ‚ΡŒ OL, Ссли Π°) Π·Π½Π°Ρ‡Π΅Π½ΠΈΠ΅ Смкости Π²Ρ‹ΡˆΠ΅ Π΄ΠΈΠ°ΠΏΠ°Π·ΠΎΠ½Π° измСрСния ΠΈΠ»ΠΈ Π±) кондСнсатор нСисправСн.

ΠžΠ±Π·ΠΎΡ€ измСрСния Смкости

УстранСниС нСисправностСй ΠΎΠ΄Π½ΠΎΡ„Π°Π·Π½Ρ‹Ρ… Π΄Π²ΠΈΠ³Π°Ρ‚Π΅Π»Π΅ΠΉ — ΠΎΠ΄Π½ΠΎ ΠΈΠ· Π½Π°ΠΈΠ±ΠΎΠ»Π΅Π΅ практичСских ΠΏΡ€ΠΈΠΌΠ΅Π½Π΅Π½ΠΈΠΉ Ρ„ΡƒΠ½ΠΊΡ†ΠΈΠΈ Смкости Ρ†ΠΈΡ„Ρ€ΠΎΠ²ΠΎΠ³ΠΎ ΠΌΡƒΠ»ΡŒΡ‚ΠΈΠΌΠ΅Ρ‚Ρ€Π°.

ΠžΠ΄Π½ΠΎΡ„Π°Π·Π½Ρ‹ΠΉ Π΄Π²ΠΈΠ³Π°Ρ‚Π΅Π»ΡŒ с кондСнсаторным пуском, ΠΊΠΎΡ‚ΠΎΡ€Ρ‹ΠΉ Π½Π΅ запускаСтся, являСтся ΠΏΡ€ΠΈΠ·Π½Π°ΠΊΠΎΠΌ нСисправного кондСнсатора. Π’Π°ΠΊΠΈΠ΅ Π΄Π²ΠΈΠ³Π°Ρ‚Π΅Π»ΠΈ Π±ΡƒΠ΄ΡƒΡ‚ ΠΏΡ€ΠΎΠ΄ΠΎΠ»ΠΆΠ°Ρ‚ΡŒ Ρ€Π°Π±ΠΎΡ‚Π°Ρ‚ΡŒ послС запуска, Ρ‡Ρ‚ΠΎ затрудняСт поиск ΠΈ устранСниС нСисправностСй. ΠžΡ‚ΠΊΠ°Π· кондСнсатора ТСсткого пуска компрСссоров HVAC — Ρ…ΠΎΡ€ΠΎΡˆΠΈΠΉ ΠΏΡ€ΠΈΠΌΠ΅Ρ€ этой ΠΏΡ€ΠΎΠ±Π»Π΅ΠΌΡ‹.Π”Π²ΠΈΠ³Π°Ρ‚Π΅Π»ΡŒ компрСссора ΠΌΠΎΠΆΠ΅Ρ‚ Π·Π°ΠΏΡƒΡΡ‚ΠΈΡ‚ΡŒΡΡ, Π½ΠΎ вскорС пСрСгрССтся, Ρ‡Ρ‚ΠΎ ΠΏΡ€ΠΈΠ²Π΅Π΄Π΅Ρ‚ ΠΊ ΡΡ€Π°Π±Π°Ρ‚Ρ‹Π²Π°Π½ΠΈΡŽ Π²Ρ‹ΠΊΠ»ΡŽΡ‡Π°Ρ‚Π΅Π»Ρ.

ΠžΠ΄Π½ΠΎΡ„Π°Π·Π½Ρ‹Π΅ Π΄Π²ΠΈΠ³Π°Ρ‚Π΅Π»ΠΈ с Ρ‚Π°ΠΊΠΈΠΌΠΈ ΠΏΡ€ΠΎΠ±Π»Π΅ΠΌΠ°ΠΌΠΈ ΠΈ ΡˆΡƒΠΌΠ½Ρ‹Π΅ ΠΎΠ΄Π½ΠΎΡ„Π°Π·Π½Ρ‹Π΅ Π΄Π²ΠΈΠ³Π°Ρ‚Π΅Π»ΠΈ с кондСнсаторами Π½ΡƒΠΆΠ΄Π°ΡŽΡ‚ΡΡ Π² ΠΌΡƒΠ»ΡŒΡ‚ΠΈΠΌΠ΅Ρ‚Ρ€Π΅ для ΠΏΡ€ΠΎΠ²Π΅Ρ€ΠΊΠΈ ΠΏΡ€Π°Π²ΠΈΠ»ΡŒΠ½ΠΎΠ³ΠΎ функционирования кондСнсаторов. ΠŸΠΎΡ‡Ρ‚ΠΈ всС ΠΌΠΎΡ‚ΠΎΡ€Π½Ρ‹Π΅ кондСнсаторы ΠΈΠΌΠ΅ΡŽΡ‚ Π·Π½Π°Ρ‡Π΅Π½ΠΈΠ΅ Π² ΠΌΠΈΠΊΡ€ΠΎΡ„Π°Ρ€Π°Π΄Π°Ρ…, ΡƒΠΊΠ°Π·Π°Π½Π½ΠΎΠ΅ Π½Π° кондСнсаторС.

Π’Ρ€Π΅Ρ…Ρ„Π°Π·Π½Ρ‹Π΅ кондСнсаторы ΠΊΠΎΡ€Ρ€Π΅ΠΊΡ†ΠΈΠΈ коэффициСнта мощности ΠΎΠ±Ρ‹Ρ‡Π½ΠΎ Π·Π°Ρ‰ΠΈΡ‰Π΅Π½Ρ‹ ΠΏΠ»Π°Π²ΠΊΠΈΠΌΠΈ прСдохранитСлями. Если ΠΎΠ΄ΠΈΠ½ ΠΈΠ»ΠΈ нСсколько ΠΈΠ· этих кондСнсаторов Π²Ρ‹ΠΉΠ΄ΡƒΡ‚ ΠΈΠ· строя, это ΠΏΡ€ΠΈΠ²Π΅Π΄Π΅Ρ‚ ΠΊ нСэффСктивности систСмы, скорСС всСго, увСличатся счСта Π·Π° ΠΊΠΎΠΌΠΌΡƒΠ½Π°Π»ΡŒΠ½Ρ‹Π΅ услуги ΠΈ ΠΌΠΎΠ³ΡƒΡ‚ ΠΏΡ€ΠΎΠΈΠ·ΠΎΠΉΡ‚ΠΈ Π½Π΅ΠΏΡ€Π΅Π΄Π½Π°ΠΌΠ΅Ρ€Π΅Π½Π½Ρ‹Π΅ ΠΎΡ‚ΠΊΠ»ΡŽΡ‡Π΅Π½ΠΈΡ оборудования.Если ΠΏΡ€Π΅Π΄ΠΎΡ…Ρ€Π°Π½ΠΈΡ‚Π΅Π»ΡŒ кондСнсатора ΠΏΠ΅Ρ€Π΅Π³ΠΎΡ€Π΅Π», Π½Π΅ΠΎΠ±Ρ…ΠΎΠ΄ΠΈΠΌΠΎ ΠΈΠ·ΠΌΠ΅Ρ€ΠΈΡ‚ΡŒ ΠΏΡ€Π΅Π΄ΠΏΠΎΠ»Π°Π³Π°Π΅ΠΌΠΎΠ΅ Π·Π½Π°Ρ‡Π΅Π½ΠΈΠ΅ ΠΌΠΈΠΊΡ€ΠΎΡ„Π°Ρ€Π°Π΄ кондСнсатора ΠΈ ΡƒΠ±Π΅Π΄ΠΈΡ‚ΡŒΡΡ, Ρ‡Ρ‚ΠΎ ΠΎΠ½ΠΎ находится Π² ΠΏΡ€Π΅Π΄Π΅Π»Π°Ρ… Π΄ΠΈΠ°ΠΏΠ°Π·ΠΎΠ½Π°, ΡƒΠΊΠ°Π·Π°Π½Π½ΠΎΠ³ΠΎ Π½Π° кондСнсаторС.

Π‘Ρ‚ΠΎΠΈΡ‚ Π·Π½Π°Ρ‚ΡŒ ΠΎ Π½Π΅ΠΊΠΎΡ‚ΠΎΡ€Ρ‹Ρ… Π΄ΠΎΠΏΠΎΠ»Π½ΠΈΡ‚Π΅Π»ΡŒΠ½Ρ‹Ρ… Ρ„Π°ΠΊΡ‚ΠΎΡ€Π°Ρ…, связанных с Π΅ΠΌΠΊΠΎΡΡ‚ΡŒΡŽ:

  • ΠšΠΎΠ½Π΄Π΅Π½ΡΠ°Ρ‚ΠΎΡ€Ρ‹ ΠΈΠΌΠ΅ΡŽΡ‚ ΠΎΠ³Ρ€Π°Π½ΠΈΡ‡Π΅Π½Π½Ρ‹ΠΉ срок слуТбы ΠΈ часто ΡΠ²Π»ΡΡŽΡ‚ΡΡ ΠΏΡ€ΠΈΡ‡ΠΈΠ½ΠΎΠΉ нСисправности.
  • НСисправныС кондСнсаторы ΠΌΠΎΠ³ΡƒΡ‚ ΠΈΠΌΠ΅Ρ‚ΡŒ ΠΊΠΎΡ€ΠΎΡ‚ΠΊΠΎΠ΅ Π·Π°ΠΌΡ‹ΠΊΠ°Π½ΠΈΠ΅, Ρ€Π°Π·Ρ€Ρ‹Π² Ρ†Π΅ΠΏΠΈ ΠΈΠ»ΠΈ ΠΌΠΎΠ³ΡƒΡ‚ физичСски Π²Ρ‹ΠΉΡ‚ΠΈ ΠΈΠ· строя Π΄ΠΎ Ρ‚ΠΎΡ‡ΠΊΠΈ ΠΎΡ‚ΠΊΠ°Π·Π°.
  • ΠŸΡ€ΠΈ ΠΊΠΎΡ€ΠΎΡ‚ΠΊΠΎΠΌ Π·Π°ΠΌΡ‹ΠΊΠ°Π½ΠΈΠΈ кондСнсатора ΠΌΠΎΠΆΠ΅Ρ‚ ΠΏΠ΅Ρ€Π΅Π³ΠΎΡ€Π΅Ρ‚ΡŒ ΠΏΡ€Π΅Π΄ΠΎΡ…Ρ€Π°Π½ΠΈΡ‚Π΅Π»ΡŒ ΠΈΠ»ΠΈ ΠΏΠΎΠ²Ρ€Π΅Π΄ΠΈΡ‚ΡŒ Π΄Ρ€ΡƒΠ³ΠΈΠ΅ ΠΊΠΎΠΌΠΏΠΎΠ½Π΅Π½Ρ‚Ρ‹.
  • Когда кондСнсатор размыкаСтся ΠΈΠ»ΠΈ Π²Ρ‹Ρ…ΠΎΠ΄ΠΈΡ‚ ΠΈΠ· строя, Ρ†Π΅ΠΏΡŒ ΠΈΠ»ΠΈ Π΅Π΅ ΠΊΠΎΠΌΠΏΠΎΠ½Π΅Π½Ρ‚Ρ‹ ΠΌΠΎΠ³ΡƒΡ‚ Π½Π΅ Ρ€Π°Π±ΠΎΡ‚Π°Ρ‚ΡŒ.
  • Износ ΠΌΠΎΠΆΠ΅Ρ‚ Ρ‚Π°ΠΊΠΆΠ΅ ΠΈΠ·ΠΌΠ΅Π½ΠΈΡ‚ΡŒ Π·Π½Π°Ρ‡Π΅Π½ΠΈΠ΅ Смкости кондСнсатора, Ρ‡Ρ‚ΠΎ ΠΌΠΎΠΆΠ΅Ρ‚ Π²Ρ‹Π·Π²Π°Ρ‚ΡŒ ΠΏΡ€ΠΎΠ±Π»Π΅ΠΌΡ‹.

Бсылка: ΠŸΡ€ΠΈΠ½Ρ†ΠΈΠΏΡ‹ Ρ†ΠΈΡ„Ρ€ΠΎΠ²ΠΎΠ³ΠΎ ΠΌΡƒΠ»ΡŒΡ‚ΠΈΠΌΠ΅Ρ‚Ρ€Π° Π“Π»Π΅Π½Π° А. ΠœΠ°Π·ΡƒΡ€Π°, American Technical Publishers.

БвязанныС рСсурсы

ΠŸΠΎΡΠ»Π΅Π΄ΠΎΠ²Π°Ρ‚Π΅Π»ΡŒΠ½Ρ‹Π΅ ΠΈ ΠΏΠ°Ρ€Π°Π»Π»Π΅Π»ΡŒΠ½Ρ‹Π΅ кондСнсаторы: Ρ‡Ρ‚ΠΎ это Ρ‚Π°ΠΊΠΎΠ΅, Ρ„ΠΎΡ€ΠΌΡƒΠ»Π°, напряТСниС (со схСмами)

Когда Π²Ρ‹ ΠΈΠ·ΡƒΡ‡Π°Π΅Ρ‚Π΅ Ρ„ΠΈΠ·ΠΈΠΊΡƒ элСктроники ΠΈ Ρ…ΠΎΡ€ΠΎΡˆΠΎ Ρ€Π°Π·Π±ΠΈΡ€Π°Π΅Ρ‚Π΅ΡΡŒ Π² основах — Π½Π°ΠΏΡ€ΠΈΠΌΠ΅Ρ€, Π·Π½Π°Ρ‡Π΅Π½ΠΈΠ΅ ΠΊΠ»ΡŽΡ‡Π΅Π²Ρ‹Ρ… Ρ‚Π΅Ρ€ΠΌΠΈΠ½ΠΎΠ², Ρ‚Π°ΠΊΠΈΡ… ΠΊΠ°ΠΊ напряТСниС , Ρ‚ΠΎΠΊ ΠΈ сопротивлСниС , Π° Ρ‚Π°ΠΊΠΆΠ΅ Π²Π°ΠΆΠ½Ρ‹Π΅ уравнСния, Ρ‚Π°ΠΊΠΈΠ΅ ΠΊΠ°ΠΊ Π·Π°ΠΊΠΎΠ½ Ома — ΠΈΠ·ΡƒΡ‡Π΅Π½ΠΈΠ΅ Ρ‚ΠΎΠ³ΠΎ, ΠΊΠ°ΠΊ Ρ€Π°Π±ΠΎΡ‚Π°ΡŽΡ‚ Ρ€Π°Π·Π»ΠΈΡ‡Π½Ρ‹Π΅ ΠΊΠΎΠΌΠΏΠΎΠ½Π΅Π½Ρ‚Ρ‹ схСмы, являСтся ΡΠ»Π΅Π΄ΡƒΡŽΡ‰ΠΈΠΌ шагом ΠΊ овладСнию ΠΏΡ€Π΅Π΄ΠΌΠ΅Ρ‚ΠΎΠΌ.

ΠšΠΎΠ½Π΄Π΅Π½ΡΠ°Ρ‚ΠΎΡ€ — ΠΎΠ΄ΠΈΠ½ ΠΈΠ· Π½Π°ΠΈΠ±ΠΎΠ»Π΅Π΅ Π²Π°ΠΆΠ½Ρ‹Ρ… ΠΊΠΎΠΌΠΏΠΎΠ½Π΅Π½Ρ‚ΠΎΠ² для понимания, ΠΏΠΎΡ‚ΠΎΠΌΡƒ Ρ‡Ρ‚ΠΎ ΠΎΠ½ΠΈ ΡˆΠΈΡ€ΠΎΠΊΠΎ ΠΈΡΠΏΠΎΠ»ΡŒΠ·ΡƒΡŽΡ‚ΡΡ практичСски Π²ΠΎ всСх областях элСктроники. ΠžΡ‚ кондСнсаторов связи ΠΈ развязки Π΄ΠΎ кондСнсаторов, ΠΊΠΎΡ‚ΠΎΡ€Ρ‹Π΅ Π·Π°ΡΡ‚Π°Π²Π»ΡΡŽΡ‚ Ρ€Π°Π±ΠΎΡ‚Π°Ρ‚ΡŒ Π²ΡΠΏΡ‹ΡˆΠΊΡƒ ΠΊΠ°ΠΌΠ΅Ρ€Ρ‹ ΠΈΠ»ΠΈ ΠΈΠ³Ρ€Π°ΡŽΡ‚ ΠΊΠ»ΡŽΡ‡Π΅Π²ΡƒΡŽ Ρ€ΠΎΠ»ΡŒ Π² выпрямитСлях, Π½Π΅ΠΎΠ±Ρ…ΠΎΠ΄ΠΈΠΌΡ‹Ρ… для прСобразования ΠΏΠ΅Ρ€Π΅ΠΌΠ΅Π½Π½ΠΎΠ³ΠΎ Ρ‚ΠΎΠΊΠ° Π² постоянный, Ρ‚Ρ€ΡƒΠ΄Π½ΠΎ ΠΏΠ΅Ρ€Π΅ΠΎΡ†Π΅Π½ΠΈΡ‚ΡŒ ΡˆΠΈΡ€ΠΎΠΊΠΈΠΉ спСктр примСнСния кондСнсаторов. Π’ΠΎΡ‚ ΠΏΠΎΡ‡Π΅ΠΌΡƒ Ρ‚Π°ΠΊ Π²Π°ΠΆΠ½ΠΎ Π·Π½Π°Ρ‚ΡŒ, ΠΊΠ°ΠΊ Ρ€Π°ΡΡΡ‡ΠΈΡ‚Π°Ρ‚ΡŒ Π΅ΠΌΠΊΠΎΡΡ‚ΡŒ ΠΈ ΠΎΠ±Ρ‰ΡƒΡŽ Π΅ΠΌΠΊΠΎΡΡ‚ΡŒ кондСнсаторов Ρ€Π°Π·Π»ΠΈΡ‡Π½ΠΎΠΉ ΠΊΠΎΠ½Ρ„ΠΈΠ³ΡƒΡ€Π°Ρ†ΠΈΠΈ.

Π§Ρ‚ΠΎ Ρ‚Π°ΠΊΠΎΠ΅ кондСнсатор?

ΠšΠΎΠ½Π΄Π΅Π½ΡΠ°Ρ‚ΠΎΡ€ — это простой элСктричСский ΠΊΠΎΠΌΠΏΠΎΠ½Π΅Π½Ρ‚, состоящий ΠΈΠ· Π΄Π²ΡƒΡ… ΠΈΠ»ΠΈ Π±ΠΎΠ»Π΅Π΅ проводящих пластин, ΠΊΠΎΡ‚ΠΎΡ€Ρ‹Π΅ ΡƒΠ΄Π΅Ρ€ΠΆΠΈΠ²Π°ΡŽΡ‚ΡΡ ΠΏΠ°Ρ€Π°Π»Π»Π΅Π»ΡŒΠ½ΠΎ Π΄Ρ€ΡƒΠ³ Π΄Ρ€ΡƒΠ³Ρƒ ΠΈ Ρ€Π°Π·Π΄Π΅Π»Π΅Π½Ρ‹ Π²ΠΎΠ·Π΄ΡƒΡ…ΠΎΠΌ ΠΈΠ»ΠΈ ΠΈΠ·ΠΎΠ»ΠΈΡ€ΡƒΡŽΡ‰ΠΈΠΌ слоСм. Π”Π²Π΅ пластины ΠΎΠ±Π»Π°Π΄Π°ΡŽΡ‚ ΡΠΏΠΎΡΠΎΠ±Π½ΠΎΡΡ‚ΡŒΡŽ Π½Π°ΠΊΠ°ΠΏΠ»ΠΈΠ²Π°Ρ‚ΡŒ элСктричСский заряд, ΠΊΠΎΠ³Π΄Π° ΠΎΠ½ΠΈ ΠΏΠΎΠ΄ΠΊΠ»ΡŽΡ‡Π΅Π½Ρ‹ ΠΊ источнику питания: ΠΎΠ΄Π½Π° пластина Π²Ρ‹Ρ€Π°Π±Π°Ρ‚Ρ‹Π²Π°Π΅Ρ‚ ΠΏΠΎΠ»ΠΎΠΆΠΈΡ‚Π΅Π»ΡŒΠ½Ρ‹ΠΉ заряд, Π° другая — ΠΎΡ‚Ρ€ΠΈΡ†Π°Ρ‚Π΅Π»ΡŒΠ½Ρ‹ΠΉ.

По сути, кондСнсатор ΠΏΠΎΡ…ΠΎΠΆ Π½Π° Π½Π΅Π±ΠΎΠ»ΡŒΡˆΡƒΡŽ Π±Π°Ρ‚Π°Ρ€Π΅ΡŽ, ΡΠΎΠ·Π΄Π°ΡŽΡ‰ΡƒΡŽ Ρ€Π°Π·Π½ΠΎΡΡ‚ΡŒ ΠΏΠΎΡ‚Π΅Π½Ρ†ΠΈΠ°Π»ΠΎΠ² (Ρ‚.Π΅.Π΅., напряТСниС) ΠΌΠ΅ΠΆΠ΄Ρƒ двумя пластинами, Ρ€Π°Π·Π΄Π΅Π»Π΅Π½Π½Ρ‹ΠΌΠΈ ΠΈΠ·ΠΎΠ»ΠΈΡ€ΡƒΡŽΡ‰ΠΈΠΌ Ρ€Π°Π·Π΄Π΅Π»ΠΈΡ‚Π΅Π»Π΅ΠΌ, Π½Π°Π·Ρ‹Π²Π°Π΅ΠΌΡ‹ΠΌ диэлСктриком (ΠΊΠΎΡ‚ΠΎΡ€Ρ‹ΠΉ ΠΌΠΎΠΆΠ΅Ρ‚ Π±Ρ‹Ρ‚ΡŒ ΠΈΠ· ΠΌΠ½ΠΎΠ³ΠΈΡ… ΠΌΠ°Ρ‚Π΅Ρ€ΠΈΠ°Π»ΠΎΠ², Π½ΠΎ часто ΠΈΠ· ΠΊΠ΅Ρ€Π°ΠΌΠΈΠΊΠΈ, стСкла, Π²ΠΎΡ‰Π΅Π½ΠΎΠΉ Π±ΡƒΠΌΠ°Π³ΠΈ ΠΈΠ»ΠΈ ΡΠ»ΡŽΠ΄Ρ‹), Ρ‡Ρ‚ΠΎ ΠΏΡ€Π΅Π΄ΠΎΡ‚Π²Ρ€Π°Ρ‰Π°Π΅Ρ‚ ΠΏΡ€ΠΎΡ…ΠΎΠΆΠ΄Π΅Π½ΠΈΠ΅ Ρ‚ΠΎΠΊΠ° ΠΎΡ‚ ΠΎΠ΄Π½ΠΎΠΉ пластину ΠΊ Π΄Ρ€ΡƒΠ³ΠΎΠΉ, Ρ‚Π΅ΠΌ самым сохраняя Π½Π°ΠΊΠΎΠΏΠ»Π΅Π½Π½Ρ‹ΠΉ заряд.

Для Π΄Π°Π½Π½ΠΎΠ³ΠΎ кондСнсатора, Ссли ΠΎΠ½ ΠΏΠΎΠ΄ΠΊΠ»ΡŽΡ‡Π΅Π½ ΠΊ Π±Π°Ρ‚Π°Ρ€Π΅Π΅ (ΠΈΠ»ΠΈ Π΄Ρ€ΡƒΠ³ΠΎΠΌΡƒ источнику напряТСния) с напряТСниСм Π’, , ΠΎΠ½ Π±ΡƒΠ΄Π΅Ρ‚ Ρ…Ρ€Π°Π½ΠΈΡ‚ΡŒ элСктричСский заряд Q . Π­Ρ‚Π° ΡΠΏΠΎΡΠΎΠ±Π½ΠΎΡΡ‚ΡŒ Π±ΠΎΠ»Π΅Π΅ Ρ‡Π΅Ρ‚ΠΊΠΎ опрСдСляСтся Β«Π΅ΠΌΠΊΠΎΡΡ‚ΡŒΡŽΒ» кондСнсатора.

Π§Ρ‚ΠΎ Ρ‚Π°ΠΊΠΎΠ΅ Π΅ΠΌΠΊΠΎΡΡ‚ΡŒ?

ИмСя это Π² Π²ΠΈΠ΄Ρƒ, Π·Π½Π°Ρ‡Π΅Π½ΠΈΠ΅ Смкости являСтся ΠΌΠ΅Ρ€ΠΎΠΉ способности кондСнсатора Π½Π°ΠΊΠ°ΠΏΠ»ΠΈΠ²Π°Ρ‚ΡŒ ΡΠ½Π΅Ρ€Π³ΠΈΡŽ Π² Π²ΠΈΠ΄Π΅ заряда. Π’ Ρ„ΠΈΠ·ΠΈΠΊΠ΅ ΠΈ элСктроникС Π΅ΠΌΠΊΠΎΡΡ‚ΡŒ обозначаСтся символом C ΠΈ опрСдСляСтся ΠΊΠ°ΠΊ:

C = \ frac {Q} {V}

Π“Π΄Π΅ Q — это заряд, хранящийся Π² пластинах, Π° Π’ — это Ρ€Π°Π·Π½ΠΎΡΡ‚ΡŒ ΠΏΠΎΡ‚Π΅Π½Ρ†ΠΈΠ°Π»ΠΎΠ² ΠΏΠΎΠ΄ΠΊΠ»ΡŽΡ‡Π΅Π½Π½ΠΎΠ³ΠΎ ΠΊ Π½ΠΈΠΌ источника напряТСния. ΠšΠΎΡ€ΠΎΡ‡Π΅ говоря, Π΅ΠΌΠΊΠΎΡΡ‚ΡŒ — это ΠΌΠ΅Ρ€Π° ΠΎΡ‚Π½ΠΎΡˆΠ΅Π½ΠΈΡ заряда ΠΊ Π½Π°ΠΏΡ€ΡΠΆΠ΅Π½ΠΈΡŽ, ΠΈ поэтому Π΅Π΄ΠΈΠ½ΠΈΡ†Π°ΠΌΠΈ Смкости ΡΠ²Π»ΡΡŽΡ‚ΡΡ ΠΊΡƒΠ»ΠΎΠ½Ρ‹ заряда / Π²ΠΎΠ»ΡŒΡ‚ разности ΠΏΠΎΡ‚Π΅Π½Ρ†ΠΈΠ°Π»ΠΎΠ².ΠšΠΎΠ½Π΄Π΅Π½ΡΠ°Ρ‚ΠΎΡ€ с Π±ΠΎΠ»Π΅Π΅ высокой Π΅ΠΌΠΊΠΎΡΡ‚ΡŒΡŽ сохраняСт большС заряда ΠΏΡ€ΠΈ Π·Π°Π΄Π°Π½Π½ΠΎΠΌ Π·Π½Π°Ρ‡Π΅Π½ΠΈΠΈ напряТСния.

ΠŸΠΎΠ½ΡΡ‚ΠΈΠ΅ Смкости Π½Π°ΡΡ‚ΠΎΠ»ΡŒΠΊΠΎ Π²Π°ΠΆΠ½ΠΎ, Ρ‡Ρ‚ΠΎ Ρ„ΠΈΠ·ΠΈΠΊΠΈ Π΄Π°Π»ΠΈ Π΅ΠΌΡƒ ΡƒΠ½ΠΈΠΊΠ°Π»ΡŒΠ½ΡƒΡŽ Π΅Π΄ΠΈΠ½ΠΈΡ†Ρƒ, Π½Π°Π·Π²Π°Π½Π½ΡƒΡŽ Ρ„Π°Ρ€Π°Π΄ (Π² Ρ‡Π΅ΡΡ‚ΡŒ британского Ρ„ΠΈΠ·ΠΈΠΊΠ° Майкла ЀарадСя), Π³Π΄Π΅ 1 F = 1 C / V. НСмного ΠΏΠΎΡ…ΠΎΠΆΠ΅ Π½Π° ΠΊΡƒΠ»ΠΎΠ½ для заряда, Ρ„Π°Ρ€Π°Π΄ — это довольно большая Π΅ΠΌΠΊΠΎΡΡ‚ΡŒ, ΠΏΡ€ΠΈ этом Π±ΠΎΠ»ΡŒΡˆΠΈΠ½ΡΡ‚Π²ΠΎ Π·Π½Π°Ρ‡Π΅Π½ΠΈΠΉ Смкости кондСнсаторов находятся Π² Π΄ΠΈΠ°ΠΏΠ°Π·ΠΎΠ½Π΅ ΠΎΡ‚ ΠΏΠΈΠΊΠΎΡ„Π°Ρ€Π°Π΄Π° (ΠΏΠ€ = 10 βˆ’12 Π€) Π΄ΠΎ ΠΌΠΈΠΊΡ€ΠΎΡ„Π°Ρ€Π°Π΄Π° (ΠΌΠΊΠ€ = 10 βˆ’6 ). F).

ЭквивалСнтная Π΅ΠΌΠΊΠΎΡΡ‚ΡŒ ΠΏΠΎΡΠ»Π΅Π΄ΠΎΠ²Π°Ρ‚Π΅Π»ΡŒΠ½Ρ‹Ρ… кондСнсаторов

Π’ ΠΏΠΎΡΠ»Π΅Π΄ΠΎΠ²Π°Ρ‚Π΅Π»ΡŒΠ½ΠΎΠΉ схСмС всС ΠΊΠΎΠΌΠΏΠΎΠ½Π΅Π½Ρ‚Ρ‹ располоТСны Π½Π° ΠΎΠ΄Π½ΠΎΠΌ ΠΈ Ρ‚ΠΎΠΌ ΠΆΠ΅ ΠΏΡƒΡ‚ΠΈ Π²ΠΎΠΊΡ€ΡƒΠ³ ΠΊΠΎΠ½Ρ‚ΡƒΡ€Π°, ΠΈ Ρ‚Π°ΠΊΠΈΠΌ ΠΆΠ΅ ΠΎΠ±Ρ€Π°Π·ΠΎΠΌ ΠΏΠΎΡΠ»Π΅Π΄ΠΎΠ²Π°Ρ‚Π΅Π»ΡŒΠ½Ρ‹Π΅ кондСнсаторы ΠΏΠΎΠ΄ΠΊΠ»ΡŽΡ‡Π°ΡŽΡ‚ΡΡ ΠΎΠ΄ΠΈΠ½ Π·Π° Π΄Ρ€ΡƒΠ³ΠΈΠΌ ΠΏΠΎ Π΅Π΄ΠΈΠ½ΠΎΠΌΡƒ ΠΏΡƒΡ‚ΠΈ Π²ΠΎΠΊΡ€ΡƒΠ³ схСмы. ΠžΠ±Ρ‰Π°Ρ Π΅ΠΌΠΊΠΎΡΡ‚ΡŒ для ряда кондСнсаторов, Π²ΠΊΠ»ΡŽΡ‡Π΅Π½Π½Ρ‹Ρ… ΠΏΠΎΡΠ»Π΅Π΄ΠΎΠ²Π°Ρ‚Π΅Π»ΡŒΠ½ΠΎ, ΠΌΠΎΠΆΠ΅Ρ‚ Π±Ρ‹Ρ‚ΡŒ Π²Ρ‹Ρ€Π°ΠΆΠ΅Π½Π° ΠΊΠ°ΠΊ Π΅ΠΌΠΊΠΎΡΡ‚ΡŒ ΠΎΠ΄Π½ΠΎΠ³ΠΎ эквивалСнтного кондСнсатора.

Π€ΠΎΡ€ΠΌΡƒΠ»Ρƒ для этого ΠΌΠΎΠΆΠ½ΠΎ вывСсти ΠΈΠ· основного выраТСния для Смкости ΠΈΠ· ΠΏΡ€Π΅Π΄Ρ‹Π΄ΡƒΡ‰Π΅Π³ΠΎ Ρ€Π°Π·Π΄Π΅Π»Π°, пСрСставлСнного ΡΠ»Π΅Π΄ΡƒΡŽΡ‰ΠΈΠΌ ΠΎΠ±Ρ€Π°Π·ΠΎΠΌ:

Π’ = \ frac {Q} {C}

ΠŸΠΎΡΠΊΠΎΠ»ΡŒΠΊΡƒ Π·Π°ΠΊΠΎΠ½ ΠšΠΈΡ€Ρ…Π³ΠΎΡ„Π° ΠΏΠΎ Π½Π°ΠΏΡ€ΡΠΆΠ΅Π½ΠΈΡŽ ΡƒΡ‚Π²Π΅Ρ€ΠΆΠ΄Π°Π΅Ρ‚, Ρ‡Ρ‚ΠΎ сумма напряТСний ΠΏΠ°Π΄Π΅Π½ΠΈΠ΅ напряТСния Π²ΠΎΠΊΡ€ΡƒΠ³ ΠΏΠΎΠ»Π½ΠΎΠ³ΠΎ ΠΊΠΎΠ½Ρ‚ΡƒΡ€Π° Ρ†Π΅ΠΏΠΈ Π΄ΠΎΠ»ΠΆΠ½ΠΎ Π±Ρ‹Ρ‚ΡŒ Ρ€Π°Π²Π½ΠΎ Π½Π°ΠΏΡ€ΡΠΆΠ΅Π½ΠΈΡŽ ΠΎΡ‚ источника питания, для ряда кондСнсаторов n напряТСния Π΄ΠΎΠ»ΠΆΠ½Ρ‹ ΡΠΊΠ»Π°Π΄Ρ‹Π²Π°Ρ‚ΡŒΡΡ ΡΠ»Π΅Π΄ΡƒΡŽΡ‰ΠΈΠΌ ΠΎΠ±Ρ€Π°Π·ΠΎΠΌ:

V_ {tot} = V_1 + V_2 + V_3 + … Π’_Π½

Π“Π΄Π΅ Π’ Π΄ΠΎ — ΠΏΠΎΠ»Π½ΠΎΠ΅ напряТСниС ΠΎΡ‚ источника питания, Π° Π’ 1 , Π’ 2 , Π’ 3 ΠΈ Ρ‚Π°ΠΊ ΠΆΠ΅ ΠΏΠ°Π΄Π΅Π½ΠΈΠ΅ напряТСния Π½Π° ΠΏΠ΅Ρ€Π²ΠΎΠΌ кондСнсаторС, Π²Ρ‚ΠΎΡ€ΠΎΠΌ кондСнсаторС, Ρ‚Ρ€Π΅Ρ‚ΡŒΠ΅ΠΌ кондСнсаторС ΠΈ Ρ‚Π°ΠΊ Π΄Π°Π»Π΅Π΅. Π’ сочСтании с ΠΏΡ€Π΅Π΄Ρ‹Π΄ΡƒΡ‰ΠΈΠΌ ΡƒΡ€Π°Π²Π½Π΅Π½ΠΈΠ΅ΠΌ это ΠΏΡ€ΠΈΠ²ΠΎΠ΄ΠΈΡ‚ ΠΊ:

\ frac {Q_ {tot}} {C_ {tot}} = \ frac {Q_1} {C_1} + \ frac {Q_2} {C_2} + \ frac {Q_3 } {C_3} +… \ frac {Q_n} {C_n}

Π“Π΄Π΅ Π½ΠΈΠΆΠ½ΠΈΠ΅ индСксы ΠΈΠΌΠ΅ΡŽΡ‚ Ρ‚ΠΎ ΠΆΠ΅ Π·Π½Π°Ρ‡Π΅Π½ΠΈΠ΅, Ρ‡Ρ‚ΠΎ ΠΈ Ρ€Π°Π½ΡŒΡˆΠ΅. Однако заряд Π½Π° ΠΊΠ°ΠΆΠ΄ΠΎΠΉ ΠΈΠ· ΠΎΠ±ΠΊΠ»Π°Π΄ΠΎΠΊ кондСнсатора (Ρ‚.Π΅. значСния Q ) исходит ΠΎΡ‚ сосСднСй пластины (Ρ‚ΠΎ Π΅ΡΡ‚ΡŒ ΠΏΠΎΠ»ΠΎΠΆΠΈΡ‚Π΅Π»ΡŒΠ½Ρ‹ΠΉ заряд Π½Π° ΠΎΠ΄Π½ΠΎΠΉ сторонС пластины 1 Π΄ΠΎΠ»ΠΆΠ΅Π½ ΡΠΎΠΎΡ‚Π²Π΅Ρ‚ΡΡ‚Π²ΠΎΠ²Π°Ρ‚ΡŒ ΠΎΡ‚Ρ€ΠΈΡ†Π°Ρ‚Π΅Π»ΡŒΠ½ΠΎΠΌΡƒ заряду Π½Π° блиТайшСй сторонС пластины 2. ΠΈ Ρ‚Π°ΠΊ Π΄Π°Π»Π΅Π΅), поэтому Π²Ρ‹ ΠΌΠΎΠΆΠ΅Ρ‚Π΅ Π½Π°ΠΏΠΈΡΠ°Ρ‚ΡŒ:

Q_ {tot} = Q_1 = Q_2 = Q_3 = Q_n

Π’Π°ΠΊΠΈΠΌ ΠΎΠ±Ρ€Π°Π·ΠΎΠΌ, начислСния Π°Π½Π½ΡƒΠ»ΠΈΡ€ΡƒΡŽΡ‚ΡΡ, Π² Ρ€Π΅Π·ΡƒΠ»ΡŒΡ‚Π°Ρ‚Π΅ Ρ‡Π΅Π³ΠΎ остаСтся:

\ frac {1} {C_ {tot}} = \ frac {1} {C_1} + \ frac {1} {C_2} + \ frac {1} {C_3} +… \ frac {1} {C_n}

ΠŸΠΎΡΠΊΠΎΠ»ΡŒΠΊΡƒ Π΅ΠΌΠΊΠΎΡΡ‚ΡŒ ΠΊΠΎΠΌΠ±ΠΈΠ½Π°Ρ†ΠΈΠΈ Ρ€Π°Π²Π½Π° эквивалСнтной Смкости ΠΎΠ΄ΠΈΠ½ΠΎΡ‡Π½Ρ‹ΠΉ кондСнсатор, это ΠΌΠΎΠΆΠ½ΠΎ Π·Π°ΠΏΠΈΡΠ°Ρ‚ΡŒ Ρ‚Π°ΠΊ:

\ frac {1} {C_ {eq}} = \ frac {1} {C_1} + \ frac {1} {C_2} + \ frac {1} {C_3} +… \ frac {1} {C_n}

для любого количСства кондСнсаторов n . {βˆ’6} \ text {F} \\ & = 1.41 \ text {ΠΌΠΊΠ€} \ end {Π²Ρ‹Ρ€Π°Π²Π½ΠΈΠ²Π°Π½ΠΈΠ΅}

ЭквивалСнтная Π΅ΠΌΠΊΠΎΡΡ‚ΡŒ ΠΏΠ°Ρ€Π°Π»Π»Π΅Π»ΡŒΠ½Ρ‹Ρ… кондСнсаторов

Для ΠΏΠ°Ρ€Π°Π»Π»Π΅Π»ΡŒΠ½Ρ‹Ρ… кондСнсаторов Π°Π½Π°Π»ΠΎΠ³ΠΈΡ‡Π½Ρ‹ΠΉ Ρ€Π΅Π·ΡƒΠ»ΡŒΡ‚Π°Ρ‚ получаСтся ΠΈΠ· Q = VC, Ρ‚ΠΎΠ³ΠΎ Ρ„Π°ΠΊΡ‚Π°, Ρ‡Ρ‚ΠΎ ΠΏΠ°Π΄Π΅Π½ΠΈΠ΅ напряТСния Π½Π° всСх кондСнсаторах, ΠΏΠΎΠ΄ΠΊΠ»ΡŽΡ‡Π΅Π½Π½Ρ‹Ρ… ΠΏΠ°Ρ€Π°Π»Π»Π΅Π»ΡŒΠ½ΠΎ (ΠΈΠ»ΠΈ Π»ΡŽΠ±Ρ‹Ρ… ΠΊΠΎΠΌΠΏΠΎΠ½Π΅Π½Ρ‚Π°Ρ… Π² ΠΏΠ°Ρ€Π°Π»Π»Π΅Π»ΡŒΠ½ΠΎΠΉ Ρ†Π΅ΠΏΠΈ), ΠΎΠ΄ΠΈΠ½Π°ΠΊΠΎΠ²ΠΎ, ΠΈ Ρ‚ΠΎΡ‚ Ρ„Π°ΠΊΡ‚, Ρ‡Ρ‚ΠΎ заряд Π½Π° ΠΎΠ΄Π½ΠΎΠΌ эквивалСнтном кондСнсаторС Π±ΡƒΠ΄Π΅Ρ‚ ΠΏΠΎΠ»Π½Ρ‹ΠΌ зарядом всСх ΠΎΡ‚Π΄Π΅Π»ΡŒΠ½Ρ‹Ρ… кондСнсаторов Π² ΠΏΠ°Ρ€Π°Π»Π»Π΅Π»ΡŒΠ½ΠΎΠΉ ΠΊΠΎΠΌΠ±ΠΈΠ½Π°Ρ†ΠΈΠΈ. Π Π΅Π·ΡƒΠ»ΡŒΡ‚Π°Ρ‚ΠΎΠΌ являСтся Π±ΠΎΠ»Π΅Π΅ простоС Π²Ρ‹Ρ€Π°ΠΆΠ΅Π½ΠΈΠ΅ для ΠΎΠ±Ρ‰Π΅ΠΉ Смкости ΠΈΠ»ΠΈ эквивалСнтной Смкости:

C_ {eq} = C_1 + C_2 + C_3 +… C_n

, Π³Π΄Π΅ снова n — ΠΎΠ±Ρ‰Π΅Π΅ количСство кондСнсаторов.{βˆ’5} \ text {F} \\ & = 15 \ text {ΠΌΠΊΠ€} \ end {align}

ΠšΠΎΠΌΠ±ΠΈΠ½Π°Ρ†ΠΈΠΈ кондСнсаторов: ΠΏΡ€ΠΎΠ±Π»Π΅ΠΌΠ° пСрвая

НахоТдСниС эквивалСнтной Смкости для ΠΊΠΎΠΌΠ±ΠΈΠ½Π°Ρ†ΠΈΠΉ кондСнсаторов, располоТСнных ΠΏΠΎΡΠ»Π΅Π΄ΠΎΠ²Π°Ρ‚Π΅Π»ΡŒΠ½ΠΎ ΠΈ упорядочСнных просто ΠΏΠ°Ρ€Π°Π»Π»Π΅Π»ΡŒΠ½ΠΎ Π²ΠΊΠ»ΡŽΡ‡Π°Π΅Ρ‚ Π² сСбя ΠΏΡ€ΠΈΠΌΠ΅Π½Π΅Π½ΠΈΠ΅ этих Π΄Π²ΡƒΡ… Ρ„ΠΎΡ€ΠΌΡƒΠ» ΠΏΠΎ ΠΎΡ‡Π΅Ρ€Π΅Π΄ΠΈ. НапримСр, ΠΏΡ€Π΅Π΄ΡΡ‚Π°Π²ΡŒΡ‚Π΅ ΠΊΠΎΠΌΠ±ΠΈΠ½Π°Ρ†ΠΈΡŽ кондСнсаторов с двумя ΠΏΠΎΡΠ»Π΅Π΄ΠΎΠ²Π°Ρ‚Π΅Π»ΡŒΠ½ΠΎ Π²ΠΊΠ»ΡŽΡ‡Π΅Π½Π½Ρ‹ΠΌΠΈ кондСнсаторами: C 1 = 3 Γ— 10 βˆ’3 F ΠΈ C 2 = 1 Γ— 10 βˆ’3 F. , ΠΈ Π΅Ρ‰Π΅ ΠΎΠ΄ΠΈΠ½ кондСнсатор ΠΏΠ°Ρ€Π°Π»Π»Π΅Π»ΡŒΠ½ΠΎ с C 3 = 8 Γ— 10 βˆ’3 F.{βˆ’3} \ text {F} \ end {align}

ΠšΠΎΠΌΠ±ΠΈΠ½Π°Ρ†ΠΈΠΈ кондСнсаторов: ΠŸΡ€ΠΎΠ±Π»Π΅ΠΌΠ° Π”Π²Π°

Для Π΄Ρ€ΡƒΠ³ΠΎΠΉ ΠΊΠΎΠΌΠ±ΠΈΠ½Π°Ρ†ΠΈΠΈ кондСнсаторов, Ρ‚Ρ€ΠΈ ΠΏΡ€ΠΈ ΠΏΠ°Ρ€Π°Π»Π»Π΅Π»ΡŒΠ½ΠΎΠΌ ΠΏΠΎΠ΄ΠΊΠ»ΡŽΡ‡Π΅Π½ΠΈΠΈ (со значСниями C 1 = 3 ΠΌΠΊΠ€ , C 2 = 8 ΠΌΠΊΠ€ ΠΈ C 3 = 12 ΠΌΠΊΠ€) ΠΈ ΠΎΠ΄ΠΈΠ½ с ΠΏΠΎΡΠ»Π΅Π΄ΠΎΠ²Π°Ρ‚Π΅Π»ΡŒΠ½Ρ‹ΠΌ ΠΏΠΎΠ΄ΠΊΠ»ΡŽΡ‡Π΅Π½ΠΈΠ΅ΠΌ (с C 4 = 20 ΠΌΠΊΠ€):

ΠŸΠΎΠ΄Ρ…ΠΎΠ΄ Π² основном Ρ‚ΠΎ ΠΆΠ΅ самоС, Ρ‡Ρ‚ΠΎ ΠΈ Π² послСднСм ΠΏΡ€ΠΈΠΌΠ΅Ρ€Π΅, Π·Π° ΠΈΡΠΊΠ»ΡŽΡ‡Π΅Π½ΠΈΠ΅ΠΌ Ρ‚ΠΎΠ³ΠΎ, Ρ‡Ρ‚ΠΎ сначала Π²Ρ‹ ΠΎΠ±Ρ€Π°Π±Π°Ρ‚Ρ‹Π²Π°Π΅Ρ‚Π΅ ΠΏΠ°Ρ€Π°Π»Π»Π΅Π»ΡŒΠ½Ρ‹Π΅ кондСнсаторы.{βˆ’1}} \\ & = 10.7 \ text {ΠΌΠΊΠ€} \ end {align}

ΠžΠ±Ρ€Π°Ρ‚ΠΈΡ‚Π΅ Π²Π½ΠΈΠΌΠ°Π½ΠΈΠ΅, Ρ‡Ρ‚ΠΎ, ΠΏΠΎΡΠΊΠΎΠ»ΡŒΠΊΡƒ всС ΠΎΡ‚Π΄Π΅Π»ΡŒΠ½Ρ‹Π΅ Смкости Π±Ρ‹Π»ΠΈ Π² ΠΌΠΈΠΊΡ€ΠΎΡ„Π°Ρ€Π°Π΄Π°Ρ…, вСсь расчСт ΠΌΠΎΠΆΠ΅Ρ‚ Π±Ρ‹Ρ‚ΡŒ Π²Ρ‹ΠΏΠΎΠ»Π½Π΅Π½ Π² ΠΌΠΈΠΊΡ€ΠΎΡ„Π°Ρ€Π°Π΄Π°Ρ… Π±Π΅Π· прСобразования — Ссли Π²Ρ‹ ΠΏΠΎΠΌΠ½ΠΈΡ‚Π΅ ΠΏΡ€ΠΈ Ρ†ΠΈΡ‚ΠΈΡ€ΠΎΠ²Π°Π½ΠΈΠΈ Π²Π°ΡˆΠΈΡ… ΠΎΠΊΠΎΠ½Ρ‡Π°Ρ‚Π΅Π»ΡŒΠ½Ρ‹Ρ… ΠΎΡ‚Π²Π΅Ρ‚ΠΎΠ²!

Π‘Ρ‚Ρ€Π°Π½ΠΈΡ†Π° Π½Π΅ Π½Π°ΠΉΠ΄Π΅Π½Π° | MIT

ΠŸΠ΅Ρ€Π΅ΠΉΡ‚ΠΈ ΠΊ ΡΠΎΠ΄Π΅Ρ€ΠΆΠ°Π½ΠΈΡŽ ↓

  • ΠžΠ±Ρ€Π°Π·ΠΎΠ²Π°Π½ΠΈΠ΅
  • ИсслСдованиС
  • Π˜Π½Π½ΠΎΠ²Π°Ρ†ΠΈΠΈ
  • ΠŸΡ€ΠΈΠ΅ΠΌ + ΠΏΠΎΠΌΠΎΡ‰ΡŒ
  • БтудСнчСская Тизнь
  • Новости
  • Выпускников
  • О MIT
  • ΠŸΠΎΠ΄Ρ€ΠΎΠ±Π½Π΅Π΅ ↓

    • ΠŸΡ€ΠΈΠ΅ΠΌ + ΠΏΠΎΠΌΠΎΡ‰ΡŒ
    • БтудСнчСская Тизнь
    • Новости
    • Выпускников
    • О MIT

МСню ↓

Поиск

МСню

Ой, ΠΏΠΎΡ…ΠΎΠΆΠ΅, ΠΌΡ‹ Π½Π΅ смогли Π½Π°ΠΉΡ‚ΠΈ Ρ‚ΠΎ, Ρ‡Ρ‚ΠΎ Π²Ρ‹ искали!
ΠŸΠΎΠΏΡ€ΠΎΠ±ΡƒΠΉΡ‚Π΅ ΠΏΠΎΠΈΡΠΊΠ°Ρ‚ΡŒ Ρ‡Ρ‚ΠΎ-Π½ΠΈΠ±ΡƒΠ΄ΡŒ Π΅Ρ‰Π΅!

Π§Ρ‚ΠΎ Π²Ρ‹ ΠΈΡ‰Π΅Ρ‚Π΅?

Π£Π²ΠΈΠ΄Π΅Ρ‚ΡŒ большС Ρ€Π΅Π·ΡƒΠ»ΡŒΡ‚Π°Ρ‚ΠΎΠ²

ΠŸΡ€Π΅Π΄Π»ΠΎΠΆΠ΅Π½ΠΈΡ ΠΈΠ»ΠΈ ΠΎΡ‚Π·Ρ‹Π²Ρ‹?

РасчСт ΠΎΠ±Ρ‰Π΅ΠΉ Смкости — Π€ΠΈΠ·ΠΈΠΊΠ° срСднСй ΡˆΠΊΠΎΠ»Ρ‹

Если Π²Ρ‹ считаСтС, Ρ‡Ρ‚ΠΎ ΠΊΠΎΠ½Ρ‚Π΅Π½Ρ‚, доступный Ρ‡Π΅Ρ€Π΅Π· Π’Π΅Π±-сайт (ΠΊΠ°ΠΊ ΠΎΠΏΡ€Π΅Π΄Π΅Π»Π΅Π½ΠΎ Π² Π½Π°ΡˆΠΈΡ… Условиях обслуТивания), Π½Π°Ρ€ΡƒΡˆΠ°Π΅Ρ‚
ΠΈΠ»ΠΈ большС Π²Π°ΡˆΠΈΡ… авторских ΠΏΡ€Π°Π², сообщитС Π½Π°ΠΌ, ΠΎΡ‚ΠΏΡ€Π°Π²ΠΈΠ² письмСнноС ΡƒΠ²Π΅Π΄ΠΎΠΌΠ»Π΅Π½ΠΈΠ΅ (Β«Π£Π²Π΅Π΄ΠΎΠΌΠ»Π΅Π½ΠΈΠ΅ ΠΎ Π½Π°Ρ€ΡƒΡˆΠ΅Π½ΠΈΠΈΒ»), содСрТащСС
Ρ‚ΠΎ
ΠΈΠ½Ρ„ΠΎΡ€ΠΌΠ°Ρ†ΠΈΡŽ, ΠΎΠΏΠΈΡΠ°Π½Π½ΡƒΡŽ Π½ΠΈΠΆΠ΅, Π½Π°Π·Π½Π°Ρ‡Π΅Π½Π½ΠΎΠΌΡƒ Π½ΠΈΠΆΠ΅ Π°Π³Π΅Π½Ρ‚Ρƒ. Если Ρ€Π΅ΠΏΠ΅Ρ‚ΠΈΡ‚ΠΎΡ€Ρ‹ Π²ΡƒΠ·Π° ΠΏΡ€Π΅Π΄ΠΏΡ€ΠΈΠΌΡƒΡ‚ дСйствия Π² ΠΎΡ‚Π²Π΅Ρ‚ Π½Π°
Π°Π½
Π£Π²Π΅Π΄ΠΎΠΌΠ»Π΅Π½ΠΈΠ΅ ΠΎ Π½Π°Ρ€ΡƒΡˆΠ΅Π½ΠΈΠΈ, ΠΎΠ½ΠΎ ΠΏΡ€Π΅Π΄ΠΏΡ€ΠΈΠΌΠ΅Ρ‚ Π΄ΠΎΠ±Ρ€ΠΎΡΠΎΠ²Π΅ΡΡ‚Π½ΡƒΡŽ ΠΏΠΎΠΏΡ‹Ρ‚ΠΊΡƒ ΡΠ²ΡΠ·Π°Ρ‚ΡŒΡΡ со стороной, которая прСдоставила Ρ‚Π°ΠΊΠΎΠΉ ΠΊΠΎΠ½Ρ‚Π΅Π½Ρ‚
срСдствами самого послСднСго адрСса элСктронной ΠΏΠΎΡ‡Ρ‚Ρ‹, Ссли Ρ‚Π°ΠΊΠΎΠ²ΠΎΠΉ имССтся, прСдоставлСнного Ρ‚Π°ΠΊΠΎΠΉ стороной Varsity Tutors.

Π’Π°ΡˆΠ΅ Π£Π²Π΅Π΄ΠΎΠΌΠ»Π΅Π½ΠΈΠ΅ ΠΎ Π½Π°Ρ€ΡƒΡˆΠ΅Π½ΠΈΠΈ ΠΌΠΎΠΆΠ΅Ρ‚ Π±Ρ‹Ρ‚ΡŒ Π½Π°ΠΏΡ€Π°Π²Π»Π΅Π½ΠΎ сторонС, ΠΏΡ€Π΅Π΄ΠΎΡΡ‚Π°Π²ΠΈΠ²ΡˆΠ΅ΠΉ доступ ΠΊ ΠΊΠΎΠ½Ρ‚Π΅Π½Ρ‚Ρƒ, ΠΈΠ»ΠΈ Ρ‚Ρ€Π΅Ρ‚ΡŒΠΈΠΌ Π»ΠΈΡ†Π°ΠΌ, Ρ‚Π°ΠΊΠΈΠΌ ΠΊΠ°ΠΊ
Π² качСствС
ChillingEffects.org.

ΠžΠ±Ρ€Π°Ρ‚ΠΈΡ‚Π΅ Π²Π½ΠΈΠΌΠ°Π½ΠΈΠ΅, Ρ‡Ρ‚ΠΎ Π²Ρ‹ Π±ΡƒΠ΄Π΅Ρ‚Π΅ нСсти ΠΎΡ‚Π²Π΅Ρ‚ΡΡ‚Π²Π΅Π½Π½ΠΎΡΡ‚ΡŒ Π·Π° ΡƒΡ‰Π΅Ρ€Π± (Π²ΠΊΠ»ΡŽΡ‡Π°Ρ расходы ΠΈ Π³ΠΎΠ½ΠΎΡ€Π°Ρ€Ρ‹ Π°Π΄Π²ΠΎΠΊΠ°Ρ‚Π°ΠΌ), Ссли Π²Ρ‹
ΠΈΡΠΊΠ°ΠΆΠ°Ρ‚ΡŒ ΠΈΠ½Ρ„ΠΎΡ€ΠΌΠ°Ρ†ΠΈΡŽ ΠΎ Ρ‚ΠΎΠΌ, Ρ‡Ρ‚ΠΎ ΠΏΡ€ΠΎΠ΄ΡƒΠΊΡ‚ ΠΈΠ»ΠΈ дСйствиС Π½Π°Ρ€ΡƒΡˆΠ°Π΅Ρ‚ ваши авторскиС ΠΏΡ€Π°Π²Π°.Π’Π°ΠΊΠΈΠΌ ΠΎΠ±Ρ€Π°Π·ΠΎΠΌ, Ссли Π²Ρ‹ Π½Π΅ ΡƒΠ²Π΅Ρ€Π΅Π½Ρ‹, Ρ‡Ρ‚ΠΎ ΠΊΠΎΠ½Ρ‚Π΅Π½Ρ‚ находится
Π½Π° Π²Π΅Π±-сайтС ΠΈΠ»ΠΈ ΠΏΠΎ ссылкС с Π½Π΅Π³ΠΎ Π½Π°Ρ€ΡƒΡˆΠ°Π΅Ρ‚ ваши авторскиС ΠΏΡ€Π°Π²Π°, Π²Π°ΠΌ слСдуСт сначала ΠΎΠ±Ρ€Π°Ρ‚ΠΈΡ‚ΡŒΡΡ ΠΊ ΡŽΡ€ΠΈΡΡ‚Ρƒ.

Π§Ρ‚ΠΎΠ±Ρ‹ ΠΎΡ‚ΠΏΡ€Π°Π²ΠΈΡ‚ΡŒ ΡƒΠ²Π΅Π΄ΠΎΠΌΠ»Π΅Π½ΠΈΠ΅, Π²Ρ‹ΠΏΠΎΠ»Π½ΠΈΡ‚Π΅ ΡΠ»Π΅Π΄ΡƒΡŽΡ‰ΠΈΠ΅ дСйствия:

Π’Ρ‹ Π΄ΠΎΠ»ΠΆΠ½Ρ‹ Π²ΠΊΠ»ΡŽΡ‡ΠΈΡ‚ΡŒ ΡΠ»Π΅Π΄ΡƒΡŽΡ‰Π΅Π΅:

ЀизичСская ΠΈΠ»ΠΈ элСктронная подпись Π²Π»Π°Π΄Π΅Π»ΡŒΡ†Π° авторских ΠΏΡ€Π°Π² ΠΈΠ»ΠΈ Π»ΠΈΡ†Π°, ΡƒΠΏΠΎΠ»Π½ΠΎΠΌΠΎΡ‡Π΅Π½Π½ΠΎΠ³ΠΎ Π΄Π΅ΠΉΡΡ‚Π²ΠΎΠ²Π°Ρ‚ΡŒ ΠΎΡ‚ ΠΈΡ… ΠΈΠΌΠ΅Π½ΠΈ;
Π˜Π΄Π΅Π½Ρ‚ΠΈΡ„ΠΈΠΊΠ°Ρ†ΠΈΡ авторских ΠΏΡ€Π°Π², ΠΊΠΎΡ‚ΠΎΡ€Ρ‹Π΅, ΠΊΠ°ΠΊ утвСрТдаСтся, Π±Ρ‹Π»ΠΈ Π½Π°Ρ€ΡƒΡˆΠ΅Π½Ρ‹;
ОписаниС Ρ…Π°Ρ€Π°ΠΊΡ‚Π΅Ρ€Π° ΠΈ Ρ‚ΠΎΡ‡Π½ΠΎΠ³ΠΎ располоТСния ΠΊΠΎΠ½Ρ‚Π΅Π½Ρ‚Π°, ΠΊΠΎΡ‚ΠΎΡ€Ρ‹ΠΉ, ΠΏΠΎ Π²Π°ΡˆΠ΅ΠΌΡƒ мнСнию, Π½Π°Ρ€ΡƒΡˆΠ°Π΅Ρ‚ ваши авторскиС ΠΏΡ€Π°Π²Π°, Π² \
достаточно подробностСй, Ρ‡Ρ‚ΠΎΠ±Ρ‹ ΠΏΠΎΠ·Π²ΠΎΠ»ΠΈΡ‚ΡŒ Ρ€Π΅ΠΏΠ΅Ρ‚ΠΈΡ‚ΠΎΡ€Π°ΠΌ унивСрситСтских школ Π½Π°ΠΉΡ‚ΠΈ ΠΈ Ρ‚ΠΎΡ‡Π½ΠΎ ΠΈΠ΄Π΅Π½Ρ‚ΠΈΡ„ΠΈΡ†ΠΈΡ€ΠΎΠ²Π°Ρ‚ΡŒ этот ΠΊΠΎΠ½Ρ‚Π΅Π½Ρ‚; Π½Π°ΠΏΡ€ΠΈΠΌΠ΅Ρ€ Π½Π°ΠΌ трСбуСтся
Π°
ссылка Π½Π° ΠΊΠΎΠ½ΠΊΡ€Π΅Ρ‚Π½Ρ‹ΠΉ вопрос (Π° Π½Π΅ Ρ‚ΠΎΠ»ΡŒΠΊΠΎ Π½Π° Π½Π°Π·Π²Π°Π½ΠΈΠ΅ вопроса), ΠΊΠΎΡ‚ΠΎΡ€Ρ‹ΠΉ содСрТит содСрТаниС ΠΈ описаниС
ΠΊ ΠΊΠ°ΠΊΠΎΠΉ ΠΊΠΎΠ½ΠΊΡ€Π΅Ρ‚Π½ΠΎΠΉ части вопроса — ΠΈΠ·ΠΎΠ±Ρ€Π°ΠΆΠ΅Π½ΠΈΡŽ, ссылкС, тСксту ΠΈ Ρ‚. Π΄. — относится ваша ΠΆΠ°Π»ΠΎΠ±Π°;
Π’Π°ΡˆΠ΅ имя, адрСс, Π½ΠΎΠΌΠ΅Ρ€ Ρ‚Π΅Π»Π΅Ρ„ΠΎΠ½Π° ΠΈ адрСс элСктронной ΠΏΠΎΡ‡Ρ‚Ρ‹; ΠΈ
Π’Π°ΡˆΠ΅ заявлСниС: (Π°) Ρ‡Ρ‚ΠΎ Π²Ρ‹ добросовСстно считаСтС, Ρ‡Ρ‚ΠΎ использованиС ΠΊΠΎΠ½Ρ‚Π΅Π½Ρ‚Π°, ΠΊΠΎΡ‚ΠΎΡ€Ρ‹ΠΉ, ΠΏΠΎ Π²Π°ΡˆΠ΅ΠΌΡƒ мнСнию, Π½Π°Ρ€ΡƒΡˆΠ°Π΅Ρ‚
ваши авторскиС ΠΏΡ€Π°Π²Π° Π½Π΅ Ρ€Π°Π·Ρ€Π΅ΡˆΠ΅Π½Ρ‹ Π·Π°ΠΊΠΎΠ½ΠΎΠΌ, Π²Π»Π°Π΄Π΅Π»ΡŒΡ†Π΅ΠΌ авторских ΠΏΡ€Π°Π² ΠΈΠ»ΠΈ Π΅Π³ΠΎ Π°Π³Π΅Π½Ρ‚ΠΎΠΌ; (Π±) Ρ‡Ρ‚ΠΎ всС
информация, содСрТащаяся Π² вашСм Π£Π²Π΅Π΄ΠΎΠΌΠ»Π΅Π½ΠΈΠΈ ΠΎ Π½Π°Ρ€ΡƒΡˆΠ΅Π½ΠΈΠΈ, являСтся Ρ‚ΠΎΡ‡Π½ΠΎΠΉ, ΠΈ (c) ΠΏΠΎΠ΄ страхом наказания Π·Π° Π»ΠΆΠ΅ΡΠ²ΠΈΠ΄Π΅Ρ‚Π΅Π»ΡŒΡΡ‚Π²ΠΎ, Ρ‡Ρ‚ΠΎ Π²Ρ‹
Π»ΠΈΠ±ΠΎ Π²Π»Π°Π΄Π΅Π»Π΅Ρ† авторских ΠΏΡ€Π°Π², Π»ΠΈΠ±ΠΎ Π»ΠΈΡ†ΠΎ, ΡƒΠΏΠΎΠ»Π½ΠΎΠΌΠΎΡ‡Π΅Π½Π½ΠΎΠ΅ Π΄Π΅ΠΉΡΡ‚Π²ΠΎΠ²Π°Ρ‚ΡŒ ΠΎΡ‚ ΠΈΡ… ΠΈΠΌΠ΅Π½ΠΈ.

ΠžΡ‚ΠΏΡ€Π°Π²ΡŒΡ‚Π΅ ΠΆΠ°Π»ΠΎΠ±Ρƒ Π½Π°ΡˆΠ΅ΠΌΡƒ ΡƒΠΏΠΎΠ»Π½ΠΎΠΌΠΎΡ‡Π΅Π½Π½ΠΎΠΌΡƒ Π°Π³Π΅Π½Ρ‚Ρƒ ΠΏΠΎ адрСсу:

Π§Π°Ρ€Π»ΡŒΠ· Кон
Varsity Tutors LLC
101 S. Hanley Rd, Suite 300
St. Louis, MO 63105

Или Π·Π°ΠΏΠΎΠ»Π½ΠΈΡ‚Π΅ Ρ„ΠΎΡ€ΠΌΡƒ Π½ΠΈΠΆΠ΅:

Как Ρ€Π°ΡΡΡ‡ΠΈΡ‚Π°Ρ‚ΡŒ кондСнсаторы, ΠΏΠΎΠ΄ΠΊΠ»ΡŽΡ‡Π΅Π½Π½Ρ‹Π΅ ΠΏΠΎΡΠ»Π΅Π΄ΠΎΠ²Π°Ρ‚Π΅Π»ΡŒΠ½ΠΎ ΠΈ ΠΏΠ°Ρ€Π°Π»Π»Π΅Π»ΡŒΠ½ΠΎ — Kitronik Ltd

ΠŸΠ°Ρ€Π°Π»Π»Π΅Π»ΡŒΠ½Ρ‹Π΅ кондСнсаторы

Когда кондСнсаторы ΠΏΠΎΠ΄ΠΊΠ»ΡŽΡ‡Π°ΡŽΡ‚ΡΡ Π΄Ρ€ΡƒΠ³ ΠΊ Π΄Ρ€ΡƒΠ³Ρƒ (Π±ΠΎΠΊ ΠΎ Π±ΠΎΠΊ), это называСтся ΠΏΠ°Ρ€Π°Π»Π»Π΅Π»ΡŒΠ½Ρ‹ΠΌ ΠΏΠΎΠ΄ΠΊΠ»ΡŽΡ‡Π΅Π½ΠΈΠ΅ΠΌ. Π­Ρ‚ΠΎ ΠΏΠΎΠΊΠ°Π·Π°Π½ΠΎ Π½ΠΈΠΆΠ΅. Π§Ρ‚ΠΎΠ±Ρ‹ Ρ€Π°ΡΡΡ‡ΠΈΡ‚Π°Ρ‚ΡŒ ΠΎΠ±Ρ‰ΡƒΡŽ ΠΎΠ±Ρ‰ΡƒΡŽ Π΅ΠΌΠΊΠΎΡΡ‚ΡŒ ряда кондСнсаторов, ΠΏΠΎΠ΄ΠΊΠ»ΡŽΡ‡Π΅Π½Π½Ρ‹Ρ… Ρ‚Π°ΠΊΠΈΠΌ ΠΎΠ±Ρ€Π°Π·ΠΎΠΌ, Π²Ρ‹ складываСтС ΠΎΡ‚Π΄Π΅Π»ΡŒΠ½Ρ‹Π΅ Смкости ΠΏΠΎ ΡΠ»Π΅Π΄ΡƒΡŽΡ‰Π΅ΠΉ Ρ„ΠΎΡ€ΠΌΡƒΠ»Π΅: CTotal = C1 + C2 + C3 ΠΈ Ρ‚. Π”. ΠŸΡ€ΠΈΠΌΠ΅Ρ€: Π§Ρ‚ΠΎΠ±Ρ‹ Ρ€Π°ΡΡΡ‡ΠΈΡ‚Π°Ρ‚ΡŒ ΠΎΠ±Ρ‰ΡƒΡŽ Π΅ΠΌΠΊΠΎΡΡ‚ΡŒ для этих Ρ‚Ρ€Π΅Ρ… кондСнсаторов, ΠΏΠΎΠ΄ΠΊΠ»ΡŽΡ‡Π΅Π½Π½Ρ‹Ρ… ΠΏΠ°Ρ€Π°Π»Π»Π΅Π»ΡŒΠ½ΠΎ. CΠΎΠ±Ρ‰ = C1 + C2 + C3 = 10F + 22F + 47F = 79F


Π—Π°Π΄Π°Ρ‡Π° 1:

РассчитайтС ΠΎΠ±Ρ‰ΡƒΡŽ Π΅ΠΌΠΊΠΎΡΡ‚ΡŒ ΡΠ»Π΅Π΄ΡƒΡŽΡ‰ΠΈΡ… кондСнсаторов, Π²ΠΊΠ»ΡŽΡ‡Π΅Π½Π½Ρ‹Ρ… ΠΏΠ°Ρ€Π°Π»Π»Π΅Π»ΡŒΠ½ΠΎ.

ΠšΠΎΠ½Π΄Π΅Π½ΡΠ°Ρ‚ΠΎΡ€Ρ‹ сСрии

Когда кондСнсаторы ΠΏΠΎΠ΄ΠΊΠ»ΡŽΡ‡Π°ΡŽΡ‚ΡΡ Π΄Ρ€ΡƒΠ³ Π·Π° Π΄Ρ€ΡƒΠ³ΠΎΠΌ, это называСтся ΠΏΠΎΡΠ»Π΅Π΄ΠΎΠ²Π°Ρ‚Π΅Π»ΡŒΠ½Ρ‹ΠΌ соСдинСниСм.Π­Ρ‚ΠΎ ΠΏΠΎΠΊΠ°Π·Π°Π½ΠΎ Π½ΠΈΠΆΠ΅. Π§Ρ‚ΠΎΠ±Ρ‹ Ρ€Π°ΡΡΡ‡ΠΈΡ‚Π°Ρ‚ΡŒ ΠΎΠ±Ρ‰ΡƒΡŽ ΠΎΠ±Ρ‰ΡƒΡŽ Π΅ΠΌΠΊΠΎΡΡ‚ΡŒ Π΄Π²ΡƒΡ… кондСнсаторов, ΠΏΠΎΠ΄ΠΊΠ»ΡŽΡ‡Π΅Π½Π½Ρ‹Ρ… Ρ‚Π°ΠΊΠΈΠΌ ΠΎΠ±Ρ€Π°Π·ΠΎΠΌ, Π²Ρ‹ ΠΌΠΎΠΆΠ΅Ρ‚Π΅ ΠΈΡΠΏΠΎΠ»ΡŒΠ·ΠΎΠ²Π°Ρ‚ΡŒ ΡΠ»Π΅Π΄ΡƒΡŽΡ‰ΡƒΡŽ Ρ„ΠΎΡ€ΠΌΡƒΠ»Ρƒ:

CΠΎΠ±Ρ‰. = C1 x C2 ΠΈ Ρ‚Π°ΠΊ Π΄Π°Π»Π΅Π΅
C1 + C2

ΠŸΡ€ΠΈΠΌΠ΅Ρ€: Ρ€Π°ΡΡΡ‡ΠΈΡ‚Π°Ρ‚ΡŒ ΠΎΠ±Ρ‰ΡƒΡŽ Π΅ΠΌΠΊΠΎΡΡ‚ΡŒ для этих Π΄Π²ΡƒΡ… ΠΏΠΎΡΠ»Π΅Π΄ΠΎΠ²Π°Ρ‚Π΅Π»ΡŒΠ½ΠΎ соСдинСнных кондСнсаторов.


Π—Π°Π΄Π°Ρ‡Π° 2:

РассчитайтС ΠΎΠ±Ρ‰ΡƒΡŽ Π΅ΠΌΠΊΠΎΡΡ‚ΡŒ ΡΠ»Π΅Π΄ΡƒΡŽΡ‰ΠΈΡ… ΠΏΠΎΡΠ»Π΅Π΄ΠΎΠ²Π°Ρ‚Π΅Π»ΡŒΠ½ΠΎ Π²ΠΊΠ»ΡŽΡ‡Π΅Π½Π½Ρ‹Ρ… кондСнсаторов.

Π’Ρ€ΠΈ ΠΈΠ»ΠΈ Π±ΠΎΠ»Π΅Π΅ кондСнсатора ΠΏΠΎΡΠ»Π΅Π΄ΠΎΠ²Π°Ρ‚Π΅Π»ΡŒΠ½ΠΎ

Π§Ρ‚ΠΎΠ±Ρ‹ Ρ€Π°ΡΡΡ‡ΠΈΡ‚Π°Ρ‚ΡŒ ΠΎΠ±Ρ‰ΡƒΡŽ ΠΎΠ±Ρ‰ΡƒΡŽ Π΅ΠΌΠΊΠΎΡΡ‚ΡŒ Ρ‚Ρ€Π΅Ρ… ΠΈΠ»ΠΈ Π±ΠΎΠ»Π΅Π΅ кондСнсаторов, ΠΏΠΎΠ΄ΠΊΠ»ΡŽΡ‡Π΅Π½Π½Ρ‹Ρ… Ρ‚Π°ΠΊΠΈΠΌ ΠΎΠ±Ρ€Π°Π·ΠΎΠΌ, Π²Ρ‹ ΠΌΠΎΠΆΠ΅Ρ‚Π΅ ΠΈΡΠΏΠΎΠ»ΡŒΠ·ΠΎΠ²Π°Ρ‚ΡŒ ΡΠ»Π΅Π΄ΡƒΡŽΡ‰ΡƒΡŽ Ρ„ΠΎΡ€ΠΌΡƒΠ»Ρƒ: ΠΈ Ρ‚Π°ΠΊ Π΄Π°Π»Π΅Π΅.ΠŸΡ€ΠΈΠΌΠ΅Ρ€: Π²Ρ‹Ρ‡ΠΈΡΠ»ΠΈΡ‚ΡŒ ΠΎΠ±Ρ‰ΡƒΡŽ Π΅ΠΌΠΊΠΎΡΡ‚ΡŒ для этих Ρ‚Ρ€Π΅Ρ… ΠΏΠΎΡΠ»Π΅Π΄ΠΎΠ²Π°Ρ‚Π΅Π»ΡŒΠ½ΠΎ соСдинСнных кондСнсаторов.


Π—Π°Π΄Π°Ρ‡Π° 3:

РассчитайтС ΠΎΠ±Ρ‰ΡƒΡŽ Π΅ΠΌΠΊΠΎΡΡ‚ΡŒ ΡΠ»Π΅Π΄ΡƒΡŽΡ‰ΠΈΡ… ΠΏΠΎΡΠ»Π΅Π΄ΠΎΠ²Π°Ρ‚Π΅Π»ΡŒΠ½ΠΎ Π²ΠΊΠ»ΡŽΡ‡Π΅Π½Π½Ρ‹Ρ… кондСнсаторов.


ΠΎΡ‚Π²Π΅Ρ‚ΠΎΠ²

Π—Π°Π΄Π°Ρ‡Π° 1

1 = 232,2F 2 = 169,0F 3 = 7,0F

Π—Π°Π΄Π°Ρ‡Π° 2

1 = 2,48F 2 = 14,99F 3 = 4,11F

Π—Π°Π΄Π°Ρ‡Π° 3

1 = 3,33F 2 = 1,167F 3 = 0,35F ΠŸΡ€ΠΈΠΌΠ΅Ρ‡Π°Π½ΠΈΠ΅ ЗначСния кондСнсаторов Π² этом листС ΠΏΠΎΠ΄Π΄Π΅Ρ€ΠΆΠΈΠ²Π°ΡŽΡ‚ΡΡ высокими (Π±Π»ΠΈΠ·ΠΊΠΈΠΌΠΈ ΠΊ Π΅Π΄ΠΈΠ½ΠΈΡ†Π΅ ΠΈΠ»ΠΈ большС). Π­Ρ‚ΠΎ сдСлано для упрощСния обучСния.На самом Π΄Π΅Π»Π΅ типичная Π΅ΠΌΠΊΠΎΡΡ‚ΡŒ кондСнсатора Π½Π°ΠΌΠ½ΠΎΠ³ΠΎ мСньшС Π΅Π΄ΠΈΠ½ΠΈΡ†Ρ‹. Π—Π°Π³Ρ€ΡƒΠ·ΠΈΡ‚Π΅ pdf-Π²Π΅Ρ€ΡΠΈΡŽ этой страницы здСсь. Π£Π·Π½Π°Ρ‚ΡŒ большС ΠΎΠ± Π°Π²Ρ‚ΠΎΡ€Π΅ ΠΏΠΎΠ΄Ρ€ΠΎΠ±Π½Π΅Π΅ »Если Π²Ρ‹ нашли эту ΡΡ‚Π°Ρ‚ΡŒΡŽ ΠΏΠΎΠ»Π΅Π·Π½ΠΎΠΉ ΠΈ Ρ…ΠΎΡ‚Π΅Π»ΠΈ Π±Ρ‹ ΠΏΠΎΠ»ΡƒΡ‡Π°Ρ‚ΡŒ ΠΎΡ‚ нас обновлСния ΠΏΡ€ΠΎΠ΄ΡƒΠΊΡ‚ΠΎΠ² ΠΈ бСсплатныС элСктронныС рСсурсы, Π·Π°Ρ€Π΅Π³ΠΈΡΡ‚Ρ€ΠΈΡ€ΡƒΠΉΡ‚Π΅ΡΡŒ здСсь.

Π”ΠΎΠ±Π°Π²ΠΈΡ‚ΡŒ ΠΊΠΎΠΌΠΌΠ΅Π½Ρ‚Π°Ρ€ΠΈΠΉ

Π’Π°Ρˆ адрСс email Π½Π΅ Π±ΡƒΠ΄Π΅Ρ‚ ΠΎΠΏΡƒΠ±Π»ΠΈΠΊΠΎΠ²Π°Π½. ΠžΠ±ΡΠ·Π°Ρ‚Π΅Π»ΡŒΠ½Ρ‹Π΅ поля ΠΏΠΎΠΌΠ΅Ρ‡Π΅Π½Ρ‹ *